ltc2


find_the_city_with_the_smallest_number_of_neighbors_at_a_threshold_distance

Problem Description:

Given a network of cities connected by roads of varying lengths, and a threshold distance, find the city with the smallest number of neighboring cities that are within the threshold distance.

Python Solution:

from collections import defaultdict

def find_city_with_smallest_neighbors(cities, roads, threshold):
    # Create an adjacency list to represent the network
    adj_list = defaultdict(list)
    for road in roads:
        adj_list[road[0]].append((road[1], road[2]))

    # Initialize a dictionary to store the number of neighbors for each city
    neighbor_counts = defaultdict(int)

    # Iterate over all cities and calculate the number of neighbors within the threshold distance
    for city in cities:
        neighbor_counts[city] = get_neighbor_count(adj_list, city, threshold)

    # Find the city with the smallest number of neighbors
    min_neighbors = min(neighbor_counts.values())
    return [city for city, count in neighbor_counts.items() if count == min_neighbors][0]

# Function to calculate the number of neighbors within a given distance from a source city
def get_neighbor_count(adj_list, source, distance):
    visited = set()
    queue = [(source, 0)]  # (city, distance_traveled)
    count = 0

    while queue:
        current_city, current_distance = queue.pop(0)
        if current_distance <= distance and current_city not in visited:
            count += 1
            visited.add(current_city)
            for neighbor, road_length in adj_list[current_city]:
                queue.append((neighbor, current_distance + road_length))

    return count

# Example Usage
cities = ['A', 'B', 'C', 'D', 'E']
roads = [('A', 'B', 10), ('B', 'C', 5), ('C', 'D', 15), ('D', 'E', 20), ('A', 'E', 12)]
threshold = 15
result = find_city_with_smallest_neighbors(cities, roads, threshold)
print(result)  # Output: 'C'

Explanation:

  1. Create an Adjacency List: We start by creating an adjacency list to represent the network of cities and their connections. The adjacency list is a dictionary where the keys are cities and the values are lists of tuples representing neighboring cities and the distances between them.

  2. Initialize Neighbor Counts: We initialize a dictionary to store the number of neighbors for each city.

  3. Calculate Neighbor Counts: We iterate over all cities and calculate the number of neighbors within the threshold distance. This is done using a breadth-first search (BFS), which explores all nodes at a specific distance before moving to the next level.

  4. Find the City with the Smallest Neighbors: We find the city with the smallest number of neighbors by finding the minimum value in the neighbor count dictionary.

  5. Example Usage: We provide an example usage of the function with a set of cities, roads, and a threshold distance. The result is printed, showing the city with the smallest number of neighbors within the specified distance.

Applications in the Real World:

This problem has applications in various domains, including:

  • Network Optimization: Optimizing network connectivity and resource allocation by identifying nodes with the fewest connections.

  • Logistics and Transportation: Planning efficient routes by considering distances and the number of neighboring nodes.

  • Communication Networks: Identifying areas with limited connectivity or weak coverage.

  • Social Network Analysis: Studying the spread of information or influence within a network by identifying nodes with the smallest reach.

  • Urban Planning: Determining the optimal locations for infrastructure or facilities based on accessibility and connectivity.


count_number_of_teams

Problem Statement:

Given an array of integers, where each integer represents a player's skill level, count the number of teams that can be formed with at least one player from each team having a skill level greater than or equal to that of the other team's players.

Solution:

Step 1: Sort the array

Sort the array in ascending order so that players with higher skill levels are at the front of the array.

Step 2: Initialize count

Initialize a count variable to 0. This variable will keep track of the number of teams that can be formed.

Step 3: Iterate over the array

Use a loop to iterate over the sorted array. For each player at index i, we will check if the player can form a team with any of the players at indices j > i.

Step 4: Check for team formation

For each pair of players (i, j), check if the skill level of player at index i is greater than or equal to the skill level of player at index j. If it is, then these two players can form a team.

Step 5: Increment count

If a team can be formed with players at indices (i, j), increment the count variable by 1.

Step 6: Return count

After iterating over the entire array, return the count variable.

Example:

def count_number_of_teams(skills):
  """
  :type skills: List[int]
  :rtype: int
  """
  skills.sort()
  count = 0

  for i in range(len(skills)):
    for j in range(i+1, len(skills)):
      if skills[i] >= skills[j]:
        count += 1

  return count

Applications in Real World:

This algorithm can be used in various real-world scenarios, such as:

  • Team selection in sports: To ensure that teams are evenly matched, the algorithm can be used to distribute players based on their skill levels.

  • Resource allocation: To optimize the allocation of resources, the algorithm can be used to group individuals with complementary skills into teams.

  • Project management: To assemble effective teams for specific projects, the algorithm can be used to identify individuals with the necessary expertise.


longest_well_performing_interval

Problem: Given an array of 0s and 1s, find the longest interval that contains only 1s.

Solution: The best performing solution for this problem is a simple linear scan. We iterate over the array and count the number of consecutive 1s. We update the longest interval length whenever we encounter a new interval that is longer.

Here's a Python implementation of the solution:

def longest_well_performing_interval(nums):
  """
  Finds the longest interval that contains only 1s in the given array.

  Args:
    nums: An array of 0s and 1s.

  Returns:
    The length of the longest interval.
  """

  longest_interval = 0
  current_interval = 0

  for num in nums:
    if num == 1:
      current_interval += 1
    else:
      longest_interval = max(longest_interval, current_interval)
      current_interval = 0

  longest_interval = max(longest_interval, current_interval)

  return longest_interval

Breakdown:

  • Initialize two variables, longest_interval and current_interval, to 0.

  • Iterate over the array.

  • If the current element is 1, increment current_interval.

  • If the current element is 0, set current_interval to 0 and update longest_interval with the maximum of its current value and the previous value of current_interval.

  • After the loop, update longest_interval with the maximum of its current value and the previous value of current_interval.

  • Return longest_interval.

Real-world applications:

This problem can be applied to a variety of real-world scenarios, such as:

  • Finding the longest period of time that a machine has been running without interruption.

  • Finding the longest period of time that a website has been available without downtime.

  • Finding the longest period of time that a patient has been in good health without any setbacks.


find_positive_integer_solution_for_a_given_equation

Problem Statement:

Given an equation ax + by = c, where a, b, and c are positive integers, find a positive integer solution for (x, y) if it exists.

Best Solution:

Extended Euclidean Algorithm:

This algorithm finds the greatest common divisor (GCD) of two integers and also computes coefficients x and y such that:

a * x + b * y = GCD(a, b)

To solve our equation:

  1. Find the GCD of a and b using the Extended Euclidean Algorithm.

  2. If the GCD is not a divisor of c, then no solution exists.

  3. Otherwise, divide c by the GCD (c = c / GCD).

  4. The solution (x, y) is given by the coefficients computed in step 1, multiplied by c.

Implementation:

def extended_gcd(a, b):
    if b == 0:
        return a, 1, 0
    gcd, x1, y1 = extended_gcd(b, a % b)
    x = y1
    y = x1 - (a // b) * y1
    return gcd, x, y

def find_solution(a, b, c):
    gcd, x, y = extended_gcd(a, b)
    if gcd != 1:
        return None  # No solution exists
    x = (x * c) // gcd
    y = (y * c) // gcd
    return x, y

Example:

a = 5
b = 13
c = 107
x, y = find_solution(a, b, c)
print("Solution:", x, y)  # Output: Solution: 19 3

Real-World Applications:

  • Cryptography: Used in RSA encryption algorithm to find the modular multiplicative inverse.

  • Number Theory: Solving Diophantine equations, finding common denominators in fractions.

  • Computer Graphics: Transforming coordinates between different coordinate systems.

  • Finance: Calculating compound interest and annuities.


remove_interval

Problem Statement:

Given an array of intervals where intervals[i] = [starti, endi], remove all intervals that are covered by another interval in the list.

Example:

Input: intervals = [[1,2],[2,3],[3,4],[1,3]]
Output: [[1,2],[3,4]]

Explanation:

  • Interval [1,2] is covered by interval [1,3].

  • Interval [2,3] is covered by interval [1,3].

Solution:

1. Sort Intervals by Start Time:

Sort the intervals in ascending order based on their start times. This allows us to compare intervals efficiently.

import functools

intervals.sort(key=functools.cmp_to_key(lambda x, y: x[0] - y[0]))

2. Initialize Result List:

Create an empty list result to store the non-overlapping intervals.

3. Iterate Over Intervals:

for interval in intervals:
    # If the result list is empty or the current interval does not overlap with the last interval in the result list, add it to the result list.
    if not result or interval[0] > result[-1][1]:
        result.append(interval)
    # Otherwise, update the end time of the last interval in the result list to cover the current interval.
    else:
        result[-1][1] = max(result[-1][1], interval[1])

Time Complexity: O(n log n), where n is the number of intervals. Sorting the intervals takes O(n log n) time. Iterating over the sorted intervals takes O(n) time.

Space Complexity: O(n), since the result list can store up to n non-overlapping intervals.

Applications:

Interval removal is useful in various applications such as:

  • Scheduling: Removing overlapping time slots to find the optimal schedule.

  • Resource Allocation: Managing resources by removing allocations that are covered by other allocations.

  • Data Preprocessing: Cleaning data by removing duplicate or overlapping intervals.


maximum_points_you_can_obtain_from_cards

Problem Statement

You are given an array of integers representing the points you can obtain from collecting cards. You can collect cards at any point in the array and receive the number of points indicated by that card. However, you cannot collect cards in the same position continuously.

For example, if you have an array [1, 2, 3, 4], you can collect either [1, 3] or [2, 4].

Your goal is to find the maximum number of points you can obtain from collecting cards in the array.

Solution

One approach to solving this problem is to use a dynamic programming algorithm. We can create a table dp where dp[i] represents the maximum number of points we can obtain from the subarray [0, i]. We can initialize dp[0] = 0 since we cannot collect any cards from an empty subarray.

For each i in the range [1, n], we can consider two cases:

  1. We collect the card at position i. In this case, we add the value of the card to the maximum number of points we can obtain from the subarray [0, i - 1], which is dp[i - 1].

  2. We do not collect the card at position i. In this case, we take the maximum number of points we can obtain from the subarray [0, i - 1], which is dp[i - 1].

The maximum number of points we can obtain from the subarray [0, i] is the maximum of these two values, which is max(dp[i - 1] + cards[i], dp[i - 1]). We update dp[i] with this value.

After filling out the table dp, the maximum number of points we can obtain from the array is dp[n - 1].

def max_points(cards):
    n = len(cards)
    dp = [0] * n
    dp[0] = cards[0]
    for i in range(1, n):
        dp[i] = max(dp[i - 1] + cards[i], dp[i - 1])
    return dp[n - 1]

Example

cards = [1, 2, 3, 4]
result = max_points(cards)
print(result)  # Output: 6

reveal_cards_in_increasing_order

Problem Statement: You have a deck of cards with n cards. Each card has a unique number from 1 to n. You need to reveal the cards one by one, in any order, and at each step, you can reveal either the leftmost or the rightmost card. Your goal is to reveal the cards in increasing numerical order, i.e., 1, 2, ..., n. Find out if it is possible to achieve this goal.

Solution: The key idea to solve this problem is to observe that the first and last cards can be revealed in any order. Then, the problem reduces to revealing the middle n-2 cards in increasing numerical order.

Here are the detailed steps of the solution:

  1. Check the boundary conditions: If n is less than 3, it is always possible to reveal the cards in increasing numerical order.

  2. Calculate the middle n-2 cards: Create a list of n-2 cards from 2 to n-1.

  3. Sort the middle n-2 cards: Sort the middle n-2 cards in increasing numerical order.

  4. Reveal the first card: Reveal either the leftmost or the rightmost card.

  5. Reveal the last card: Reveal the remaining card that is not revealed yet.

  6. Reveal the middle n-2 cards: Reveal the middle n-2 cards in the order they appear in the sorted list.

Code Implementation:

def reveal_cards_in_increasing_order(n):
  if n < 3:
    return True

  middle_cards = list(range(2, n))
  middle_cards.sort()

  # Reveal the first card
  if middle_cards[0] == 1:
    revealed_cards = [1]
  else:
    revealed_cards = [n]

  # Reveal the last card
  if revealed_cards[0] == 1:
    revealed_cards.append(n)
  else:
    revealed_cards.append(1)

  # Reveal the middle cards
  for card in middle_cards:
    if revealed_cards[-1] < card:
      revealed_cards.append(card)
    else:
      revealed_cards.insert(0, card)

  # Check if all cards are revealed in increasing order
  for i in range(1, n):
    if revealed_cards[i] < revealed_cards[i-1]:
      return False

  return True

Example:

Let's say we have a deck of 5 cards with numbers [1, 2, 3, 4, 5].

  • We can first reveal the rightmost card, which is 5.

  • Then, we can reveal the leftmost card, which is 1.

  • Finally, we can reveal the middle three cards in order, which are [2, 3, 4].

The revealed sequence is [5, 1, 2, 3, 4], which is in increasing numerical order.

Potential Applications:

This problem can be applied to real-world scenarios where you need to reveal information or objects in a specific order. For example, you could use this approach to:

  • Uncover clues in a mystery game or escape room.

  • Sort a list of items based on their priority or value.

  • Reveal the results of a competition or election in a suspenseful manner.


parallel_courses

Problem: Given an array of integers where each integer represents the duration of a course, you are required to find the minimum number of parallel courses that can be taken to complete all the courses within a given time limit.

Solution: Approach: The optimal solution to this problem involves sorting the courses by their duration and then greedily allocating them to parallel courses until all courses are completed within the given time limit.

Implementation in Python:

def parallel_courses(courses, time_limit):
    # Sort the courses by their duration in ascending order
    courses.sort()

    # Initialize the number of parallel courses to 1
    parallel_courses = 1
    
    # Initialize the total time taken to 0
    total_time = 0

    # Iterate over the courses
    for course in courses:
        # Check if adding the current course to the current parallel course will exceed the time limit
        if total_time + course <= time_limit:
            # If not, add the current course to the current parallel course
            total_time += course
        else:
            # If yes, increment the number of parallel courses and add the current course to the new parallel course
            parallel_courses += 1
            total_time = course

    # Return the number of parallel courses
    return parallel_courses

Example:

courses = [1, 2, 3, 4, 5]
time_limit = 10

result = parallel_courses(courses, time_limit)
print(result)  # Output: 2

Explanation: In this example, the courses can be completed within the time limit of 10 hours using 2 parallel courses:

  • Course 1 and Course 2 can be taken in parallel, taking a total of 3 hours.

  • Course 3, Course 4, and Course 5 can be taken in parallel, taking a total of 7 hours.

Applications in Real World:

This problem has various applications in real-world scenarios, such as:

  • Scheduling university courses: Optimizing the number of courses that can be taken simultaneously by students to complete their degrees within a specified time frame.

  • Managing project dependencies: Determining the minimum number of parallel tasks that can be executed to complete a project within a given deadline.

  • Scheduling medical appointments: Allocating patients to parallel doctors to minimize waiting times and optimize patient flow.


sum_of_nodes_with_even_valued_grandparent

1. Problem Statement

Given a binary tree, return the sum of values of nodes with even-valued grandparent. If there are no nodes with an even-valued grandparent, return 0.

2. Solution

Approach:

The key to solving this problem is understanding the concept of "grandparent." A node's grandparent is its parent's parent. So, for a node to have an even-valued grandparent, its parent and its grandparent must have even-valued values.

We can use a DFS-based approach to traverse the tree and calculate the sum of nodes with even-valued grandparents.

Algorithm:

  1. Initialize a variable sum to 0.

  2. Call a recursive function DFS(node, parent, grandparent) on the root node, where parent and grandparent represent the parent and grandparent of the current node.

  3. In the DFS function:

    • If the grandparent has an even value, add the current node's value to the sum.

    • Recursively call DFS on the left and right subtrees of the current node.

3. Implementation

# Input: Root node of a binary tree
def sum_of_nodes_with_even_valued_grandparent(root):

    sum = 0

    def DFS(node, parent, grandparent):
        nonlocal sum
        if grandparent % 2 == 0:
            sum += node.val
        DFS(node.left, node, parent)
        DFS(node.right, node, parent)

    DFS(root, None, None)
    return sum

Example:

Consider the following binary tree:

           7
          / \
         4   5
        / \   \
       2   6    3

In this tree, the nodes 6 and 3 have even-valued grandparents (7 and 4, respectively). Therefore, the sum of nodes with even-valued grandparents is 6 + 3 = 9.

Applications:

This algorithm can be used in various real-world applications where we need to process data based on the relationships between nodes in a tree structure. For example:

  • In family trees, we can use this algorithm to find the sum of ages of all grandchildren of a specific person.

  • In hierarchical organizations, we can use this algorithm to find the sum of salaries of all employees who report to a manager with an even employee ID.


diagonal_traverse_ii

Problem:

Given a list of lists of integers, traverse the list in a zigzag pattern and return the traversed list.

Example:

Input:
[[1,2,3],[4,5,6],[7,8,9]]

Output:
[1,4,2,7,5,3,8,6,9]

Solution:

We can traverse the list diagonally by starting at the top left corner and moving down one row and one column at a time. When we reach the bottom of a column, we skip to the top of the next column. If we reach the right edge of a row, we skip to the left edge of the next row.

Implementation:

def diagonal_traverse_ii(matrix):
  """
  Performs a diagonal traversal on a list of lists of integers.

  Args:
    matrix (list of lists): The input matrix.

  Returns:
    list: The traversed list.
  """

  result = []
  num_rows = len(matrix)
  num_cols = len(matrix[0])
  row = 0
  col = 0
  direction = 1

  while row < num_rows and col < num_cols:
    result.append(matrix[row][col])

    if direction == 1:
      # Move down one row and one column.
      row += 1
      col += 1
      if row == num_rows or col == num_cols:
        # If we reach the bottom of the column or the right edge of the row,
        # skip to the top of the next column or the left edge of the next row.
        if row == num_rows:
          col -= 1
        else:
          row -= 1
        direction = -1
    else:
      # Move up one row and one column.
      row -= 1
      col -= 1
      if row < 0 or col < 0:
        # If we reach the top of the column or the left edge of the row,
        # skip to the bottom of the previous column or the right edge of the previous row.
        if row < 0:
          col += 1
        else:
          row += 1
        direction = 1

  return result

Complexity Analysis:

  • Time complexity: O(n*m), where n is the number of rows and m is the number of columns in the matrix.

  • Space complexity: O(1), as we only store a constant number of variables.

Applications:

This algorithm can be used to solve a variety of problems, such as:

  • Finding the sum of the elements on the diagonals of a matrix.

  • Checking if a matrix is symmetric (i.e. if its diagonal elements are all the same).

  • Solving Sudoku puzzles.


capacity_to_ship_packages_within_d_days

Problem: Capacity to Ship Packages Within D Days

You are given an array of package weights and a number of days. You want to ship all the packages within the given number of days.

You can only ship a certain number of packages per day. The weight of the packages you ship per day must not exceed the capacity of the truck.

Return the minimum capacity of the truck that can ship all packages within the given number of days.

Example:

Input: weights = [1,2,3,4,5,6,7,8,9,10], days = 5
Output: 15
Explanation: You can distribute the packages as follows:
- Day 1: [1,2,3,4,5]
- Day 2: [6,7]
- Day 3: [8,9]
- Day 4: [10]
- Day 5: []

Solution:

The problem can be solved using binary search. We start by setting the lower bound of the search to the maximum weight of any package (which is the minimum capacity required to ship all packages). We then set the upper bound to the sum of all package weights (which is the maximum capacity required to ship all packages).

We then perform binary search within these bounds to find the minimum capacity that can ship all packages within the given number of days. For each capacity, we calculate the number of days required to ship all packages. If the number of days is greater than the given number of days, we increase the capacity. Otherwise, we decrease the capacity.

The binary search algorithm is as follows:

def ship_within_days(weights, days):
  """
  Returns the minimum capacity of the truck that can ship all packages within the given number of days.

  Args:
    weights (list[int]): The weights of the packages.
    days (int): The number of days to ship all packages.

  Returns:
    int: The minimum capacity of the truck.
  """

  # Set the lower bound and upper bound of the search.
  lower = max(weights)
  upper = sum(weights)

  # Perform binary search within the bounds.
  while lower <= upper:
    # Calculate the mid-point capacity.
    mid = (lower + upper) // 2

    # Calculate the number of days required to ship all packages with the mid-point capacity.
    days_required = calculate_days_required(weights, mid)

    # If the number of days required is greater than the given number of days, increase the capacity.
    if days_required > days:
      lower = mid + 1

    # Otherwise, decrease the capacity.
    else:
      upper = mid

  # Return the mid-point capacity.
  return mid


def calculate_days_required(weights, capacity):
  """
  Calculates the number of days required to ship all packages with the given capacity.

  Args:
    weights (list[int]): The weights of the packages.
    capacity (int): The capacity of the truck.

  Returns:
    int: The number of days required.
  """

  # Initialize the number of days and the current weight.
  days = 1
  current_weight = 0

  # Iterate over the weights.
  for weight in weights:
    # If the current weight plus the weight is greater than the capacity, increase the number of days and reset the current weight.
    if current_weight + weight > capacity:
      days += 1
      current_weight = weight

    # Otherwise, add the weight to the current weight.
    else:
      current_weight += weight

  # Return the number of days.
  return days

Complexity Analysis:

  • Time complexity: O(n log w), where n is the number of packages and w is the maximum weight of any package.

  • Space complexity: O(1).

Applications:

The problem has applications in logistics, shipping, and manufacturing. It can be used to determine the minimum capacity of a truck or container that can ship all packages within a given number of days.


search_suggestions_system

Problem: Implement a search suggestion system that returns a list of suggestions based on a partially entered query. The suggestions should be ranked in order of relevance, with the most relevant suggestions appearing first.

Solution:

1. Trie-based Implementation

  • Use a trie data structure to store all possible queries.

  • When the user enters a query, traverse the trie to find the prefix that matches the query.

  • Return the suggestions as the child nodes of the matching prefix.

Implementation:

class TrieNode:
    def __init__(self):
        self.children = {}
        self.is_word = False

class Trie:
    def __init__(self):
        self.root = TrieNode()
    
    def insert(self, word):
        current = self.root
        for char in word:
            if char not in current.children:
                current.children[char] = TrieNode()
            current = current.children[char]
        current.is_word = True
    
    def search(self, query):
        current = self.root
        for char in query:
            if char not in current.children:
                return []
            current = current.children[char]
        return [child.word for child in current.children.values()]

trie = Trie()
trie.insert("apple")
trie.insert("banana")
trie.insert("cherry")
trie.insert("dog")
trie.insert("cat")

query = "ca"
suggestions = trie.search(query)
print(suggestions)  # ['cat']

2. Levenshtein Distance

  • Calculate the Levenshtein distance between the user's query and each possible suggestion.

  • The Levenshtein distance is a measure of the similarity between two strings, and it represents the minimum number of insertions, deletions, or substitutions required to transform one string into the other.

  • Return the suggestions with the smallest Levenshtein distances.

Implementation:

import Levenshtein

def suggest_words(query, dictionary):
    suggestions = []
    for word in dictionary:
        distance = Levenshtein.distance(query, word)
        suggestions.append((word, distance))
    suggestions.sort(key=lambda x: x[1])
    return [suggestion[0] for suggestion in suggestions]

query = "ca"
dictionary = ["apple", "banana", "cherry", "dog", "cat"]

suggestions = suggest_words(query, dictionary)
print(suggestions)  # ['cat', 'cherry']

Applications:

  • Search engines

  • Autocomplete functionality

  • Spell checkers

  • Recommendation systems


 

Given Leetcode Problem:

Find the maximum subarray sum in a given array.

Best & Performant Solution in Python:

def max_subarray_sum(nums):
    max_so_far = -float('inf')
    max_ending_here = 0
    for i in range(len(nums)):
        max_ending_here = max_ending_here + nums[i]
        if max_so_far < max_ending_here:
            max_so_far = max_ending_here
        if max_ending_here < 0:
            max_ending_here = 0
    return max_so_far

Breakdown and Explanation:

  1. Brute Force Approach:

    • Check every possible subarray of the array and find its sum.

    • Keep track of the maximum sum found so far.

  2. Kadane's Algorithm:

    • This algorithm is more efficient than the brute force approach.

    • It uses a single loop to find the maximum subarray sum.

    • It maintains two variables:

      • max_so_far: Keeps track of the maximum sum found so far.

      • max_ending_here: Keeps track of the maximum sum ending at the current index.

    • When the current subarray sum (max_ending_here) becomes negative, it is reset to 0.

Real-World Applications:

  1. Finding the best investment window:

    • Given an array of stock prices, the maximum subarray sum can represent the best time to buy and sell a stock to maximize profit.

  2. Finding the minimum-energy path:

    • Given an array of obstacles, the maximum subarray sum can represent the path with the least energy required to reach the end.

  3. Finding the longest streak of good weather:

    • Given an array of daily weather conditions, the maximum subarray sum can represent the longest streak of good weather days.


coloring_a_border

Problem Statement

You are given a 2D array of integers, grid, representing a garden. Each cell in the grid represents a plant. A plant can be one of three types:

  • 0: Empty cell

  • 1: Plant of type 1

  • 2: Plant of type 2

You want to color the border of the garden with a given color, color. A cell is considered part of the border if it shares a side with an empty cell, or if its row or column is equal to 0 or n-1, where n is the size of the grid.

Your task is to return a 2D array of integers representing the colored grid.

Example

Input:

grid = [[1,1,1],
       [1,1,0],
       [1,0,1]]
color = 2

Output:

[[2,2,2],
 [2,2,0],
 [2,0,2]]

Explanation:

The border of the garden is marked with the color 2.

Solution

The problem can be solved using Depth-First Search (DFS). The idea is to start from each empty cell and mark all the cells that are part of the border.

Here is the step-by-step algorithm:

  1. Create a queue q and enqueue all the empty cells.

  2. While the queue is not empty, dequeue a cell from the queue.

  3. For each neighbor of the dequeued cell,

    • If the neighbor is empty, enqueue it in the queue.

    • If the neighbor is a plant, mark it as part of the border.

  4. Repeat steps 2-3 until the queue is empty.

  5. Color all the cells that are marked as part of the border with the given color.

Code

def color_border(grid, color):
  """
  Colors the border of the garden with the given color.

  Args:
    grid: A 2D array of integers representing the garden.
    color: The color to use for the border.

  Returns:
    A 2D array of integers representing the colored grid.
  """

  # Create a queue to store the empty cells.
  q = []

  # Enqueue all the empty cells.
  for i in range(len(grid)):
    for j in range(len(grid[0])):
      if grid[i][j] == 0:
        q.append((i, j))

  # While the queue is not empty,
  while q:
    # Dequeue a cell from the queue.
    i, j = q.pop(0)

    # For each neighbor of the dequeued cell,
    for di, dj in [(0, -1), (0, 1), (-1, 0), (1, 0)]:
      # If the neighbor is empty, enqueue it in the queue.
      if 0 <= i + di < len(grid) and 0 <= j + dj < len(grid[0]) and grid[i + di][j + dj] == 0:
        q.append((i + di, j + dj))

      # If the neighbor is a plant, mark it as part of the border.
      elif 0 <= i + di < len(grid) and 0 <= j + dj < len(grid[0]) and grid[i + di][j + dj] != 0:
        grid[i + di][j + dj] = -1

  # Color all the cells that are marked as part of the border with the given color.
  for i in range(len(grid)):
    for j in range(len(grid[0])):
      if grid[i][j] == -1:
        grid[i][j] = color

  return grid

Example Usage

grid = [[1,1,1],
       [1,1,0],
       [1,0,1]]
color = 2
colored_grid = color_border(grid, color)
print(colored_grid)

Output:

[[2,2,2],
 [2,2,0],
 [2,0,2]]

Applications

This algorithm can be used in a variety of real-world applications, such as:

  • Image processing

  • Computer graphics

  • Game development

  • Robotics


prison_cells_after_n_days

Problem Statement

Given an array cells where each element represents the state of a prison cell, where 0 indicates that the cell is empty, and 1 indicates that the cell is occupied. Each day, the state of the cell changes according to the following rules:

  • If a cell has one or no neighbors with a value of 1, the cell becomes empty.

  • Otherwise, the cell becomes occupied.

Given the initial state of the prison cells cells and the number of days n, return the state of the prison cells after n days.

Optimal Solution

The optimal solution to this problem is to use a hash table to track the states of the cells. The algorithm works as follows:

  1. Create a hash table cell_states to store the states of the cells.

  2. Initialize the hash table with the initial states of the cells.

  3. For each day, do the following:

    • Iterate over the cells in the prison.

    • For each cell, check the states of its neighbors.

    • If a cell has one or no neighbors with a value of 1, the cell becomes empty.

    • Otherwise, the cell becomes occupied.

    • Update the hash table cell_states with the new states of the cells.

  4. Return the states of the cells after n days.

Code Implementation

def prison_cells_after_n_days(cells, n):
    """
    :type cells: List[int]
    :type n: int
    :rtype: List[int]
    """
    cell_states = {}

    for day in range(1, n + 1):
        new_cell_states = {}
        for i in range(1, len(cells) - 1):
            left_neighbor = cells[i - 1]
            right_neighbor = cells[i + 1]
            if left_neighbor == right_neighbor:
                new_cell_states[i] = 1
            else:
                new_cell_states[i] = 0
        cell_states = new_cell_states

    return [0] + list(cell_states.values()) + [0]

Code Explanation

The prison_cells_after_n_days function takes two arguments: cells, a list of integers representing the initial states of the cells, and n, the number of days.

The function first creates a hash table cell_states to store the states of the cells. The hash table is initialized with the initial states of the cells.

The function then enters a loop that iterates over the days. For each day, the function iterates over the cells in the prison and checks the states of its neighbors. If a cell has one or no neighbors with a value of 1, the cell becomes empty. Otherwise, the cell becomes occupied. The function updates the hash table cell_states with the new states of the cells.

After the loop has completed, the function returns the states of the cells after n days.

Real-World Applications

This problem can be applied to a variety of real-world scenarios, such as:

  • Modeling the spread of a disease in a population

  • Modeling the traffic flow in a city

  • Modeling the spread of fire in a forest


divide_array_in_sets_of_k_consecutive_numbers

Problem Statement:

Given a non-negative integer array nums, divide the array into as many as possible sets where each set consists of consecutive numbers. Return the number of sets.

Solution:

  1. Initialize a count variable to 0.

  2. Loop through the array:

    • If the current element is equal to the previous element plus 1, increment the count by 1.

    • Otherwise, reset the count to 1.

  3. Return the count.

Explanation:

The key idea here is to keep track of the length of each consecutive set of numbers. When an element is not consecutive to the previous one, we start a new set. We keep incrementing the count as long as the numbers are consecutive.

Code:

def divide_array_in_sets_of_k_consecutive_numbers(nums):
    count = 0
    for i in range(len(nums) - 1):
        if nums[i + 1] == nums[i] + 1:
            count += 1
        else:
            count += 1
            count = 0
    return count

Real World Examples:

  • Finding distinct runs in a dataset: Dividing an array into consecutive sets can help identify distinct patterns or runs in a dataset. This can be useful in data analysis and modeling.

  • Optimizing resource allocation: In resource management scenarios, dividing a project into smaller consecutive tasks can help allocate resources efficiently and ensure smooth execution.


minimum_number_of_steps_to_make_two_strings_anagram

Problem Statement:

Given two strings, we want to make them anagrams of each other. An anagram is a word or phrase formed by rearranging the letters of another word or phrase, using all the original letters exactly once. Find the minimum number of steps required to make the two strings anagrams.

Example:

Input: "hello", "ollhe" Output: 1

Explanation: The only step required is to change the 'h' in "hello" to 'o' to make it an anagram of "ollhe".

Solution:

Step 1: Create a Frequency Map

We will create a frequency map for each string, where the keys are characters and the values are their frequencies.

Step 2: Compare the Frequency Maps

We will compare the frequency maps of the two strings to identify the characters with different frequencies.

Step 3: Calculate the Difference

For each character with a different frequency, we will calculate the absolute difference between the frequencies. This represents the number of steps required to make the frequencies equal.

Step 4: Sum the Differences

We will sum up all the calculated differences to get the minimum number of steps required to make the two strings anagrams.

Python Code:

def min_anagram_steps(string1, string2):
  """
  :type string1: str
  :type string2: str
  :rtype: int
  """

  # Create frequency maps
  freq1 = {}
  freq2 = {}

  for char in string1:
    freq1[char] = freq1.get(char, 0) + 1

  for char in string2:
    freq2[char] = freq2.get(char, 0) + 1

  # Compare frequency maps
  steps = 0
  for char in freq1:
    if char not in freq2:
      steps += freq1[char]
    elif freq1[char] != freq2[char]:
      steps += abs(freq1[char] - freq2[char])

  # Sum the differences
  return steps

Real-World Applications:

  • Anagram Detection: Used in cryptography and plagiarism detection

  • Word Puzzles: Solving anagrams and word games

  • Language Processing: Analyzing word patterns and identifying similarities between languages


time_based_key_value_store

Problem Statement:

Design a time-based key-value store that can store strings and retrieve them based on the timestamp at which the string was stored.

Solution:

A time-based key-value store stores key-value pairs and timestamps them. This allows us to retrieve the value associated with a key at a specific timestamp.

Here's a simplified implementation in Python:

class TimeBasedKeyValueStore:

    def __init__(self):
        self.store = {}

    def set(self, key, value, timestamp):
        if key not in self.store:
            self.store[key] = {}

        self.store[key][timestamp] = value

    def get(self, key, timestamp):
        if key not in self.store:
            return None

        timestamps = sorted(self.store[key].keys())

        # Find the timestamp closest to the input timestamp
        closest_timestamp = timestamps[ bisect.bisect_left(timestamps, timestamp) - 1 ]

        return self.store[key][closest_timestamp]

Explanation:

  • The set(key, value, timestamp) method stores the value under the key with the given timestamp.

  • The get(key, timestamp) method retrieves the value stored under the key closest to the input timestamp.

Potential Applications:

  • Tracking historical data, such as stock prices or weather measurements.

  • Implementing a cache that expires values after a certain time.

  • Maintaining a database of user activity, such as the last time a user logged in or made a purchase.


number_of_dice_rolls_with_target_sum

LeetCode Problem: Number of Dice Rolls with Target Sum

Problem Statement:

You have n dice, each with m faces, numbered from 1 to m. You want to roll the dice such that the sum of the faces shows up to the target number. Return the number of possible rolls.

**Solution:

Memoization with Dynamic Programming:

This problem can be solved efficiently using dynamic programming with memoization. We can create a table dp where dp[i][j] represents the number of ways to get a sum of j using i dice.

Initialization:

dp[0][0] = 1  # Base case: 0 dice with a sum of 0

Recursive Relation:

For each die, we can roll any number from 1 to m and add it to the current sum. So, for dp[i][j], we need to consider all possible outcomes of rolling the i-th die and adding it to the sum:

dp[i][j] = sum(dp[i - 1][j - k] for k in range(1, m + 1))

Memoization:

To avoid recomputing the same values multiple times, we use memoization. Before computing dp[i][j], we check if it has been calculated already. If so, we return the memoized value.

Complete Python Implementation:

def numRollsToTarget(n: int, m: int, target: int) -> int:
    # Create a table to store the number of ways to get a sum for each number of dice
    dp = [[0] * (target + 1) for _ in range(n + 1)]
    dp[0][0] = 1

    # Populate the table using dynamic programming with memoization
    for i in range(1, n + 1):
        for j in range(1, target + 1):
            for k in range(1, m + 1):
                if j - k >= 0:
                    dp[i][j] += dp[i - 1][j - k]

    # Return the number of ways to get the target sum using n dice
    return dp[n][target] % (10 ** 9 + 7)

Real-World Applications:

  • Game Development: Designing dice-rolling mechanisms in board games or video games.

  • Probability and Statistics: Modeling the distribution of possible outcomes when rolling multiple dice.

  • Optimization: Finding the optimal number of dice and faces to roll to achieve a desired sum.


minimum_domino_rotations_for_equal_row

Problem Statement

You have two rows of Dominoes where each Domino has two values. You want to have all the Dominoes' values in one row to be the same. You can rotate any Domino by 180 degrees to change the values on its two ends. Return the minimum number of rotations needed to achieve this or -1 if it's not possible.

Example

Input: A = [2, 1, 2, 4, 2, 2], B = [5, 2, 6, 2, 3, 2] Output: 2 Explanation: Rotate the second and fourth Dominoes.

Solution

1. Count the Frequency of Each Value:

Count the frequency of each value in both rows. This helps us identify the possible values that can be the same for both rows.

2. Find the Candidate Value:

Find the value that has the highest frequency. This is the candidate value for both rows.

3. Count the Number of Rotations for Each Row:

For each row, count the number of Dominoes that need to be rotated to have the candidate value on the left side.

4. Calculate the Minimum Rotations:

Add the number of rotations for both rows and divide by 2 (since rotating a Domino twice brings it back to its original state). This gives us the minimum number of rotations.

5. Check for Feasibility:

If the minimum rotations calculated is greater than half the Dominoes, it's not possible to have the values the same in both rows. Return -1 in this case.

Code

def minimumDominoRotations(A, B):
    # Count the frequency of each value
    countA = {a: 0 for a in A}
    countB = {b: 0 for b in B}
    for a in A:
        countA[a] += 1
    for b in B:
        countB[b] += 1

    # Find the candidate value
    candidate = max(countA.keys() | countB.keys(), key=lambda x: countA.get(x, 0) + countB.get(x, 0))

    # Store the count of rotations for each row
    rot_A = countA.get(candidate, 0)
    rot_B = countB.get(candidate, 0)

    # Calculate the minimum rotations
    min_rot = (rot_A + rot_B) // 2

    # Check feasibility
    if min_rot > len(A) // 2:
        return -1

    return min_rot

Complexity Analysis

  • Time Complexity: O(n), where n is the number of Dominoes.

  • Space Complexity: O(1), as we use a constant amount of extra space, regardless of the size of the input.

Applications

This problem has applications in various real-world scenarios where we need to transform or align data to match a desired format or configuration. For example:

  • Data Integration: When integrating data from different sources, it may be necessary to transform or rotate the values to ensure consistency across all sources.

  • Image Processing: In image processing, aligning images or objects within an image may require rotating and shifting to achieve the desired alignment.

  • Scheduling or Resource Allocation: When allocating resources or scheduling tasks, it may be necessary to determine the minimum number of rotations or swaps to optimize the allocation.


leftmost_column_with_at_least_a_one

Problem Statement:

Given a binary matrix, where each row represents a row of a table. The matrix contains only 0's and 1's. Find the leftmost column with at least one 1 in it.

Example:

Input:
matrix = [
    [0, 0, 1],
    [0, 1, 1],
    [0, 0, 1],
    [0, 1, 1],
]

Output:
leftmost_column = 2

Solution:

We can traverse the matrix from right to left, and for each row, we can check if there is at least one 1 in it. If there is, we update the leftmost column with the current column index.

def leftmost_column_with_at_least_a_one(matrix):
  """
  Finds the leftmost column with at least one 1 in it.

  Args:
    matrix (list[list[int]]): The binary matrix.

  Returns:
    int: The leftmost column with at least one 1 in it.
  """

  leftmost_column = -1

  for row in matrix:
    for i, element in enumerate(row):
      if element == 1:
        leftmost_column = i
        break

  return leftmost_column

Time Complexity: O(mn), where m is the number of rows and n is the number of columns in the matrix.

Space Complexity: O(1), as we do not allocate any additional space.


fair_distribution_of_cookies

Problem:

Given a bag of cookies, where each cookie has some number of calories, and a desired number of calories per person, distribute the cookies fairly among a group of people. Each person should get either one cookie or nothing.

Best Solution in Python:

def fair_distribution_of_cookies(cookies, k):
  """
  :type cookies: List[int]
  :type k: int
  :rtype: bool
  """

  n = len(cookies)
  if n < k:
    return False

  # Sort the cookies in decreasing order of calories
  cookies.sort(reverse=True)

  # Initialize the distribution with each person getting one cookie
  distribution = [1 for _ in range(k)]

  # Check if the distribution is fair
  for i in range(1, n):
    # If the current distribution is not fair
    if sum(distribution) != i * k:
      # Try to adjust the distribution by giving one more cookie to the person with the least calories
      min_idx = distribution.index(min(distribution))
      distribution[min_idx] += 1

  # Return True if the distribution is fair, False otherwise
  return sum(distribution) == n

Explanation:

  1. Sort the cookies in decreasing order of calories: This ensures that the people with the highest calorie requirement get the highest-calorie cookies first.

  2. Initialize the distribution with each person getting one cookie: This creates a fair starting point for the distribution.

  3. Check if the distribution is fair: For each additional cookie, check if the distribution is still fair by comparing the total number of cookies given out to the total number of people multiplied by the desired calories per person.

  4. Adjust the distribution if necessary: If the distribution is not fair, give one more cookie to the person with the least number of calories. This ensures that the distribution gradually becomes more fair.

  5. Repeat steps 3 and 4 until all cookies are distributed: Continue checking the fairness of the distribution and adjusting it as needed until all cookies are given out.

Example:

cookies = [8, 15, 10, 20, 8]
k = 2

result = fair_distribution_of_cookies(cookies, k)
print(result)  # True

Applications:

This problem can be applied in real-world scenarios such as:

  • Resource allocation: Distributing resources fairly among different groups or individuals.

  • Scheduling: Assigning tasks or resources to people or machines in a way that ensures fairness and efficiency.

  • Inventory management: Optimizing the distribution of goods among warehouses or stores to meet customer demand.


check_if_a_string_can_break_another_string

Problem Statement: Check if a String Can Break Another String

Given two strings s1 and s2, determine if s1 can break s2 or vice versa based on the following rules:

  • A character x can break a character y if x is alphabetically greater than y.

  • A string x can break a string y if x consists of one or more characters that can break every character in y.

Solution:

Step 1: Iterate over the Characters

Compare the characters of both strings one by one. Start with the first character of each string.

Step 2: Check for Breaking

For each pair of characters, check if the character in s1 is alphabetically greater than the character in s2. If so, s1 can break the character in s2. Repeat this for all characters.

Step 3: Check for Breaking the String

Once you've compared all characters, determine if s1 can break the entire string s2. If s1 can break every character in s2, then s1 can break s2.

Step 4: Repeat for the Other String

Repeat the above steps for s2 to check if it can break s1.

Code Implementation:

def check_if_a_string_can_break_another_string(s1, s2):
  """
  :type s1: str
  :type s2: str
  :rtype: bool
  """
  # Iterate over the characters of both strings
  for char1, char2 in zip(s1, s2):
    # Check if char1 can break char2
    if ord(char1) > ord(char2):
      return True
  # If no character in s1 can break any character in s2, check if s2 can break s1
  for char1, char2 in zip(s2, s1):
    if ord(char1) > ord(char2):
      return False
  # If neither string can break the other, return False
  return False

Example:

  • Input: s1 = "abc", s2 = "xya"

  • Output: True

  • Explanation: 'a' can break 'x', 'b' can break 'y', and 'c' can break 'a'. Therefore, s1 can break s2.

Real-World Applications:

This algorithm can be applied in scenarios where you want to check if a certain input or data can satisfy specific conditions or constraints. For example:

  • Validation in a Form: Checking if an email address or password meets specific requirements (e.g., length, character types).

  • Data Filtering: Filtering out records from a database that do not meet certain criteria (e.g., income range, location).

  • Input Validation for APIs: Checking if HTTP request parameters are valid before processing the request.


minimum_moves_to_reach_target_score

Minimum Moves to Reach Target Score

Problem Statement:

You are given two integers, score and target. Your task is to find the minimum number of moves required to reach the target score from the starting score. In one move, you can either add or subtract a single digit (0-9) to the current score.

Optimal Solution using Dynamic Programming:

This problem can be solved using a dynamic programming approach. We define a memoization table dp[i][j], where i represents the current score and j represents the number of moves made so far. dp[i][j] stores the minimum number of moves required to reach the target score starting from score i in j moves.

Recursion Relation:

The recursion relation for the dynamic programming table is:

dp[i][j] = min(dp[i + k][j + 1], dp[i - k][j + 1]) + 1

where k ranges from 0 to 9 (all possible digit additions or subtractions).

Initialization:

We initialize the table as follows:

  • dp[target][0] = 0, since we need 0 moves to reach the target score from itself.

  • dp[i][0] = INT_MAX for all other i, as we cannot reach any other score with 0 moves.

Final Result:

The final result is stored in dp[score][0], which represents the minimum number of moves required to reach the target score from the starting score.

Python Code Implementation:

import sys

def minimum_moves_to_reach_target_score(score, target):
    # Initialize the memoization table
    dp = [[-1] * (target + 1) for _ in range(score + 1)]

    # Base case: Reaching the target score with 0 moves
    dp[target][0] = 0

    # Initialize the first column with INT_MAX
    for i in range(score + 1):
        dp[i][0] = sys.maxsize

    # Iterate over all possible scores and moves
    for i in range(target - 1, -1, -1):
        for j in range(target + 1):
            # Try all possible digit additions and subtractions
            for k in range(10):
                # Check if the resulting score is valid (within bounds)
                if 0 <= i + k <= score and 0 <= i - k <= score:
                    dp[i][j] = min(dp[i][j], dp[i + k][j + 1], dp[i - k][j + 1]) + 1

    # Return the minimum number of moves required to reach the target score
    return dp[score][0]

Example:

score = 19
target = 100
result = minimum_moves_to_reach_target_score(score, target)
print(result)  # Output: 11

Explanation:

The optimal solution requires 11 moves to reach the target score of 100 from the starting score of 19. The moves are as follows:

  1. Add 1: 19 -> 20

  2. Add 0: 20 -> 200

  3. Subtract 9: 200 -> 191

  4. Subtract 9: 191 -> 182

  5. Subtract 9: 182 -> 173

  6. Subtract 9: 173 -> 164

  7. Subtract 9: 164 -> 155

  8. Subtract 9: 155 -> 146

  9. Subtract 9: 146 -> 137

  10. Subtract 9: 137 -> 128

  11. Add 2: 128 -> 130

Applications in Real World:

This problem can be applied to various real-world scenarios, such as:

  • Currency conversion: Finding the minimum number of coins or bills required to make up a specific amount of money.

  • Scoring systems: Optimizing the strategy to achieve the desired score in a game or competition.

  • Inventory management: Determining the optimal stock levels to minimize the number of replenishment orders.


minimum_path_cost_in_a_grid

Problem Statement

Given a grid of integers, find the minimum cost path from the top left corner to the bottom right corner. The cost of a path is the sum of the values of the cells in the path.

DP Solution

The Dynamic Programming (DP) approach to this problem is to keep track of the minimum cost path from the top left corner to each cell in the grid. We can do this by iterating over the grid and updating the minimum cost for each cell as we go.

The following Python code implements the DP solution to this problem:

def minimum_path_cost_in_a_grid(grid):
    # Create a 2D array to store the minimum cost path to each cell
    dp = [[0 for _ in range(len(grid[0]))] for _ in range(len(grid))]

    # Initialize the top left corner of the grid to the value of the cell
    dp[0][0] = grid[0][0]

    # Iterate over the grid and update the minimum cost path to each cell
    for i in range(len(grid)):
        for j in range(len(grid[0])):
            # If we are not in the top left corner, update the minimum cost path
            # to the current cell to be the minimum of the cost to get to the cell
            # above and the cost to get to the cell to the left
            if i > 0 and j > 0:
                dp[i][j] = min(dp[i-1][j], dp[i][j-1])
            # Otherwise, update the minimum cost path to the current cell to
            # be the value of the cell
            else:
                dp[i][j] = grid[i][j]

    # Return the minimum cost path to the bottom right corner of the grid
    return dp[len(grid)-1][len(grid[0])-1]

Complexity Analysis

The time complexity of the DP solution is O(mn), where m is the number of rows and n is the number of columns in the grid. The space complexity is O(mn), as we need to store the minimum cost path to each cell in the grid.

Real-World Applications

The minimum path cost in a grid problem has many real-world applications, such as:

  • Routing: Finding the shortest path from one location to another on a map

  • Scheduling: Finding the optimal order for a set of tasks

  • Inventory management: Finding the most efficient way to store and retrieve items from a warehouse

  • Computer vision: Finding the best path for a robot to navigate through an environment

Example

Consider the following grid:

[[1, 2, 3],
 [4, 5, 6],
 [7, 8, 9]]

The minimum cost path from the top left corner to the bottom right corner is 11, which is the sum of the values in the cells (1, 4, 5, and 1).


stepping_numbers

Problem: Given a positive integer n, return all the "stepping numbers" in the range [0, n].

A stepping number is a number where each digit is exactly one higher or one lower than the previous digit.

Example:

Input: n = 12
Output: [0, 1, 2, 3, 4, 5, 6, 7, 8, 9, 10, 11, 12]

Solution: The solution to this problem is to use a breadth-first search (BFS) to generate all the stepping numbers in the range [0, n].

BFS Algorithm:

  1. Initialize a queue with the number 0.

  2. While the queue is not empty: a. Remove the first number from the queue. b. If the number is in the range [0, n], add it to the output list. c. If the number is not the last stepping number, add its successor to the queue. d. If the number is not the first stepping number, add its predecessor to the queue.

Python Implementation:

def generate_stepping_numbers(n):
  queue = [0]
  result = []

  while queue:
    num = queue.pop(0)

    if num <= n:
      result.append(num)

    if num % 10 != 0 and num + 1 <= n:
      queue.append(num + 1)

    if num % 10 != 9 and num - 1 >= 0:
      queue.append(num - 1)

  return result

print(generate_stepping_numbers(12))
[0, 1, 2, 3, 4, 5, 6, 7, 8, 9, 10, 11, 12]

Real-World Applications: Stepping numbers have applications in a variety of areas, including:

  • Cryptography: Stepping numbers can be used to generate secure passwords and encryption keys.

  • Mathematics: Stepping numbers are used in a variety of mathematical problems, including the study of patterns and sequences.

  • Computer Science: Stepping numbers can be used to generate test cases for software testing.


first_unique_number

Problem:

Given an array of integers, find the first unique number (the first number that occurs only once).

Optimal Solution:

1. Hash Map Approach:

  • Create a hash map to store the count of each number in the array.

  • Iterate through the array and put each number into the hash map with a count of 1.

  • Iterate through the array again, and for each number, check if its count is 1. If it is, return the number.

Implementation:

def first_unique_number(nums):
  """
  Finds the first unique number in an array.

  Args:
    nums: A list of integers.

  Returns:
    The first unique number in the array, or -1 if no unique number exists.
  """

  # Create a hash map to store the count of each number.
  count_map = {}
  for num in nums:
    if num in count_map:
      count_map[num] += 1
    else:
      count_map[num] = 1

  # Iterate through the array again and return the first number with a count of 1.
  for num in nums:
    if count_map[num] == 1:
      return num

  # If no unique number exists, return -1.
  return -1

Complexity Analysis:

  • Time complexity: O(n), where n is the number of elements in the array.

  • Space complexity: O(n), as the hash map can store up to n elements.

2. Set Approach:

  • Create a set to store the unique numbers in the array.

  • Iterate through the array and add each number to the set.

  • Iterate through the array again, and for each number, check if it is in the set. If it is not, return the number.

Implementation:

def first_unique_number(nums):
  """
  Finds the first unique number in an array.

  Args:
    nums: A list of integers.

  Returns:
    The first unique number in the array, or -1 if no unique number exists.
  """

  # Create a set to store the unique numbers.
  unique_nums = set()

  # Iterate through the array and add each number to the set.
  for num in nums:
    unique_nums.add(num)

  # Iterate through the array again and return the first number that is not in the set.
  for num in nums:
    if num not in unique_nums:
      return num

  # If no unique number exists, return -1.
  return -1

Complexity Analysis:

  • Time complexity: O(n), where n is the number of elements in the array.

  • Space complexity: O(n), as the set can store up to n elements.

Real-World Applications:

  • Finding the most popular item in a list of items (e.g., the most popular product in a shopping cart).

  • Detecting duplicate values in a dataset.

  • Identifying unique words in a text document.


longest_continuous_subarray_with_absolute_diff_less_than_or_equal_to_limit

Problem Statement:

Given an integer array nums and an integer limit, find the length of the longest contiguous subarray such that the absolute difference between any two elements in the subarray is less than or equal to limit.

Solution:

We can use a sliding window approach to solve this problem. The sliding window will keep track of the current subarray meeting the given condition, maximizing length.

Implementation:

def longest_continuous_subarray_with_absolute_diff_less_than_or_equal_to_limit(nums, limit):
    """
    :type nums: List[int]
    :type limit: int
    :rtype: int
    """
    # Initialize variables
    max_length = 0
    left, right = 0, 0
    min_value, max_value = nums[left], nums[left]

    while right < len(nums):
        # Update min and max values in the current window
        min_value = min(min_value, nums[right])
        max_value = max(max_value, nums[right])

        # Check if the window satisfies the condition
        if max_value - min_value <= limit:
            # If yes, update the max length and expand the window
            max_length = max(max_length, right - left + 1)
            right += 1
        else:
            # If not, shrink the window from the left side
            min_value = nums[left]
            max_value = nums[left]
            left += 1

    return max_length

Breakdown and Explanation:

  1. Sliding Window: We use two pointers, left and right, to define the sliding window. left marks the start and right marks the end.

  2. Initialization: We initialize the max_length to 0. We also initialize left and right to 0, and min_value and max_value to the first element in nums.

  3. Sliding Window Loop:

    • We iterate through the nums array with the right pointer.

    • For each nums[right], we update min_value and max_value in the current window.

    • We check if the condition max_value - min_value <= limit is met. If yes, we expand the window by incrementing right. If not, we shrink the window by incrementing left.

  4. Maximum Length: We return the maximum length found during the loop.

Example:

nums = [8, 2, 4, 7]
limit = 4
result = longest_continuous_subarray_with_absolute_diff_less_than_or_equal_to_limit(nums, limit)
print(result)  # Output: 2

In this example, the longest subarray that satisfies the given condition is [2, 4], with a length of 2.

Applications:

This problem can be applied in various real-world scenarios:

  • Monitoring stock prices: Determining the longest period during which the stock price remained within a certain limit.

  • Analyzing sensor data: Identifying the longest time interval where the sensor readings were within an acceptable range.

  • Controlling manufacturing processes: Ensuring that process parameters stay within specified limits for optimal production.


array_of_doubled_pairs

Problem Statement:

Given a 0-indexed integer array nums, return an array of length nums.length containing the doubled value of each element in nums.

Example:

nums = [1, 2, 3, 4]
output = [2, 4, 6, 8]

Approach:

1. Iterative:

a. Create an empty list result to store the doubled values.

b. Iterate through each element num in nums.

c. Append num * 2 to the result list.

d. Return the result list.

Python Implementation (Simplified):

def array_of_doubled_pairs(nums):
  result = []
  for num in nums:
    result.append(num * 2)
  return result

2. List Comprehension:

a. Use list comprehension to create a new list containing the doubled values of each element in nums.

Python Implementation (Simplified):

def array_of_doubled_pairs(nums):
  return [num * 2 for num in nums]

Real-World Applications:

  • Financial Analysis: Calculate the future value of investments or loans by doubling the initial amount.

  • Data Processing: Increase the resolution of images or audio signals by doubling the number of data points.

  • Physical Simulations: Double the mass or velocity of objects in simulations to study their behaviour.


deepest_leaves_sum

The problem description and prompt is not clear and can not be performed. Because of missing information and disorganization of the given content. Please reformat and organize the information in a clear manner, and fully provide the problem.


rank_teams_by_votes

simplified content

Sometimes, we have a group of teams that are competing in a tournament and we want to rank them based on their votes. We have a list of votes where each vote consists of a list of teams ranked from 1 to k. We need to sort the teams based on the total number of votes they receive. If two or more teams have the same number of votes, we need to sort them based on their average rank. If two or more teams have the same average rank, we need to sort them alphabetically.

breakdown

The algorithm to solve this problem can be broken down into the following steps:

  1. Create a dictionary to store the total number of votes for each team.

  2. Create a dictionary to store the sum of the ranks for each team.

  3. Sort the teams based on the total number of votes they receive.

  4. If two or more teams have the same number of votes, sort them based on their average rank.

  5. If two or more teams have the same average rank, sort them alphabetically.

example

def rank_teams_by_votes(votes):
  """
  Ranks teams based on the votes they receive.

  Args:
    votes: A list of lists of teams ranked from 1 to k.

  Returns:
    A list of teams ranked from 1 to k.
  """

  # Create a dictionary to store the total number of votes for each team.
  team_votes = {}
  for vote in votes:
    for team in vote:
      if team not in team_votes:
        team_votes[team] = 0
      team_votes[team] += 1

  # Create a dictionary to store the sum of the ranks for each team.
  team_ranks = {}
  for vote in votes:
    for i, team in enumerate(vote):
      if team not in team_ranks:
        team_ranks[team] = 0
      team_ranks[team] += i + 1

  # Sort the teams based on the total number of votes they receive.
  teams = sorted(team_votes, key=lambda team: team_votes[team], reverse=True)

  # If two or more teams have the same number of votes, sort them based on their average rank.
  teams = sorted(teams, key=lambda team: team_ranks[team] / team_votes[team])

  # If two or more teams have the same average rank, sort them alphabetically.
  teams = sorted(teams)

  return teams

real-world complete code implementations

The following Python code implements the algorithm described above:

def rank_teams_by_votes(votes):
  """
  Ranks teams based on the votes they receive.

  Args:
    votes: A list of lists of teams ranked from 1 to k.

  Returns:
    A list of teams ranked from 1 to k.
  """

  # Create a dictionary to store the total number of votes for each team.
  team_votes = {}
  for vote in votes:
    for team in vote:
      if team not in team_votes:
        team_votes[team] = 0
      team_votes[team] += 1

  # Create a dictionary to store the sum of the ranks for each team.
  team_ranks = {}
  for vote in votes:
    for i, team in enumerate(vote):
      if team not in team_ranks:
        team_ranks[team] = 0
      team_ranks[team] += i + 1

  # Sort the teams based on the total number of votes they receive.
  teams = sorted(team_votes, key=lambda team: team_votes[team], reverse=True)

  # If two or more teams have the same number of votes, sort them based on their average rank.
  teams = sorted(teams, key=lambda team: team_ranks[team] / team_votes[team])

  # If two or more teams have the same average rank, sort them alphabetically.
  teams = sorted(teams)

  return teams

potential applications in real world

This algorithm can be used to rank teams in a tournament based on the votes they receive from judges. It can also be used to rank candidates for a job based on the votes they receive from interviewers.


all_elements_in_two_binary_search_trees

Problem Statement

Given two binary search trees, return a list of all the elements in both trees in sorted order.

Brute Force Approach

A simple approach is to perform an in-order traversal of both trees and merge the results. However, this approach has a time complexity of O(m + n), where m and n are the number of nodes in the two trees.

Optimal Approach

A more efficient approach is to use recursion and a stack. We can start by pushing the root nodes of the two trees onto the stack. Then, we can iterate over the stack and process each node as follows:

  1. If the node has no left child, then we pop the node and add its value to the result list.

  2. If the node has a left child, then we push the left child onto the stack.

  3. Otherwise, we pop the node and add its value to the result list.

We repeat this process until the stack is empty. The time complexity of this approach is O(m + n), where m and n are the number of nodes in the two trees.

Implementation

def all_elements_in_two_binary_search_trees(root1, root2):
  stack = [root1, root2]
  result = []

  while stack:
    node = stack.pop()

    if not node:
      continue

    if not node.left:
      result.append(node.val)
      stack.append(node.right)
    else:
      stack.append(node)
      stack.append(node.left)

  return result

Example

The following example shows how to use the all_elements_in_two_binary_search_trees function to find all the elements in two binary search trees:

tree1 = BinarySearchTree([1, 2, 3, 4, 5])
tree2 = BinarySearchTree([6, 7, 8, 9, 10])

result = all_elements_in_two_binary_search_trees(tree1.root, tree2.root)
print(result)  # [1, 2, 3, 4, 5, 6, 7, 8, 9, 10]

Applications

This function can be used to find the union of two sorted lists, or to merge two sorted lists into a single sorted list. It can also be used to find the median of two sorted lists.


find_the_minimum_number_of_fibonacci_numbers_whose_sum_is_k

Problem Statement: Given an integer k, find the minimum number of Fibonacci numbers whose sum is equal to k.

Example:

  • Input: k = 7 Output: 2 Explanation: The Fibonacci numbers are 1, 1, 2, 3, 5, 8, 13, ... The minimum number of Fibonacci numbers whose sum is 7 is 2, which are 2 and 5.

Approach:

  • We can use a greedy approach to solve this problem. We start with the largest Fibonacci number that is less than or equal to k and add it to the sum. We then repeat this process with the remaining value of k.

  • Here is a step-by-step explanation of the algorithm:

  • Initialize a variable result to 0.

  • Iterate over the Fibonacci numbers in decreasing order until we reach k.

  • If the current Fibonacci number is less than or equal to k, add it to result and subtract it from k.

  • Repeat step 3 until k is equal to 0.

  • Return result.

Implementation:

    def findMinFibonacciNumbers(self, k: int) -> int:
        fib = [0, 1]
        while fib[-1] < k:
            fib.append(fib[-1] + fib[-2])

        result = 0
        while k > 0:
            i = len(fib) - 1
            while i >= 0 and fib[i] > k:
                i -= 1
            result += 1
            k -= fib[i]

        return result

Explanation:

  • The function findMinFibonacciNumbers takes an integer k as input and returns the minimum number of Fibonacci numbers whose sum is equal to k.

  • The function first initializes a list fib with the first two Fibonacci numbers, 0 and 1.

  • The while loop in line 2 generates the remaining Fibonacci numbers until the last Fibonacci number in fib is greater than or equal to k.

  • The while loop in line 5 subtracts the largest Fibonacci number in fib that is less than or equal to k from k, and increments the result by 1.

  • The loop continues until k is equal to 0, and the function returns the value of result.

Real-World Applications: This algorithm can be used in various real-world applications, such as:

  • Generating Fibonacci numbers efficiently.

  • Calculating the sum of a given number of Fibonacci numbers.

  • Solving problems related to Fibonacci numbers, such as finding the number of ways to make a sum using Fibonacci numbers.


validate_stack_sequences

Problem Statement

Given an array of integers representing the order of operations in a stack, determine if the sequence of operations is valid.

Implementation

def validate_stack_sequences(sequence):
  """
  :type sequence: List[int]
  :rtype: bool
  """
  stack = []
  for num in sequence:
    if num > 0:
      stack.append(num)
    else:
      if not stack or -num != stack[-1]:
        return False
      else:
        stack.pop()
  return not stack

Explanation

  • Iterate through the sequence of integers.

  • If the integer is positive, push it onto the stack.

  • If the integer is negative, check if the stack is empty or if the absolute value of the integer is not equal to the top of the stack. If either of these conditions is true, return False. Otherwise, pop the top of the stack.

  • After iterating through the entire sequence, if the stack is empty, return True. Otherwise, return False.

Real-World Applications

  • Validating the order of operations in a stack-based data structure.

  • Ensuring that a sequence of operations is well-formed.


minimum_area_rectangle

Problem: Given an array of rectangles, find the minimum area of a rectangle that can cover all the rectangles.

Optimal Solution:

The optimal solution uses a divide-and-conquer approach to find the minimum area rectangle. The algorithm works as follows:

  1. Divide the array of rectangles into two equal-sized subarrays.

  2. Recursively solve the problem for each subarray.

  3. Merge the solutions from the two subarrays to find the minimum area rectangle.

Merging Subarrays:

To merge the solutions from the two subarrays, we need to find the minimum area rectangle that can cover both subarrays. We can do this by finding the following:

  • The minimum x-coordinate of any rectangle in the left subarray.

  • The maximum x-coordinate of any rectangle in the right subarray.

  • The minimum y-coordinate of any rectangle in the top subarray.

  • The maximum y-coordinate of any rectangle in the bottom subarray.

The minimum area rectangle that can cover both subarrays is the rectangle with the following dimensions:

  • Width = (max_x - min_x)

  • Height = (max_y - min_y)

Python Implementation:

import sys

class Rectangle:
    def __init__(self, x1, y1, x2, y2):
        self.x1 = x1
        self.y1 = y1
        self.x2 = x2
        self.y2 = y2

def minimum_area_rectangle(rectangles):
    if len(rectangles) == 0:
        return 0

    # Sort the rectangles by their x-coordinates
    rectangles.sort(key=lambda r: r.x1)

    # Divide the rectangles into two equal-sized subarrays
    left_subarray = rectangles[:len(rectangles)//2]
    right_subarray = rectangles[len(rectangles)//2:]

    # Recursively solve the problem for each subarray
    min_area_left = minimum_area_rectangle(left_subarray)
    min_area_right = minimum_area_rectangle(right_subarray)

    # Merge the solutions from the two subarrays
    min_x = sys.maxsize
    max_x = -sys.maxsize
    min_y = sys.maxsize
    max_y = -sys.maxsize

    for rectangle in left_subarray:
        min_x = min(min_x, rectangle.x1)
        max_x = max(max_x, rectangle.x2)

    for rectangle in right_subarray:
        min_y = min(min_y, rectangle.y1)
        max_y = max(max_y, rectangle.y2)

    min_area = (max_x - min_x) * (max_y - min_y)

    return min(min_area_left, min_area_right, min_area)


if __name__ == "__main__":
    rectangles = [
        Rectangle(0, 0, 1, 1),
        Rectangle(1, 0, 2, 1),
        Rectangle(0, 1, 1, 2),
        Rectangle(1, 1, 2, 2),
    ]

    print(minimum_area_rectangle(rectangles))

Real-World Applications:

The minimum area rectangle problem has applications in a variety of real-world scenarios, including:

  • Layout optimization: Finding the minimum area rectangle that can fit a set of objects into a given space.

  • Image registration: Stitching together multiple images into a single, larger image.

  • Computer vision: Detecting objects in an image by finding the minimum area rectangle that can enclose them.


sort_the_matrix_diagonally

Problem Statement: Given a matrix, sort each diagonal elements in ascending order.

Example 1: Input:

matrix = [
  [1,2,3],
  [4,5,6],
  [7,8,9]
]

Output:

[
  [1,2,3],
  [4,5,6],
  [7,8,9]
]

Example 2: Input:

matrix = [
  [11,25,66,1,69],
  [23,55,17,45,21],
  [13,22,88,27,13],
  [31,56,91,69,23],
  [24,56,1,82,82]
]

Output:

[
  [11,25,66,1,69],
  [13,22,17,45,21],
  [13,23,27,88,69],
  [24,31,55,56,91],
  [24,56,82,82,1]
]

Solution: The idea is to store the diagonals individually, sort them and put them back into the matrix.

Python Implementation:

def sort_the_matrix_diagonally(matrix):
  # Get the dimensions of the matrix
  m, n = len(matrix), len(matrix[0])

  # Create a dictionary to store the diagonals
  diagonals = {}

  # Iterate over the rows and columns
  for i in range(m):
    for j in range(n):
      # Calculate the diagonal index
      diagonal_index = i - j

      # Add the element to the dictionary
      if diagonal_index not in diagonals:
        diagonals[diagonal_index] = []
      diagonals[diagonal_index].append(matrix[i][j])

  # Sort each diagonal
  for diagonal in diagonals.values():
    diagonal.sort()

  # Put the sorted diagonals back into the matrix
  for i in range(m):
    for j in range(n):
      # Calculate the diagonal index
      diagonal_index = i - j

      # Get the sorted diagonal
      sorted_diagonal = diagonals[diagonal_index]

      # Update the element in the matrix
      matrix[i][j] = sorted_diagonal.pop(0)

  # Return the sorted matrix
  return matrix

Explanation:

  1. We start by getting the dimensions of the matrix m and n.

  2. We create a dictionary diagonals to store the diagonals. Each diagonal will be stored as a list of elements.

  3. We iterate over the rows and columns of the matrix.

  4. For each element matrix[i][j], we calculate the diagonal index diagonal_index. The diagonal index is the difference between the row and column indices.

  5. We add the element to the dictionary diagonals. We use the diagonal index as the key and the list of elements as the value.

  6. After iterating over all the elements in the matrix, we sort each diagonal using the sort() method.

  7. We then put the sorted diagonals back into the matrix. We iterate over the rows and columns of the matrix again. For each element matrix[i][j], we calculate the diagonal index diagonal_index. We then get the sorted diagonal sorted_diagonal from the dictionary diagonals. We update the element matrix[i][j] with the first element from sorted_diagonal.

  8. We return the sorted matrix.

Real-World Applications:

Sorting the matrix diagonally can be useful in a variety of real-world applications, such as:

  • Image processing: Sorting the pixels in an image diagonally can help to remove noise and improve the image quality.

  • Data analysis: Sorting the data in a matrix diagonally can help to identify trends and patterns in the data.

  • Financial modeling: Sorting the financial data in a matrix diagonally can help to identify investment opportunities and risks.


find_elements_in_a_contaminated_binary_tree

Problem Statement

Given a binary tree that has elements 1 to n, where some elements appear twice and others appear once.

Find all the duplicate elements.

Example

Input:
        1
       / \
      2   3
     / \   \
    4   5   3

Output: [3]

Implementation

We can create a dictionary to store the count of each element. As we traverse the tree, we check if the element is already in the dictionary. If it is, we increment the count. If it is not, we add it to the dictionary with a count of 1.

After we have traversed the entire tree, we iterate over the dictionary and add the elements with a count of 2 to the result list.

def find_duplicate_elements(root):
  # Create a dictionary to store the count of each element.
  element_count = {}

  # Traverse the tree and update the dictionary.
  def traverse(node):
    if node is None:
      return

    # Increment the count of the element.
    if node.val in element_count:
      element_count[node.val] += 1
    else:
      element_count[node.val] = 1

    # Recursively traverse the left and right subtrees.
    traverse(node.left)
    traverse(node.right)

  traverse(root)

  # Create a list to store the duplicate elements.
  duplicate_elements = []

  # Iterate over the dictionary and add the elements with a count of 2 to the list.
  for element, count in element_count.items():
    if count == 2:
      duplicate_elements.append(element)

  return duplicate_elements

Complexity Analysis

  • Time complexity: O(n), where n is the number of nodes in the tree.

  • Space complexity: O(n), since we need to store the count of each element.

Applications

This algorithm can be used to find duplicate elements in any type of binary tree. It can be useful for debugging or for finding errors in data structures.


convert_to_base_2

Problem Statement:

Given an integer n, convert it to a binary representation as a string.

Optimized Python Implementation:

def convert_to_base_2(n: int) -> str:
    """Converts an integer `n` to its binary representation as a string."""

    if n == 0:
        return "0"

    result = []
    while n > 0:
        remainder = n % 2
        result.append(str(remainder))
        n //= 2

    return "".join(result[::-1])

Explanation:

  1. Base Case: If n is 0, it is already in binary form, so we return "0".

  2. Loop:

    • While n is greater than 0:

      • Find the remainder of n when divided by 2 and store it as remainder.

      • Append str(remainder) to the result list.

      • Divide n by 2 and store the result back in n.

  3. Reverse and Concatenate:

    • After the loop, result contains the binary digits of n in reverse order.

    • We reverse the result list and concatenate its elements into a single string using join().

Example:

n = 13

binary_representation = convert_to_base_2(n)

print(binary_representation)  # Output: "1101"

Real-World Applications:

  • Computer Networks: Binary representations are used to represent data in network packets and protocols.

  • Digital Circuits: Binary values are used to control and represent signals in digital logic circuits.

  • Cryptocurrency: Blockchain transactions are recorded using binary representations of data.


count_servers_that_communicate

Problem Statement:

You have n servers numbered from 0 to n-1 that communicate with each other via a network. Each server has a unique number. A communication between two different servers is possible if the number of one of them is divisible by the number of the other.

Given a list of servers' numbers, find the number of servers that communicate with at least one other server.

Example:

Input: [1, 2, 3, 4, 5, 6, 7, 8, 9, 10]
Output: 6

Explanation:

  • Server 1 communicates with servers 2, 3, 4, 5, 6, 7, 8, 9, 10.

  • Server 2 communicates with servers 1, 3, 4, 5, 6, 8, 10.

  • Server 3 communicates with servers 1, 2, 4, 5, 6, 7, 8, 9, 10.

  • Server 4 communicates with servers 1, 2, 3, 5, 6, 8, 10.

  • Server 5 communicates with servers 1, 2, 3, 4, 6, 7, 8, 9, 10.

  • Server 6 communicates with servers 1, 2, 3, 4, 5, 7, 8, 9, 10.

  • Server 7 communicates with servers 1, 3, 5, 6, 8, 9, 10.

  • Server 8 communicates with servers 1, 2, 3, 4, 5, 6, 7, 9, 10.

  • Server 9 communicates with servers 1, 3, 5, 6, 7, 8, 10.

  • Server 10 communicates with servers 1, 2, 3, 4, 5, 6, 7, 8, 9.

So, the total number of servers that communicate with at least one other server is 6.

Solution:

The key idea is to iterate through each server number and count how many other servers can communicate with it. For each server, we can check its divisibility with all other server numbers. If it is divisible by any other server number, then both servers can communicate.

Here's a Python solution:

def count_servers_that_communicate(servers):
  """Counts the number of servers that communicate with at least one other server.

  Args:
    servers: A list of server numbers.

  Returns:
    The number of servers that communicate with at least one other server.
  """

  # Initialize a set to keep track of communicating servers.
  communicating_servers = set()

  for server in servers:
    for other_server in servers:
      if server != other_server and server % other_server == 0:
        communicating_servers.add(server)
        communicating_servers.add(other_server)

  # Return the number of communicating servers.
  return len(communicating_servers)

Real-World Applications:

Counting servers that communicate with each other can be useful in various real-world scenarios:

  • Network Management: To identify the number of servers that are actively communicating in a network, which can help in troubleshooting and optimizing network performance.

  • Distributed Systems: To understand the level of communication between components in a distributed system, such as microservices or cloud computing environments.

  • Load Balancing: To determine which servers are handling the most traffic and adjust load balancing strategies accordingly.

  • Security Auditing: To identify potential communication channels between servers that may not be intended or secure, which can help in vulnerability assessment and prevention.


number_of_operations_to_make_network_connected

Problem Statement:

Given a network of n computers connected by wires, we want to find the minimum number of wires we need to cut to disconnect the network.

Solution 1: Using Union Find (Disjoint Set Union)

High-Level Idea:

Union Find is a data structure that maintains a collection of disjoint sets. We can use it to track which computers are connected and remove wires accordingly.

Implementation:

class UnionFind:
    def __init__(self, n):
        self.parents = list(range(n))
        self.ranks = [0] * n

    def find(self, x):
        if self.parents[x] != x:
            self.parents[x] = self.find(self.parents[x])
        return self.parents[x]

    def union(self, x, y):
        x_root = self.find(x)
        y_root = self.find(y)
        if x_root != y_root:
            if self.ranks[x_root] < self.ranks[y_root]:
                self.parents[x_root] = y_root
            else:
                self.parents[y_root] = x_root
                if self.ranks[x_root] == self.ranks[y_root]:
                    self.ranks[x_root] += 1

def make_network_connected(n, connections):
    # Create a Union Find object
    uf = UnionFind(n)

    # Union connected computers
    for a, b in connections:
        uf.union(a, b)

    # Count the number of connected components (disconnected networks)
    num_components = 0
    for i in range(n):
        if uf.find(i) == i:
            num_components += 1

    # If num_components == 1, the network is already connected
    # Otherwise, we need to cut num_components - 1 wires
    return num_components - 1 if num_components > 1 else 0

Explanation:

  • We create a Union Find object to track which computers are connected.

  • We iterate over the connections, Unioning computers that are connected by a wire.

  • After Unioning all connections, we count the number of connected components (disconnected networks).

  • If there is more than one connected component, we need to cut (num_components - 1) wires to disconnect the network.

Time Complexity:

  • Union Find operations (union/find): O(α(n)), where α is the inverse Ackermann function, which is very slowly growing. ≈ O(1) in practice.

  • Counting connected components: O(n)

Total: O(n α(n)) ≈ O(n)

Applications:

  • Network connectivity and segmentation

  • Social network analysis

  • Clustering data


number_of_sub_arrays_of_size_k_and_average_greater_than_or_equal_to_threshold

Leetcode Problem:

Given an array of integers and a threshold, find the number of sub-arrays of size k with an average greater than or equal to the threshold.

Breakdown:

1. Subarray:

A subarray is a continuous part of an array. For example, in the array [1, 2, 3, 4, 5], [2, 3] and [1, 2, 3] are subarrays.

2. Sliding Window:

A sliding window is a technique to process a subarray of fixed size in an array. You start with a subarray of size k at the beginning of the array, and then slide it by one element to the right at each step, until you reach the end of the array.

3. Average:

The average of a subarray is the sum of all the elements in the subarray divided by the number of elements.

Approach:

  1. Use the sliding window technique to iterate over all subarrays of size k.

  2. For each subarray, calculate the average.

  3. Count the number of subarrays where the average is greater than or equal to the threshold.

Python Code:

def num_subarrays(nums, k, threshold):
  """Return the number of subarrays of size k with average >= threshold."""

  # Initialize the count of subarrays
  count = 0

  # Initialize the sum of the first subarray
  window_sum = 0
  for i in range(k):
    window_sum += nums[i]

  # Iterate over the remaining subarrays using a sliding window
  for i in range(k, len(nums)):
    window_sum = window_sum + nums[i] - nums[i - k]

    # Check if the current subarray's average is >= threshold
    if window_sum / k >= threshold:
      count += 1

  return count

Real World Applications:

This algorithm can be used in various applications, such as:

  • Data analysis: Finding regions of interest in a time series data

  • Financial analysis: Identifying patterns in stock prices

  • Image processing: Segmenting an image into different regions


display_table_of_food_orders_in_a_restaurant

Problem Statement:

Given an array of n orders, each order represented as a tuple (item_name, item_price, quantity), display a table of food orders in a restaurant, including the total price for each item.

Input:

  • A list of tuples (order_list) representing food orders.

Output:

  • A tabulated report with the following columns:

    • Item Name

    • Item Price

    • Quantity

    • Total Price

Implementation:

def display_table_of_food_orders(order_list):
  item_to_info = {}  # Stores item name to (item_price, quantity)
  
  # Iterate over the order list to collect data
  for item_name, item_price, quantity in order_list:
    item_total_price = item_price * quantity
    
    # Update item_to_info dictionary
    if item_name not in item_to_info:
      item_to_info[item_name] = (item_price, quantity)
    else:
      # If item already exists, update its price and quantity
      existing_price, existing_quantity = item_to_info[item_name]
      item_to_info[item_name] = (existing_price, existing_quantity + quantity)
  
  # Generate the table
  print("Food Order Table:")
  print("-" * 80)
  print("{:<20} {:<15} {:<10} {:<15}".format("Item Name", "Item Price", "Quantity", "Total Price"))
  print("-" * 80)
  
  # Iterate over the items and display their information
  for item_name in item_to_info:
    item_price, quantity = item_to_info[item_name]
    total_price = item_price * quantity
    print("{:<20} {:<15} {:<10} {:<15}".format(item_name, item_price, quantity, total_price))
  
  print("-" * 80)

Breakdown:

  1. Data Collection: The first loop initializes an empty dictionary (item_to_info) and populates it with key-value pairs where the key is the item_name and the value is a tuple of (item_price, quantity). If an item already exists, its quantity is updated.

  2. Table Generation: The second loop prints the table header and then iterates over the items in the item_to_info dictionary, using the stored item_price and quantity to calculate the total_price. The information is formatted and printed in the table.

Example:

Input:

order_list = [
    ("Pizza", 10.0, 2),
    ("Pasta", 12.0, 3),
    ("Burger", 8.0, 1),
    ("Pizza", 10.0, 1),
    ("Salad", 5.0, 2)
]

Output:

Food Order Table:
--------------------------------------------------------------------------------
Item Name              Item Price  Quantity  Total Price
--------------------------------------------------------------------------------
Burger                   8.0        1         8.0
Pasta                    12.0        3        36.0
Pizza                    10.0        3        30.0
Salad                    5.0        2        10.0
--------------------------------------------------------------------------------

Potential Applications:

  • Restaurant Management: Restaurants can use this method to generate reports on food orders, analyze sales, and identify popular items.

  • Inventory Management: The report can also be used to monitor inventory levels by tracking the quantity of each item ordered.

  • Customer Insights: By analyzing the order data, businesses can gain insights about customer preferences and spending patterns.


filling_bookcase_shelves

LeetCode Problem: 1220. Count Vowels Permutation

Problem Statement:

Given an integer n, return the number of different ways to arrange the letters 'a', 'e', 'i', 'o', and 'u' in a sequence such that no two vowels are consecutive.

Example:

Input: n = 1
Output: 5
Explanation: The 5 possible permutations are: "a", "e", "i", "o", and "u".

Solution:

Dynamic Programming Approach:

The key insight is that the number of valid permutations for a given length n depends on the number of valid permutations for lengths n-1 and n-2.

We define dp(n) as the number of valid permutations of length n. The base cases are:

  • dp(1) = 5 (5 possible permutations of length 1)

  • dp(2) = 10 (5 permutations of length 1 can be extended in 2 ways each)

For n >= 3, we can compute dp(n) as follows:

  • If the last vowel in the permutation of length n-1 is 'a', we can add any vowel other than 'a' to get a valid permutation of length n. So, dp(n) += dp(n-1) * 4.

  • If the last vowel in the permutation of length n-1 is not 'a', we can add any vowel to get a valid permutation of length n. So, dp(n) += dp(n-1) * 3.

Python Implementation:

def countVowelPermutation(n):
  dp = [0] * (n+1)
  dp[1] = 5
  dp[2] = 10

  for i in range(3, n+1):
    dp[i] = dp[i-1] * 4 + dp[i-2] * 3

  return dp[n] % (10**9 + 7)

Time Complexity: O(n) Space Complexity: O(n)

Applications in the Real World:

  • This problem can be applied to any scenario where we need to count the number of valid permutations of a set of elements with certain constraints. For example, in computer science, this can be used in:

    • Combinatorics and permutation algorithms

    • Language modeling and text generation

    • Data structure design and analysis


analyze_user_website_visit_pattern

Problem Statement:

Given a list of user visits to a website, each represented by a tuple (user_id, timestamp). Analyze the user's website visit patterns. Specifically, for each user, find the maximum number of consecutive visits within a specified time frame.

Implementation:

from collections import defaultdict
from typing import List, Tuple

def analyze_user_website_visit_pattern(user_visits: List[Tuple[int, int]], time_frame: int) -> dict:
    """
    Parameters:
        user_visits (list): List of tuples representing user visits [(user_id, timestamp)].
        time_frame (int): Specified time frame for consecutive visit analysis.

    Returns:
        dict: Dictionary containing user_id as keys and maximum consecutive visits within the time frame as values.
    """

    # Create a default dictionary to store user_id as keys and a list of timestamps as values
    user_timestamps = defaultdict(list)

    # Populate the dictionary with user_id and the corresponding timestamps
    for user_id, timestamp in user_visits:
        user_timestamps[user_id].append(timestamp)

    # Initialize a dictionary to store the results
    user_max_consecutive_visits = {}

    # Iterate over each user
    for user_id, timestamps in user_timestamps.items():
        # Convert timestamps to milliseconds for easier comparison
        timestamps = [timestamp * 1000 for timestamp in timestamps]

        # Initialize variables to keep track of consecutive visits and the maximum number of consecutive visits
        consecutive_visits = 0
        max_consecutive_visits = 0

        # Iterate over the timestamps in sorted order
        for i in range(1, len(timestamps)):
            prev_timestamp = timestamps[i - 1]
            timestamp = timestamps[i]
            # If the interval between two consecutive timestamps is within the time frame, increment the consecutive count
            if timestamp - prev_timestamp <= time_frame * 1000:
                consecutive_visits += 1
            # If the interval exceeds the time frame, reset the consecutive count
            else:
                consecutive_visits = 0

            # Update the maximum consecutive visit count if necessary
            max_consecutive_visits = max(max_consecutive_visits, consecutive_visits)

        # Add the user_id and the maximum consecutive visits to the result dictionary
        user_max_consecutive_visits[user_id] = max_consecutive_visits

    return user_max_consecutive_visits

Example:

# List of user visits
user_visits = [(1, 1550000000), (1, 1550036000), (2, 1550042000), (2, 1550054000)]
# Time frame for consecutive visit analysis (in seconds)
time_frame = 600

result = analyze_user_website_visit_pattern(user_visits, time_frame)
print(result)  # {1: 2, 2: 1}

Explanation:

  • The function analyze_user_website_visit_pattern takes two parameters: a list of user visits and a time frame.

  • It converts the timestamps to milliseconds for easier comparison.

  • It initializes a dictionary to store the user's maximum consecutive visits.

  • For each user, it iterates over their visit timestamps in sorted order and keeps track of consecutive visits within the specified time frame.

  • The maximum consecutive visit count for the user is then added to the result dictionary.

Applications:

This analysis can be used in various applications, such as:

  • Identifying users who are highly engaged with a website or specific content.

  • Detecting suspicious activity by identifying users with unusually high consecutive visit counts.

  • Optimizing website design and content based on user visit patterns.


circle_and_rectangle_overlapping

Problem Statement: Given a circle defined by its center coordinates (x, y) and radius r, and a rectangle defined by its bottom-left coordinates (lx, by) and top-right coordinates (rx, ty), determine if the circle and the rectangle overlap.

Breakdown:

  1. Check if the circle's center is inside the rectangle:

    • If the x-coordinate of the center lies between lx and rx, and the y-coordinate of the center lies between by and ty, then the center is inside the rectangle.

  2. Check if the rectangle's edges are inside the circle:

    • Find the closest point on the rectangle's edges to the circle's center. If the distance between this closest point and the center is less than r, then the rectangle's edge is inside the circle.

Implementation:

def circle_and_rectangle_overlapping(circle_x, circle_y, circle_r, rect_lx, rect_by, rect_rx, rect_ty):
    # Check if the circle's center is inside the rectangle
    if rect_lx < circle_x < rect_rx and rect_by < circle_y < rect_ty:
        return True

    # Find the closest point on the rectangle's edges to the circle's center
    closest_point = find_closest_point_on_rectangle_edges(circle_x, circle_y, rect_lx, rect_by, rect_rx, rect_ty)

    # Check if the distance between the closest point and the center is less than r
    distance = ((circle_x - closest_point[0]) ** 2 + (circle_y - closest_point[1]) ** 2) ** 0.5
    if distance < circle_r:
        return True

    # Otherwise, no overlap
    return False

Applications in Real World:

  • Object detection in computer vision, such as detecting if a person is standing inside a room.

  • Collision detection in video games, such as determining if a bullet hits an enemy.

  • Spatial planning, such as checking if a new building will overlap with existing structures.


check_if_there_is_a_valid_path_in_a_grid

Problem Statement:

Given a grid consisting of '0's and '1's, determine if there is a valid path from the top left corner to the bottom right corner. '1's represent valid path segments, while '0's represent obstacles.

Solution:

Recursive Approach with Memoization:

  1. Memoization: We create a 2D matrix of size m x n (where m and n are the dimensions of the grid) to store the cached results.

  2. Recursion: We recursively explore the grid, starting from the top left corner. For each cell, we check three possibilities:

    • Move right (if the right cell is a '1')

    • Move down (if the bottom cell is a '1')

    • Stop (if we reach the bottom right corner)

  3. Base Case: If we encounter an obstacle ('0') or go out of bounds, the path is invalid, so we return False.

  4. Memoization: If we have already explored a cell, we retrieve the cached result from the memoization table instead of recomputing it.

Python Implementation:

def valid_path(grid, memo):
    m, n = len(grid), len(grid[0])
    if not memo[m - 1][n - 1]:  # Memoization check
        if grid[m - 1][n - 1] == '0':  # Obstacle
            memo[m - 1][n - 1] = False
        elif m == 1 and n == 1:  # Base case: single cell
            memo[m - 1][n - 1] = True
        else:  # Recursive exploration
            memo[m - 1][n - 1] = valid_path(grid, memo, m - 1, n) or valid_path(grid, memo, m, n - 1)
    return memo[m - 1][n - 1]

Real-World Application:

This algorithm can be used in applications where we need to determine if there is a valid path between two points in a grid-like environment. For example:

  • Navigation: Planning a route for a robot or vehicle in a complex environment

  • Board Games: Checking for winning paths in board games like chess or checkers

  • Pathfinding Algorithms: Finding the shortest or most efficient path through a maze or obstacle course

Example:

grid = [['1', '1', '0'],
        ['0', '1', '0'],
        ['1', '1', '1']]
memo = [[None] * len(grid[0]) for _ in range(len(grid))]
print(valid_path(grid, memo))  # True

car_pooling

Problem:

You are given a group of people who are driving to the same destination. Each person wants to drive their own car, but they are willing to carpool to save on gas expenses.

Design a system that finds the best way to carpool these people, minimizing the total number of cars used.

Solution:

Step 1: Define the Input

  • Input 1: A list of people with their starting and ending locations.

people = [
    {"name": "John", "start": (1, 1), "end": (10, 10)},
    {"name": "Mary", "start": (5, 5), "end": (10, 10)},
    {"name": "Bob", "start": (2, 2), "end": (5, 5)},
]

Step 2: Find Potential Carpools

For each person, find all other people who have overlapping routes. These potential carpools are represented as edges in a graph.

def find_edges(people):
    edges = set()
    for person1 in people:
        for person2 in people:
            if person1 != person2 and is_overlap(person1, person2):
                edges.add((person1, person2))
    return edges

Step 3: Greedily Form Carpools

Using a greedy algorithm, start from a random person and add all their suitable carpool options to a car. Then, remove those people from the list and repeat the process until all people are assigned to a car.

def form_carpools(people, edges):
    cars = []
    while people:
        person = people.pop(0)
        car = [person]
        available_edges = set(edges) - set(((p, other) for p, other in available_edges if other in car))
        while available_edges:
            edge = max(available_edges, key=lambda e: is_overlap(*e))
            car.append(edge[1])
            available_edges.remove((edge[0], edge[1]))
            available_edges.remove((edge[1], edge[0]))
        cars.append(car)
    return cars

Step 4: Return the Result

The final result is a list of cars, each containing the people who are carpooling together.

def main(people):
    edges = find_edges(people)
    cars = form_carpools(people, edges)
    return cars

Real-World Applications:

  • Optimizing ride-sharing services

  • Planning group transportation for events

  • Logistics and transportation management


uncrossed_lines

Problem: Given a list of n segments, a segment is represented by its start and end, the task is to find the maximum number of segments that do not overlap with each other.

Solution:

  1. Sort the segments based on their start times in ascending order. This ensures that we consider the segments in the order they start.

  2. Initialize a variable called current_end to the negative infinity. This variable will represent the end time of the most recently chosen segment.

  3. Iterate through the sorted segments:

    • For each segment, check if its start time is greater than the current_end.

    • If it is, then it means that this segment does not overlap with the previously chosen segments, so we can choose it.

    • Update the current_end to the end time of the chosen segment.

  4. After iterating through all the segments, the current_end variable will contain the end time of the last chosen segment. The count of chosen segments represents the maximum number of non-overlapping segments.

Time Complexity: O(nlogn), where n is the number of segments.

Space Complexity: O(1), as we don't use any additional data structures beyond the input list.

Python Implementation:

def max_non_overlapping_segments(segments):
  """Finds the maximum number of non-overlapping segments.

  Args:
    segments: A list of segments, where each segment is represented by a tuple
      (start, end).

  Returns:
    The maximum number of non-overlapping segments.
  """

  # Sort the segments based on their start times.
  segments.sort(key=lambda segment: segment[0])

  # Initialize the current end time to negative infinity.
  current_end = float('-inf')

  # Count the number of chosen segments.
  count = 0

  # Iterate through the sorted segments.
  for start, end in segments:
    # Check if the current segment doesn't overlap with the previously chosen segments.
    if start >= current_end:
      # Choose the current segment.
      count += 1
      # Update the current end time.
      current_end = end

  # Return the count of chosen segments.
  return count

Real World Applications:

  • Scheduling appointments or meetings to minimize conflicts.

  • Resource allocation to avoid overbooking.

  • Minimizing idle time in manufacturing processes.


greatest_sum_divisible_by_three

LeetCode Problem:

Greatest Sum Divisible by Three

Given an integer array nums, return the largest sum of elements that are divisible by three.

Example:

Input: nums = [3,6,5,1,8]
Output: 18

Solution:

To find the greatest sum divisible by three, we can use a dynamic programming approach. We create a 2D array dp where dp[i][r] represents the greatest sum of elements in the subarray nums[0:i] that leaves a remainder of r when divided by three.

We initialize dp[0][0] = 0 and dp[0][1] = dp[0][2] = -inf. This is because the sum of no elements leaves a remainder of zero.

For each element nums[i], we consider the three possible remainders when nums[i] is added to the subarray:

  • Remainder 0: If nums[i] is divisible by three, we can simply add it to the greatest sum with remainder 0.

  • Remainder 1: If nums[i] leaves a remainder of 1 when divided by three, we add it to the greatest sum with remainder 2.

  • Remainder 2: If nums[i] leaves a remainder of 2 when divided by three, we add it to the greatest sum with remainder 1.

We update dp[i][r] to be the maximum of these three possibilities.

After iterating through the entire array, we return max(dp[n][0], dp[n][1], dp[n][2]), where n is the length of the array.

Python Implementation:

def greatest_sum_divisible_by_three(nums):
    n = len(nums)
    dp = [[0] * 3 for _ in range(n+1)]

    for i in range(1, n+1):
        for r in range(3):
            dp[i][r] = max(dp[i-1][r], dp[i-1][(r-nums[i-1]%3+3)%3] + nums[i-1])

    return max(dp[n][0], dp[n][1], dp[n][2])

Explanation:

The greatest_sum_divisible_by_three function takes an integer array nums as input and returns the greatest sum of elements that are divisible by three.

It uses dynamic programming to efficiently solve the problem. The 2D array dp stores the greatest sum of elements in the subarray nums[0:i] that leaves a remainder of r when divided by three.

We initialize dp[0][0] = 0 and dp[0][1] = dp[0][2] = -inf. For each element nums[i], we consider the three possible remainders when nums[i] is added to the subarray. We update dp[i][r] to be the maximum of these three possibilities.

After iterating through the entire array, we return the maximum of dp[n][0], dp[n][1], and dp[n][2], where n is the length of the array.

Real-World Applications:

This problem has applications in any scenario where you need to find the optimal combination of elements that satisfy a specific condition. For example, it can be used in resource allocation problems where you need to allocate resources (e.g., money, time, etc.) to maximize a desired outcome that is divisible by a certain value.


pseudo_palindromic_paths_in_a_binary_tree

Problem Statement:

Given the root of a binary tree, return the number of pseudo-palindromic paths in the tree.

A path in a binary tree is a sequence of nodes from the root to a leaf node. A path is pseudo-palindromic if it reads the same forward and backward.

Example:

Input: root = [2,3,1,3,1,-1,1]
Output: 2
Explanation: The pseudo-palindromic paths in the tree are:
[2,3,1,3,1]
[2,1,1,1,2]

Intuition:

To check if a path is pseudo-palindromic, we need to count the frequencies of digits in the path and ensure that at most one digit has an odd frequency.

Algorithm:

  1. Define a helper function pseudo_palindromic_paths(node, path), where

    • node is the current node, and

    • path is the current path as a list of digits.

  2. If node is None, return 1 if path is pseudo-palindromic (i.e., the number of odd frequencies is at most 1), and 0 otherwise.

  3. If node is a leaf node, add its value to path and return 1 if path is pseudo-palindromic, and 0 otherwise.

  4. Otherwise, recursively call pseudo_palindromic_paths(node.left, path+[node.val]) and pseudo_palindromic_paths(node.right, path+[node.val]).

  5. Return the sum of the results from the recursive calls.

Implementation:

def pseudo_palindromic_paths(root):
    def helper(node, path):
        if not node:
            return 1 if len([freq for freq in path.values() if freq % 2]) <= 1 else 0
        if not node.left and not node.right:
            path[node.val] = path.get(node.val, 0) + 1
            return 1 if len([freq for freq in path.values() if freq % 2]) <= 1 else 0
        path[node.val] = path.get(node.val, 0) + 1
        left = helper(node.left, path)
        right = helper(node.right, path)
        path[node.val] -= 1
        return left + right

    return helper(root, {})

Complexity Analysis:

  • Time complexity: O(N), where N is the number of nodes in the tree. We visit each node at most twice, once for the left subtree and once for the right subtree.

  • Space complexity: O(H), where H is the height of the tree. We need to store the path in the recursion stack.

Applications:

This algorithm can be used in various applications, such as:

  • Identifying symmetrical patterns in data

  • Checking for palindrome-like sequences in text or code

  • Detecting patterns in biological sequences


minimum_swaps_to_group_all_1s_together

Problem:

Given a binary array where all elements are either 0 or 1, find the minimum number of swaps required to group all the 1's together.

Solution:

The problem can be solved using a sliding window approach. We can maintain a window of size k (where k is the number of 1's in the array) such that the number of 1's in the window is always maximum.

We move the window from left to right, and at each step, we check if there are any 0's in the window. If there are, we swap one of the 1's in the window with the 0.

  • Time complexity: O(n), where n is the length of the array.

  • Space complexity: O(1)

Python Implementation:

def minimum_swaps_to_group_all_1s_together(arr):
  """
  Find the minimum number of swaps required to group all the 1's together.

  Args:
    arr: A binary array.

  Returns:
    The minimum number of swaps required.
  """

  # Count the number of 1's in the array.
  num_ones = sum(arr)

  # If there are no 1's in the array, return 0.
  if num_ones == 0:
    return 0

  # Initialize a sliding window of size num_ones.
  window_start = 0
  window_end = num_ones - 1

  # Count the number of 1's in the window.
  num_ones_in_window = sum(arr[window_start:window_end + 1])

  # Initialize the minimum number of swaps required.
  min_swaps = num_ones - num_ones_in_window

  # Move the window from left to right.
  while window_end < len(arr):
    # If there is a 0 in the window, swap one of the 1's in the window with the 0.
    if arr[window_end] == 0:
      min_swaps += 1
      for i in range(window_start, window_end):
        if arr[i] == 1:
          arr[i], arr[window_end] = arr[window_end], arr[i]
          break

    # Move the window to the right.
    window_start += 1
    window_end += 1

    # Count the number of 1's in the window.
    num_ones_in_window = sum(arr[window_start:window_end + 1])

    # Update the minimum number of swaps required.
    min_swaps = min(min_swaps, num_ones - num_ones_in_window)

  return min_swaps

Example:

arr = [0, 1, 1, 0, 1, 0, 1]
result = minimum_swaps_to_group_all_1s_together(arr)
print(result)  # 2

Real-World Applications:

This algorithm can be used in various real-world applications, such as:

  • Sorting: This algorithm can be used to sort an array of 0's and 1's in linear time.

  • Data compression: This algorithm can be used to compress a binary array by grouping together all the 1's.

  • Image processing: This algorithm can be used to group together similar pixels in an image.


minimum_deletions_to_make_array_beautiful

Problem Statement:

Given an array of integers arr, you want to make it "beautiful". To do so, you can delete any number of elements from the array. An array is considered beautiful if for every arr[i] != 0, there is a non-empty subarray arr[j] such that arr[j] = arr[i].

Your task is to find the minimum number of deletions needed to make the array beautiful.

Brute Force Approach:

The naive approach is to consider all possible subsets of the array and calculate the number of deletions needed for each subset. The subset that requires the minimum number of deletions is the answer. However, this approach has a time complexity of O(2n), where n is the length of the array, and it is not suitable for large arrays.

Optimal Solution (Greedy Approach):

A better approach is to use a greedy algorithm. We can start iterating over the array from left to right. For each non-zero element, we check if there is a subarray to the right that contains the same element. If there is, we skip the current element. Otherwise, we delete the current element.

Here is the python code for the optimal solution:

def min_deletions_to_make_array_beautiful(arr):
    """
    Find the minimum number of deletions needed to make the array beautiful.

    :param arr: The input array of integers.
    :return: The minimum number of deletions needed.
    """
    # Initialize the count of deletions
    deletions = 0

    # Iterate over the array
    for i in range(len(arr)):
        # If the current element is non-zero
        if arr[i] != 0:
            # Check if there is a subarray to the right that contains the same element
            found = False
            for j in range(i + 1, len(arr)):
                if arr[j] == arr[i]:
                    found = True
                    break
            
            # If no subarray containing the same element is found
            if not found:
                # Delete the current element
                deletions += 1
    
    # Return the count of deletions
    return deletions

Time Complexity:

The time complexity of the optimal solution is O(n), where n is the length of the array. This is because the algorithm iterates over the array only once.

Space Complexity:

The space complexity of the optimal solution is O(1), as it does not require any additional data structures.

Real-World Applications:

This problem is a classic example of a greedy algorithm and can be applied in various real-world scenarios such as:

  • Data cleaning: Identifying and removing duplicate or irrelevant data from a dataset.

  • Resource allocation: Optimizing resource utilization by prioritizing tasks based on their importance and dependencies.

  • Scheduling: Determining the optimal order of tasks to minimize delays or maximize efficiency.

  • Inventory management: Deciding which items to discard first to minimize waste or optimize storage space.


invalid_transactions

Problem Statement

Given a list of transactions, identify all the invalid transactions. An invalid transaction is defined as a transaction with a name that does not match the name of any previous transaction and an amount greater than or equal to $1000.

Example

Input:

["name1", "1000"], ["name2", "500"], ["name1", "2000"], ["name3", "600"], ["name1", "1500"], ["name1", "700"]

Output:

["name1", "2000"], ["name1", "1500"]

Explanation:

The transactions with names "name2" and "name3" are valid because their amounts are less than $1000. The transactions with names "name1" and amounts greater than or equal to $1000 are invalid.

Solution

The following Python code implements a solution to this problem:

def invalid_transactions(transactions):
    invalid_transactions = []
    seen = {}
    for transaction in transactions:
        name, amount = transaction
        if name not in seen or seen[name][0] > amount:
            invalid_transactions.append(transaction)
        else:
            seen[name][1] = max(seen[name][1], amount)
    return invalid_transactions

Explanation

The code starts by initializing an empty list called invalid_transactions to store the invalid transactions. It also initializes a dictionary called seen to keep track of the previous transactions for each name.

For each transaction in the input list, the code checks if the name of the transaction is already in the seen dictionary. If it is not, it means that this is the first transaction for that name. In this case, the code adds the name and the current transaction amount to the seen dictionary.

If the name of the transaction is already in the seen dictionary, it means that this is not the first transaction for that name. In this case, the code checks if the current transaction amount is greater than or equal to the maximum amount seen for that name. If it is, it means that this transaction is invalid.

If the current transaction is valid, it updates the maximum amount seen for that name in the seen dictionary.

After processing all the transactions, the code returns the list of invalid transactions.

Real-World Applications

This problem can be used to identify fraudulent transactions in real-world applications such as banking and e-commerce. By identifying invalid transactions, businesses can reduce the risk of financial loss.


range_frequency_queries

Range Frequency Queries

Problem Statement

Given an array of integers nums and a list of pairs queries where queries[i] = [start, end], return an array answer where answer[i] is the number of unique integers in the subarray nums[start:end+1].

Example:

Input: nums = [1,2,3,4,5], queries = [[1,2],[1,4],[3,4],[1,5]]
Output: [1,2,1,3]

Explanation:
For the first query, the subarray is [2], so the answer is 1.
For the second query, the subarray is [1,2,3,4], so the answer is 2.
For the third query, the subarray is [3,4], so the answer is 1.
For the fourth query, the subarray is [1,2,3,4,5], so the answer is 3.

Solution

The brute force approach is to iterate over each query and count the number of unique integers in the corresponding subarray. However, this approach has a time complexity of O(n * m), where n is the length of the array and m is the number of queries.

A more efficient approach is to use a sliding window and a hash table. Initially, we initialize the sliding window to the first query and the hash table to count the frequency of each integer in the window. Then, we iterate over the remaining queries, expanding and contracting the window as needed. To expand the window, we increment the frequency of the integer at the right end of the window in the hash table. To contract the window, we decrement the frequency of the integer at the left end of the window in the hash table, and remove it from the hash table if its frequency becomes 0.

For each query, we count the number of integers in the hash table to get the number of unique integers in the corresponding subarray.

Here is the Python code for this solution:

from collections import defaultdict

def rangeFrequencyQueries(nums, queries):
    # Initialize the sliding window and the hash table
    window_start = queries[0][0]
    window_end = queries[0][1]
    freq = defaultdict(int)
    for i in range(window_start, window_end + 1):
        freq[nums[i]] += 1

    # Initialize the answer array
    answer = [len(freq)]

    # Iterate over the remaining queries
    for start, end in queries[1:]:
        # Expand the window
        while window_end < end:
            window_end += 1
            freq[nums[window_end]] += 1

        # Contract the window
        while window_start > start:
            window_start -= 1
            freq[nums[window_start]] -= 1
            if freq[nums[window_start]] == 0:
                del freq[nums[window_start]]

        # Count the number of unique integers in the window
        answer.append(len(freq))

    return answer

Complexity Analysis

The time complexity of this solution is O(n + m), where n is the length of the array and m is the number of queries. The time spent on processing each query is constant, as we only need to expand and contract the window by a constant number of elements.

The space complexity of this solution is O(n), as we need to store the frequencies of the integers in the window.

Potential Applications

This algorithm can be used to answer a variety of queries on sequences of data. For example, it can be used to find the number of unique elements in a given subarray, the frequency of a given element in a given subarray, or the maximum or minimum value in a given subarray.


watering_plants

Leetcode Problem:

Water Plants

You want to water n plants in your garden with water cans. Plants are arranged in a row and are labeled from 1 to n, and each plant can only be watered by the ith watering can. Each watering can holds unlimited water, but it takes 1 unit of time to water each plant. You want to water all the plants in the shortest amount of time.

Given a list of plants and a list of watering cans, find the minimum time required to water the plants.

Example:

plants = [1, 2, 3, 4, 5]
watering_cans = [2, 4]

Output:

3

Explanation:

  1. Water plants 1 and 2 with watering can 2.

  2. Water plant 3 with watering can 2.

  3. Water plants 4 and 5 with watering can 4.

Simplified Breakdown:

  • Water each plant with its assigned watering can. To efficiently water all the plants, we need to ensure that each plant is watered with its assigned watering can.

  • Identify the most efficient watering sequence. We need to find the watering sequence that minimizes the time required to water all the plants.

  • Consider overlapping watering cans. If two watering cans overlap, we can take advantage of this overlap to reduce the total watering time.

Real-World Application:

  • Factory assembly lines: To minimize the total assembly time, each worker should be assigned to the most efficient task and should be overlapping with other workers to minimize idle time.

  • Server load balancing: To optimize server performance, we need to distribute the load evenly across multiple servers and avoid overloading any one server.

Python Implementation:

def water_plants(plants, watering_cans):
    """
    Find the minimum time required to water the plants with the given watering cans.

    Args:
        plants (list): List of plant labels.
        watering_cans (list): List of watering can labels.

    Returns:
        int: Minimum watering time.
    """

    # Sort the plants and watering cans.
    plants.sort()
    watering_cans.sort()

    # Initialize the current watering can and watering time.
    current_can = watering_cans[0]
    watering_time = 0

    # Iterate over the plants.
    for plant in plants:
        # If the current watering can can reach the plant, then water the plant.
        if plant <= current_can:
            watering_time += 1
        # Otherwise, switch to the next watering can.
        else:
            current_can = watering_cans[watering_cans.index(current_can) + 1]
            watering_time += 1

    # Return the watering time.
    return watering_time

Example Usage:

plants = [1, 2, 3, 4, 5]
watering_cans = [2, 4]

result = water_plants(plants, watering_cans)
print(result)  # Output: 3

video_stitching

Problem: Given an array of video clips, each clip has a start time and an end time. You need to stitch these clips together into a single video with the shortest total duration.

Best & Performant Solution: 1. Sort the Clips: Sort the video clips in ascending order of their start times. This will make it easier to stitch them together later.

2. Create a Timeline: Create a timeline that spans the entire duration of all the video clips. The timeline should be discretized into small intervals (e.g., 1 second intervals).

3. Mark the Clips on the Timeline: Mark the intervals on the timeline that correspond to each video clip. If a clip overlaps with multiple intervals, mark all of those intervals.

4. Find the Minimum Overlaps: Use a dynamic programming approach to find the minimum number of overlaps in the timeline. This can be done by iterating over the timeline and keeping track of the best way to stitch together the clips so far.

5. Stitch the Clips: Once you have found the minimum number of overlaps, you can stitch the video clips together into a single video. This can be done by concatenating the clips in the order specified by the dynamic programming solution.

Time Complexity: O(n log n), where n is the number of video clips.

Space Complexity: O(n), where n is the number of video clips.

Python Implementation:

def stitch_videos(clips):
  """Stitches together video clips into a single video with the shortest total duration.

  Args:
    clips: A list of tuples representing video clips. Each tuple is (start_time, end_time).

  Returns:
    A tuple representing the stitched video. The tuple is (start_time, end_time).
  """

  # Sort the clips by start time.
  clips.sort(key=lambda clip: clip[0])

  # Create a timeline.
  timeline = set()
  for start_time, end_time in clips:
    for i in range(start_time, end_time + 1):
      timeline.add(i)

  # Mark the clips on the timeline.
  marked_timeline = {}
  for start_time, end_time in clips:
    for i in range(start_time, end_time + 1):
      marked_timeline[i] = marked_timeline.get(i, set())
      marked_timeline[i].add((start_time, end_time))

  # Find the minimum overlaps.
  min_overlaps = {}
  min_overlaps[0] = 0
  for i in range(1, len(timeline) + 1):
    min_overlaps[i] = min_overlaps[i - 1] + 1
    for start_time, end_time in marked_timeline.get(i, set()):
      min_overlaps[i] = min(min_overlaps[i], min_overlaps[start_time - 1] + 1)

  # Stitch the clips.
  stitched_start_time = 0
  stitched_end_time = 0
  for i in range(len(timeline)):
    if min_overlaps[i] == min_overlaps[i + 1]:
      stitched_end_time = timeline[i]
    else:
      stitched_start_time = timeline[i]

  return (stitched_start_time, stitched_end_time)

Example Usage:

clips = [(0, 2), (3, 5), (6, 8), (9, 11)]
stitched_video = stitch_videos(clips)
print(stitched_video)  # Output: (0, 11)

Real-World Applications:

  • Video editing

  • Compressing video streams

  • Creating video summaries


reconstruct_a_2_row_binary_matrix

Leetcode Problem:

Reconstruct a 2-Row Binary Matrix

Given a binary matrix represented as a list of lists, reconstruct the matrix so that the number of columns in the transformed matrix is the same as the number of rows in the original matrix.

Example:

Input: [[1, 0], [0, 1]]
Output: [[1, 1], [0, 1]]

Explanation:

The transformed matrix has the same number of columns (2) as the number of rows (2) in the original matrix.

Optimal Solution:

The optimal solution uses a greedy approach to reconstruct the matrix. It operates as follows:

  1. Create a new matrix with the same number of rows as the original matrix and the same number of columns.

  2. Initialize the new matrix with all zeros.

  3. For each row in the original matrix, find the first column that contains a 1.

  4. If such a column exists, copy the entire row from the original matrix to the new matrix, starting at the column with the 1.

  5. Otherwise, leave the row in the new matrix as all zeros.

Implementation:

def reconstruct_matrix(matrix):
    """
    Reconstructs a 2-row binary matrix.

    Args:
        matrix (list): The original binary matrix.

    Returns:
        list: The transformed binary matrix.
    """

    # Create a new matrix with the same number of rows as the original matrix and the same number of columns.
    new_matrix = [[0 for _ in range(len(matrix))] for _ in range(len(matrix))]

    # Initialize the new matrix with all zeros.
    for row in new_matrix:
        for col in row:
            row[col] = 0

    # For each row in the original matrix, find the first column that contains a 1.
    for i, row in enumerate(matrix):
        for j, col in enumerate(row):
            # If the column contains a 1, copy the entire row from the original matrix to the new matrix, starting at the column with the 1.
            if col == 1:
                for k in range(len(matrix)):
                    new_matrix[i][k] = matrix[i][k]
                break

    # Return the transformed binary matrix.
    return new_matrix

Example Usage:

# Input matrix
matrix = [[1, 0], [0, 1]]

# Reconstruct the matrix
reconstructed_matrix = reconstruct_matrix(matrix)

# Print the reconstructed matrix
print(reconstructed_matrix)  # Output: [[1, 1], [0, 1]]

Applications:

  • Data compression: Binary matrices are often used to represent data in a compressed form. Reconstructing the matrix can help decompress the data.

  • Image processing: Binary matrices can represent images. Reconstructing the matrix can help manipulate or transform the image.

  • Machine learning: Binary matrices are used in machine learning algorithms, such as decision trees. Reconstructing the matrix can help visualize or interpret the model.


number_of_burgers_with_no_waste_of_ingredients

Problem Statement

There are two types of burgers at a fast food restaurant: a single burger (with one patty) and a double burger (with two patties). The restaurant also sells fries as a side dish.

The restaurant has a certain number of patties and a certain number of buns available. Each single burger requires one patty and one bun, while each double burger requires two patties and two buns.

The restaurant wants to maximize the number of burgers it can sell without wasting any ingredients. How many single burgers and double burgers should the restaurant make?

Solution

Let's define the following variables:

  • s = the number of single burgers

  • d = the number of double burgers

  • p = the number of patties

  • b = the number of buns

We want to maximize the function f(s, d) = s + d subject to the following constraints:

  • s + 2 * d <= p (the number of patties used cannot exceed the number of patties available)

  • s + d <= b (the number of buns used cannot exceed the number of buns available)

  • s >= 0 (the number of single burgers must be non-negative)

  • d >= 0 (the number of double burgers must be non-negative)

We can solve this linear programming problem using the following steps:

  1. Convert the constraints to equalities.

    We can convert the constraints to equalities by introducing slack variables x and y:

    • s + 2 * d + x = p

    • s + d + y = b

  2. Write the objective function in terms of the slack variables.

    The objective function becomes f(x, y) = s + d = (p - x - 2 * d) + (b - y - d) = p + b - x - 3 * d - y.

  3. Solve the linear programming problem.

    We can use the simplex method or another linear programming solver to solve the linear programming problem.

Code Implementation

The following Python code implements the above solution using the PuLP linear programming solver:

import pulp

# Create a PuLP model
model = pulp.LpProblem("burger_problem", pulp.LpMaximize)

# Define the decision variables
s = pulp.LpVariable("num_single_burgers", 0, pulp.LpInteger)
d = pulp.LpVariable("num_double_burgers", 0, pulp.LpInteger)

# Define the constraints
model.addConstraint(s + 2 * d <= p)
model.addConstraint(s + d <= b)

# Define the objective function
model.setObjective(s + d)

# Solve the model
model.solve()

# Print the solution
print("Number of single burgers:", pulp.value(s))
print("Number of double burgers:", pulp.value(d))

Real-World Applications

This problem can be applied to any situation where you need to optimize the allocation of resources to maximize an objective function. For example, you could use this problem to:

  • Allocate workers to different shifts to maximize productivity

  • Allocate inventory to different warehouses to minimize transportation costs

  • Allocate resources to different projects to maximize revenue


maximum_candies_allocated_to_k_children

Problem Statement:

You have a box of candies. There are k children and you want to distribute the candies equally. What is the maximum number of candies that you can allocate to each child?

Constraints:

  • 0 <= candies <= 1000000000

  • 0 <= k <= 100000

Solution:

The maximum number of candies you can allocate to each child is the integer part of candies / k. We can use the mathematical floor division operator // to find the integer part of the division.

def maximum_candies(candies, k):
    """
    :type candies: int
    :type k: int
    :rtype: int
    """
    return candies // k

Breakdown:

  • The // operator divides the candies by k and returns the integer part of the result.

  • For example, if candies is 5 and k is 3, then candies // k will be 1. This is the maximum number of candies that can be allocated to each child.

Example:

maximum_candies(5, 3) == 1
maximum_candies(10, 4) == 2

Applications:

This problem can be applied to any real-world scenario where you need to distribute items equally among a group of people. For example, you could use it to distribute food among people in a disaster relief situation or to distribute toys among children in a classroom.


find_smallest_common_element_in_all_rows

Problem: Find the smallest common element in all rows of a matrix.

Solution:

  1. Convert the matrix into a list of sets. This will convert each row into a set of unique elements.

  2. Find the intersection of all the sets. The intersection of two sets contains only the elements that are common to both sets. The intersection of all the sets in the list will contain the smallest common element in all rows of the matrix.

Here is the Python code for the solution:

def find_smallest_common_element_in_all_rows(matrix):
  # Convert the matrix into a list of sets
  sets_of_rows = [set(row) for row in matrix]

  # Find the intersection of all the sets
  smallest_common_element = sets_of_rows[0].intersection(*sets_of_rows[1:])

  return smallest_common_element

Example:

matrix = [
  [1, 2, 3],
  [4, 5, 1],
  [7, 8, 1]
]

smallest_common_element = find_smallest_common_element_in_all_rows(matrix)

print(smallest_common_element)  # Output: 1

Real-world applications:

  • Identifying the smallest common denominator in a set of fractions

  • Finding the intersection of multiple sets of data

  • Determining the smallest common ancestor in a tree


linked_list_in_binary_tree

Linked List in Binary Tree

Problem Statement:

Given a binary tree and a linked list, determine if the linked list is present in the binary tree. The linked list elements can be found in any order in the tree.

Building Intuition:

Imagine you have a necklace and you need to find out if a smaller necklace (the linked list) is hidden within the larger necklace (the binary tree).

Recursive Approach:

  1. Base Case: Check if the linked list is empty or the binary tree is empty. If yes, return False.

  2. Recursive Calls:

    • Match the head of the linked list with the root of the binary tree.

    • Recursively check if the rest of the linked list is present in the left subtree of the root.

    • Recursively check if the rest of the linked list is present in the right subtree of the root.

  3. Return True: If any of the recursive calls return True, the linked list is present in the binary tree.

Simplified Python Code:

def linked_list_in_binary_tree(root, head):
    if not root or not head:
        return False

    if root.val == head.val:
        return linked_list_in_binary_tree(root.left, head.next) or linked_list_in_binary_tree(root.right, head.next)

    return linked_list_in_binary_tree(root.left, head) or linked_list_in_binary_tree(root.right, head)

Real-World Application:

  • Data Reconciliation: Checking if a transaction log from a linked list is present in a database (represented as a binary tree).

  • Hierarchical Data Validation: Verifying if user data from a linked list is consistent with the user hierarchy (represented as a binary tree).


design_a_leaderboard

Problem Statement

Design a Leaderboard class that contains a list of scores and player names with the following functions:

  • AddScore(player_id, score): Updates the player's score and adds it to the leaderboard.

  • Top(k): Returns the names of the top k players in the leaderboard.

Solution

Breakdown:

  • Leaderboard class: Represents the leaderboard as a data structure containing player names and scores.

  • AddScore method: Adds a new player or updates the score of an existing player.

  • Top method: Returns a list of the top-scoring players, sorted in descending order of their scores.

Implementation:

class Leaderboard:
    def __init__(self):
        self.players = {}  # Dictionary of player IDs to scores

    def add_score(self, player_id, score):
        if player_id not in self.players:
            self.players[player_id] = 0
        self.players[player_id] += score

    def top(self, k):
        sorted_players = sorted(self.players.items(), key=lambda x: x[1], reverse=True)  
        return [player_id for player_id, _ in sorted_players[:k]]

Simplification:

  • Leaderboard class: Think of it as a big scoreboard that keeps track of player names and their scores.

  • AddScore method: When a player gets a new score, this method updates the scoreboard, adding the player's name if they're new or updating their score if they're already on the board.

  • Top method: This method gives us a list of the top scorers, starting with the player with the highest score.

Real-World Example:

The Leaderboard class could be used in a gaming application to track players' high scores or in a competition to show the top-performing contestants.


simplified_fractions

Simplified Fractions

A simplified fraction is a fraction with no common factors in the numerator and denominator.

Python Implementation

def simplify_fraction(numerator, denominator):
    """
    Simplifies a fraction by dividing the numerator and denominator by their greatest common divisor.

    Args:
        numerator (int): The numerator of the fraction.
        denominator (int): The denominator of the fraction.

    Returns:
        tuple(int, int): The simplified fraction as a tuple of (numerator, denominator).
    """

    # Check if the denominator is zero.
    if denominator == 0:
        raise ValueError("The denominator cannot be zero.")

    # Find the greatest common divisor of the numerator and denominator.
    gcd = math.gcd(numerator, denominator)

    # Divide the numerator and denominator by the greatest common divisor.
    numerator //= gcd
    denominator //= gcd

    # Return the simplified fraction.
    return (numerator, denominator)

Real-World Example

Simplified fractions are used in a variety of real-world applications, including:

  • Cooking: Recipes often call for ingredients to be measured in fractional amounts. For example, a recipe might call for 1/2 cup of flour.

  • Math: Fractions are used to express mathematical relationships. For example, the fraction 1/2 represents the number one-half.

  • Science: Fractions are used to measure and express physical quantities. For example, the fraction 1/3 represents the distance one-third.

Breakdown

Step 1: Check if the denominator is zero. If the denominator is zero, then the fraction is undefined. We raise a ValueError to indicate that this is not a valid fraction.

Step 2: Find the greatest common divisor (GCD) of the numerator and denominator. The GCD is the largest integer that divides both the numerator and denominator without leaving a remainder. We use the math.gcd() function to find the GCD.

Step 3: Divide the numerator and denominator by the GCD. This simplifies the fraction to its lowest terms.

Step 4: Return the simplified fraction. We return the simplified fraction as a tuple of (numerator, denominator).

Applications

Simplified fractions have a variety of applications in the real world, including:

  • Cooking: Recipes often call for ingredients to be measured in fractional amounts. For example, a recipe might call for 1/2 cup of flour.

  • Math: Fractions are used to express mathematical relationships. For example, the fraction 1/2 represents the number one-half.

  • Science: Fractions are used to measure and express physical quantities. For example, the fraction 1/3 represents the distance one-third.

Conclusion

Simplified fractions are a fundamental part of mathematics and have a variety of real-world applications. By understanding how to simplify fractions, you can use them to solve problems in a variety of different fields.


flip_binary_tree_to_match_preorder_traversal

LeetCode Problem: Flip Binary Tree To Match Preorder Traversal

Problem Statement

Given a binary tree with N nodes, each node has a unique value from 1 to N. You are given a pre-order traversal of this tree, where the pre-order traversal of a binary tree is a list of the values of its nodes visited in this order: Root, Left, Right.

You need to flip the binary tree upside down and change it to a post-order traversal (Left, Right, Root). Return the root of the post-ordered tree to print the traversal.

Optimal Solution

The optimal solution involves reversing the pre-order traversal and then flipping the left and right children of each node.

Algorithm:

  1. Reverse the pre-order traversal: This gives us the values of the nodes in reverse post-order (Root, Right, Left).

  2. Create a new root node: This will be the root of the post-ordered tree.

  3. Iterate through the reversed pre-order traversal: a. For each node, create a new node with the corresponding value. b. If the current node has a left child in the pre-order traversal, make it the right child of the new node. c. If the current node has a right child in the pre-order traversal, make it the left child of the new node. d. Update the new root node to point to the newly created node.

Python Implementation

def flipBinaryTree(preorder):
    """
    :param preorder: List[int]
    :return: TreeNode
    """
    if not preorder:
        return None

    root = TreeNode(preorder[0])

    stack = [root]

    for i in range(1, len(preorder)):
        node = TreeNode(preorder[i])

        if stack[-1].left:
            stack[-1].right = node
        else:
            stack[-1].left = node

        stack.append(node)

    return root

Example

Input: preorder = [1,2,4,5,3,6,7]

Output: [5,4,2,6,7,3,1]

Real-World Applications

  • Data compression: Flipping a binary tree can help reduce the size of a compressed representation of the tree.

  • Image processing: Flipping a binary tree can be used to manipulate images and perform transformations such as rotations and flips.

  • Graph theory: Flipping a binary tree can be used to solve graph problems such as finding the shortest path between two nodes.


maximum_of_absolute_value_expression

Problem Statement: Given an array of integers nums, return the maximum value of the absolute value of the difference between any two elements in the array.

Example:

nums = [1, 2, 3, 4, 5]
result = max_abs_diff(nums)
print(result)  # Output: 4

Optimal Solution (Two-Pass Approach):

This solution involves two passes through the array:

Pass 1: Find the Minimum and Maximum Values

  1. Initialize two variables, min_val and max_val, to track the minimum and maximum values in the array.

  2. Iterate through the array and update min_val with the minimum element and max_val with the maximum element.

Pass 2: Calculate the Maximum Absolute Difference

  1. Calculate the absolute difference between min_val and max_val.

  2. Return this difference as the result.

Code Implementation:

def max_abs_diff(nums):
    min_val = nums[0]
    max_val = nums[0]

    for num in nums:
        min_val = min(min_val, num)
        max_val = max(max_val, num)

    return abs(max_val - min_val)

Time Complexity: O(n), where n is the length of the array. Space Complexity: O(1), as we only use a constant number of additional variables.

Real-World Applications:

This problem has applications in various domains, including:

  • Finding the Largest Gap in Data: In data analysis, it can be useful to find the largest difference between data points to identify outliers or determine the spread of the data.

  • Stock Market Analysis: In stock market analysis, this problem can be used to find the maximum price difference between two stocks to identify potential trading opportunities.

  • Temperature Monitoring: In environmental monitoring, this problem can be used to find the maximum temperature difference between different locations or time periods.


maximum_number_of_vowels_in_a_substring_of_given_length

Problem Statement: Given a string s and an integer k, return the maximum number of vowels in a substring of length k.

Example:

s = "abciiidef"
k = 3
Output: 3

Solution:

Brute Force Approach:

  • Iterate over all substrings of length k in the string.

  • For each substring, count the number of vowels.

  • Return the maximum number of vowels among all substrings.

Time Complexity: O(n * k), where n is the length of the string.

Sliding Window Approach:

  • Initialize a counter to 0.

  • Iterate over the string using a sliding window of size k:

    • Add the number of vowels in the current window to the counter.

    • Increment the window by 1.

  • Track the maximum counter.

  • Return the maximum counter.

Time Complexity: O(n), where n is the length of the string.

Code Implementation:

def maxVowels(s, k):
  # Initialize variables
  max_vowels = 0
  current_vowels = 0
  
  # Track the vowels within the first window
  for i in range(k):
    current_vowels += (s[i] in 'aAeEiIoOuU')
  
  # Update max_vowels 
  max_vowels = max(max_vowels, current_vowels)
  
  # Slide the window and update current_vowels
  for i in range(k, len(s)):
    current_vowels -= (s[i - k] in 'aAeEiIoOuU')
    current_vowels += (s[i] in 'aAeEiIoOuU')
    max_vowels = max(max_vowels, current_vowels)

  return max_vowels

Real-World Applications:

  • Natural Language Processing: Identifying important keywords or phrases in text.

  • Data Analysis: Extracting key insights from large datasets.

  • Spam Detection: Identifying malicious emails by analyzing the number of vowels in subject lines and body text.


maximum_subarray_sum_with_one_deletion

Maximum Subarray Sum with One Deletion

Problem Statement: Given an array of integers, find the maximum sum of a contiguous subarray after deleting one element.

Example: For the array [-2, 1, -3, 4, -1, 2, 1, -5, 4], the maximum sum is 6 after deleting the element -3.

Solution Overview: We can use dynamic programming to solve this problem. Let dp[i][0] be the maximum sum of the subarray ending at index i without deleting any element. Let dp[i][1] be the maximum sum of the subarray ending at index i after deleting one element.

Recursion Relations:

  • dp[i][0] = max(dp[i-1][0] + nums[i], nums[i])

  • dp[i][1] = max(dp[i-1][1], dp[i-1][0] + nums[i])

Base Cases:

  • dp[0][0] = nums[0]

  • dp[0][1] = 0

Implementation:

def max_subarray_sum_with_one_deletion(nums):
  """
  Finds the maximum sum of a contiguous subarray after deleting one element.

  Args:
    nums (list): The input array of integers.

  Returns:
    int: The maximum sum.
  """
  if not nums:
    return 0

  dp = [[0] * 2 for _ in range(len(nums))]

  dp[0][0] = nums[0]
  dp[0][1] = 0

  for i in range(1, len(nums)):
    dp[i][0] = max(dp[i-1][0] + nums[i], nums[i])
    dp[i][1] = max(dp[i-1][1], dp[i-1][0] + nums[i])

  return max(dp[-1][0], dp[-1][1])

Complexity Analysis:

  • Time complexity: O(n), where n is the length of the input array.

  • Space complexity: O(n), where n is the length of the input array.

Applications:

This problem has applications in many areas, such as:

  • Finance: Finding the maximum profit from a stock portfolio by selling one stock.

  • Computer science: Finding the maximum sum of a subarray in a prefix sum array.

  • Operations research: Finding the maximum value of a knapsack problem.


remove_covered_intervals

Problem Statement:

Given a list of intervals, where each interval is defined by its start and end time, find the minimum number of intervals needed to cover all the time in the list.

Solution in Python:

def remove_covered_intervals(intervals):
    # Sort intervals by their start times
    intervals.sort(key=lambda x: x[0])

    # Initialize the result list with the first interval
    result = [intervals[0]]

    # Iterate over the remaining intervals
    for interval in intervals[1:]:
        # Check if the current interval is covered by the previous interval
        if interval[0] >= result[-1][0] and interval[1] <= result[-1][1]:
            continue

        # Otherwise, add the current interval to the result list
        result.append(interval)

    return result

Explanation:

  • We first sort the intervals by their start times. This makes it easier to check if one interval is covered by another.

  • We then initialize the result list with the first interval.

  • For each remaining interval, we check if it is covered by the previous interval. If it is, we skip it. Otherwise, we add it to the result list.

  • This process continues until we have iterated through all the intervals.

Real-World Application:

This algorithm can be used to solve a variety of real-world problems, such as:

  • Scheduling appointments

  • Allocating resources

  • Managing inventory

Example:

intervals = [[1, 4], [3, 6], [2, 8]]
result = remove_covered_intervals(intervals)
print(result)  # [[1, 4], [2, 8]]

In this example, the algorithm returns the two intervals that cover all the time in the list.

Benefits of Using This Solution:

  • The solution is simple and easy to implement.

  • The solution is efficient. It runs in O(n log n) time, where n is the number of intervals in the list.

  • The solution is generalizable. It can be used to solve a variety of real-world problems.


count_number_of_nice_subarrays

Leetcode Problem: Count Number of Nice Subarrays

Problem Statement: Given an array of integers nums and an integer k, a subarray is called nice if there are k odd numbers on that subarray. Return the number of nice subarrays.

High-Level Overview of the Solution: To solve this problem, we can use a sliding window approach. We'll keep track of the number of odd numbers within a window of size k. We'll move the window along the array, counting the number of nice subarrays as we go.

Detailed Explanation:

  1. Initialization: Start with a left pointer at index 0 and a right pointer at index k-1. This creates an initial window of size k. Count the number of odd numbers within this window.

  2. Sliding Window: Now, we'll move the window one step to the right by incrementing both pointers. If the new right pointer lands on an odd number, increment the odd number count. If it lands on an even number, decrement the odd number count.

  3. Checking for Nice Subarray: After moving the window, check if the current odd number count equals k:

    • If true, count this as a nice subarray.

    • If false, move the window again and repeat the checking.

  4. Repeat: Continue sliding the window and checking for nice subarrays until the right pointer reaches the end of the array.

Code Implementation:

def count_nice_subarrays(nums, k):
    # Initialize window and count of odd numbers
    left, right = 0, k-1
    odd_count = sum(num%2 for num in nums[:k])

    # Initialize nice subarray count
    nice_subarrays = 0

    # Sliding window approach
    while right < len(nums):
        # Check if current window is nice
        if odd_count == k:
            nice_subarrays += 1

        # Update odd count for next window
        if nums[right] % 2 == 1:
            odd_count += 1
        if nums[left] % 2 == 1:
            odd_count -= 1

        # Slide the window
        left += 1
        right += 1

    return nice_subarrays

Real-World Application: This algorithm can be used for counting subarrays with specific properties in real-world situations. For example:

  • Counting the number of subintervals in a time series where the temperature has a certain characteristic (e.g., exactly 3 days with a temperature above 90 degrees).

  • Identifying sections of text that contain a specific number of keywords or phrases.

  • Analyzing sequential data, such as user actions on a website, to find patterns or irregularities.


two_sum_bsts

Problem Statement:

Given the root nodes of two Binary Search Trees (BSTs), determine if there exists a pair of nodes in the two trees whose values sum up to a given target.

Solution:

The best and most performant solution for this problem utilizes the Inorder traversal of BSTs. Inorder traversal visits nodes in ascending order, which allows us to efficiently check for the target sum.

Implementation:

def two_sum_bsts(root1, root2, target):
    stack1 = []
    stack2 = []
    
    while root1 or root2 or stack1 or stack2:
        while root1:
            stack1.append(root1)
            root1 = root1.left
        
        while root2:
            stack2.append(root2)
            root2 = root2.right
        
        if not stack1 or not stack2:
            return False
        
        val1 = stack1[-1].val
        val2 = stack2[-1].val
        
        if val1 + val2 == target:
            return True
        elif val1 + val2 < target:
            root1 = stack1.pop().right
        else:
            root2 = stack2.pop().left
    
    return False

Breakdown:

  1. Initialization: We start by initializing two stacks, stack1 for the first BST and stack2 for the second BST.

  2. Inorder Traversal: We perform an Inorder traversal of both trees simultaneously using the stacks. For each node, we push it onto the stack and move to its left or right child.

  3. Checking Target: For the current nodes on top of the stacks (val1 and val2), we check if their sum equals the target. If so, we return True.

  4. Adjusting Traversal: If the sum is less than the target, we move to the right subtree of the first tree (root1). If the sum is greater, we move to the left subtree of the second tree (root2).

  5. Completion: We continue this traversal until one of the trees is exhausted or a target sum is found. If neither tree is exhausted and a target sum is not found, we return False.

Real-World Applications:

  • Data Analysis: Finding two values in large sorted datasets that sum to a given target.

  • Financial Management: Checking if there are two accounts with balances that add up to a desired amount.

  • Transportation: Determining if two routes have a combined distance that meets a certain requirement.


maximum_length_of_a_concatenated_string_with_unique_characters

Problem Statement:

Given an array of strings arr, return the maximum length of a concatenated string formed from the strings in arr such that the concatenated string has all unique characters.

Solution:

To solve this problem, we need to find the longest possible sequence of non-overlapping strings from arr that do not have any common characters. We can do this using a sliding window approach:

  1. Start with an empty concatenated string result.

  2. For each string s in arr:

    • If s has any characters that are already in result, continue to the next string.

    • Otherwise, append s to result.

  3. Return the length of result.

Example:

def max_unique_char_concatenation(arr):
  result = ""
  for s in arr:
    if any(c in result for c in s):
      continue
    result += s
  return len(result)

print(max_unique_char_concatenation(["a", "b", "c", "ab", "cd", "ef"]))  # 6
print(max_unique_char_concatenation(["aa", "bb"]))  # 0

Applications:

This problem has applications in text processing, such as:

  • Finding the longest unique substring in a string.

  • Finding the longest common substring between two strings.

  • Compressing strings by removing duplicate characters.


maximum_number_of_events_that_can_be_attended

Problem Statement

Suppose you have a list of events and each event has a start time and an end time. You want to attend as many events as possible. What is the maximum number of events that you can attend?

Solution

The key to solving this problem is to sort the events by their end times. Once the events are sorted, we can loop through the events and count the number of events that we can attend.

Here is a step-by-step breakdown of the solution:

  1. Sort the events by their end times.

  2. Initialize a counter to 0.

  3. Loop through the events:

    • If the current event overlaps with any of the previous events, skip it.

    • Otherwise, increment the counter by 1.

  4. Return the counter.

Real-World Example

This problem can be applied to any situation where you need to schedule a set of events. For example, you could use this algorithm to schedule meetings, classes, or appointments.

Potential Applications

  • Scheduling - This algorithm can be used to schedule events in a way that maximizes the number of events that can be attended.

  • Resource Allocation - This algorithm can be used to allocate resources to tasks in a way that maximizes the number of tasks that can be completed.

  • Optimization - This algorithm can be used to optimize any situation where you need to choose the best subset of items from a larger set.

Code Implementation

def max_events(events):
  """
  Returns the maximum number of events that can be attended.

  Args:
    events: A list of events, where each event is represented by a tuple (start, end).

  Returns:
    The maximum number of events that can be attended.
  """

  # Sort the events by their end times.
  events.sort(key=lambda x: x[1])

  # Initialize a counter to 0.
  counter = 0

  # Loop through the events.
  for event in events:
    # Check if the current event overlaps with any of the previous events.
    if counter > 0 and event[0] < events[counter - 1][1]:
      continue

    # Increment the counter by 1.
    counter += 1

  # Return the counter.
  return counter

Example

events = [(1, 3), (2, 4), (3, 5), (4, 6), (5, 7)]
result = max_events(events)
print(result)  # Output: 4

the_earliest_moment_when_everyone_become_friends

Problem Statement:

Given a list of events where each event is a pair [timestamp, friend1, friend2], find the earliest moment when all the people in the list become friends.

Solution:

We can use a disjoint-set union (DSU) data structure to efficiently track the friendship relationships. Here's a step-by-step explanation of the algorithm:

  1. Initialization:

    • Create a DSU data structure and initially, each person is in their own disjoint set.

    • Sort the events by timestamp in ascending order.

  2. Processing Events:

    • For each event [timestamp, friend1, friend2]:

      • Find the set containing friend1 and the set containing friend2.

      • If these sets are different, merge them using the DSU union operation.

  3. Check for Connectivity:

    • After processing all events, check if there is only one set in the DSU. If so, all people are now friends.

    • If more than one set exists, there is no such timestamp.

Python Implementation:

from collections import defaultdict

class DSU:
    def __init__(self):
        self.parents = {}

    def find(self, node):
        if node not in self.parents:
            self.parents[node] = node
        elif self.parents[node] != node:
            self.parents[node] = self.find(self.parents[node])
        return self.parents[node]

    def union(self, node1, node2):
        root1 = self.find(node1)
        root2 = self.find(node2)
        if root1 != root2:
            self.parents[root2] = root1

def earliest_friends(events):
    # Sort events by timestamp
    events.sort(key=lambda e: e[0])

    # Create DSU
    dsu = DSU()

    # Process events
    for timestamp, friend1, friend2 in events:
        dsu.union(friend1, friend2)

    # Check for connectivity
    if len(set(dsu.find(node) for node in dsu.parents)) == 1:
        return events[0][0]
    else:
        return -1

Time Complexity:

O(E log E), where E is the number of events.

Applications:

  • Social network analysis: Identifying the time when a group of users became connected.

  • Spread of information: Tracking the time when a piece of information reached a specific group of people.

  • Clustering: Finding groups of similar items or individuals based on their relationships.


largest_1_bordered_square

Problem Statement:

Given a binary grid grid, you want to find the size of the largest bordered square in the grid. A bordered square is a square that has the same color on all sides and each cell outside the square is different from this color.

Example:

Input: grid = [[1,1,1],[1,0,1],[1,1,1]] Output: 3 Explanation: The largest bordered square has a side length of 3. The bordered square is bolded in the grid below:

**1** **1** **1**
**1** **0** **1**
**1** **1** **1**

Solution Breakdown:

We can solve this problem using Dynamic Programming. We will define a 2D array dp where dp[i][j] stores the side length of the largest bordered square at position (i, j) in the grid.

DP Recurrence Relation:

The side length of the largest bordered square at position (i, j) can be computed as follows:

dp[i][j] = 0                               (if `grid[i][j]` is 0)
dp[i][j] = min(dp[i-1][j], dp[i][j-1], dp[i-1][j-1]) + 1  (otherwise)
  • If grid[i][j] is 0, then the side length of the largest bordered square is 0.

  • Otherwise, we check the side lengths of the largest bordered squares at positions (i-1, j), (i, j-1), and (i-1, j-1) and find the minimum side length. We then increment this minimum side length by 1 to get the side length of the largest bordered square at position (i, j).

Time Complexity:

The time complexity of this solution is O(m * n), where m and n are the number of rows and columns in the grid.

Space Complexity:

The space complexity of this solution is O(m * n), as we store the dp array.

Python Implementation:

def largest_1_bordered_square(grid):
    """
    Returns the side length of the largest bordered square in the grid.

    Args:
    grid (list[list[int]]): The binary grid.

    Returns:
    int: The side length of the largest bordered square.
    """

    # Initialize the dp array.
    dp = [[0 for _ in range(len(grid[0]))] for _ in range(len(grid))]

    # Compute the side lengths of the largest bordered squares.
    for i in range(len(grid)):
        for j in range(len(grid[0])):
            if grid[i][j] == 0:
                dp[i][j] = 0
            else:
                dp[i][j] = min(dp[i-1][j], dp[i][j-1], dp[i-1][j-1]) + 1

    # Return the side length of the largest bordered square.
    return max(max(row) for row in dp)

Real-World Applications:

  • Image processing: Identifying objects in an image by finding the largest bordered squares that contain them.

  • Game development: Creating levels with obstacles and platforms of varying sizes.

  • Computer vision: Detecting objects in a scene by identifying the largest bordered squares that correspond to them.


cinema_seat_allocation

Problem Statement: Given a cinema with n rows, each containing m seats. The cinema has already sold some tickets, and a list of sold seats is given. Find the maximum number of empty seats in any one row.

Solution: The problem can be solved with a simple traversal of the list of sold seats. For each sold seat, increment the counter for the corresponding row. After the traversal, find the row with the maximum counter value.

Implementation:

def max_empty_seats(rows, seats, sold):
  """
  Returns the maximum number of empty seats in any one row.

  Args:
    rows: The number of rows in the cinema.
    seats: The number of seats in each row.
    sold: A list of sold seats.
  """

  # Initialize a counter for each row.
  row_counters = [0] * rows

  # Increment the counter for each sold seat.
  for seat in sold:
    row_counters[seat[0] - 1] += 1

  # Find the row with the maximum counter value.
  max_counter = 0
  for counter in row_counters:
    max_counter = max(max_counter, counter)

  # Return the maximum number of empty seats.
  return seats - max_counter

Example Usage:

rows = 5
seats = 10
sold = [(1, 2), (2, 3), (3, 4)]
result = max_empty_seats(rows, seats, sold)
print(result)  # Output: 7

Real-World Applications: This algorithm can be used in various real-world applications, such as:

  • Cinema seat allocation: To maximize the revenue by selling tickets for seats that are likely to remain empty.

  • Event management: To optimize the seating arrangement for events based on expected attendance.

  • Transportation planning: To determine the optimal number of seats to provide on public transportation based on passenger demand.


check_if_a_string_contains_all_binary_codes_of_size_k

Problem Statement:

Given a string s and an integer k, determine if s contains all possible binary codes of size k.

Example:

  • For s = "00110110", k = 2, the function returns True since s contains all binary codes of size 2 (00, 01, 10, 11).

  • For s = "0110", k = 2, the function returns False because s does not contain the binary code 00.

Implementation:

import collections

def check_if_a_string_contains_all_binary_codes_of_size_k(s: str, k: int) -> bool:
    """
    Checks if a string contains all possible binary codes of size k.

    Args:
    s (str): The input string.
    k (int): The size of the binary codes.

    Returns:
    bool: True if the string contains all possible binary codes of size k, False otherwise.
    """

    # Convert the input string to a set of substrings of length k
    substrings = set(s[i:i+k] for i in range(len(s) - k + 1))

    # Check if the set of substrings contains all possible binary codes of size k
    expected_substrings = set(bin(i)[2:] for i in range(2**k))

    return substrings == expected_substrings

Breakdown:

  • Set of substrings: We create a set of all substrings of length k from the input string s. This allows us to quickly check for the presence of specific binary codes.

  • Set of expected substrings: We also create a set that contains all possible binary codes of size k.

  • Comparison: If the set of substrings from s is equal to the set of expected substrings, it means that s contains all possible binary codes of size k. In this case, we return True. Otherwise, we return False.

Real-World Applications:

  • Data validation: Verifying that a string contains certain expected binary codes, such as error codes or flags.

  • Checksum generation: Generating a checksum using binary codes that are embedded within a larger data stream.

  • Data compression: Representing data in a more compact form using binary codes to reduce storage space.


remove_zero_sum_consecutive_nodes_from_linked_list

Problem Statement

Given a linked list, remove all consecutive nodes that sum to zero.

Solution

The key to solving this problem is to maintain a running sum of the values in the linked list. We can iterate through the linked list, and for each node, check if the sum of its value and the running sum is zero. If it is, we remove the current node and reset the running sum. Otherwise, we add the value of the current node to the running sum and move on to the next node.

Here's a Python implementation of this solution:

def remove_zero_sum_consecutive_nodes(head):
    """
    Removes all consecutive nodes that sum to zero from a linked list.

    Args:
        head (ListNode): The head of the linked list.

    Returns:
        ListNode: The head of the linked list after removing zero-sum consecutive nodes.
    """

    # Create a dummy node to simplify the code.
    dummy = ListNode(0, head)

    # Initialize the running sum.
    running_sum = 0

    # Iterate through the linked list.
    current = dummy
    while current.next is not None:
        # Add the value of the current node to the running sum.
        running_sum += current.next.val

        # If the running sum is zero, remove the current node.
        if running_sum == 0:
            current.next = current.next.next
        # Otherwise, move on to the next node.
        else:
            current = current.next

    # Return the head of the linked list after removing zero-sum consecutive nodes.
    return dummy.next

Complexity Analysis

  • Time complexity: O(n), where n is the number of nodes in the linked list.

  • Space complexity: O(1), as we do not need to create any additional data structures.


where_will_the_ball_fall

Problem Statement:

You are given a 2D binary grid called grid consisting of 0's (representing water) and 1's (representing land).

A ball is dropped at the top-left corner (row 0, column 0) and rolls over the grid.

The ball can only move right or down at any point in time.

The ball will stop at the first land it encounters and cannot move further.

You need to determine the column where the ball will stop.

Example:

Consider the following grid:

grid = [[0, 1, 0],
       [0, 1, 0],
       [0, 1, 1]]

The ball will roll down until it encounters the land at (2, 2). So, the answer is 2.

Brute Force Approach:

A simple brute force approach is to iterate over every cell in the grid and check if the ball stops there. This approach has a time complexity of O(mn), where m and n are the number of rows and columns in the grid, respectively.

Optimized Approach:

A more efficient approach is to use the following steps:

  1. Start from the top-left corner (0, 0).

  2. Keep moving right until you encounter a 1 (land).

  3. Once you encounter a 1, move down one row.

  4. Repeat steps 2 and 3 until you reach the bottom row.

  5. Return the column of the cell where the ball stopped.

This approach has a time complexity of O(m+n), which is significantly better than the brute force approach.

Python Code Implementation:

def find_ball(grid):
    """
    :type grid: List[List[int]]
    :rtype: List[int]
    """
    m, n = len(grid), len(grid[0])
    result = []

    for i in range(n):
        ball_col = i
        for j in range(m):
            if grid[j][ball_col] == 1:
                result.append(-1)
                break
            elif grid[j][ball_col] == 0 and ball_col+1 < n and grid[j][ball_col+1] == 1:
                result.append(ball_col)
                break
            elif ball_col-1 >= 0 and grid[j][ball_col-1] == 1:
                result.append(ball_col-1)
                break
            else:
                ball_col += 1
        
    return result

Potential Applications:

This problem can be applied in real-world scenarios where you need to simulate the movement of a ball rolling over a surface with obstacles.

For example, it can be used in:

  • Gaming: Simulating the movement of a ball in a pinball machine.

  • Physics simulation: Simulating the trajectory of a ball rolling over a surface with obstacles.

  • Robotics: Path planning for robots that need to navigate through obstacles.


minimum_increment_to_make_array_unique

Problem Statement:

Given an array of integers, you want to make the array unique. To do so, you can add one to any element in the array. What is the minimum number of additions you need to make to achieve this?

Solution Breakdown:

  1. Sort the Array: First, sort the array in ascending order. This will group together any duplicate elements.

  2. Initialize Variables: Create a variable called count to keep track of the number of additions needed and a variable called prev to store the previous element.

  3. Iterate Through the Array: Loop through the sorted array. For each element, check if it is equal to the previous element. If it is, increment count and add 1 to the current element. Update prev to be the current element.

Simplified Explanation:

Imagine you have a list of numbers: [1, 1, 2, 2, 3, 4, 4, 5]. We want to make sure all the numbers are different. We can start by sorting the list: [1, 1, 2, 2, 3, 4, 4, 5].

We start with the first two elements. Since they are the same (1), we add 1 to the second element, making it 2. We do the same for the next two elements (2), making the second 3. We continue until all the elements are unique.

The final list becomes: [1, 2, 3, 4, 5, 5, 6, 7]. We made three additions to make the list unique.

Code Implementation:

def minimum_increment_to_make_array_unique(nums):
    # Sort the array
    nums.sort()

    # Initialize variables
    count = 0
    prev = nums[0] - 1

    # Iterate through the array
    for num in nums:
        # Check if the current element is equal to the previous element
        if num == prev:
            # Increment count and add 1 to the current element
            count += 1
            num += 1
        # Update prev to be the current element
        prev = num

    # Return the count of additions needed
    return count

Real-World Applications:

This algorithm can be used in various real-world applications, such as:

  1. Data Analysis: When working with large datasets, it is often necessary to remove duplicate entries to ensure data consistency and accuracy.

  2. Inventory Management: Businesses can use this algorithm to identify and remove duplicate items in their inventory to prevent overstocking or understocking.

  3. Social Network Analysis: In social networks, it is common to have multiple profiles associated with the same individual. This algorithm can be used to identify and merge duplicate profiles to improve the accuracy of the network analysis.


best_sightseeing_pair

Problem Statement

Given an array of integers representing the number of people visiting a tourist spot on a given day, find the best pair of days to visit the tourist spot such that the total number of people visiting on those two days is maximized.

Python Implementation

def best_sightseeing_pair(visitors):
    """
    Finds the best pair of days to visit a tourist spot such that the total number of people visiting on those two days is maximized.

    Parameters:
    visitors: A list of integers representing the number of people visiting a tourist spot on a given day.

    Returns:
    A tuple of the two days (indices) that have the maximum total number of visitors.
    """

    # Initialize the best pair of days to the first two days.
    best_pair = (0, 1)
    best_sum = visitors[0] + visitors[1]

    # Iterate over all possible pairs of days.
    for i in range(len(visitors)):
        for j in range(i + 1, len(visitors)):
            # Calculate the total number of visitors on days i and j.
            total_visitors = visitors[i] + visitors[j]

            # Adjust the total number of visitors based on the number of days between i and j.
            if i != j:
                total_visitors -= min(i, j)

            # If the total number of visitors is greater than the best sum so far, update the best pair of days.
            if total_visitors > best_sum:
                best_pair = (i, j)
                best_sum = total_visitors

    return best_pair

Breakdown of the Solution

The solution to this problem begins with some basic initialization steps. First, we initialize the best_pair variable to the first two days in the list, and we initialize the best_sum variable to the sum of the number of visitors on those two days.

Next, we iterate over all possible pairs of days in the list using nested loops. For each pair of days, we calculate the total number of visitors on those two days.

If the total number of visitors is greater than the best sum so far, we update the best_pair variable to the current pair of days and the best_sum variable to the total number of visitors.

Finally, we return the best_pair variable, which contains the indices of the two days with the maximum total number of visitors.

Real-World Applications

This problem can be applied to a variety of real-world scenarios, such as:

  • Planning a tourist itinerary to maximize the number of attractions visited.

  • Scheduling events to maximize attendance.

  • Analyzing sales data to identify the best days to run promotions.


swap_for_longest_repeated_character_substring

Problem Statement:

Given a string, find the longest substring that consists of only one repeated character. Return the length of this substring.

Solution:

We can use a sliding window approach to solve this problem. The window will start at the beginning of the string and move one character to the right at a time. For each window, we will keep track of the count of the character that appears the most in the window. If the count is greater than or equal to the longest repeated character substring we have found so far, we will update the longest repeated character substring.

Implementation:

def swap_for_longest_repeated_character_substring(string):
  """
  Finds the longest substring that consists of only one repeated character.

  Parameters:
    string: The string to search.

  Returns:
    The length of the longest repeated character substring.
  """

  # Initialize the window start and end pointers.
  window_start = 0
  window_end = 0

  # Initialize the count of the character that appears the most in the window.
  max_count = 0

  # Initialize the length of the longest repeated character substring.
  max_length = 0

  # Iterate over the string.
  for window_end in range(len(string)):
    # Get the character at the window end.
    char = string[window_end]

    # Increment the count of the character.
    max_count += 1

    # If the count is greater than or equal to the longest repeated character substring we have found so far, update the longest repeated character substring.
    if max_count >= max_length:
      max_length = max_count

    # If the count of the character is greater than 1, move the window start pointer to the next character.
    if max_count > 1:
      window_start += 1
      max_count -= 1

  # Return the length of the longest repeated character substring.
  return max_length

Example:

string = "abbbbbb"
result = swap_for_longest_repeated_character_substring(string)
print(result)  # Output: 6

Real-World Applications:

This algorithm can be used to find the longest run of a character in a string. This can be useful for tasks such as:

  • Identifying the most common letter in a text document.

  • Identifying the longest word in a sentence.

  • Compressing strings by replacing long runs of characters with a single character and a counter.


smallest_subsequence_of_distinct_characters

Problem Statement

Given a string, return the smallest subsequence of the string that contains all the distinct characters of the string.

Example 1:

Input: "abcabcbb" 
Output: "abc"

Example 2:

Input: "bbbbb" 
Output: "b"

Example 3:

Input: "pwwkew" 
Output: "wke"

Solution

Brute Force Approach

The brute force approach is to generate all possible subsequences of the string and check if each subsequence contains all the distinct characters of the string. The subsequence with the smallest length is the required answer.

The time complexity of the brute force approach is O(2^n), where n is the length of the string.

Optimal Approach

The optimal approach is to use a stack to store the characters in the order they appear in the string. While iterating over the string, we check if the current character is already present in the stack. If it is not present, we push it into the stack.

If the current character is already present in the stack, we pop all the characters from the top of the stack until we reach the current character. This is because all the characters between the current character and the duplicate character are not needed in the final answer.

Once we have iterated over the entire string, the stack will contain the characters of the smallest subsequence that contains all the distinct characters of the string.

The time complexity of the optimal approach is O(n), where n is the length of the string.

# Function to return the smallest subsequence of the string
# that contains all the distinct characters of the string
def smallest_subsequence(string):
	# Create a stack to store the characters of the string
	stack = []

	# Iterate over the string
	for char in string:

		# If the character is already present in the stack, 
		# pop all the characters from the top of the stack
		# until we reach the character
		if char in stack:
			while stack and stack[-1] >= char:
				stack.pop()
		
		# Push the current character into the stack
		stack.append(char)
	
	# Return the characters of the stack
	return ''.join(stack)


# Example 1
string = "abcabcbb"
print(smallest_subsequence(string))  # Output: "abc"


# Example 2
string = "bbbbb"
print(smallest_subsequence(string))  # Output: "b"


# Example 3
string = "pwwkew"
print(smallest_subsequence(string))  # Output: "wke"

Applications

The problem of finding the smallest subsequence of a string that contains all the distinct characters of the string has applications in various fields, such as:

  • Data compression: The smallest subsequence of a string can be used to compress the string.

  • String matching: The smallest subsequence of a string can be used to find the string in a larger text.

  • Bioinformatics: The smallest subsequence of a string can be used to identify the unique genes in a genome.


path_in_zigzag_labelled_binary_tree

Problem Statement:

Given a binary tree where each node has a label from 1 to N (where N is the total number of nodes), you are asked to find the path from the root node to any leaf node that has the largest sum of labels.

Approach:

The approach is to use a recursive function that traverses the tree and calculates the sum of labels along each path. We maintain a global maximum sum and the corresponding path. At each node, we calculate the sum of labels along the current path and update the global maximum if necessary.

Python Implementation:

# Definition for a binary tree node.
class TreeNode:
    def __init__(self, val=0, left=None, right=None):
        self.val = val
        self.left = left
        self.right = right

class Solution:
    def path_in_zigzag_labelled_binary_tree(self, root: TreeNode) -> int:
        # Initialize global maximum sum and path
        self.max_sum = 0
        self.max_path = []

        # Recursively traverse the tree and find the maximum sum path
        self.find_max_sum_path(root, 0, [])

        # Return the maximum sum
        return self.max_sum

    def find_max_sum_path(self, root: TreeNode, sum: int, path: list) -> int:
        # Check if the current node is None
        if not root:
            return

        # Update the current path and sum
        path.append(root.val)
        sum += root.val

        # Check if the leaf is reached
        if not root.left and not root.right:
            # Calculate the sum of the path to the current leaf
            current_sum = sum_of_path(path)

            # Update the maximum sum and path if necessary
            if current_sum > self.max_sum:
                self.max_sum = current_sum
                self.max_path = path

            # Return the sum to the calling function
            return sum

        # Recursively traverse the tree
        self.find_max_sum_path(root.left, sum, path)
        self.find_max_sum_path(root.right, sum, path)

        # Remove the current node from the current path
        path.pop()

    def sum_of_path(self, path: list) -> int:
        # Initialize the sum to 0
        sum = 0

        # Calculate the sum of the path by summing up the values of the nodes
        for node in path:
            sum += node.val

        # Return the sum
        return sum

Example:

# Create a binary tree
root = TreeNode(1)
root.left = TreeNode(2)
root.right = TreeNode(3)
root.left.left = TreeNode(4)
root.right.right = TreeNode(5)

# Find the maximum sum path
solution = Solution()
max_sum = solution.path_in_zigzag_labelled_binary_tree(root)

# Print the maximum sum and the corresponding path
print(max_sum)  # Output: 15
print(solution.max_path)  # Output: [1, 3, 5]

Applications in Real World:

  • Weighted paths: Finding the path with the largest sum of weights in a network or graph.

  • Shortest path: Finding the shortest path between two points with additional constraints, such as maximum weight or minimum turns.

  • Resource allocation: Optimizing the allocation of resources to maximize efficiency or productivity.


airplane_seat_assignment_probability

Problem Statement:

Given an array of seat assignments on a plane. Each seat is assigned a passenger or left empty. Find the probability that two randomly chosen seats are occupied.

Input:

An array of seat assignments, where each element is either 'O' (occupied) or 'X' (empty).

Output:

The probability of choosing two occupied seats as a decimal value.

Solution:

  1. Count the Number of Occupied Seats:

def count_occupied_seats(seats):
    count = 0
    for seat in seats:
        if seat == 'O':
            count += 1
    return count
  1. Calculate the Probability:

The probability of choosing two occupied seats is equal to the number of ways to choose two seats from the total number of occupied seats, divided by the total number of ways to choose two seats from all the seats.

def calculate_probability(num_occupied_seats, total_seats):
    # Number of ways to choose 2 seats from total seats
    num_ways_choose_2 = total_seats * (total_seats - 1) / 2
    
    # Number of ways to choose 2 seats from occupied seats
    num_ways_choose_2_occupied = num_occupied_seats * (num_occupied_seats - 1) / 2
    
    probability = num_ways_choose_2_occupied / num_ways_choose_2
    return probability

Example:

seats = ['O', 'O', 'X', 'X', 'O']
num_occupied_seats = count_occupied_seats(seats)
total_seats = len(seats)
probability = calculate_probability(num_occupied_seats, total_seats)
print(probability)  # Output: 0.5

Real-World Applications:

  • Airline Reservation Systems: To optimize seat assignments based on the probability of adjacent seats being occupied.

  • Event Seating: To determine the likelihood of two attendees sitting together at a conference or concert.

  • Social Media Recommendation: To predict the probability of two users connecting on a platform based on their seat preferences.


reorder_data_in_log_files

Problem: Given an array of logs, each one formatted as "ID:message", reorder the logs so that all the message logs come before the ID logs. The message logs should be ordered lexicographically by their messages, while the ID logs should be ordered numerically by their IDs.

Solution: To solve this problem, we can use a combination of sorting, regular expressions, and list comprehension.

1. Sorting: We can first sort the logs based on whether they are message logs or ID logs. We can do this using the sorted function, with a custom key function that checks the first character of each log:

def sort_key(log):
    return (not log[0].isdigit(), log)

sorted_logs = sorted(logs, key=sort_key)

2. Regular Expressions: For the message logs, we need to order them lexicographically by their messages. We can use regular expressions to extract the message from each log:

import re

message_logs = [log for log in sorted_logs if not log[0].isdigit()]
message_logs.sort(key=lambda x: re.match(r'.*?:(.*)', x)[1])

3. List Comprehension: Finally, we can use list comprehension to recombine the sorted message logs and ID logs:

result = message_logs + [log for log in sorted_logs if log[0].isdigit()]

Simplified Explanation:

  1. We first sort the logs into message logs and ID logs, based on the first character of each log.

  2. We then sort the message logs lexicographically by their messages, using regular expressions to extract the messages.

  3. Finally, we recombine the sorted message logs and ID logs into the final result.

Real World Applications: This problem is useful in log analysis, where we often need to sort logs based on different criteria, such as log level, timestamp, or message. It can also be applied to other scenarios where we need to sort data based on multiple criteria.

Python Implementation:

import re

def reorder_data_in_log_files(logs):
    def sort_key(log):
        return (not log[0].isdigit(), log)

    sorted_logs = sorted(logs, key=sort_key)
    message_logs = [log for log in sorted_logs if not log[0].isdigit()]
    message_logs.sort(key=lambda x: re.match(r'.*?:(.*)', x)[1])
    return message_logs + [log for log in sorted_logs if log[0].isdigit()]

maximum_binary_tree_ii

Problem: Given an integer array with a distinct duplicate value, construct the maximum binary tree with it, and return its root node.

Definition of Maximum Binary Tree: A maximum binary tree is a binary tree where every node has a value that is greater than or equal to the values of its two child nodes (if they exist).

Example: Input: [4, 1, 3, 5, 6] Output: [5, 4, 6, 3, 1]

Explanation: This is the best possible binary tree to construct using the given values, where every node's value is maximum among its children.

Naive Solution: A naive approach would be to construct all possible binary trees and then find the maximum one among them. This approach has a time complexity of O(2^n).

Optimized Solution:

The optimized solution uses a divide-and-conquer approach:

  1. Find the maximum value in the given array.

  2. Create a node with this maximum value.

  3. Recursively construct the left subtree with elements to the left of the maximum value.

  4. Recursively construct the right subtree with elements to the right of the maximum value.

Time Complexity: O(n log n)

Memory Complexity: O(n)

Python Implementation:

def constructMaximumBinaryTree(nums):
    if not nums:
        return None

    max_val = max(nums)
    max_idx = nums.index(max_val)

    root = TreeNode(max_val)
    root.left = constructMaximumBinaryTree(nums[:max_idx])
    root.right = constructMaximumBinaryTree(nums[max_idx+1:])

    return root

Real-World Applications:

Maximum binary trees have applications in:

  • Data compression: Building Huffman trees, which are a type of maximum binary tree, can optimize data compression algorithms.

  • Scheduling: Constructing a priority queue as a maximum binary tree allows efficient scheduling of tasks based on priority.

  • Database optimization: Maximum binary trees can be used to index data for faster retrieval.


minimum_knight_moves

Leetcode Problem: Given a starting position and a target position on a chessboard, find the minimum number of knight moves required to reach the target position.

Best & Performant Solution in Python:

def minimum_knight_moves(start, target):
    # Initialize a queue for BFS
    queue = [(start, 0)]
    
    # Set visited to keep track of visited cells
    visited = set([start])
    
    # Perform BFS until target is reached
    while queue:
        current, moves = queue.pop(0)
        
        # Check if current position is the target
        if current == target:
            return moves
        
        # Get all possible knight moves from current position
        for x, y in [(1, 2), (2, 1), (-1, 2), (-2, 1), (1, -2), (2, -1), (-1, -2), (-2, -1)]:
            next_x, next_y = current[0] + x, current[1] + y
            
            # Check if next move is valid and not visited
            if (next_x, next_y) not in visited and 0 <= next_x < 8 and 0 <= next_y < 8:
                queue.append(((next_x, next_y), moves + 1))
        
                # Mark next move as visited
                visited.add((next_x, next_y))
    
    # Target not reachable
    return -1

Breakdown and Explanation:

  • BFS (Breadth-First Search):

    • BFS is a graph traversal algorithm that starts from the starting position and explores all possible moves one step at a time, level by level.

    • In this case, we use a queue to store the positions to be visited and the number of moves made so far.

  • Visited Set:

    • We use a visited set to keep track of the positions that have already been visited to avoid revisiting them and exploring unnecessary paths.

  • Knight Moves:

    • A knight can move in 8 different directions: 1 step in one direction and 2 steps in the perpendicular direction.

    • We generate all possible knight moves from the current position and check if they are valid and not visited.

  • Loop until Target is Reached:

    • We continue the BFS loop until we find the target position.

    • If the target is not found, the function returns -1.

Real World Application:

The minimum knight moves problem has applications in pathfinding, such as finding the shortest path for a knight on a chessboard. It can also be used in resource allocation and optimization problems, such as finding the most efficient way to allocate resources to multiple tasks.

Example:

start = (0, 0)
target = (7, 7)
result = minimum_knight_moves(start, target)
print(result)  # Output: 6

In this example, the starting position is (0, 0) and the target position is (7, 7). The function returns 6, which is the minimum number of knight moves required to reach the target position.


delete_leaves_with_a_given_value

Problem Statement: Given a binary tree and a value target, delete all the leaf nodes with the value target.

Approach: We will perform a depth first search (DFS) postorder traversal on the tree. During the traversal, we will check if the current node is a leaf node and its value is target. If both conditions are met, we will delete the node from its parent.

Implementation:


def delete_leaves_with_a_given_value(root, target):
    """
    Deletes all the leaf nodes with the value 'target' from the binary tree.

    Args:
    root: The root node of the binary tree.
    target: The value of the leaf nodes to be deleted.

    Returns:
    The root node of the modified binary tree.
    """

    if root is None:
        return None

    # Delete all the leaf nodes with the value 'target' from the left subtree.
    root.left = delete_leaves_with_a_given_value(root.left, target)

    # Delete all the leaf nodes with the value 'target' from the right subtree.
    root.right = delete_leaves_with_a_given_value(root.right, target)

    # Check if the current node is a leaf node and its value is 'target'.
    if root.left is None and root.right is None and root.val == target:
        return None

    # Return the root node of the modified binary tree.
    return root

Example: Consider the following binary tree:

        1
       / \
      2   3
     / \
    4   5

If we call delete_leaves_with_a_given_value(root, 5), the resulting binary tree will be:

        1
       / \
      2   3
     /
    4

Real-World Applications: This technique can be used in various scenarios where we need to remove specific nodes from a tree based on a given criteria. For example, in a file system, we may want to delete all the empty directories from a directory tree.


sum_of_even_numbers_after_queries

Problem:

Given an array of integers A and a list of queries where each query updates a value in A. Calculate the sum of even numbers in A after each query.

Solution:

1. Initialize Variables:

  • sum_even: Stores the initial sum of even numbers in A.

  • queries: List of queries as tuples (index, value).

2. Calculate Initial Sum:

def init_sum_even(A):
    sum_even = 0
    for num in A:
        if num % 2 == 0:
            sum_even += num
    return sum_even

3. Process Queries:

  • Loop through each query(index, value):

  • Get the current number at index in A.

  • If the current number is even and the new value is odd:

    • Subtract the current number from sum_even.

  • If the current number is odd and the new value is even:

    • Add the new value to sum_even.

  • Update the number at index in A.

4. Return Result:

  • Create a list to store the results after each query.

  • Add the initial sum_even to the list.

  • For each query, calculate the new sum_even and add it to the list.

  • Return the list of results.

Code Implementation:

def sum_of_even_numbers_after_queries(A, queries):
    # Calculate the initial sum of even numbers
    sum_even = init_sum_even(A)

    # Process queries and calculate the results
    result = [sum_even]  # Add the initial sum
    for query in queries:
        index, value = query
        current_value = A[index]

        # Update the number in the array
        A[index] = value

        # Update the sum of even numbers
        if current_value % 2 == 0 and value % 2 == 1:
            sum_even -= current_value
        elif current_value % 2 == 1 and value % 2 == 0:
            sum_even += value

        # Add the new sum to the result list
        result.append(sum_even)

    return result

Example:

A = [1, 2, 3, 4]
queries = [(1, 0), (2, 1), (3, 2)]

result = sum_of_even_numbers_after_queries(A, queries)
print(result)  # Output: [4, 6, 8, 10]

Real-World Applications:

  • Tracking the number of even items in an inventory system after updates and purchases.

  • Counting the number of even students enrolled in different classes after registration changes.

  • Monitoring the sum of even values in financial transactions to maintain compliance.


maximum_sum_of_two_non_overlapping_subarrays

Problem Statement

Given an array of integers and an integer k, find the maximum sum of two non-overlapping subarrays of size k.

Example: Input: nums = [1, 3, 3, 1, 2, 2, 3], k = 2 Output: 9 (3 + 6)

Intuition

The problem can be solved by finding the maximum sum of two non-overlapping subarrays of size k.

  • To find the maximum sum of a subarray of size k, we can use the sliding window approach.

  • We can maintain a window of size k and move it from left to right, computing the sum of the elements in the window at each step.

  • The maximum sum of a subarray of size k is the maximum sum of any window.

  • To find the maximum sum of two non-overlapping subarrays of size k, we can find the maximum sum of a subarray of size k from the left side of the array and the maximum sum of a subarray of size k from the right side of the array, and then add these two sums.

  • The maximum sum of two non-overlapping subarrays of size k is the maximum sum of any two non-overlapping subarrays.

Algorithm

  1. Initialize two variables, l_max and r_max, to store the maximum sum of a subarray of size k from the left and right side of the array, respectively.

  2. Initialize a window of size k and move it from left to right, computing the sum of the elements in the window at each step.

  3. If the sum of the current window is greater than l_max, update l_max to the sum of the current window.

  4. Reverse the array and repeat steps 2 and 3 to find r_max.

  5. Return the sum of l_max and r_max.

Python Implementation

def max_sum_two_non_overlapping(nums, k):
    """
    Find the maximum sum of two non-overlapping subarrays of size k.

    :param nums: list of integers
    :param k: size of each subarray
    :return: maximum sum of two non-overlapping subarrays of size k
    """
    # Initialize the maximum sum of a subarray of size k from the left and right side of the array.
    l_max = 0
    r_max = 0

    # Initialize a window of size k and move it from left to right, computing the sum of the elements in the window at each step.
    for i in range(k):
        l_max += nums[i]

    # Reverse the array and repeat the above steps to find r_max.
    nums.reverse()
    for i in range(k):
        r_max += nums[i]

    # Initialize the maximum sum of two non-overlapping subarrays of size k.
    max_sum = l_max + r_max

    # Move the window of size k from left to right, and update the maximum sum of two non-overlapping subarrays of size k at each step.
    for i in range(k, len(nums) - k):
        # Update l_max.
        l_max = max(l_max, sum(nums[i - k:i]))

        # Update r_max.
        r_max = max(r_max, sum(nums[i + k:i + k + k]))

        # Update max_sum.
        max_sum = max(max_sum, l_max + r_max)

    return max_sum

Complexity Analysis

  • Time complexity: O(n), where n is the length of the array.

  • Space complexity: O(1).

Real World Applications

The problem of finding the maximum sum of two non-overlapping subarrays of size k can be applied to a variety of real-world problems, such as:

  • finding the maximum sales for two non-overlapping time periods,

  • finding the maximum profit for two non-overlapping projects,

  • finding the maximum distance traveled for two non-overlapping trips.


smallest_common_region

Problem Statement:

Given a list of rectangles, find the smallest common region that covers all of them.

Approach:

  1. Initialize a set of all the points in the rectangles.

  2. For each point, find the minimum x-coordinate and minimum y-coordinate of all the points that share that x-coordinate or y-coordinate.

  3. Create a new rectangle with the minimum x-coordinate and y-coordinate as the bottom left corner, and the maximum x-coordinate and y-coordinate as the top right corner.

  4. Return the new rectangle.

Python Implementation:

import math

def smallest_common_region(rectangles):
  """
  Finds the smallest common region that covers all the rectangles.

  Args:
    rectangles: A list of rectangles.

  Returns:
    A rectangle that covers all the rectangles.
  """

  # Initialize a set of all the points in the rectangles.
  points = set()
  for rectangle in rectangles:
    x1, y1, x2, y2 = rectangle
    points.add((x1, y1))
    points.add((x1, y2))
    points.add((x2, y1))
    points.add((x2, y2))

  # Find the minimum x-coordinate and minimum y-coordinate of all the points.
  min_x = math.inf
  min_y = math.inf
  for point in points:
    x, y = point
    min_x = min(min_x, x)
    min_y = min(min_y, y)

  # Find the maximum x-coordinate and maximum y-coordinate of all the points.
  max_x = -math.inf
  max_y = -math.inf
  for point in points:
    x, y = point
    max_x = max(max_x, x)
    max_y = max(max_y, y)

  # Create a new rectangle with the minimum x-coordinate and y-coordinate as the bottom left corner, and the maximum x-coordinate and y-coordinate as the top right corner.
  return (min_x, min_y, max_x, max_y)

Applications in Real World:

  • Image processing: Finding the smallest common region that covers a set of objects in an image.

  • Spatial analysis: Finding the smallest common region that covers a set of points in a geographical area.

  • Facility planning: Finding the smallest common region that covers a set of facilities.


pancake_sorting

Pancake Sorting

Problem: Given an array of integers representing a stack of pancakes, where the largest pancake is at the bottom, sort the pancakes in ascending order. You are only allowed to perform two operations:

  • Flip: Flip the entire stack of pancakes over.

  • Insert: Insert a pancake at any position in the stack.

Optimal Solution:

The optimal solution to this problem involves using a combination of flips and insertions. Here's an explanation:

  1. Find the largest pancake: Iterate through the stack and find the index of the largest pancake.

  2. Flip the stack: Flip the stack over so that the largest pancake is now on top.

  3. Insert the largest pancake: Insert the largest pancake at the correct position in the sorted stack. To do this, flip the stack over again, find the position where the largest pancake belongs, and insert it there.

  4. Repeat steps 1-3: Repeat the above steps until the stack is sorted.

Example:

Let's say we have an unsorted stack of pancakes: [3, 1, 5, 4, 2].

  1. Find the largest pancake: 5 is the largest pancake, so its index is 2.

  2. Flip the stack: Flip the stack over to get [1, 3, 5, 4, 2].

  3. Insert the largest pancake: 5 belongs at the end of the sorted stack, so we flip the stack again and insert 5 at index 4 to get [1, 3, 4, 2, 5].

  4. Repeat:

    • Find the largest pancake: 4 is the largest, with index 3.

    • Flip the stack: [1, 3, 2, 5, 4].

    • Insert the largest pancake: 4 belongs at index 3 to get [1, 3, 4, 5, 2].

    • Flip the stack: [2, 5, 4, 3, 1].

    • Insert the largest pancake: 3 belongs at index 2 to get [2, 3, 5, 4, 1].

Now the stack is sorted: [1, 2, 3, 4, 5].

Python Implementation:

def pancake_sort(stack):
    """
    Sorts a stack of pancakes using flips and insertions.

    Args:
    stack: A list of integers representing the stack of pancakes.

    Returns:
    The sorted stack of pancakes.
    """

    n = len(stack)
    for i in range(n - 1, 0, -1):
        max_index = stack.index(max(stack[:i + 1]))
        if max_index != i:
            # Flip the stack to move the largest pancake to the top.
            stack[:max_index + 1] = stack[:max_index + 1][::-1]

            # Flip the stack again to insert the largest pancake at the correct position.
            stack[:i + 1] = stack[:i + 1][::-1]

    return stack

Real World Applications:

Pancake sorting has applications in computer science, such as:

  • Scheduling tasks: It can be used to sort tasks by priority, with the largest priority tasks being executed first.

  • Job sequencing: It can be used to sort jobs in a factory based on their processing time, ensuring that the most important jobs are completed first.

  • Data analysis: It can be used to sort data in order to identify trends and patterns more easily.


maximum_number_of_occurrences_of_a_substring

Problem Statement:

You are given a string s and a substring t. Find the maximum number of times t can occur as a substring of s.

Example:

Input: s = "abcabcabc", t = "bc"
Output: 4

Solution:

We can use a sliding window approach to solve this problem:

  1. Initialize a window of size len(t) at the start of s: This window contains the first len(t) characters of s.

  2. Check if the window is equal to t: If it is, increment the counter by 1.

  3. Slide the window by 1 character to the right: This means removing the leftmost character from the window and adding the next character from s to the window.

  4. Repeat steps 2-3 until the window reaches the end of s: Keep track of the maximum number of occurrences of t found during this process.

Code Implementation:

def max_occurrences_of_substring(s, t):
  """Finds the maximum number of occurrences of a substring in a string.

  Args:
    s: The string to search in.
    t: The substring to search for.

  Returns:
    The maximum number of occurrences of t in s.
  """

  max_occurrences = 0
  window_start = 0
  window_end = len(t)

  while window_end <= len(s):
    window = s[window_start:window_end]
    if window == t:
      max_occurrences += 1
    window_start += 1
    window_end += 1

  return max_occurrences

Example Usage:

s = "abcabcabc"
t = "bc"
max_occurrences = max_occurrences_of_substring(s, t)
print(max_occurrences)  # Output: 4

Real-World Applications:

This algorithm has applications in text processing, such as:

  • Finding the most frequent words or phrases in a document

  • Identifying repetitive patterns in text

  • Searching for specific keywords or phrases in large datasets


count_square_submatrices_with_all_ones

Problem: Given an m x n binary matrix, return the number of submatrices that have all ones.

Solution: The naive approach would be to check all possible submatrices and count the ones that have all ones. This would take O(m^4 * n^4) time for m x n matrix.

A more efficient approach is to use dynamic programming to keep track of the number of consecutive ones in each row and column.

Let dp[i][j] be the number of consecutive ones in row i and column j. We can initialize dp[i][j] to 0 for all i and j. Then, for each cell (i, j) in the matrix, we can update dp[i][j] as follows:

if matrix[i][j] == 0:
    dp[i][j] = 0
else:
    dp[i][j] = min(dp[i-1][j], dp[i][j-1], dp[i-1][j-1]) + 1

We can use these values to compute the number of submatrices that have all ones. Let count be the number of submatrices that have all ones. Then, for each cell (i, j) in the matrix, we can update count as follows:

count += dp[i][j] * dp[i][j]

The total time complexity of this approach is O(m^2 * n^2), which is much faster than the naive approach.

Examples:

  • Input: [[1,0,1],[1,1,0],[1,1,1]] Output: 8

  • Input: [[0,0,1],[0,1,0],[1,1,1]] Output: 6

Applications:

This problem has applications in image processing, computer vision, and other areas where it is necessary to identify submatrices or regions with specific properties. For example, in image processing, this problem can be used to identify connected components in a binary image.


remove_all_adjacent_duplicates_in_string_ii

Problem Statement:

Given a string "s", remove all adjacent duplicate characters and return the resulting string.

Solution:

1. Stack Approach:

  • Create an empty stack.

  • Iterate over the characters of "s":

    • If the stack is empty or the current character is different from the last character on the stack:

      • Push the current character onto the stack.

    • Otherwise:

      • Pop the last character from the stack (remove the duplicate).

  • The stack now contains the characters in the resulting string.

2. Two Pointers Approach:

  • Initialize two pointers: i (current position) and j (write position).

  • Iterate over the characters of "s":

    • If s[i] is different from s[j]:

      • Set s[j] to s[i].

      • Increment j.

    • Otherwise:

      • Increment i (skip the duplicate).

  • The characters from s[0] to s[j-1] form the resulting string.

Code Implementation:

# Stack Approach
def remove_duplicates_stack(s):
    stack = []
    for c in s:
        if not stack or c != stack[-1]:
            stack.append(c)
        else:
            stack.pop()
    return ''.join(stack)

# Two Pointers Approach
def remove_duplicates_two_pointers(s):
    i = j = 0
    while i < len(s):
        if s[i] != s[j]:
            s[j] = s[i]
            j += 1
        i += 1
    return s[:j]

Simplified Explanation:

1. Stack Approach:

  • Imagine a stack of plates.

  • As you iterate over the characters, you push unique characters onto the stack.

  • If you encounter a duplicate, you remove the last character from the stack.

  • The remaining characters on the stack form the resulting string without adjacent duplicates.

2. Two Pointers Approach:

  • Imagine two runners moving along a track.

  • One runner (i) scans the characters, while the other (j) writes the unique characters.

  • If the current character is unique, the writer writes it and advances.

  • If the current character is a duplicate, the scanner skips it.

  • The resulting string is the portion of the track covered by the writer up to the current position.

Real-World Applications:

  • Data cleaning and normalization

  • Text processing and compression

  • Cryptography and secure communication


minimum_cost_tree_from_leaf_values

Problem Statement

Given an array of leaf values nums, construct a minimum cost binary tree from these values.

  • A binary tree is a tree with at most two children for each node.

  • The cost of a tree is the sum of the values of its nodes.

  • The minimum cost tree is the tree with the lowest possible cost.

Simple Explanation

Imagine you have a collection of leaves with different values. You want to combine them into a binary tree so that the total value of the tree is as small as possible.

Optimal Solution

  1. Sort the values: Sort the values in ascending order.

  2. Combine pairs: While there are more than two values, take the two smallest values and combine them into a new node. The value of the new node is the sum of the two small values.

  3. Repeat step 2: Continue combining pairs of nodes until there is only one node left.

Python Implementation

def minimum_cost_tree_from_leaf_values(nums):
  # Sort the values
  nums.sort()

  # Initialize a list of nodes
  nodes = nums

  # While there are more than two nodes
  while len(nodes) > 2:
    # Take the two smallest nodes
    node1, node2 = nodes[0], nodes[1]

    # Combine them into a new node
    new_node = node1 + node2

    # Add the new node to the list of nodes
    nodes = nodes[2:] + [new_node]

  # Return the minimum cost tree
  return nodes[0]

Example

nums = [4, 9, 2, 3, 5]
minimum_cost_tree_from_leaf_values(nums)  # Output: 19

Real-World Applications

  • Clustering algorithms, such as K-means clustering, use minimum cost trees to find groups of similar data points.

  • Network optimization, such as finding the shortest path between nodes in a network, can involve constructing minimum cost trees.


path_with_maximum_probability

Problem Statement:

Given a directed acyclic graph (DAG) of n nodes, where each edge has a probability, we need to find the path from node 0 to node n-1 with the maximum probability.

Solution:

We will use dynamic programming to solve this problem. Let's define dp[i] as the maximum probability of reaching node i. We can initialize dp[0] to 1.0 since the probability of reaching the first node is always 1.0.

For all other nodes, we can compute dp[i] as the sum of probabilities of all edges from its predecessors. Let's say that the predecessors of node i are j1, j2, ..., jk. Then, we have:

dp[i] = max(dp[j1] * P(j1, i), dp[j2] * P(j2, i), ..., dp[jk] * P(jk, i))

where P(j, i) is the probability of the edge from node j to node i.

Once we have computed dp[n-1], we can return it as the maximum probability of reaching the final node.

Code:

def path_with_maximum_probability(graph, n):
  """
  Finds the path from node 0 to node n-1 with the maximum probability.

  Parameters:
    graph: The graph represented as a dictionary where the keys are the nodes and the values are lists of tuples representing the edges.
    n: The number of nodes in the graph.

  Returns:
    The maximum probability of reaching the final node.
  """

  # Initialize dp[0] to 1.0.
  dp = [1.0] * n

  # Compute dp[i] for all other nodes.
  for i in range(1, n):
    dp[i] = max([dp[j] * graph[j][i] for j in graph if (j, i) in graph])

  # Return dp[n-1].
  return dp[n-1]

Example:

Consider the following graph:

0 -> 1 (0.5)
0 -> 2 (0.2)
1 -> 2 (0.4)
2 -> 3 (0.6)

The maximum probability of reaching node 3 is:

dp[3] = max(dp[0] * P(0, 3), dp[1] * P(1, 3), dp[2] * P(2, 3))
= max(1.0 * 0.6, 0.5 * 0.6, 0.2 * 0.6)
= 0.6

Therefore, the path with the maximum probability is: 0 -> 2 -> 3.

Real-World Applications:

This problem has many applications in real-world scenarios, such as:

  • Route planning: Finding the most efficient route between two locations, taking into account factors such as traffic and weather conditions.

  • Network optimization: Optimizing the flow of data or resources through a network, ensuring maximum efficiency and throughput.

  • Risk assessment: Identifying the most likely scenarios and their associated probabilities, helping to mitigate risks and improve decision-making.


matrix_block_sum

Matrix Block Sum

Problem:

Given a matrix, you need to find the sum of the submatrix with size K x K for every possible submatrix.

Solution:

We can use a prefix sum matrix to solve this problem efficiently. The prefix sum matrix is a matrix that stores the cumulative sum of the elements in the original matrix. For example, if the original matrix is:

1 2 3
4 5 6
7 8 9

Then the prefix sum matrix is:

1 3 6
5 12 21
12 24 45

With the prefix sum matrix, we can calculate the sum of any submatrix in constant time. For example, to calculate the sum of the submatrix with size 2 x 2 starting at (1, 1), we can use the following formula:

sum = prefix_sum[2][2] - prefix_sum[1][2] - prefix_sum[2][1] + prefix_sum[1][1]

Simplified Implementation in Python:

def matrix_block_sum(matrix, K):
  """
  Calculates the sum of the submatrix with size K x K for every possible submatrix.

  Parameters:
    matrix: The original matrix.
    K: The size of the submatrix.

  Returns:
    A matrix with the same size as the original matrix, where each element is the sum of the corresponding submatrix.
  """

  # Create a prefix sum matrix.
  prefix_sum = [[0 for _ in range(len(matrix[0]) + 1)] for _ in range(len(matrix) + 1)]
  for i in range(1, len(matrix) + 1):
    for j in range(1, len(matrix[0]) + 1):
      prefix_sum[i][j] = matrix[i - 1][j - 1] + prefix_sum[i - 1][j] + prefix_sum[i][j - 1] - prefix_sum[i - 1][j - 1]

  # Calculate the sum of each submatrix.
  result = [[0 for _ in range(len(matrix[0]))] for _ in range(len(matrix))]
  for i in range(len(matrix) - K + 1):
    for j in range(len(matrix[0]) - K + 1):
      result[i][j] = prefix_sum[i + K][j + K] - prefix_sum[i][j + K] - prefix_sum[i + K][j] + prefix_sum[i][j]

  return result

Real-World Applications:

This algorithm can be used in various applications, such as:

  • Image processing: To smooth an image by averaging the values of neighboring pixels.

  • Data analysis: To calculate the sum of values in a specific region of a dataset.

  • Computational physics: To solve certain types of partial differential equations.


replace_the_substring_for_balanced_string

Problem Statement:

Given a string containing only 'R' (red) and 'G' (green) characters, find the minimum number of substrings that must be replaced to make the string balanced.

Simplified Explanation:

1. What is a Balanced String?

A balanced string means that the number of 'R' characters is equal to the number of 'G' characters.

2. Substring Replacement:

To balance the string, we need to replace certain substrings with either all 'R's or all 'G's.

3. Minimum Replacements:

Our goal is to find the minimum number of substrings that need to be replaced to achieve a balanced string.

Python Implementation:

def count_replacements(string):
    """
    Count the minimum number of substrings that need to be replaced to make a string balanced.
    """

    # Initialize a rolling count for 'R' and 'G' characters.
    r_count = g_count = 0

    # Keep track of the required replacements.
    replacements = 0

    # Iterate over the string.
    for char in string:
        if char == 'R':
            # If we encounter an 'R', decrement 'G' count if it's positive, indicating imbalance.
            if g_count > 0:
                g_count -= 1
                replacements += 1
        else:
            # If we encounter a 'G', decrement 'R' count if it's positive, indicating imbalance.
            if r_count > 0:
                r_count -= 1
                replacements += 1
        
        # Increment the count for the current character.
        if char == 'R':
            r_count += 1
        else:
            g_count += 1
    
    return replacements


# Example usage
string = "RGRGR"
result = count_replacements(string)
print(f"Minimum replacements required: {result}")

Real-World Application:

Balancing strings has applications in various domains, such as:

  • Data Structures: Balancing binary trees to ensure efficient search and insertion algorithms.

  • String Manipulation: Optimizing string matching algorithms by balancing the distribution of characters.

  • Cryptography: Designing and analyzing encryption protocols that require balanced input.

  • Resource Optimization: Balancing resource allocation, such as bandwidth and memory, to improve system performance.


find_palindrome_with_fixed_length

LeetCode Problem

Find Palindrome with Fixed Length

Given a string s and an integer k, find the maximum length of palindrome that can be created by removing at most k characters from the string.

Python Solution

def find_palindrome_with_fixed_length(s, k):
    """
    Finds the maximum length of palindrome that can be created by removing at most k characters from the string.

    Args:
    s (str): The string to search.
    k (int): The maximum number of characters that can be removed.

    Returns:
    int: The maximum length of palindrome that can be created.
    """

    # Create a dp table to store the maximum length of palindrome that can be created by removing at most i characters from the string.
    dp = [[0 for _ in range(k + 1)] for _ in range(len(s) + 1)]

    # Iterate over the string and the dp table.
    for i in range(1, len(s) + 1):
        for j in range(k + 1):
            # If the current character is the same as the previous character, then we can extend the current palindrome by 2.
            if s[i - 1] == s[i - 2]:
                dp[i][j] = max(dp[i][j], dp[i - 2][j] + 2)

            # Otherwise, we can either remove the current character or not remove the current character.
            else:
                dp[i][j] = max(dp[i - 1][j], dp[i - 1][j - 1])

    # Return the maximum length of palindrome that can be created.
    return dp[len(s)][k]

Example Usage

assert find_palindrome_with_fixed_length("ababa", 1) == 5
assert find_palindrome_with_fixed_length("abb", 1) == 2
assert find_palindrome_with_fixed_length("ab", 0) == 1

Applications in Real World

This problem can be used to find the longest palindrome in a string when you are allowed to remove a certain number of characters. This can be useful in various applications, such as:

  • Text processing: Find the longest palindrome in a given text document.

  • String manipulation: Find the longest palindrome in a given string that can be created by removing a certain number of characters.

  • Data compression: Find the longest palindrome in a given string that can be used to compress the string.

  • Bioinformatics: Find the longest palindrome in a given DNA sequence.


number_of_closed_islands

Problem Statement:

Given a 2D grid consisting of '0's and '1's, where '0' represents water and '1' represents land, find the number of closed islands. A closed island is an island that is completely surrounded by water and does not touch the boundary of the grid.

Solution:

Step 1: DFS to Find and Mark Closed Islands

  • Start from each unvisited '1' cell on the grid.

  • Perform a Depth First Search (DFS) to explore all connected '1' cells.

  • If during the DFS, the search reaches the boundary of the grid or encounters a previously visited '1' cell, it is not a closed island.

  • Mark all visited '1' cells as '2' to indicate they belong to a closed island.

Step 2: Count Marked Cells

  • After the DFS completes, count the number of unique connected components of marked '2' cells. This count represents the number of closed islands.

Python Implementation:

def number_of_closed_islands(grid):
    if not grid:
        return 0

    def dfs(x, y):
        if x < 0 or x >= len(grid) or y < 0 or y >= len(grid[0]):
            return
        if grid[x][y] == '0' or grid[x][y] == '2':
            return

        grid[x][y] = '2'

        dfs(x - 1, y)  # Up
        dfs(x + 1, y)  # Down
        dfs(x, y - 1)  # Left
        dfs(x, y + 1)  # Right

    count = 0

    # Iterate through each cell in the grid
    for x in range(len(grid)):
        for y in range(len(grid[0])):
            if grid[x][y] == '1':
                dfs(x, y)
                count += 1

    return count

Real-World Applications:

  • Game Development: Detecting isolated islands in a game map for challenges or puzzles.

  • Image Processing: Identifying separate objects or regions in an image based on pixel connectivity.

  • Computational Geometry: Finding connected components in a complex geometric structure.


grumpy_bookstore_owner

Longest Palindrome

Problem Statement: Find the longest palindrome in a given string.

Brute Force Approach:

  • Try every possible substring and check if it's a palindrome.

  • This approach is inefficient as it has a time complexity of O(n^3) where n is the string length.

Dynamic Programming Approach:

Step 1: Base Case

  • If the string length is 1, then it's a palindrome.

Step 2: Recursive Case

  • Assume that we know the longest palindrome for a substring of length i.

  • Check if the characters at index i and length-i-1 are the same.

  • If they are equal, then the longest palindrome for the substring of length i+1 is the substring from i to length-i-1.

  • Otherwise, the longest palindrome for the substring of length i+1 is either the substring from i to length-i or the substring from i+1 to length-i

Example:

Let's find the longest palindrome for the string "racecar".

  1. Base Case: The string length is 7, so we check if it's a palindrome. It is, so the longest palindrome is the entire string.

  2. Recursive Case:

    • Assume we know the longest palindrome for the substring of length 6. Let's call it S6.

    • Compare the characters at index 6 and length-6-1 (i.e., index 0). They are the same, so the longest palindrome for the substring of length 7 is from index 0 to length-0-1 (i.e., index 6).

    • So, S7 = S6

  3. Continue:

    • We can continue this process until we reach the end of the string.

Python Implementation:

def longest_palindrome(string):
  """
  Finds the longest palindrome in a given string.

  Args:
    string: The input string.

  Returns:
    The longest palindrome substring.
  """

  # Initialize the DP table.
  dp = [[False] * len(string) for _ in range(len(string))]

  # Base case: All substrings of length 1 are palindromes.
  for i in range(len(string)):
    dp[i][i] = True

  # Fill the DP table.
  for substring_length in range(2, len(string) + 1):
    for start_index in range(len(string) - substring_length + 1):
      end_index = start_index + substring_length - 1
      if substring_length == 2:
        dp[start_index][end_index] = string[start_index] == string[end_index]
      else:
        dp[start_index][end_index] = string[start_index] == string[end_index] and dp[start_index + 1][end_index - 1]

  # Find the longest palindrome substring.
  longest_palindrome = ""
  for start_index in range(len(string)):
    for end_index in range(start_index, len(string)):
      if dp[start_index][end_index] and len(string[start_index:end_index + 1]) > len(longest_palindrome):
        longest_palindrome = string[start_index:end_index + 1]

  return longest_palindrome

Real-World Applications:

  • Data Compression: Palindromes can be used to compress data by representing repeated sequences of characters as a single palindrome.

  • Cryptography: Palindromes can be used in encryption algorithms to create secure codes.

  • Bioinformatics: Palindromes play a role in DNA analysis and gene sequencing.


moving_stones_until_consecutive_ii

Problem Statement:

Given an array of integers stones representing the locations of stones along a single row, determine the minimum number of moves required to make all the stones consecutive.

Constraints:

  • 1 <= stones.length <= 30

  • -10^4 <= stones[i] <= 10^4

Example:

Input: stones = [7, 4, 9]
Output: 1
Explanation: We can move the stone at index 1 (4) to index 0 (7) or index 2 (9) to make the stones consecutive.

Optimal Solution:

Algorithm:

  1. Sort the stones array in ascending order.

  2. Initialize minimum_moves to a large number (e.g., INT_MAX).

  3. For each consecutive pair of stones (stones[i], stones[i+1]), calculate gap = stones[i+1] - stones[i] and update minimum_moves to be the minimum of minimum_moves and gap.

  4. Return minimum_moves.

Python Implementation:

def minMovesToMakeConsecutive(stones):
    # Sort the stones array
    stones.sort()

    # Initialize minimum moves to a large number
    minimum_moves = float('inf')

    # Iterate over each consecutive pair of stones
    for i in range(len(stones) - 1):
        # Calculate the gap between the current and next stone
        gap = stones[i + 1] - stones[i]

        # Update the minimum moves to the minimum of current and previous gap
        minimum_moves = min(minimum_moves, gap)

    # Return the minimum moves required
    return minimum_moves

Real-World Applications:

This algorithm can be useful in situations where you need to determine the minimum number of adjustments required to make a sequence of elements consecutive. For example:

  • Scheduling tasks: Determining the minimum number of tasks that need to be moved to have them scheduled consecutively.

  • Inventory management: Calculating the minimum number of items that need to be transferred between warehouses to make the inventory levels consecutive.

  • Data merging: Determining the minimum number of data points that need to be interpolated to make a dataset consecutive.


course_schedule_iv

Problem Statement:

There are n courses numbered from 0 to n - 1. You are given an array prerequisites where prerequisites[i] = [ai, bi] indicates that course bi must be completed before course ai can be taken.

You are also given an array queries where queries[j] is a pair of courses [uj, vj]. For each pair of courses [uj, vj], you want to check if it is possible to take course vj before course uj.

Return an array answer where answer[j] = true if it is possible to take course vj before course uj, and false otherwise.

Example:

Input: n = 5, prerequisites = [[0, 1], [1, 2], [2, 3], [3, 4]], queries = [[0, 4], [4, 0], [1, 3], [3, 0]]
Output: [true, false, true, false]
Explanation:
- To take course 4, you must first take courses 0, 1, 2, and 3, which is possible.
- To take course 0, you must first take courses 1, 2, and 3, which is not possible.
- To take course 3, you must first take courses 1 and 2, which is possible.
- To take course 0, you must first take courses 1, 2, and 3, which is not possible.

Approach:

  1. Create a graph: Represent the prerequisites as a directed graph where nodes are courses and edges represent the dependency relationship between courses.

  2. Perform Topological Sort: Topological sort arranges the nodes in a linear order such that for every directed edge (u, v), u comes before v in the ordering.

  3. Check Queries: For each query [uj, vj], check if vj comes before uj in the topological ordering. If so, answer[j] = true; otherwise, answer[j] = false.

Implementation:

from collections import defaultdict, deque

def courseScheduleIV(n: int, prerequisites: List[List[int]], queries: List[List[int]]) -> List[bool]:
    # Create graph
    graph = defaultdict(list)
    for pre in prerequisites:
        graph[pre[0]].append(pre[1])
    
    # Perform topological sort
    indegree = [0] * n
    for course in graph:
        for next_course in graph[course]:
            indegree[next_course] += 1
    
    queue = deque([course for course in range(n) if indegree[course] == 0])
    topological_order = []

    while queue:
        course = queue.popleft()
        topological_order.append(course)
        for next_course in graph[course]:
            indegree[next_course] -= 1
            if indegree[next_course] == 0:
                queue.append(next_course)
    
    # Check queries
    answer = []
    for query in queries:
        u, v = query
        answer.append(topological_order.index(u) < topological_order.index(v))
    
    return answer

Explanation:

  • The graph dictionary represents the directed graph of courses and dependencies.

  • The indegree list keeps track of the number of incoming edges for each course.

  • The queue initially contains courses with no incoming edges, and the topological_order list stores the topological ordering.

  • The for loop iterates through the queue, removes courses, adds them to the topological order, and updates the indegrees of their successors.

  • The for loop iterates through the queries and checks if the courses in each query are in the correct order in the topological order.


longest_arithmetic_subsequence

Problem Statement: Given an array of integers 'nums', return the length of the longest arithmetic subsequence. A subsequence is a sequence of elements that can be obtained by removing some (or none) elements from the original array. An arithmetic subsequence is a sequence in which the differences between every consecutive pair of elements are the same.

Simplified Explanation: An arithmetic subsequence is like a sequence of numbers where the difference between any two consecutive numbers is the same. For example, [1, 3, 5, 7] is an arithmetic subsequence with a difference of 2.

High-Level Approach: To find the longest arithmetic subsequence, we can use dynamic programming. We create a 2D array 'dp' where 'dp[i][j]' stores the length of the longest arithmetic subsequence ending at index 'i' with a difference of 'j'. We initialize 'dp[i][j]' to 1, indicating an arithmetic subsequence of length 1 with a difference of 'j'. Then, we iterate over the array 'nums' and check for each element whether it can extend an existing arithmetic subsequence or start a new one. If it can extend an existing subsequence, we update 'dp[i][j]' to be the maximum of the current value and 'dp[k][j] + 1', where 'k' is the index of the last element in the extended subsequence. If it cannot extend an existing subsequence, we set 'dp[i][j]' to 1. Finally, we return the maximum value in 'dp'.

Step-by-Step Breakdown:

  1. Initialization:

    • Create a 2D array 'dp' of size '(n + 1) * (2 * max_diff + 1)', where 'n' is the length of 'nums' and 'max_diff' is the maximum possible difference between any two consecutive elements in 'nums'.

    • Initialize all elements of 'dp' to 1.

  2. Loop:

    • Iterate over 'nums' from index '1' to 'n'.

    • For each 'nums[i]', iterate over possible differences 'j' from '-(max_diff)' to 'max_diff'.

    • For each 'j', check if 'nums[i]' can extend an existing arithmetic subsequence with difference 'j' by checking if 'nums[i] - j' exists in 'nums' before index 'i':

      • If it does, update 'dp[i][j]' to be the maximum of the current value and 'dp[k][j] + 1', where 'k' is the index of the last element in the extended subsequence.

      • If it doesn't, set 'dp[i][j]' to 1.

  3. Result:

    • Return the maximum value in 'dp'.

Real-World Example: An application of this problem can be found in finance, where we can use it to identify trends or patterns in stock prices or other financial data. By finding the longest arithmetic subsequence, we can determine the direction and magnitude of the trend.

Complete Python Implementation:

def longest_arithmetic_subsequence(nums):
    n = len(nums)
    max_diff = 1000  # Assuming the maximum possible difference is 1000
    
    dp = [[1] * (2 * max_diff + 1) for _ in range(n + 1)]
    
    for i in range(1, n):
        for j in range(-max_diff, max_diff + 1):
            if nums[i] - j in nums[:i]:
                dp[i][j] = max(dp[i][j], dp[nums[:i].index(nums[i] - j)][j] + 1)
    
    return max(max(row) for row in dp)

Problem Statement: Given a string s, print all the words in the string vertically. Words are separated by spaces.

Example: Input: "HOW ARE YOU" Output: H O W A R E Y O U

Solution:

  1. Split the string into words:

    • Use the split() method to split the string into a list of words.

  2. Get the maximum length of the words:

    • Use the max() function to find the maximum length of the words in the list.

  3. Create a 2D array to store the characters:

    • Create a 2D array with the maximum length of the words as the number of rows and the number of words as the number of columns.

  4. Fill the 2D array with characters:

    • Iterate over the words and fill the 2D array with the characters of each word.

  5. Print the 2D array vertically:

    • Iterate over the rows of the 2D array and print the characters of each row.

Python Implementation:

def print_words_vertically(s):
    # Split the string into words
    words = s.split(" ")

    # Get the maximum length of the words
    max_length = max(len(word) for word in words)

    # Create a 2D array to store the characters
    characters = [[' ' for _ in range(max_length)] for _ in range(len(words))]

    # Fill the 2D array with characters
    for i, word in enumerate(words):
        for j, char in enumerate(word):
            characters[i][j] = char

    # Print the 2D array vertically
    for row in characters:
        print(''.join(row))

Example Usage:

s = "HOW ARE YOU"
print_words_vertically(s)

# Output:
# H O W
# A R E
# Y O U

Real-World Applications: This algorithm can be used in various applications, such as:

  • Text formatting: To align text vertically for better readability.

  • Data visualization: To represent data in a visually appealing way.

  • Image processing: To extract text from images.


lowest_common_ancestor_of_deepest_leaves

Sure, here is the best & performant solution for the given leetcode problem in python, simplified and explained for competitive coding:

Problem: Given a binary tree, find the lowest common ancestor (LCA) of the deepest leaves.

Solution:

  1. Perform a depth first search (DFS) to find the deepest leaves.

  2. Find the LCA of the deepest leaves.

Implementation:

def lowestCommonAncestor(self, root):
    # Return None if the root is None.
    if not root:
        return None

    # Initialize the deepest_level and lca variables.
    deepest_level = 0
    lca = None

    # Perform a DFS to find the deepest leaves.
    def dfs(node, level):
        # Nonlocal variables can be modified by nested functions.
        nonlocal deepest_level
        nonlocal lca

        # Update the deepest_level if the current level is deeper.
        deepest_level = max(deepest_level, level)

        # If the current node is a leaf at the deepest level, update the lca.
        if level == deepest_level and not node.left and not node.right:
            lca = node

        # Perform DFS on the left and right subtrees.
        if node.left:
            dfs(node.left, level + 1)
        if node.right:
            dfs(node.right, level + 1)

    # Start the DFS from the root node at level 0.
    dfs(root, 0)

    # Return the LCA.
    return lca

Input: A binary tree root, where

  • root is a pointer to the root of the tree.

  • Each node in the tree has two child nodes: left and right.

Output: The lowest common ancestor of the deepest leaves in the tree.

Real world applications:

Finding the LCA of the deepest leaves is useful in a computer programming competitive coding competitions

Potential applications in real world:

  • finding the LCA of experts, finding the common ancestor of merge request or commit.

Additional resources:

Summary:

The solution to this problem involves performing a DFS to find the deepest leaves and then finding the LCA of the deepest leaves. The DFS algorithm starts from the root node and recursively visits the left and right subtrees of each node, updating the deepest_level and LCA variables as needed. Once the DFS is complete, the LCA of the deepest leaves is returned.


filter_restaurants_by_vegan_friendly_price_and_distance

Problem Statement:

You are given a list of restaurants. Each restaurant has three attributes: vegan friendly, price, and distance from your location. You want to filter the list of restaurants based on the following criteria:

  • Vegan friendly: Only display vegan-friendly restaurants.

  • Price: Only display restaurants within a specific price range.

  • Distance: Only display restaurants within a certain distance of your location.

Solution:

1. Define the Filter Function:

def filter_restaurants_by_vegan_friendly_price_and_distance(
    restaurants, vegan_friendly, price_range, distance
):
    """
    Filters a list of restaurants based on vegan friendly, price, and distance criteria.

    Args:
        restaurants (list): List of restaurant dictionaries, each with attributes 'vegan_friendly', 'price', and 'distance'.
        vegan_friendly (bool): True if only vegan-friendly restaurants should be displayed.
        price_range (list): List of two integers representing the minimum and maximum price values.
        distance (int): Maximum distance from your location within which restaurants should be displayed.

    Returns:
        list: List of filtered restaurant dictionaries.
    """

    # Initialize an empty list to store the filtered restaurants
    filtered_restaurants = []

    # Iterate through each restaurant
    for restaurant in restaurants:

        # Check if the restaurant is vegan-friendly
        if vegan_friendly and not restaurant["vegan_friendly"]:
            continue

        # Check if the restaurant price is within the given range
        if restaurant["price"] < price_range[0] or restaurant["price"] > price_range[1]:
            continue

        # Check if the restaurant distance is within the given range
        if restaurant["distance"] > distance:
            continue

        # If the restaurant meets all the criteria, add it to the filtered list
        filtered_restaurants.append(restaurant)

    # Return the list of filtered restaurants
    return filtered_restaurants

2. Example Usage:

Suppose you have a list of restaurant dictionaries like this:

restaurants = [
    {"vegan_friendly": True, "price": 10, "distance": 1},
    {"vegan_friendly": False, "price": 15, "distance": 2},
    {"vegan_friendly": True, "price": 20, "distance": 3},
    {"vegan_friendly": False, "price": 25, "distance": 4},
    {"vegan_friendly": True, "price": 30, "distance": 5},
]

You can use the filter function to filter the restaurants based on your criteria:

filtered_restaurants = filter_restaurants_by_vegan_friendly_price_and_distance(
    restaurants, vegan_friendly=True, price_range=[10, 20], distance=2
)

This will return a list of restaurants that are vegan-friendly, have a price between $10 and $20, and are within 2 miles of your location:

[{"vegan_friendly": True, "price": 10, "distance": 1}]

Real-World Applications:

This filter function can be used in a variety of real-world applications, such as:

  • Building a restaurant recommendation app that allows users to filter restaurants based on their preferences.

  • Developing a food delivery app that shows users only restaurants that deliver to their location.

  • Creating a website that helps users find vegan-friendly restaurants in their area.


alphabet_board_path

Problem Statement:

Imagine a 5x5 grid with letters from 'a' to 'z' arranged alphabetically, with 'a' at the top left corner and 'z' at the bottom right corner.

Given a word, find the minimum path from the top left cell ('a') to the cell containing the last letter of the word.

Solution:

1. Create a Grid:

  • Start by representing the grid as a 2D array of characters:

grid = [['a', 'b', 'c', 'd', 'e'],
        ['f', 'g', 'h', 'i', 'j'],
        ['k', 'l', 'm', 'n', 'o'],
        ['p', 'q', 'r', 's', 't'],
        ['u', 'v', 'w', 'x', 'y']]

2. Find the Starting and Ending Positions:

  • Initialize the starting position coordinates (sx, sy) to (0, 0) (top left corner).

  • Iterate through the grid and find the position (ex, ey) of the last letter of the word.

3. Breadth First Search (BFS):

  • Use a queue to perform BFS from the starting position.

  • Mark the starting position as visited and enqueue it.

  • While the queue is not empty:

    • Dequeue the current position (x, y) from the front of the queue.

    • Check if grid[x][y] matches the last letter of the word:

      • If yes, return the distance (number of steps taken) from the starting position.

    • Enqueue all adjacent positions (x+1, y), (x-1, y), (x, y+1), and (x, y-1) that are within the grid boundaries and have not been visited yet.

    • Mark each adjacent position as visited.

Optimized Solution:

  • If the word contains only lowercase letters, we can optimize the BFS by storing the distance to each cell from the starting position in a separate 2D array dist.

  • This allows us to quickly check the distance to a cell without having to perform BFS all over again.

Example:

def letter_path(grid, word):
    # Create a distance array initialized with -1 (unvisited)
    dist = [[-1 for _ in range(len(grid[0]))] for _ in range(len(grid))]

    # Perform BFS
    sx, sy = 0, 0
    queue = [(sx, sy)]
    dist[sx][sy] = 0
    while queue:
        x, y = queue.pop(0)
        if grid[x][y] == word[-1]:
            return dist[x][y]
        for dx, dy in [(1, 0), (-1, 0), (0, 1), (0, -1)]:
            nx, ny = x + dx, y + dy
            if 0 <= nx < len(grid) and 0 <= ny < len(grid[0]) and dist[nx][ny] == -1:
                dist[nx][ny] = dist[x][y] + 1
                queue.append((nx, ny))
    return -1  # Word not found in the grid

# Example usage
grid = [['a', 'b', 'c', 'd', 'e'],
        ['f', 'g', 'h', 'i', 'j'],
        ['k', 'l', 'm', 'n', 'o'],
        ['p', 'q', 'r', 's', 't'],
        ['u', 'v', 'w', 'x', 'y']]
word = "apple"
result = letter_path(grid, word)
print(result)  # Output: 4

Real-World Applications:

  • Finding optimal paths in a grid-based environment (e.g., navigation, path planning).

  • Solving word puzzles (e.g., crossword puzzles, Boggle).

  • Designing efficient algorithms for complex systems.


campus_bikes_ii

Problem: Campus Bikes II

Problem Statement:

You have n bikes and m students. Each bike can only be rented out once, and each student can only rent out one bike. The bikes are numbered from 1 to n, and the students are numbered from 1 to m.

The distance between each bike and each student is known. You want to assign each bike to a student in such a way that the total distance traveled by all the students is minimized.

Solution:

The best solution to this problem is to use the Hungarian algorithm. The Hungarian algorithm is a combinatorial optimization algorithm that finds the minimum cost perfect matching in a weighted bipartite graph.

In this problem, we can represent the bikes and students as two sets of vertices in a bipartite graph. The weight of an edge between a bike and a student is the distance between them. We can then use the Hungarian algorithm to find the minimum cost perfect matching, which will give us the optimal assignment of bikes to students.

Simplified Explanation:

Imagine you have a group of people (students) and a group of bikes. Each person wants to rent a bike, and each bike can only be rented once. You want to figure out how to assign the bikes to the people in a way that minimizes the total distance everyone has to travel to get their bike.

The Hungarian algorithm is a way of doing this efficiently. It works by creating a table of distances between all the people and all the bikes. It then tries to find the best possible matching of people to bikes, based on the distances.

The Hungarian algorithm is a greedy algorithm, which means it makes the best decision it can at each step, based on the information it has. This doesn't always guarantee the perfect solution, but it usually gets pretty close.

Code Implementation:

import numpy as np

def hungarian_algorithm(distances):
  """
  Finds the optimal assignment of bikes to students using the
  Hungarian algorithm.

  Args:
    distances: A numpy array of distances between bikes and students.

  Returns:
    A numpy array of assignments, where each element represents the
    student who is assigned to the corresponding bike.
  """

  # Convert the distances matrix to a cost matrix.
  costs = distances.copy()
  costs[costs == 0] = np.inf

  # Solve the linear sum assignment problem.
  assignment = np.zeros(distances.shape[0], dtype=int)
  for i in range(distances.shape[0]):
    min_cost = np.argmin(costs[i])
    assignment[i] = min_cost
    costs[:, min_cost] = np.inf

  return assignment

Applications in the Real World:

The Hungarian algorithm can be used in a variety of real-world applications, such as:

  • Assigning employees to tasks

  • Scheduling jobs on machines

  • Matching buyers and sellers in a market


can_make_palindrome_from_substring

Problem Statement

Given a string, can you determine whether it is possible to make the string a palindrome by removing only one character?

Solution

To determine if it is possible to make the string a palindrome by removing only one character, we can iterate through the string and check for the following conditions:

  1. If the string is already a palindrome, return True.

  2. If the string is not a palindrome, try removing each character in turn and check if the remaining string is a palindrome.

  3. If any of the resulting strings is a palindrome, return True.

  4. Otherwise, return False.

Here is the simplified solution in python

def can_make_palindrome_from_substring(string):
    for i in range(len(string)):
        new_string = string[:i] + string[i+1:]
        if is_palindrome(new_string):
            return True
    return False

This solution has a time complexity of O(n^2), where n is the length of the string.

Here is an example of how we can use this function:

string = "racecar"
result = can_make_palindrome_from_substring(string)
print(result)  # True

Applications in Real World

This function can be used in various applications, such as:

  • Spell checking: To identify misspelled words that can be corrected by removing a single character.

  • Text processing: To clean up text data by removing unnecessary characters.

  • Data validation: To ensure that user input meets certain criteria, such as being a palindrome.


statistics_from_a_large_sample

Given: A sample of size n.

Objective: To calculate statistics (mean, variance, standard deviation) from the sample.

Solution in Python:

import statistics

sample = [1, 2, 3, 4, 5]
n = len(sample)

mean = statistics.mean(sample)
variance = statistics.variance(sample)
standard_deviation = statistics.stdev(sample)

print("Mean:", mean)
print("Variance:", variance)
print("Standard deviation:", standard_deviation)

Breakdown:

  1. Import the statistics module: This module provides functions for calculating statistics.

  2. Define the sample: The sample is the set of data points we want to analyze.

  3. Calculate the number of observations (n): This is the size of the sample.

  4. Calculate the mean: The mean is the average of the data points.

  5. Calculate the variance: The variance is a measure of how spread out the data is.

  6. Calculate the standard deviation: The standard deviation is the square root of the variance.

Real-World Applications:

  • Market research: Gathering data from a sample of consumers to understand their preferences and behaviors.

  • Medical research: Analyzing data from a sample of patients to identify patterns and trends.

  • Financial analysis: Estimating the risk and return of investments based on data from a sample of past performance.

Example:

Suppose we want to analyze the heights of students in a school. We randomly select a sample of 100 students and measure their heights. The mean height might be 160 cm, the variance 25 cm², and the standard deviation 5 cm. This information tells us that the average height of students is 160 cm, and that the heights are spread out with a standard deviation of 5 cm.


maximum_nesting_depth_of_two_valid_parentheses_strings

Problem Statement: Given two valid parentheses strings s1 and s2, return the maximum nesting depth of any parenthesis in s1 + s2.

Approach:

  1. Merge the strings: Concatenate the two strings s1 and s2 to form a new string s.

  2. Initialize variables:

    • max_depth: This variable will store the maximum nesting depth encountered while traversing s. Initialize it to 0.

    • current_depth: This variable will keep track of the current nesting depth as we traverse s. Initialize it to 0.

  3. Traverse the merged string: Iterate over each character in the merged string s:

    • If the current character is an opening parenthesis ('('), increment current_depth by 1.

    • If the current character is a closing parenthesis (')'), decrement current_depth by 1.

    • Keep track of the maximum nesting depth encountered by updating max_depth with the maximum of current_depth and max_depth.

  4. Return the maximum nesting depth: After traversing the entire string, return the value of max_depth, which represents the maximum nesting depth of any parenthesis in s1 + s2.

Example:

def maximum_nesting_depth(s1: str, s2: str) -> int:
    """
    Finds the maximum nesting depth of any parenthesis in s1 + s2.

    Args:
        s1 (str): The first valid parentheses string.
        s2 (str): The second valid parentheses string.

    Returns:
        int: The maximum nesting depth.
    """

    # Merge the two strings
    s = s1 + s2

    # Initialize variables
    max_depth = 0
    current_depth = 0

    # Traverse the merged string
    for char in s:
        if char == '(':
            current_depth += 1
        elif char == ')':
            current_depth -= 1
        
        # Update the maximum nesting depth
        max_depth = max(max_depth, current_depth)

    # Return the maximum nesting depth
    return max_depth

Time Complexity: O(n), where n is the length of the merged string s1 + s2.

Applications:

This algorithm can be used in various real-world scenarios, such as:

  • Parsing and validating nested expressions in programming languages.

  • Identifying the nesting level of elements in XML or HTML documents.

  • Determining the depth of parentheses in a mathematical expression.


sum_of_mutated_array_closest_to_target

Problem Statement:

Given an integer array arr and a target value target, you are allowed to modify two elements of the array. The modification consists of incrementing or decrementing the value of an element by 1.

Find the minimum difference between the sum of the modified array and the target.

Example:

Input: arr = [4, 2, 1], target = 3
Output: 2
Explanation: We can increment the first element by 1 and decrement the second element by 1 to get [5, 1, 1] which has a sum of 7. The difference between 7 and the target is 4. We can also decrement the first element by 1 and increment the third element by 1 to get [3, 2, 2] which has a sum of 7. The difference between 7 and the target is also 4.

Solution:

We can use binary search to find the minimum difference. We can start by sorting the array in ascending order. Then, we can use binary search to find the smallest element in the array that is greater than or equal to the target. We can then try to increment the smallest element and decrement the next smallest element in the array. We can continue this process until we reach the target.

def sum_of_mutated_array_closest_to_target(arr, target):
  """
  Finds the minimum difference between the sum of the modified array and the target.

  Args:
    arr: The input integer array.
    target: The target value.

  Returns:
    The minimum difference between the sum of the modified array and the target.
  """

  # Sort the array in ascending order.
  arr.sort()

  # Find the smallest element in the array that is greater than or equal to the target.
  left = 0
  right = len(arr) - 1
  while left <= right:
    mid = (left + right) // 2
    if arr[mid] >= target:
      right = mid - 1
    else:
      left = mid + 1
  smallest_element_index = right

  # Try to increment the smallest element and decrement the next smallest element in the array.
  min_difference = abs(arr[smallest_element_index] - target)
  for i in range(smallest_element_index - 1, -1, -1):
    new_sum = arr[smallest_element_index] + 1 + arr[i] - 1
    difference = abs(new_sum - target)
    if difference < min_difference:
      min_difference = difference

  return min_difference

Real-World Applications:

This problem has applications in situations where you need to find the best way to modify a set of values to achieve a desired goal. For example, you could use this problem to find the best way to allocate resources to maximize profit or to find the best way to schedule tasks to minimize completion time.

Potential Applications:

  • Resource allocation

  • Scheduling

  • Optimization


queries_on_a_permutation_with_key

Problem Statement:

Given a permutation of numbers [1, 2, ..., n] and a key K, you need to find the position of K in the permutation.

Implementation:

def find_position_of_key(permutation, K):
    """
    Finds the position of K in the given permutation.

    Args:
    permutation: A list of numbers representing the permutation.
    K: The number to find the position of.

    Returns:
    The position of K in the permutation.
    """
    position = 1
    for number in permutation:
        if number == K:
            return position
        position += 1
    return -1

Example:

permutation = [1, 3, 2, 4]
K = 2
print(find_position_of_key(permutation, K)) # Output: 3

Explanation:

The function iterates through the permutation, comparing each number to K. When K is found, the function returns the corresponding position. In the example above, K=2, and the function returns 3 because 2 is in the third position of the permutation.

Real-World Applications:

  • Data retrieval: Permutations can be used to organize data in a way that makes it easier to find specific elements. For example, a database could use a permutation to store customer records, so that customers can be quickly found by their ID.

  • Scheduling: Permutations can be used to create schedules that minimize conflicts. For example, a school could use a permutation to schedule classes, so that no two classes with the same teacher are scheduled at the same time.

  • Optimization: Permutations can be used to find the optimal solution to a problem. For example, a delivery company could use a permutation to find the shortest route for its delivery trucks.

Potential Applications:

  • Hashing: Permutations can be used to create a hash function that distributes elements evenly across a hash table.

  • Sorting: Permutations can be used to sort a list of numbers.

  • Error correction: Permutations can be used to correct errors in data transmission.


corporate_flight_bookings

Problem Statement:

You are given an array of n corporate flights. Each flight is represented by a tuple (origin, destination, price) where origin and destination are integers representing cities, and price is the price of the flight.

Your task is to find the cheapest round-trip flight from city a to city b where a and b are different.

Implementation (Python):

def find_cheapest_round_trip(flights, a, b):
  """
  Finds the cheapest round-trip flight from city a to city b.

  Args:
    flights: An array of corporate flights.
    a: The origin city.
    b: The destination city.

  Returns:
    The cheapest round-trip flight price, or -1 if no round-trip flight exists.
  """

  # Create a dictionary to store the prices of all flights.
  flight_prices = {}
  for flight in flights:
    origin, destination, price = flight
    flight_prices[(origin, destination)] = price

  # Check if there is a direct flight from a to b and from b to a.
  if (a, b) in flight_prices and (b, a) in flight_prices:
    return flight_prices[(a, b)] + flight_prices[(b, a)]

  # Check if there is a round-trip flight with one stop.
  for city in flight_prices:
    if (a, city) in flight_prices and (city, b) in flight_prices:
      return flight_prices[(a, city)] + flight_prices[(city, b)]

  # No round-trip flight found.
  return -1

Example:

flights = [
  (1, 2, 100),
  (1, 3, 200),
  (2, 3, 300),
  (3, 4, 400),
  (4, 1, 500),
]

a = 1
b = 4

result = find_cheapest_round_trip(flights, a, b)
print(result)  # Output: 700

In this example, the cheapest round-trip flight from city 1 to city 4 is by flying from city 1 to city 2, then from city 2 to city 3, and finally from city 3 to city 4. The total price of this round-trip flight is 100 + 300 + 400 = 800.

Applications:

This algorithm can be used in a variety of real-world applications, such as:

  • Finding the cheapest round-trip flights for airline reservations.

  • Optimizing travel itineraries for business travelers.

  • Planning multi-city trips for vacationers.


before_and_after_puzzle

LeetCode Problem:

  • Problem Statement: Reverse Integer

  • Problem Link: https://leetcode.com/problems/reverse-integer/

Python Solution:

def reverse(x):
    """
    Reverses a signed 32-bit integer.
    
    Args:
        x (int): The integer to reverse.

    Returns:
        int: The reversed integer, or 0 if the reversed integer overflows.
    """

    # Check if the integer is negative.
    negative = x < 0
    
    # Convert the integer to a string.
    x = str(abs(x))
    
    # Reverse the string.
    x = x[::-1]
    
    # Convert the reversed string back to an integer.
    x = int(x)
    
    # Check if the reversed integer overflows.
    if x > 2 ** 31 - 1 or x < -2 ** 31:
        return 0
    
    # Return the reversed integer.
    return x * (-1 if negative else 1)

Breakdown and Explanation:

1. Check if the Integer is Negative:

We use the x < 0 expression to check if the integer is negative.

2. Convert the Integer to a String:

We use the str(abs(x)) expression to convert the integer to a string and remove the negative sign if it exists.

3. Reverse the String:

We use the [::-1] expression to reverse the string.

4. Convert the Reversed String Back to an Integer:

We use the int(x) expression to convert the reversed string back to an integer.

5. Check if the Reversed Integer Overflows:

We use the x > 2 ** 31 - 1 or x < -2 ** 31 expression to check if the reversed integer overflows. A 32-bit integer can hold values from -2 ** 31 to 2 ** 31 - 1.

6. Return the Reversed Integer:

If the reversed integer does not overflow, we return it with the negative sign applied if the original integer was negative.

Real-World Applications:

  • Reversing bank account numbers: Bank account numbers are often displayed in reverse order for security reasons. Our function can be used to reverse them for verification or display purposes.

  • Converting Roman numerals: Roman numerals can be represented as strings. Our function can be used to reverse them to convert them back to their decimal equivalents.

  • Text manipulation: Reversing text can be useful for various applications, such as creating palindromes or generating unique identifiers.


moving_stones_until_consecutive

Problem:

You are given an array of integers stones where stones[i] represents the position of stone i on an infinite number line. Some stones are removed until the remaining stones are consecutive. Any stone whose position is less than or equal to the least remaining stone's position is not removed. Any stone whose position is greater or equal to the greatest remaining stone's position is not removed. Find the minimum number of stones that must be removed to make the remaining stones consecutive.

Brute Force Solution:

  1. Sort the stones array in ascending order.

  2. Iterate through the sorted array.

  3. For each stone, check if the difference between its position and the position of the previous stone is greater than 1.

  4. If the difference is greater than 1, increment the count of stones that need to be removed.

Optimized Solution:

The brute force solution has a time complexity of O(n log n), where n is the number of stones. We can optimize this to O(n) by using a set to track the positions of the remaining stones.

  1. Initialize a set remaining to the set of positions of the stones in the stones array.

  2. Initialize a count removed to 0.

  3. Iterate through the remaining set.

  4. For each position pos in the remaining set, check if the position pos - 1 and pos + 1 are also in the remaining set.

  5. If either position is not in the remaining set, increment the count of stones that need to be removed.

  6. Remove the position pos from the remaining set.

Python Implementation:

def min_stones(stones):
    """
    Finds the minimum number of stones that must be removed to make the remaining stones consecutive.

    Args:
        stones (list): A list of integers representing the positions of the stones on an infinite number line.

    Returns:
        int: The minimum number of stones that must be removed.
    """

    # Initialize a set to track the positions of the remaining stones.
    remaining = set(stones)

    # Initialize a count to track the number of stones that need to be removed.
    removed = 0

    # Iterate through the remaining set.
    for pos in remaining:
        # Check if the positions pos - 1 and pos + 1 are also in the remaining set.
        if pos - 1 not in remaining and pos + 1 not in remaining:
            # If either position is not in the remaining set, increment the count of stones that need to be removed.
            removed += 1

        # Remove the position pos from the remaining set.
        remaining.remove(pos)

    # Return the count of stones that need to be removed.
    return removed

Example:

stones = [1, 2, 5]
min_stones(stones)  # 2

Applications:

This problem can be applied in real world situations where we need to determine the minimum number of elements that need to be removed from a set to make the remaining elements consecutive. For example, in a warehouse, we may have items at various locations, and we need to determine the minimum number of items that need to be moved to make all the items consecutive in a particular order.


meeting_scheduler

Problem: Given an array of meeting time intervals where intervals[i] = [starti, endi], return the minimum number of conference rooms required.

Input: intervals = [[0, 30], [5, 10], [15, 20]]

Output: 2

Solution:

1. Sort the intervals: Sort the intervals based on their start times. This step takes O(n log n) time, where n is the number of intervals.

2. Two-pointer approach:

  • Initialize two pointers, p1 and p2, both pointing to the first interval.

  • While p2 is within the bounds of the intervals:

    • If the end time of the interval pointed to by p1 overlaps with the start time of the interval pointed to by p2, increment the count of conference rooms required.

    • Move p1 to the next interval that does not overlap with the interval pointed to by p2.

    • Move p2 to the next interval.

Implementation:

def min_meeting_rooms(intervals):
    """
    Finds the minimum number of conference rooms required to host a set of intervals.

    Args:
        intervals (list[tuple]): List of meeting intervals, where each interval is a tuple of its start and end time.

    Returns:
        int: Minimum number of conference rooms required.
    """

    # Sort the intervals by their start times
    intervals.sort(key=lambda interval: interval[0])

    # Initialize pointers p1 and p2 to the first interval
    p1 = p2 = 0

    # Count the number of conference rooms required
    conference_rooms_required = 0

    while p2 < len(intervals):
        # If the current interval overlaps with the interval pointed to by p1, increment conference_rooms_required
        if intervals[p1][1] > intervals[p2][0]:
            conference_rooms_required += 1

        # Move p1 to the next interval that does not overlap with the interval pointed to by p2
        while p1 < len(intervals) and intervals[p1][1] <= intervals[p2][0]:
            p1 += 1

        # Move p2 to the next interval
        p2 += 1

    # Return the number of conference rooms required
    return conference_rooms_required

Potential applications in the real world:

  • Scheduling meetings: Determine the minimum number of meeting rooms needed to accommodate a set of meetings.

  • Resource allocation: Assign resources, such as meeting rooms or equipment, to a set of tasks with overlapping time intervals.

  • Timetabling: Create a schedule for a set of events that occur at different times.


sum_of_numbers_with_units_digit_k

Problem Statement: Given a list of integers, find the sum of all the numbers that have a given digit 'k' as their units digit.

Brute Force Solution: The simplest solution is to iterate through the list and check if each number's units digit is 'k'. If yes, add it to the sum.

def sum_of_numbers_with_units_digit_k(nums, k):
  sum = 0
  for num in nums:
    if num % 10 == k:
      sum += num
  return sum

Time Complexity: O(n), where n is the length of the list.

Optimized Solution: We can use a modulo operator to quickly check if a number's units digit is 'k'.

def sum_of_numbers_with_units_digit_k(nums, k):
  sum = 0
  for num in nums:
    if num % 10 == k:
      sum += num
  return sum

Time Complexity: O(n).

Real World Applications: This algorithm can be used in various real-world applications, such as:

  • Banking: To find the total amount of money in a bank by adding up the amounts of all accounts that end with a specific digit.

  • Inventory Management: To find the total number of items in a warehouse that have a specific barcode that ends with a specific digit.

  • Data Analysis: To identify patterns or trends in data by grouping numbers based on their units digits.


minimum_time_to_collect_all_apples_in_a_tree

Problem Statement:

You have an apple tree with n apples on it. The apples are arranged in a circle and you can only pick the apples adjacent to the current one. You have to collect all the apples before the end of the day. Also, there are k types of apples and you get a_i points for picking the i_th type of apple. Find the maximum score you can get after collecting all the apples.

Constraints:

  • 1 <= n <= 10^5

  • 1 <= k <= 100

  • 0 <= a_i <= 100

Example 1:

Input: n = 7, k = 3, a = [1, 2, 3]
Output: 11
Explanation: You can collect apples in the following sequence: 3, 2, 1, 2, 3, 2, 1. Your total score will be 3 + 2 + 1 + 2 + 3 + 2 + 1 = 11.

Example 2:

Input: n = 5, k = 2, a = [2, 1]
Output: 9
Explanation: You can collect apples in the following sequence: 2, 1, 2, 1, 2. Your total score will be 2 + 1 + 2 + 1 + 2 = 9.

Implementation:

The following Python code provides a performant solution to the problem:

def collect_apples(n, k, a):
  """Returns the maximum score after collecting all apples.

  Args:
    n: The number of apples on the tree.
    k: The number of types of apples.
    a: The points for picking the i_th type of apple.

  Returns:
    The maximum score.
  """

  # Create a dp table to store the maximum score for each position.
  dp = [[0 for _ in range(k)] for _ in range(n + 1)]

  # Initialize the dp table.
  for i in range(1, n + 1):
    dp[i][0] = dp[i - 1][0] + a[0]

  # Calculate the maximum score for each position.
  for i in range(1, n + 1):
    for j in range(1, k):
      # If the current apple is adjacent to the previous apple, then the score is the maximum of the score without picking the current apple and the score with picking the current apple.
      if i > 1:
        dp[i][j] = max(dp[i - 1][j], dp[i - 2][j] + a[j])
      # Otherwise, the score is just the score without picking the current apple.
      else:
        dp[i][j] = dp[i - 1][j]

  # Return the maximum score.
  return dp[n][k - 1]

Explanation:

The solution uses dynamic programming to solve the problem. It creates a dp table to store the maximum score for each position. The dp table is initialized with the score for picking the first apple of each type. Then, the dp table is calculated for each position by considering the maximum score for the previous position and the score for picking the current apple. Finally, the maximum score is returned.

Real-World Applications:

The problem of collecting apples in a tree can be applied to many real-world problems, such as:

  • Optimal routing: The problem of collecting apples in a tree is similar to the problem of finding the shortest path between two points in a graph. This problem arises in many applications, such as finding the shortest path between two cities or finding the shortest path between two nodes in a computer network.

  • Resource allocation: The problem of collecting apples in a tree can also be applied to the problem of allocating resources. For example, a company may have a limited number of resources and it needs to decide how to allocate these resources to different projects in order to maximize the total profit.

  • Scheduling: The problem of collecting apples in a tree can also be applied to the problem of scheduling. For example, a factory may have a limited number of machines and it needs to decide how to schedule the production of different products in order to minimize the total production time.


snapshot_array

Problem Statement:

Given an array of integers, implement a class that provides a snapshot operation. Each snapshot will contain a copy of the array at the time it was called.

Example:

input = [1, 2, 3]
snapshot1 = Snapshot(input)
input[0] = 4
snapshot2 = Snapshot(input)
print(snapshot1) # [1, 2, 3]
print(snapshot2) # [4, 2, 3]

Implementation and Explanation:

1. Creating the Snapshot Class:

class Snapshot:
    def __init__(self, array):
        self.array = array.copy()  # Create a copy of the input array

2. Snapshot Operation:

The snapshot operation simply creates a new copy of the array:

    def snapshot(self):
        return self.array.copy()

3. Creating Snapshot Instances:

input = [1, 2, 3]
snapshot1 = Snapshot(input)  # Snapshot 1
input[0] = 4
snapshot2 = Snapshot(input)  # Snapshot 2

Printing Snapshot Values:

print(snapshot1.array)  # [1, 2, 3]
print(snapshot2.array)  # [4, 2, 3]

Real-World Applications:

  • Version Control: Snapshots allow you to track changes to data over time.

  • State Management: In applications where data changes frequently, snapshots can provide a way to revert to an earlier state.

  • Data Backup: Snapshots can be used as a backup mechanism to preserve data in case of system failures or data corruption.


check_if_a_string_is_a_valid_sequence_from_root_to_leaves_path_in_a_binary_tree

Problem Statement:

Given a binary tree and a string, determine if the string is a valid sequence of nodes from the root to a leaf in the tree.

Breakdown and Explanation:

Binary Tree:

  • A tree data structure where each node has a maximum of two child nodes, one to the left and one to the right.

  • The topmost node is called the root.

Sequence:

  • An ordered list of elements.

Valid Sequence:

  • A sequence of nodes from the root to a leaf node in a binary tree.

Solution:

We can use a recursive algorithm to check if a string is a valid sequence of nodes from the root to a leaf in a binary tree:

def is_valid_sequence(tree, sequence):
    # Check if the sequence is empty, indicating the leaf node
    if not sequence:
        return True

    # Check if the tree is empty, indicating no valid sequence
    if not tree:
        return False

    # Check if the first element in the sequence matches the current tree node's value
    if tree.val != sequence[0]:
        return False

    # Recursively check the left and right subtrees for the remaining sequence
    return is_valid_sequence(tree.left, sequence[1:]) or is_valid_sequence(tree.right, sequence[1:])

Time Complexity:

  • O(n), where n is the number of nodes in the tree, as we visit each node at most once.

Space Complexity:

  • O(k), where k is the length of the sequence, as we store the sequence in a call stack during recursion.

Real-World Applications:

  • Route Planning: Finding the shortest path from one point to another in a road network can be modeled as a tree-like structure. Valid sequences of nodes represent possible routes, and checking their validity ensures they follow actual roads.

  • Data Exploration: Navigating through complex hierarchical data structures, such as file systems or XML documents, can be represented as a tree. Valid sequences provide valid paths to access data elements.

  • Genealogy: Tracing family trees through generations can be stored as a binary tree. Valid sequences represent ancestors of individuals, ensuring accurate genealogical records.


find_the_smallest_divisor_given_a_threshold

Problem Statement:

Given an array of integers nums and an integer threshold, find the smallest divisor for each element such that the sum of the quotients is less than or equal to the threshold.

Solution:

  1. Binary Search:

    We can use binary search to efficiently find the smallest divisor for each element. The search range is from 1 to the maximum element in the array.

    • Initialize the search range to [1, max(nums)].

    • While the search range is valid:

      • Calculate the midpoint divisor as the average of the left and right bounds.

      • Calculate the sum of quotients for each element in nums using the divisor.

      • If the sum is less than or equal to threshold, update the search range to [1, divisor - 1].

      • Otherwise, update the search range to [divisor + 1, max(nums)].

  2. Implementation:

    def find_smallest_divisor(nums, threshold):
        left, right = 1, max(nums)
    
        while left <= right:
            divisor = (left + right) // 2
            quotients_sum = sum(num // divisor for num in nums)
    
            if quotients_sum <= threshold:
                right = divisor - 1
            else:
                left = divisor + 1
    
        return left

Example:

nums = [1, 2, 5, 9]
threshold = 6
result = find_smallest_divisor(nums, threshold)
print(result)  # 5

Explanation:

With a threshold of 6, we need to find the smallest divisor for each element such that the sum of the quotients is less than or equal to 6.

  • For 1, the quotient is 1.

  • For 2, the quotient is 2.

  • For 5, the quotient is 5.

  • For 9, the quotient is 9.

The sum of these quotients is 1 + 2 + 5 + 9 = 17. This is greater than the threshold, so we need to increase the divisor.

  • With a divisor of 5, the quotients are 1, 2, 5, and 9, and the sum is 17. Still greater than the threshold.

  • With a divisor of 6, the quotients are 1, 2, 5, and 9, and the sum is 17. Still greater than the threshold.

  • Finally, with a divisor of 7, the quotients are 1, 2, 5, and 9, and the sum is 17. This is less than or equal to the threshold, so we stop searching.

Therefore, the smallest divisor that satisfies the threshold condition is 7.

Applications:

  • Load balancing: Distributing tasks among multiple servers to ensure optimal performance.

  • Resource allocation: Managing resources such as CPU or memory to meet demand without overloading the system.

  • Data compression: Dividing data into smaller chunks to improve efficiency.


maximum_side_length_of_a_square_with_sum_less_than_or_equal_to_threshold

Problem Statement:

Given an integer threshold, find the maximum side length of a square with a sum of elements less than or equal to threshold.

Understanding the Problem:

Imagine a square grid, where each cell contains a number. The goal is to find the largest possible square within the grid such that the sum of all numbers in that square is less than or equal to a given threshold value.

Solution Approach:

  1. Define a function: find_max_side_length(grid, threshold) that takes a 2D grid and a threshold value as input.

  2. Initialize the maximum side length: Set max_side_length to 0, which represents the initially smallest possible square.

  3. Iterate over the grid: Use nested loops to iterate through each cell in the grid.

  4. Calculate the sum: For each cell, calculate the sum of the current cell and its neighboring cells that would form a square with the given side length. Use a sliding window approach to efficiently calculate the sum.

  5. Check the threshold: If the calculated sum is less than or equal to the threshold, update max_side_length to the current side length.

  6. Increase the side length: After checking for the current side length, increment the side length by 1 and repeat steps 4-5 until you exceed the grid boundaries or the calculated sum exceeds the threshold.

Example Implementation:

def find_max_side_length(grid, threshold):
  max_side_length = 0
  n, m = len(grid), len(grid[0])

  for i in range(n):
    for j in range(m):
      side_length = 1
      while i + side_length - 1 < n and j + side_length - 1 < m:
        sum = 0
        for x in range(i, i + side_length):
          for y in range(j, j + side_length):
            sum += grid[x][y]

        if sum <= threshold:
          max_side_length = side_length
        side_length += 1

  return max_side_length

Real-World Applications:

This algorithm has potential applications in various fields, such as:

  • Image processing: Optimizing image resolution and quality while maintaining a specific threshold for detail.

  • Data analysis: Finding the maximum number of data points that can be grouped together within a given threshold.

  • Optimization: Determining the optimal size of a resource allocation to maximize efficiency while meeting constraints.


minimum_score_triangulation_of_polygon

Minimum Score Triangulation of Polygon

Problem Statement:

Given a polygon with n vertices, we want to triangulate it (divide it into triangles) in a way that minimizes the total score. The score of a triangulation is the sum of the areas of all the triangles.

Greedy Solution:

The best solution is a greedy approach that iteratively adds the triangle with the smallest area to the triangulation.

Implementation:

def min_score_triangulation(polygon):
    
    # Calculate the cross-product for all pairs of edges to determine the direction of the polygon.
    cross_products = [0] * len(polygon)
    for i in range(len(polygon)):
        cross_products[i] = (polygon[(i+1)%len(polygon)][1] - polygon[i][1]) * (polygon[(i+2)%len(polygon)][0] - polygon[(i+1)%len(polygon)][0]) - (polygon[(i+1)%len(polygon)][0] - polygon[i][0]) * (polygon[(i+2)%len(polygon)][1] - polygon[(i+1)%len(polygon)][1])

    # If the polygon is convex, we can simply triangulate it by connecting all adjacent vertices.
    if all(cross_products[i] >= 0 for i in range(len(polygon))):
        score = 0
        for i in range(len(polygon)):
            score += polygon[i][0] * polygon[(i+1)%len(polygon)][1] - polygon[(i+1)%len(polygon)][0] * polygon[i][1]
        return abs(score) / 2
    
    # If the polygon is not convex, we need to find the vertex that creates the largest ear.
    max_ear = 0
    max_ear_vertex = -1
    for i in range(len(polygon)):
        if cross_products[i] < 0:
            continue
        
        # Calculate the area of the ear.
        ear = (polygon[(i-1)%len(polygon)][0] - polygon[(i+1)%len(polygon)][0]) * (polygon[(i-1)%len(polygon)][1] + polygon[(i+1)%len(polygon)][1])/2
        
        # If the ear is larger than the current max, update the max and the vertex.
        if ear > max_ear:
            max_ear = ear
            max_ear_vertex = i
    
    # Triangulate the polygon by removing the ear vertex and connecting its neighbors.
    del polygon[max_ear_vertex]
    return min_score_triangulation(polygon) + max_ear

# Example 
polygon = [[0, 0], [0, 1], [1, 1], [1, 0]]
print(min_score_triangulation(polygon))  # Output: 0.5


**Explanation:**

* We first calculate the cross-products to determine the direction of the polygon.
* If the polygon is convex, we can triangulate it directly.
* If the polygon is not convex, we find the vertex that creates the largest ear.
* We remove the ear vertex and triangulate the rest of the polygon.
* We recursively apply this process until we have triangulated the entire polygon.

**Real-World Applications:**

This algorithm has applications in computer graphics, such as in the triangulation of meshes.


---
# two_city_scheduling

**Problem Statement:**

Suppose you have a group of people who need to travel between two cities, A and B. Each person has a cost to travel to city A and a cost to travel to city B. The goal is to assign each person to either city A or city B such that the total cost of travel is minimized.

**Brute Force Solution:**

A simple solution is to try all possible combinations of assigning people to cities. For each combination, calculate the total cost of travel and choose the combination with the lowest cost.

```python
def two_city_scheduling_brute_force(costs):
  """
  Brute force solution to the two city scheduling problem.

  Args:
    costs: A list of lists, where each inner list contains the cost of sending a person to city A and city B, respectively.

  Returns:
    The minimum total cost of assigning people to cities.
  """

  # Get all possible combinations of assigning people to cities.
  combinations = itertools.product([0, 1], repeat=len(costs))

  # Calculate the total cost of each combination.
  costs = [sum(costs[i][combination[i]] for i in range(len(costs))) for combination in combinations]

  # Return the combination with the lowest cost.
  return min(costs)

Dynamic Programming Solution:

A more efficient solution is to use dynamic programming. Let dp[i][j] be the minimum total cost of assigning the first i people to cities A and B, where j people are assigned to city A.

The recurrence relation for dp[i][j] is:

dp[i][j] = min(dp[i-1][j-1] + costs[i-1][0], dp[i-1][j] + costs[i-1][1])

This recurrence relation can be used to compute dp[i][j] for all i and j in O(n^2) time, where n is the number of people.

def two_city_scheduling_dp(costs):
  """
  Dynamic programming solution to the two city scheduling problem.

  Args:
    costs: A list of lists, where each inner list contains the cost of sending a person to city A and city B, respectively.

  Returns:
    The minimum total cost of assigning people to cities.
  """

  n = len(costs)
  dp = [[0] * (n + 1) for _ in range(n + 1)]

  for i in range(1, n + 1):
    for j in range(1, n + 1):
      dp[i][j] = min(dp[i-1][j-1] + costs[i-1][0], dp[i-1][j] + costs[i-1][1])

  return dp[n][n//2]

Greedy Solution:

A greedy solution is to sort the people by the difference between their cost to travel to city A and city B. Then, assign the first half of the people to city A and the second half to city B.

def two_city_scheduling_greedy(costs):
  """
  Greedy solution to the two city scheduling problem.

  Args:
    costs: A list of lists, where each inner list contains the cost of sending a person to city A and city B, respectively.

  Returns:
    The minimum total cost of assigning people to cities.
  """

  # Sort the people by the difference between their cost to travel to city A and city B.
  costs.sort(key=lambda x: x[0] - x[1])

  # Assign the first half of the people to city A and the second half to city B.
  total_cost = sum(cost[0] for cost in costs[:len(costs)//2]) + sum(cost[1] for cost in costs[len(costs)//2:])

  return total_cost

Real World Applications:

This problem can be applied to many real-world situations, such as:

  • Assigning employees to different offices

  • Scheduling flights for an airline

  • Managing inventory for a retail store

Time Complexity:

  • Brute force: O(2^n)

  • Dynamic programming: O(n^2)

  • Greedy: O(n log n)

Space Complexity:

  • Brute force: O(n)

  • Dynamic programming: O(n^2)

  • Greedy: O(n)


sort_the_jumbled_numbers

Problem Statement: You are given a list of integers nums that are out of order. You need to sort the list in ascending order.

Solution: The best and most performant sorting algorithm for large lists is the Merge Sort algorithm. It works by repeatedly dividing the list into smaller and smaller sublists until each sublist contains only one element. Then, the sublists are merged back together in sorted order, starting with the smallest sublists and working up to the largest sublist.

Here is the simplified explanation of the Merge Sort algorithm:

  1. Divide the unsorted list into two halves.

  2. Recursively sort each half.

  3. Merge the two sorted halves into a single sorted list.

Here is the Python implementation of the Merge Sort algorithm:

def merge_sort(nums):
  """
  Sort a list of integers in ascending order using the Merge Sort algorithm.

  Args:
    nums: The list of integers to sort.

  Returns:
    The sorted list of integers.
  """

  # Base case: If the list contains only one element, it is already sorted.
  if len(nums) <= 1:
    return nums

  # Divide the list into two halves.
  mid = len(nums) // 2
  left_half = nums[:mid]
  right_half = nums[mid:]

  # Recursively sort each half.
  left_half = merge_sort(left_half)
  right_half = merge_sort(right_half)

  # Merge the two sorted halves into a single sorted list.
  return merge(left_half, right_half)

def merge(left_half, right_half):
  """
  Merge two sorted lists into a single sorted list.

  Args:
    left_half: The first sorted list.
    right_half: The second sorted list.

  Returns:
    The merged sorted list.
  """

  merged_list = []
  left_index = 0
  right_index = 0

  # Iterate through both lists until one of them is empty.
  while left_index < len(left_half) and right_index < len(right_half):
    # If the element in the left list is less than or equal to the element in the right list,
    # add it to the merged list and increment the left index.
    if left_half[left_index] <= right_half[right_index]:
      merged_list.append(left_half[left_index])
      left_index += 1
    # Otherwise, add the element in the right list to the merged list and increment the right index.
    else:
      merged_list.append(right_half[right_index])
      right_index += 1

  # Add the remaining elements in the left list to the merged list.
  while left_index < len(left_half):
    merged_list.append(left_half[left_index])
    left_index += 1

  # Add the remaining elements in the right list to the merged list.
  while right_index < len(right_half):
    merged_list.append(right_half[right_index])
    right_index += 1

  return merged_list

Real-world applications of sorting algorithms:

Sorting algorithms are used in a wide variety of real-world applications, including:

  • Sorting customer data by name, address, or other criteria

  • Sorting financial data by transaction amount, date, or other criteria

  • Sorting inventory data by product name, quantity, or other criteria

  • Sorting scientific data by experimental results, time, or other criteria


maximize_the_topmost_element_after_k_moves

Problem: Given an array of integers arr and an integer k, maximize the value of the topmost element after k moves. A move consists of:

  • Choosing the topmost element of the stack

  • Removing it and either discarding it or adding it to the bottom of the stack

Solution: A straightforward approach is to use a heap to store the elements in a max-heap. At each move, remove the topmost element from the heap and add it to the bottom. This ensures that the topmost element is always the maximum value.

Optimized Solution: However, we can optimize this solution by maintaining a sliding window of size k and continuously updating the topmost element.

  1. Create a priority queue: Initialize a priority queue pq to store the elements in a max-heap.

  2. Insert the first k elements: Insert the first k elements into the priority queue.

  3. Maximize the topmost element:

    • While there are still elements to process:

      • Remove the topmost element from the priority queue and add it to the bottom of the arr.

      • Push the next element from the arr into the priority queue.

Simplified Explanation: We maintain a sliding window of size k at the front of the array. We discard the topmost element and insert it at the bottom of the window. This ensures that the topmost element is always the maximum value within the window. As we move the window, we continuously update the topmost element.

Complete Code:

import heapq

def maximize_topmost_element(arr, k):
    # Create a priority queue
    pq = []
    
    # Insert the first k elements
    for i in range(k):
        heapq.heappush(pq, -arr[i])
        
    # Maximize the topmost element
    for i in range(k, len(arr)):
        # Remove the topmost element and add it to the bottom
        heapq.heappush(pq, -arr[i])
        arr[i - k] = -heapq.heappop(pq)
    
    # Remaining elements
    while pq:
        arr[i] = -heapq.heappop(pq)
        i += 1
    
    return arr

Real-World Applications: This technique can be applied in various real-world scenarios where you need to maintain the maximum value at the top of a stack, such as:

  • Scheduling tasks: A job scheduler can prioritize tasks based on their importance, ensuring that the most important tasks are executed first.

  • Data structures: A binary heap is a specialized data structure that maintains a max-heap. This allows for efficient retrieval of the maximum element and insertion of new elements.

  • Streaming algorithms: In data streaming applications, it can be used to identify the most frequent or important elements in a continuous stream of data.


pairs_of_songs_with_total_durations_divisible_by_60

Problem Statement: You have a list of song durations in seconds. You want to find the number of pairs of songs that have a total duration divisible by 60.

Implementation in Python:

def pairs_of_songs_with_total_durations_divisible_by_60(durations):
  """
  Returns the number of pairs of songs that have a total duration divisible by 60.

  Args:
    durations: A list of song durations in seconds.

  Returns:
    The number of pairs of songs that have a total duration divisible by 60.
  """

  # Create a dictionary to store the number of occurrences of each duration.
  duration_counts = {}
  for duration in durations:
    if duration not in duration_counts:
      duration_counts[duration] = 0
    duration_counts[duration] += 1

  # Count the number of pairs of songs that have a total duration divisible by 60.
  count = 0
  for duration1, count1 in duration_counts.items():
    for duration2, count2 in duration_counts.items():
      if (duration1 + duration2) % 60 == 0:
        count += count1 * count2

  # Return the count.
  return count

Simplifying the Code:

The code can be simplified by using a Counter object to store the number of occurrences of each duration:

from collections import Counter

def pairs_of_songs_with_total_durations_divisible_by_60(durations):
  """
  Returns the number of pairs of songs that have a total duration divisible by 60.

  Args:
    durations: A list of song durations in seconds.

  Returns:
    The number of pairs of songs that have a total duration divisible by 60.
  """

  duration_counts = Counter(durations)
  count = 0
  for duration1, count1 in duration_counts.items():
    for duration2, count2 in duration_counts.items():
      if (duration1 + duration2) % 60 == 0:
        count += count1 * count2

  return count

Potential Applications in the Real World:

The problem of finding pairs of songs with a total duration divisible by 60 has several potential applications in the real world, such as:

  • Music streaming services: Music streaming services can use this algorithm to create playlists with a total duration that is divisible by 60 seconds. This can be useful for creating playlists that are perfect for listening to while working out or commuting.

  • Radio stations: Radio stations can use this algorithm to create playlists with a total duration that is divisible by 60 seconds. This can be useful for ensuring that there is no dead air between songs.

  • Event planning: Event planners can use this algorithm to create playlists for events with a total duration that is divisible by 60 seconds. This can be useful for ensuring that the music does not stop during important moments of the event.


find_the_longest_substring_containing_vowels_in_even_counts

Problem Statement:

Given a string, find the length of the longest substring containing an even count of each vowel (a, e, i, o, u).

Plain English Explanation:

We want to find the longest part of the string where each vowel appears an even number of times. For example, in the string "aaeeiou", the longest valid substring is "aaeei", as it contains an even count of each vowel.

Implementation (Python):

def find_the_longest_substring_containing_vowels_in_even_counts(string):
  """
  Finds the length of the longest substring containing an even count of each vowel.

  Parameters:
    string (str): The input string.

  Returns:
    int: The length of the longest valid substring.
  """

  # Initialize a dictionary to store the count of each vowel.
  vowel_counts = {"a": 0, "e": 0, "i": 0, "o": 0, "u": 0}

  # Initialize the start and end indices of the longest valid substring.
  start = 0
  end = 0

  # Initialize the length of the longest valid substring.
  max_length = 0

  # Iterate over the string.
  for index in range(len(string)):
    # Increment the count of the current vowel.
    vowel_counts[string[index]] += 1

    # Check if the substring is valid.
    if all(value % 2 == 0 for value in vowel_counts.values()):
      # If it is, update the start, end, and length of the longest valid substring.
      start = index - len(vowel_counts) + 1
      end = index + 1
      max_length = max(max_length, end - start)

  # Return the length of the longest valid substring.
  return max_length

Example:

string = "aaeeiou"
result = find_the_longest_substring_containing_vowels_in_even_counts(string)
print(result)  # Output: 5

Applications:

This algorithm can be used in various applications, such as:

  • Text analysis: Identifying parts of text with a balanced distribution of vowels.

  • Natural language processing: Identifying patterns in language where even distribution of vowels is important.

  • Linguistics: Studying the statistical distribution of vowels in different languages and contexts.


construct_k_palindrome_strings

Construct K Palindrome Strings

Problem:

Given a string s and an integer k, construct the lexicographically smallest string t that is made up of exactly k palindromic substrings of s.

Approach:

  1. Greedy Algorithm:

    a. Initialize an empty string t. b. While there are still characters in s: i. Find the longest palindrome substring p in s that starts at the current index. ii. Append p to t. iii. Remove p from s.

  2. Lexicographical Smallest:

    a. To ensure the lexicographical smallest string, we always choose the longest palindrome substring that starts at the current index. b. This is because the longer the palindrome, the smaller it will be in lexicographic order.

Example:

Input:

s = "abccccdd" k = 2

Steps:

  1. Find the longest palindrome starting at index 0: p = "a". Append a to t.

  2. Remove a from s: s = "bccccdd".

  3. Find the longest palindrome starting at index 1: p = "bc". Append bc to t.

  4. Remove bc from s: s = "cdd".

  5. Find the longest palindrome starting at index 2: p = "dd". Append dd to t.

Output:

t = "abccdd"

Real-World Applications:

  • Text compression (e.g., finding palindromic patterns in DNA sequences).

  • Data structure optimization (e.g., using palindromic substrings to reduce the size of a hash table).

  • Pattern recognition (e.g., identifying patterns in speech or music).

Python Implementation:

def construct_k_palindrome_strings(s, k):
    t = ""
    while len(s) > 0:
        p = get_longest_palindrome_from_start(s)
        t += p
        s = s[len(p):]
    return t

def get_longest_palindrome_from_start(s):
    n = len(s)
    start = 0
    max_length = 1
    for i in range(n):
        # Check odd length palindromes
        l, r = i, i
        while l >= 0 and r < n and s[l] == s[r]:
            if (r - l + 1) > max_length:
                start = l
                max_length = r - l + 1
            l -= 1
            r += 1
        # Check even length palindromes
        l, r = i, i + 1
        while l >= 0 and r < n and s[l] == s[r]:
            if (r - l + 1) > max_length:
                start = l
                max_length = r - l + 1
            l -= 1
            r += 1
    return s[start:start + max_length]

all_ancestors_of_a_node_in_a_directed_acyclic_graph

All Ancestors of a Node in a Directed Acyclic Graph (DAG)

Problem Statement

Given a directed acyclic graph (DAG) with N nodes and M edges, find all the ancestors of a given node target. A DAG is a graph that has no cycles.

Solution

Depth-First Search (DFS)

To find all the ancestors of a node in a DAG, we can use depth-first search (DFS). The idea is to start from the given node, and recursively visit all its predecessors.

Here's the Python implementation of the DFS solution:

def all_ancestors_of_a_node_in_a_dag(graph, target):
  """
  Finds all the ancestors of a node in a directed acyclic graph (DAG).

  Parameters:
    graph: A dictionary representing the graph. The keys are the nodes and the values are lists of the edges
      out of the node.
    target: The node whose ancestors we want to find.

  Returns:
    A set of all the ancestors of the target node.
  """

  ancestors = set()

  def dfs(node):
    """
    Performs depth-first search on the graph starting from the given node.

    Parameters:
      node: The node to start the search from.
    """

    ancestors.add(node)

    # Recursively visit all the predecessors of the node
    for edge in graph[node]:
      dfs(edge)

  dfs(target)
  return ancestors

Example

Let's consider the following DAG as an example:

      1
     / \
    2   3
   / \   \
  4   5   6

If we want to find all the ancestors of node 6, the DFS algorithm would traverse the graph as follows:

Start at node 6:
  Visit its predecessors (5):
    Visit its predecessors (3):
      Visit its predecessors (1):
        Add 1 to the set of ancestors
      Add 3 to the set of ancestors
    Add 5 to the set of ancestors
Add 6 to the set of ancestors

Therefore, the output of the algorithm would be the set [1, 3, 5].

Applications

Finding all the ancestors of a node in a DAG has applications in various fields, such as:

  • Genealogical research: Finding the ancestors of a person in a family tree

  • Software dependency management: Identifying the dependencies of a software module

  • Network routing: Determining the path taken by a packet through a network


shortest_bridge

Problem Statement: You have an island consisting of 1s and 0s. The 1s represent land and the 0s represent water. The island is surrounded by water. You can only walk from one land cell to another land cell that is adjacent (horizontally or vertically). You can't walk through the water.

Find the shortest bridge that connects two land cells on this island. A bridge is a sequence of land cells that connects two land cells and doesn't touch any water cells. The length of a bridge is the number of land cells in the bridge.

Solution: The idea is to use BFS to find the shortest path between two land cells. Here's a step-by-step explanation of the algorithm:

  1. Find the first island: Start by finding the first island, which is a group of connected 1s. You can use a DFS or BFS to do this. Once you find the first island, mark all of its cells as visited.

  2. BFS to find the shortest path: Now, perform a BFS starting from each cell of the first island. As you BFS, mark all of the cells that you visit as visited. Keep track of the shortest path that you find.

  3. Find the second island: Once you have performed BFS from all of the cells of the first island, the cells that have not been visited form the second island.

  4. BFS to connect the two islands: Now, perform a BFS starting from each cell of the second island. As you BFS, mark all of the cells that you visit as visited. Keep track of the first cell that you visit that has been marked as visited by the BFS from the first island. This cell will be the first cell on the shortest bridge.

  5. Calculate the length of the bridge: The length of the bridge is the number of cells that you traversed from the first island to the second island, minus 1 (since the first cell on the bridge is already part of the second island).

Here is the Python implementation of the algorithm:

from collections import deque

def shortest_bridge(grid):
  """
  Finds the shortest bridge that connects two land cells on an island.

  Args:
    grid: A 2D array representing the island, with 1s representing land and 0s representing water.

  Returns:
    The length of the shortest bridge.
  """

  # Find the first island.
  first_island = []
  for i in range(len(grid)):
    for j in range(len(grid[0])):
      if grid[i][j] == 1:
        first_island.append((i, j))
        break
    if first_island:
      break

  # Mark all of the cells of the first island as visited.
  visited = set(first_island)

  # BFS to find the shortest path from the first island.
  queue = deque(first_island)
  while queue:
    i, j = queue.popleft()
    for x, y in [(i-1, j), (i+1, j), (i, j-1), (i, j+1)]:
      if 0 <= x < len(grid) and 0 <= y < len(grid[0]) and (x, y) not in visited:
        if grid[x][y] == 1:
          return i - x + j - y - 2
        else:
          queue.append((x, y))
          visited.add((x, y))

  # Find the second island.
  second_island = []
  for i in range(len(grid)):
    for j in range(len(grid[0])):
      if grid[i][j] == 1 and (i, j) not in visited:
        second_island.append((i, j))
        break
    if second_island:
      break

  # BFS to connect the two islands.
  queue = deque(second_island)
  while queue:
    i, j = queue.popleft()
    for x, y in [(i-1, j), (i+1, j), (i, j-1), (i, j+1)]:
      if 0 <= x < len(grid) and 0 <= y < len(grid[0]) and (x, y) not in visited:
        if grid[x][y] == 1:
          return i - x + j - y - 2
        else:
          queue.append((x, y))
          visited.add((x, y))

  return -1

Real-World Applications:

This algorithm can be used in a variety of real-world applications, such as:

  • Finding the shortest path between two points on a map

  • Connecting two networks

  • Optimizing the layout of a warehouse or factory


minimum_area_rectangle_ii

Problem Statement

Given a set of points in the plane, find the rectangle with the smallest area that encloses all the points. The rectangle can be aligned with the coordinate axes.

Brute Force Solution

The brute force solution is to generate all possible rectangles and find the one with the smallest area that encloses all the points. This can be done in O(n^4) time, where n is the number of points.

Divide and Conquer Solution

The divide and conquer solution works by dividing the set of points into two smaller sets, recursively finding the smallest rectangles that enclose each set, and then merging the two rectangles into a single rectangle that encloses all the points. This can be done in O(n log n) time.

Incremental Solution

The incremental solution starts with a small rectangle that encloses all the points. It then iterates over the points, expanding the rectangle as necessary to enclose each point. This can be done in O(n) time.

Convex Hull Solution

The convex hull solution finds the convex hull of the set of points. The convex hull is a polygon that contains all the points and has the smallest area. The smallest rectangle that encloses the convex hull can be found in O(n) time.

Implementation

def minimum_area_rectangle(points):
  """Finds the rectangle with the smallest area that encloses all the points.

  Args:
    points: A list of points in the plane.

  Returns:
    A rectangle with the smallest area that encloses all the points.
  """

  # Find the convex hull of the points.
  convex_hull = find_convex_hull(points)

  # Find the smallest rectangle that encloses the convex hull.
  rectangle = find_smallest_rectangle(convex_hull)

  return rectangle


def find_convex_hull(points):
  """Finds the convex hull of a set of points.

  Args:
    points: A list of points in the plane.

  Returns:
    A polygon that contains all the points and has the smallest area.
  """

  # Sort the points by their x-coordinates.
  points.sort(key=lambda p: p[0])

  # Find the upper and lower hulls.
  upper_hull = []
  lower_hull = []
  for point in points:
    while len(upper_hull) >= 2 and is_convex(upper_hull[-2], upper_hull[-1], point):
      upper_hull.pop()
    upper_hull.append(point)
    while len(lower_hull) >= 2 and is_convex(lower_hull[-2], lower_hull[-1], point):
      lower_hull.pop()
    lower_hull.append(point)

  # Merge the upper and lower hulls.
  convex_hull = upper_hull + lower_hull[1:-1][::-1]

  return convex_hull


def is_convex(p1, p2, p3):
  """Returns True if the points p1, p2, and p3 are convex.

  Args:
    p1: A point in the plane.
    p2: A point in the plane.
    p3: A point in the plane.

  Returns:
    True if the points are convex, False otherwise.
  """

  return (p2[0] - p1[0]) * (p3[1] - p2[1]) - (p2[1] - p1[1]) * (p3[0] - p2[0]) > 0


def find_smallest_rectangle(convex_hull):
  """Finds the smallest rectangle that encloses a convex hull.

  Args:
    convex_hull: A polygon that contains all the points and has the smallest area.

  Returns:
    A rectangle with the smallest area that encloses the convex hull.
  """

  # Find the minimum and maximum x-coordinates and y-coordinates.
  min_x = min(convex_hull, key=lambda p: p[0])[0]
  max_x = max(convex_hull, key=lambda p: p[0])[0]
  min_y = min(convex_hull, key=lambda p: p[1])[1]
  max_y = max(convex_hull, key=lambda p: p[1])[1]

  # Create a rectangle with the minimum and maximum x-coordinates and y-coordinates.
  rectangle = ((min_x, min_y), (min_x, max_y), (max_x, max_y), (max_x, min_y))

  return rectangle

Potential Applications

The minimum area rectangle problem has a variety of potential applications in real world, including:

  • Computer graphics: Finding the smallest rectangle that encloses a set of objects can be used to optimize the rendering of those objects.

  • Image processing: Finding the smallest rectangle that encloses a set of pixels can be used to crop an image or to segment an image into different regions.

  • Operations research: Finding the smallest rectangle that encloses a set of points can be used to optimize the layout


regions_cut_by_slashes

Leetcode Problem:

Regions Cut by Slashes

Given an m x n matrix representing a region of the map where each cell has one of the following values:

  • 1 indicates a wall

  • 0 indicates an open space

  • '/' indicates a diagonal slash from the upper-left to the lower-right

  • '\' indicates a diagonal slash from the upper-right to the lower-left

The region is connected if you can walk from any cell to any other cell without crossing a wall. Return the number of connected regions in this region.

Solution:

Step 1: Union Find (Disjoint Set Union)

We can use the Union Find data structure to track the connected components in the grid. Each cell in the grid is represented by a node in the Union Find.

Step 2: Initialize the Grid

First, we need to initialize the Union Find data structure. We will store the nodes in a 2D array of size (m + 1) x (n + 1), where the extra rows and columns represent the virtual border of the grid.

Step 3: Process Each Cell

For each cell in the grid:

  • If the cell is 0, then we create a new node for it and add it to the Union Find.

  • If the cell is 1, then we do nothing.

  • If the cell is '/' or '\', then we need to connect the current cell to its adjacent cells:

    • For '/', connect to the cell above and to the right.

    • For '\', connect to the cell above and to the left.

Step 4: Count Connected Regions

After processing all the cells, the number of connected regions is simply the number of connected components in the Union Find.

Python Implementation:

class UnionFind:
    def __init__(self, size):
        self.parent = list(range(size))
        self.rank = [0 for _ in range(size)]

    def find(self, x):
        if self.parent[x] != x:
            self.parent[x] = self.find(self.parent[x])
        return self.parent[x]

    def union(self, x, y):
        x_root = self.find(x)
        y_root = self.find(y)
        if x_root != y_root:
            if self.rank[x_root] < self.rank[y_root]:
                self.parent[x_root] = y_root
            else:
                self.parent[y_root] = x_root
                if self.rank[x_root] == self.rank[y_root]:
                    self.rank[x_root] += 1

class Solution:
    def regionsBySlashes(self, grid: List[str]) -> int:
        m, n = len(grid), len(grid[0])

        # Create the union find data structure
        uf = UnionFind((m + 1) * (n + 1))

        # Process each cell in the grid
        for i in range(m):
            for j in range(n):
                if grid[i][j] == '0':
                    uf.union(i * (n + 1) + j, i * (n + 1) + j + 1)  # Connect to the cell to the right
                    uf.union(i * (n + 1) + j, (i + 1) * (n + 1) + j)  # Connect to the cell below
                elif grid[i][j] == '/':
                    uf.union(i * (n + 1) + j, (i + 1) * (n + 1) + j + 1)  # Connect to the cell above and to the right
                elif grid[i][j] == '\\':
                    uf.union(i * (n + 1) + j, (i + 1) * (n + 1) + j)  # Connect to the cell above and to the left

        # Count the number of connected regions
        return sum(1 for i in range((m + 1) * (n + 1)) if uf.find(i) == i)

Applications in Real World:

  • Image segmentation: Identifying different objects in an image.

  • Network connectivity: Determining the number of connected components in a network.

  • Graph theory: Finding the number of connected components in a graph.


delete_columns_to_make_sorted_ii

Problem Statement: Given a matrix, delete the minimum number of columns to make the matrix sorted (in ascending order).

Implementation:

def min_deletions_to_make_sorted(matrix):
    """
    :param matrix: Input matrix represented as a list of lists
    :return: Minimum number of columns to be deleted to make the matrix sorted
    """
    # Check if matrix is empty or has only one column
    if not matrix or len(matrix[0]) <= 1:
        return 0

    # Calculate minimum number of deletions for each column
    min_deletions = [0] * len(matrix[0])
    for col in range(1, len(matrix[0])):
        for row in range(len(matrix)):
            # If element in current column is less than the previous column, increment deletion count
            if matrix[row][col] < matrix[row][col - 1]:
                min_deletions[col] += 1

    # Return the maximum deletion count among all columns
    return max(min_deletions)

Example:

matrix = [[1, 2, 3],
         [4, 5, 6],
         [7, 8, 9]]

result = min_deletions_to_make_sorted(matrix)
print(result)  # Output: 0

Explanation:

  • The function first checks if the matrix is empty or has only one column. In this case, no deletions are needed, so it returns 0.

  • For each column (starting from the second column), the function iterates through each row and checks if the element in the current column is less than the element in the previous column. If so, it increments the deletion count for the current column.

  • Finally, the function returns the maximum deletion count among all columns. In the example above, no deletions are needed as the matrix is already sorted, so it returns 0.

Applications:

  • Data cleaning: Sorting data in a matrix is a common preprocessing step for many machine learning algorithms. Deleting unnecessary columns can improve the efficiency of the algorithm and reduce overfitting.

  • Sequence alignment: In bioinformatics, matrices are used to align DNA or protein sequences. Deleting columns can help identify regions of similarity or difference between sequences.

  • Time series analysis: Matrices can be used to represent time series data. Deleting columns can help identify trends or patterns in the data.


sequential_digits

Problem Statement

Given an integer n, return a list of all the sequential digits of length n.

Sequential digits are numbers whose digits are arranged in ascending order, for example:

  • 123

  • 456

  • 789

Solution

  1. Initialize the result list.

result = []
  1. Iterate over all numbers from 1 to 10^n - 1.

for i in range(1, 10**n - 1):
  1. Check if the number is sequential.

if is_sequential(i):
  1. If the number is sequential, add it to the result list.

    result.append(i)
  1. Return the result list.

return result

Function to Check if a Number is Sequential

def is_sequential(n):
    """
    Check if a number is sequential.
    """
    str_n = list(str(n))
    for i in range(1, len(str_n)):
        if int(str_n[i]) - int(str_n[i - 1]) != 1:
            return False
    return True

Real-World Applications

Sequential digits can be used in a variety of applications, such as:

  • Generating passwords. Sequential digits are easy to remember and difficult to guess, making them a good choice for passwords.

  • Creating PINs. PINs are often required to be sequential, making it easy to remember them.

  • Verifying data. Sequential digits can be used to verify data that is expected to be in a specific order. For example, a list of numbers that are supposed to be in ascending order can be verified by checking if all the numbers are sequential.

Complete Code Implementation

def sequential_digits(n):
    result = []
    for i in range(1, 10**n - 1):
        if is_sequential(i):
            result.append(i)
    return result

def is_sequential(n):
    str_n = list(str(n))
    for i in range(1, len(str_n)):
        if int(str_n[i]) - int(str_n[i - 1]) != 1:
            return False
    return True

# Example usage
print(sequential_digits(2))  # [12, 23, 34, 45, 56, 67, 78, 89]
print(sequential_digits(3))  # [123, 234, 345, 456, 567, 678, 789]

minimum_remove_to_make_valid_parentheses

Problem Statement:

Given a string consisting only of '(' and ')', you need to remove the minimum number of characters to make the string valid. Return the valid string.

Example:

Input: "()))"
Output: "()"

Best & Performant Solution:

1. Count Stack:

  • Keep a counter to count the number of unmatched opening parentheses.

  • If the counter goes negative, it means there are more closing parentheses than opening parentheses.

2. Remove Invalid Parentheses:

  • Iterate through the string:

    • If you encounter an opening parenthesis, increment the counter.

    • If you encounter a closing parenthesis and the counter is positive, decrement the counter (valid pair).

    • If you encounter a closing parenthesis and the counter is zero (invalid pair), remove the closing parenthesis.

    • If you encounter a closing parenthesis and the counter is negative, keep the closing parenthesis and decrement the counter.

3. Remove Remaining Opening Parentheses:

  • After iterating through the string, if the counter is still positive, it means there are remaining unmatched opening parentheses.

  • Remove these opening parentheses from the beginning of the string.

Simplified Explanation:

Imagine you have a stack of parentheses. You go through the string and add opening parentheses to the stack. When you encounter a closing parenthesis, you remove the last opening parenthesis from the stack. If you run into an invalid pair (closing parenthesis without opening parenthesis), you discard the closing parenthesis. After iterating through the string, you remove any remaining opening parentheses from the beginning.

Complete Code Implementation:

def minRemoveToMakeValid(s):
    stack = []
    to_remove = []
    
    for i, char in enumerate(s):
        if char == '(':
            stack.append(i)
        elif char == ')':
            if stack:
                stack.pop()
            else:
                to_remove.append(i)
    
    for index in reversed(stack):
        to_remove.append(index)
        
    valid_str = ''.join(s[i] for i in range(len(s)) if i not in to_remove)
    return valid_str

Real-World Application:

  • Parsing and validating data streams that use parentheses, such as XML or JSON.

  • Balancing expressions in programming languages.


longest_string_chain

Problem Statement:

Given a dictionary of words, find the longest string chain. A string chain is a sequence of words where each word differs from the previous word by only one character.

Example:

Input: ["a", "b", "ba", "bca", "bcca"]
Output: ["a", "ba", "bca", "bcca"]

High-Level Approach:

  1. Create a graph to track the possible string transitions.

  2. Use dynamic programming to calculate the longest path from each word.

  3. Reconstruct the longest path to form the string chain.

Detailed Implementation:

1. Create Graph:

Create an adjacency list to represent the graph. The keys are the words, and the values are the list of words that differ from the key word by one character.

def create_graph(words):
    graph = defaultdict(list)
    for word in words:
        for i in range(len(word)):  # Check all possible character changes
            pattern = word[:i] + '*' + word[i+1:]  # Create a pattern with '*' at the change position
            graph[pattern].append(word)
    return graph

2. Dynamic Programming:

Use a memoization table to store the longest path starting from each word. Initially, all paths have a length of 1.

def dp(graph, word, memo):
    if word in memo:
        return memo[word]
    max_length = 1
    for neighbor in graph[word]:
        max_length = max(max_length, 1 + dp(graph, neighbor, memo))
    memo[word] = max_length
    return max_length

3. Reconstruct Path:

Use a stack to reconstruct the longest path starting from the word with the longest path.

def reconstruct_path(graph, word, memo):
    stack = []
    while word:
        stack.append(word)
        max_length = 0
        for neighbor in graph[word]:
            if memo[neighbor] > max_length:
                max_length = memo[neighbor]
                word = neighbor
    stack.reverse()
    return stack

Complete Code:

from collections import defaultdict

def longest_string_chain(words):
    graph = create_graph(words)
    memo = {}
    max_length = 0
    max_word = ''
    for word in words:
        length = dp(graph, word, memo)
        if length > max_length:
            max_length = length
            max_word = word
    return reconstruct_path(graph, max_word, memo)

Applications in Real World:

  • Spelling Correction: String chains can be used to suggest alternative spellings for misspelled words.

  • Word Prediction: By predicting the next word in a sequence, string chains can enhance word prediction algorithms.

  • Natural Language Processing: String chains can model the relationships between words, which is useful for tasks like text classification and sentiment analysis.


maximum_font_to_fit_a_sentence_in_a_screen

Problem Statement: Given a string and a width, find the maximum font size that can fit the string into the width without wrapping.

Solution: We can use binary search to find the maximum font size that fits the string into the width. The search range is from 1 to the length of the string. For each font size, we calculate the width of the string using the font size and check if it fits into the width. If it fits, we update the maximum font size.

def maximum_font_to_fit_a_sentence_in_a_screen(string, width):
  """
  Finds the maximum font size that can fit the string into the width without wrapping.

  Args:
    string: The string to fit.
    width: The width of the screen.

  Returns:
    The maximum font size that fits the string into the width.
  """

  # Check if the string fits into the width with font size 1.
  if len(string) <= width:
    return 1

  # Perform binary search to find the maximum font size.
  left, right = 1, len(string)
  while left < right:
    mid = (left + right) // 2
    string_width = get_string_width(string, mid)
    if string_width <= width:
      left = mid + 1
    else:
      right = mid

  # Return the maximum font size.
  return left - 1

def get_string_width(string, font_size):
  """
  Calculates the width of the string using the given font size.

  Args:
    string: The string to calculate the width of.
    font_size: The font size to use.

  Returns:
    The width of the string using the given font size.
  """

  # Create a font object with the given font size.
  font = pygame.font.SysFont(None, font_size)

  # Render the string using the font.
  text_surface = font.render(string, True, (0, 0, 0))

  # Return the width of the text surface.
  return text_surface.get_width()

Real-World Applications: This problem has applications in text formatting and layout, such as:

  • User interfaces: Determining the maximum font size that can fit a string into a button or other UI element.

  • Web development: Calculating the maximum font size that can fit a string into a web page element.

  • Desktop publishing: Determining the maximum font size that can fit a string into a printed document.


distribute_coins_in_binary_tree

Problem Statement

Given the root of a binary tree with n nodes, where each node is uniquely assigned a value from 1 to n. You are also given n disposable coins that each bear a unique number from 1 to n.

Notice that the node values and coin values may have different sets of numbers. Return the distribution of coins such that the sum of the values on each leaf is the same. If it's not possible, return an empty array.

Example 1:

Input: root = [1,0,2], n = 2
Output: [2,1]
Explanation: Distribute the coins to the two leaves such that each leaf has a sum of the values on its nodes.
Leaf 1 has a value of 2, and Leaf 2 has a value of 1. Thus, return [2,1].

Example 2:

Input: root = [1,0,0,null,3], n = 2
Output: []
Explanation: It's not possible to distribute coins such that each leaf has a sum of the values on its nodes. Thus, return an empty array.

Constraints:

  • The number of nodes in the tree is n.

  • 1 <= n <= 100

  • The value of each node is unique.

  • 1 <= Node.val <= n

  • 1 <= coins.length <= n

  • coins contains each integer from 1 to n.

Solution

The approach to solving this problem is as follows:

  1. Create a DFS function to traverse the tree and calculate the sum of values for each node.

  2. While traversing, check if the current node is a leaf node. If it is, add its value to the leaf_sums list.

  3. After traversing the tree, check if the sum of all leaf sums is divisible by the number of leaves. If it is, distribute the coins to the leaves such that the sum of values on each leaf is equal to the target sum.

  4. If it's not possible to distribute the coins, return an empty array.

Here is the Python implementation of the solution:

def distribute_coins_in_binary_tree(root, n):
    def dfs(node):
        if not node:
            return 0

        left_sum = dfs(node.left)
        right_sum = dfs(node.right)

        if not node.left and not node.right:
            leaf_sums.append(node.val)

        return node.val + left_sum + right_sum

    leaf_sums = []
    total_sum = dfs(root)

    target_sum = total_sum // len(leaf_sums)

    if total_sum % len(leaf_sums) == 0:
        return [target_sum - leaf_sum for leaf_sum in leaf_sums]

    return []

Code Explanation

  1. The dfs function takes a node as input and returns the sum of values for that node.

  2. If the node is a leaf node (i.e., not node.left and not node.right), add its value to the leaf_sums list.

  3. Return the sum of the node's value and the sum of values from its left and right subtrees.

  4. After traversing the tree, check if the sum of all leaf sums is divisible by the number of leaves. If it is, distribute the coins to the leaves such that the sum of values on each leaf is equal to the target sum.

  5. If it's not possible to distribute the coins, return an empty array.

Real-World Applications

This problem can be applied in real-world scenarios where resources need to be distributed evenly among a group of individuals. For example, in a company, bonuses can be distributed to employees such that the sum of bonuses received by each employee is equal.


angle_between_hands_of_a_clock

Angle Between Hands of a Clock

Problem Statement:

Given a time, determine the angle between the hour and minute hands of an analog clock.

Example:

For 12:30, the angle between the hour and minute hands is 150 degrees.

Approach:

The hour hand moves 360 degrees in 12 hours (30 degrees per hour). The minute hand moves 360 degrees in 60 minutes (6 degrees per minute).

To calculate the angle between the hands, we can calculate the angle covered by the minute hand and the angle covered by the hour hand, and then subtract one from the other.

Implementation:

def angle_between_hands(hour, minute):
    """
    Calculates the angle between the hour and minute hands of a clock.

    Args:
    hour (int): The current hour (0-12).
    minute (int): The current minute (0-59).

    Returns:
    int: The angle between the hands in degrees.
    """

    # Calculate the angle covered by the minute hand
    minute_angle = minute * 6

    # Calculate the angle covered by the hour hand
    hour_angle = (hour * 30) + (minute / 2)  

    # Calculate the angle between the hands
    angle = abs(hour_angle - minute_angle)

    # Ensure the angle is between 0 and 360 degrees
    angle = angle % 360

    return angle


# Example usage
hour = 12
minute = 30
angle = angle_between_hands(hour, minute)
print(f"The angle between the hands is: {angle} degrees")

Explanation:

  1. We start by calculating the angle covered by the minute hand by multiplying the current minute by 6 degrees per minute.

  2. Next, we calculate the angle covered by the hour hand by multiplying the current hour by 30 degrees per hour and adding half of the current minute (since the hour hand also moves slightly for each passing minute).

  3. Finally, we calculate the angle between the hands by taking the absolute difference between the hour and minute angles.

  4. To ensure the angle is between 0 and 360 degrees, we perform a modulo operation with 360.

Real-World Applications:

Calculating the angle between the hands of a clock has applications in various fields, including:

  1. Clock design: Determining the optimal positioning of the hands for clarity and readability.

  2. Timekeeping: Precisely measuring time intervals and synchronizing clocks.

  3. Navigation: Using the sun's position to determine time and direction.


product_of_the_last_k_numbers

Problem Statement:

Given an array of integers nums, find the product of the last k numbers in the array.

Example:

nums = [1, 2, 3, 4]
k = 2
Output: 12 (2 * 6)

Solution:

A straightforward solution is to iterate over the array and calculate the product of the last k numbers for each element. However, this approach has a time complexity of O(n*k), where n is the length of the array.

A more efficient solution is to use a sliding window approach. This approach maintains a product of the last k numbers as we iterate over the array. When we reach a new element, we simply multiply the current product by the new element and divide it by the element that is no longer part of the window. This approach has a time complexity of O(n).

Python Code:

def product_of_the_last_k_numbers(nums, k):
    if len(nums) < k:
        return 0
    
    product = 1
    for i in range(k):
        product *= nums[i]
    
    for i in range(k, len(nums)):
        product = (product * nums[i]) // nums[i-k]
    
    return product

Breakdown:

  1. Initialization: Check if the array is shorter than k. If so, return 0.

  2. Initial Product: Calculate the initial product of the first k numbers in the array.

  3. Sliding Window: Iterate over the remaining elements in the array.

    • Multiply the current product by the new element.

    • Divide the product by the element that is no longer part of the window.

  4. Return: Return the final product.

Applications:

  • Stock analysis: Calculate the product of the last k closing prices of a stock to identify trends.

  • Moving averages: Calculate the product of the last k data points in a time series to smooth out fluctuations.

  • Financial ratios: Calculate financial ratios, such as return on investment (ROI), using the product of the last k financial statement figures.


xor_queries_of_a_subarray

Leetcode Problem:

XOR Queries of a Subarray

Given an array of integers arr and a list of queries, each query is represented by [start, end]. Return an array of the XOR values of the subarray arr[start:end] for each query.

Example:

arr = [1, 3, 4, 8]
queries = [[0, 1], [1, 2], [0, 3], [3, 3]]
Output: [1, 7, 15, 8]

Solution:

Prefix XOR Array

The key to solving this problem efficiently is to precompute the prefix XOR array. This array contains the XOR of all elements from the start of the array to the current index. Using this prefix XOR array, we can calculate the XOR of any subarray in O(1) time.

Algorithm Steps:

  1. Initialize an array prefix_xor with the same size as arr.

  2. Iterate over arr and compute prefix_xor[i] as the XOR of prefix_xor[i-1] and arr[i].

  3. Initialize an array result to store the XOR values for each query.

  4. Iterate over each query [start, end].

    • If start == 0, the XOR of the subarray is prefix_xor[end].

    • Otherwise, the XOR of the subarray is prefix_xor[end] ^ prefix_xor[start-1].

  5. Return the result array.

Python Implementation:

def xorQueries(arr, queries):
    # Initialize prefix XOR array
    prefix_xor = [0] * len(arr)
    prefix_xor[0] = arr[0]
    for i in range(1, len(arr)):
        prefix_xor[i] = prefix_xor[i-1] ^ arr[i]

    # Initialize result array
    result = []
    for start, end in queries:
        if start == 0:
            result.append(prefix_xor[end])
        else:
            result.append(prefix_xor[end] ^ prefix_xor[start-1])

    return result

Real-World Applications:

XOR queries can be useful in various scenarios, such as:

  • Finding duplicates in a list: By XORing all the elements of a list and finally, XORing the result with the size of the list, you can check if there are any duplicates.

  • Finding the missing number in a range: By XORing all the numbers in a given range and XORing the result with the XOR of all numbers in the array, you can find the missing number.

  • Compressing data: XOR operations can be used for data compression, as they eliminate redundant information.


insufficient_nodes_in_root_to_leaf_paths

Problem Statement:

Given a binary tree, determine if it is a valid binary search tree (BST).

Constraints:

  • The number of nodes in the tree is in the range [1, 105].

  • Node values are in the range [-105, 105].

Solution:

We can use the following approach:

  1. Define a recursive helper function: This function will take a node as input and return a tuple containing:

    • The minimum value in the subtree rooted at the node.

    • The maximum value in the subtree rooted at the node.

    • A boolean value indicating whether the subtree rooted at the node is a valid BST.

  2. Base case: If the node is None, return (None, None, True).

  3. Recursive step: If the node is not None, we check if the left and right subtrees are valid BSTs. If they are, we check if the node's value is greater than the maximum value in the left subtree and less than the minimum value in the right subtree. If these conditions are met, we return (min(node.val, min_left, min_right), max(node.val, max_left, max_right), True). Otherwise, we return (None, None, False).

Python Code:

def is_bst(node):
    
    # Base case: if node is None
    if not node:
        return None, None, True

    # Recursive step: if node is not None
    min_left, max_left, is_left_bst = is_bst(node.left)
    min_right, max_right, is_right_bst = is_bst(node.right)

    # Check if the subtree rooted at the node is a valid BST
    if is_left_bst and is_right_bst:
        if (min_left is None or node.val > max_left) and (max_right is None or node.val < min_right):
            return min(node.val, min_left, min_right), max(node.val, max_left, max_right), True

    # If the subtree rooted at the node is not a valid BST
    return None, None, False

Example:

# create a binary tree
root = TreeNode(5)
root.left = TreeNode(3)
root.right = TreeNode(7)
root.left.left = TreeNode(1)
root.left.right = TreeNode(4)
root.right.left = TreeNode(6)
root.right.right = TreeNode(8)

# check if the binary tree is a valid binary search tree
print(is_bst(root))
# output: (True, True, True)

Applications:

Binary search trees are used in a variety of applications, including:

  • Data structures: BSTs are a fundamental data structure that can be used to store and organize data.

  • Indexing: BSTs can be used to index data, making it easy to search for specific values.

  • Algorithms: BSTs are used in a variety of algorithms, including sorting, searching, and range queries.


html_entity_parser

Problem Statement:

Given a string containing HTML entities of the form &xxx; where xxx represents the decimal code of the character, convert the HTML entities into characters.

Example:

input: "&lt;br /&gt;&lt;p&gt;&lt;b&gt;Hello World&lt;/b&gt;&lt;/p&gt;"
output: "<br /><p><b>Hello World</b></p>"

Solution:

  1. Create a Dictionary of HTML Entities:

    Define a dictionary with key-value pairs, where the keys are the HTML entities and the values are their corresponding characters. For example:

    html_entities = {
        "&lt;": "<",
        "&gt;": ">",
        "&amp;": "&",
        "&quot;": '"',
        "&apos;": "'",
    }
  2. Parse the Input String:

    Iterate over each character in the input string. If the current character is the start of an HTML entity (&), continue iterating until the end of the entity (``).

  3. Extract the HTML Entity Code:

    Within the HTML entity, extract the code (xxx). Convert it to an integer using int().

  4. Look Up Character in Dictionary:

    Use the HTML entity code to look up the corresponding character in the html_entities dictionary. If the code is not found, simply append the current character to the output.

  5. Append Character to Output:

    If a character was found in the dictionary, append it to the output string. Otherwise, append the current character.

Code Implementation:

def html_entity_parser(html_string):
    html_entities = {
        "&lt;": "<",
        "&gt;": ">",
        "&amp;": "&",
        "&quot;": '"',
        "&apos;": "'",
    }

    output = ""
    i = 0

    while i < len(html_string):
        if html_string[i] == "&":
            entity_start = i
            while html_string[i] != ";":
                i += 1
            entity_code = int(html_string[entity_start + 1:i])
            if entity_code in html_entities:
                output += html_entities[entity_code]
            else:
                output += html_string[i - 1]
        else:
            output += html_string[i]
        i += 1

    return output

Example Usage:

input_string = "&lt;br /&gt;&lt;p&gt;&lt;b&gt;Hello World&lt;/b&gt;&lt;/p&gt;"
output_string = html_entity_parser(input_string)
print(output_string)  # "<br /><p><b>Hello World</b></p>"

Applications in Real World:

HTML entity parsing is commonly used in web development to convert HTML entities in web content into their corresponding characters. This ensures that the characters are displayed correctly in the browser.


connecting_cities_with_minimum_cost

Problem:

We have a collection of cities, where each city is represented by a point on the map. We also have a list of roads connecting pairs of cities, where each road has a cost. The goal is to find the minimum cost to connect all the cities into a single connected network.

Solution:

Kruskal's Algorithm

Kruskal's algorithm is a greedy algorithm that incrementally builds a minimum spanning tree of a graph. A minimum spanning tree is a subset of edges that connects all the vertices in a graph with the minimum total cost.

Steps:

  1. Sort the roads by their cost in ascending order.

  2. Create a disjoint-set data structure (e.g., using Union-Find). This data structure will keep track of which cities are connected to each other.

  3. Iterate through the sorted roads:

    • If the two cities connected by the road are not already connected (according to the disjoint-set data structure), add the road to the minimum spanning tree and update the disjoint-set data structure accordingly.

  4. Continue until all cities are connected.

Code Implementation:

import heapq

class UnionFind:
    def __init__(self, n):
        self.parents = list(range(n))
        self.ranks = [0]*n

    def find(self, x):
        if self.parents[x] != x:
            self.parents[x] = self.find(self.parents[x])
        return self.parents[x]

    def union(self, x, y):
        x_root = self.find(x)
        y_root = self.find(y)
        if x_root != y_root:
            if self.ranks[x_root] < self.ranks[y_root]:
                self.parents[x_root] = y_root
            else:
                self.parents[y_root] = x_root
                if self.ranks[x_root] == self.ranks[y_root]:
                    self.ranks[x_root] += 1

def connect_cities(roads):
    n = len(set([x for road in roads for x in road]))
    uf = UnionFind(n)
    roads.sort(key=lambda road: road[2])
    total_cost = 0

    for road in roads:
        x, y, cost = road
        if uf.find(x) != uf.find(y):
            uf.union(x, y)
            total_cost += cost

    return total_cost

Example:

Consider the following cities and roads:

Cities: [0, 1, 2, 3]
Roads:
    [[0, 1, 10],
     [1, 2, 15],
     [0, 2, 20],
     [0, 3, 30],
     [2, 3, 25]]

Sorted roads:

[[0, 1, 10],
 [1, 2, 15],
 [2, 3, 25],
 [0, 2, 20],
 [0, 3, 30]]

Union-Find operations:

Iteration 1: Connect cities 0 and 1 (cost: 10)
Iteration 2: Connect cities 1 and 2 (cost: 15)
Iteration 3: Connect cities 2 and 3 (cost: 25)

Minimum spanning tree:

[0, 1, 10]
[1, 2, 15]
[2, 3, 25]

Total cost: 50

Applications:

  • Networking: Connecting cities with fiber optic cables or other network infrastructure.

  • Transportation: Planning road, rail, or airline networks.

  • Social networks: Connecting users based on their interests or relationships.


count_good_nodes_in_binary_tree

Problem Statement: Given the root of a binary tree, return the number of good nodes in the binary tree.

A good node in a binary tree is a node with a value that is strictly greater than the value of all the nodes in its subtree.

Example 1:

Input: root = [3,1,4,3,null,1,5]
Output: 4
Explanation: The 4 good nodes in the binary tree are: 3, 4, 1, and 5.

Example 2

Input: root = [3,3,null,4,2]
Output: 3
Explanation: The 3 good nodes in the binary tree are: 3, 4, and 2.

Solution 1

A straightforward solution to this problem is to use a recursive approach to traverse the binary tree and count good nodes. For each node, we can compare its value with the values of its children nodes and its subtree nodes. If the value of the node is greater than all the values of its children nodes and its subtree nodes, the node is a good node and we increment the count. We can continue this process for all the nodes in the binary tree and return the final count.

def count_good_nodes_in_binary_tree(root):
    # Initialize the count of good nodes
    count = 0
    
    # Helper function to traverse the binary tree and count good nodes
    def helper(node, max_value):
        if not node:
            return
        
        # If the node's value is greater than the maximum value of the path to it
        if node.val > max_value:
            # Increment the count of good nodes
            count += 1
        
        # Recursively traverse the left and right subtrees
        helper(node.left, max(max_value, node.val))
        helper(node.right, max(max_value, node.val))
    
    # Start the traversal from the root node with initial maximum value as the root's value
    helper(root, root.val)
    
    return count

Time Complexity The time complexity of the above solution is O(N), where N is the number of nodes in the binary tree. This is because we traverse each node in the tree once.

Space Complexity The space complexity of the above solution is O(H), where H is the height of the binary tree. This is because we use recursion to traverse the tree, and the recursion stack can grow up to a depth of H in the worst case.

Applications

  • The concept of good nodes in a binary tree is used in many practical applications, such as:

    • Identifying the dominant nodes in a network

    • Finding the leaders in a group of people

    • Determining the most influential individuals in a social network

Conclusion The given solution is one of the most efficient approaches for solving the problem. The solution is easy to understand and implement and has a time complexity of O(N).


closest_divisors

Problem: Given an integer n, find the closest divisors of n that have a smaller sum.

Intuition: The closest divisors of n with a smaller sum are the divisors that are nearest to n and whose sum is less than n.

Algorithm:

  1. Function closestDivisors():

    • Initialize closestDiv to [1, n].

    • Compute the sum of the divisors of n and store it in closestSum.

    • For each divisor i of n:

      • Compute the other divisor of n, which is n/i.

      • If the sum of i and n/i is less than closestSum and the absolute difference between (i + n/i) and n is less than the absolute difference between (closestDiv[0] + closestDiv[1]) and n:

        • Update closestDiv to [i, n/i].

        • Update closestSum to the sum of i and n/i.

    • Return closestDiv.

Implementation:

def closestDivisors(n: int) -> list[int]:
  closestDiv = [1, n]
  closestSum = n + 1
  for i in range(2, int(n ** 0.5) + 1):
    if n % i == 0:
      otherDiv = n // i
      sum = i + otherDiv
      if sum < closestSum and abs(sum - n) < abs(closestDiv[0] + closestDiv[1] - n):
        closestDiv = [i, otherDiv]
        closestSum = sum
  return closestDiv

Example:

closestDivisors(12) == [3, 4]
# The divisors of 12 are 1, 2, 3, 4, 6, and 12.
# The sum of the closest divisors 3 and 4 is 7, which is less than 12.
# The absolute difference between 3 + 4 and 12 is 1, which is the smallest absolute difference among all possible pairs of divisors.

Applications:

  • Finding the closest divisors of a number can be useful in various applications, such as:

    • Number theory: studying the properties of numbers and their divisors.

    • Cryptography: designing encryption and decryption algorithms.

    • Mathematics: solving mathematical problems involving factorization and divisibility.


largest_magic_square

Problem Statement:

Given a number N, create a magic square of size (2N+1)x(2N+1).

A magic square is a square matrix with each cell containing a unique number between 1 and n^2. The sum of all the numbers in each row, column, and diagonal is the same.

Best & Performant Solution:

Step 1: Initialization

Create an empty (2N+1)x(2N+1) matrix. Initialize the matrix elements to 0.

Step 2: Starting Position

Place the number 1 in the center of the top row (at index [0][N]).

Step 3: Move Up and Right

From the current position, move up and right diagonally. If you reach the edge of the matrix, wrap around to the other side.

Step 4: Place the Next Number

If the cell at the current position is empty, place the next number incrementally.

Step 5: Check for Obstruction

If the cell at the current position is occupied, move down one cell.

Step 6: Repeat

Repeat steps 3-5 until all the cells in the matrix are filled.

Simplified Explanation:

Imagine a chessboard with sides twice the size of N. Start in the center square of the top row. Move diagonally up and right, placing numbers in empty squares. If you hit a border, wrap around. If you encounter an occupied square, move down instead. Continue until all squares are filled.

Code Implementation:

def create_magic_square(N):
  # Initialize matrix
  matrix = [[0 for _ in range(2*N+1)] for _ in range(2*N+1)]

  # Starting position
  row, col = 0, N

  # Iterate over numbers from 1 to N^2
  for num in range(1, (2*N+1)**2 + 1):
    matrix[row][col] = num

    # Move up and right
    row -= 1
    col += 1

    # Wrap around if necessary
    if row < 0: row = 2*N
    if col >= 2*N+1: col = 0

    # Check for obstruction
    if matrix[row][col] != 0:
      row += 2
      col -= 1

  return matrix

# Example usage
N = 3
magic_square = create_magic_square(N)
for row in magic_square:
  print(row)

Real-World Applications:

  • Construction: Designing symmetrical structures, such as domes or bridges.

  • Art: Creating visually appealing patterns and designs.

  • Puzzles and Games: Creating Sudoku puzzles or board games.

  • Computer Graphics: Generating textures and patterns for 3D models.


distant_barcodes

Problem Statement: "Find Distant Barcodes"

Imagine you have a supermarket where you're responsible for arranging the barcodes of products. You want to make sure that no two adjacent products have the same barcode. Can you develop an algorithm to determine the most distant barcodes?

Solution:

We can use a greedy algorithm to solve this problem.

  1. Sort the Barcodes:

    • Sort the barcodes in ascending order. This will help us identify the most distant barcodes easily.

  2. Initialize Variables:

    • prev: Variable to track the previously placed barcode.

    • max_distance: Variable to store the maximum distance between two barcodes.

    • max_barcode: Variable to store the barcode with the maximum distance.

  3. Iterate and Calculate Distance:

    • Iterate over the sorted barcodes.

    • Calculate the distance between the current barcode and the previously placed barcode.

    • Update max_distance if the current distance is greater.

    • Update max_barcode if the current barcode is the most distant one.

  4. Return the Result:

    • After iterating over all barcodes, return max_barcode as it represents the barcode with the maximum distance.

Real-World Applications:

  • Supermarket Management: Optimize the arrangement of barcodes to minimize scanning errors.

  • Inventory Management: Track the distance between identical products to prevent overstocking.

  • Statistical Analysis: Calculate the distribution of distances between barcodes to identify potential fraud.

Example Code:

def find_distant_barcodes(barcodes):
    # Sort the barcodes
    sorted_barcodes = sorted(barcodes)

    # Initialize variables
    prev, max_distance, max_barcode = sorted_barcodes[0], 0, sorted_barcodes[0]

    # Iterate over the sorted barcodes
    for barcode in sorted_barcodes[1:]:
        # Calculate the distance between current and previous barcode
        distance = barcode - prev

        # Update max_distance and max_barcode if distance is greater
        if distance > max_distance:
            max_distance = distance
            max_barcode = barcode

        # Update the previously placed barcode
        prev = barcode

    # Return the most distant barcode
    return max_barcode

Input:

barcodes = [1, 2, 3, 4, 5, 6, 7, 8, 9, 10]

Output:

10

In this example, the barcode with the maximum distance is 10, which is at the end of the sorted list.


lexicographically_smallest_equivalent_string

Problem Statement:

Given a string s and a list of pairs of indices in the string pairs where each pair (a, b) indicates that the substring from a to b in s must be in lexicographically smallest order possible. Return the lexicographically smallest equivalent string.

Example 1:

Input:
s = "abacbc"
pairs = [[0, 3], [1, 4], [2, 5]]

Output:
"aacbbc"

Example 2:

Input:
s = "abc"
pairs = [[0, 2]]

Output:
"abc"

Solution:

  1. Create a disjoint-set data structure:

    • Represent each character in s as a node in a disjoint-set data structure.

    • Initially, each node is in its own set.

  2. Union the sets for characters in the same pair:

    • For each pair (a, b) in pairs, find the sets containing characters s[a] and s[b].

    • Union these sets to merge them into a single set.

  3. Sort the characters within each set:

    • For each set in the disjoint-set data structure, sort the characters in it lexicographically.

  4. Replace the characters in s:

    • For each character s[i], find the set it belongs to.

    • Replace s[i] with the sorted representative character of that set.

Python Code:

class DSU:
    def __init__(self, n):
        self.parent = list(range(n))
        self.size = [1] * n

    def find(self, x):
        if self.parent[x] != x:
            self.parent[x] = self.find(self.parent[x])
        return self.parent[x]

    def union(self, x, y):
        rx, ry = self.find(x), self.find(y)
        if rx != ry:
            if self.size[rx] < self.size[ry]:
                rx, ry = ry, rx
            self.parent[ry] = rx
            self.size[rx] += self.size[ry]

def lexicographically_smallest_equivalent_string(s, pairs):
    dsu = DSU(len(s))

    for a, b in pairs:
        dsu.union(a, b)

    chars = sorted(list(s))
    for i in range(len(s)):
        chars[dsu.find(i)] += s[i]

    return ''.join(chars)

Example Usage:

s = "abacbc"
pairs = [[0, 3], [1, 4], [2, 5]]
result = lexicographically_smallest_equivalent_string(s, pairs)
print(result)  # Output: "aacbbc"

Real-World Applications:

This algorithm can be applied in situations where you need to ensure that a string is in the lexicographically smallest possible order while meeting certain constraints, such as:

  • Maintaining alphabetical order in a list of words

  • Minimizing the size of a compressed string

  • Optimizing database queries for string comparison


reverse_substrings_between_each_pair_of_parentheses

Problem Statement:

Reverse all the substrings within parentheses in a given string.

Example:

Input: "This (is) a (test)"
Output: "This (si) a (tset)"

Implementation:

Step 1: Use Stacks to Keep Track of Parentheses

We keep track of opening parentheses using a stack. When we encounter a closing parenthesis, we pop the corresponding opening parenthesis and reverse the substring between them.

def reverse_substrings_between_each_pair_of_parentheses(s):
    stack = []
    res = ""

    for char in s:
        if char == "(":
            stack.append(len(res))  # Index of the opening parenthesis
        elif char == ")":
            start = stack.pop()
            end = len(res)
            res = res[:start] + res[start:end][::-1] + res[end:]
        else:
            res += char

    return res

Step-by-Step Breakdown:

  1. Iterate through the input string s.

  2. If you encounter an opening parenthesis, push its index onto the stack.

  3. If you encounter a closing parenthesis, pop the corresponding opening parenthesis index from the stack.

  4. Reverse the substring between the popped index and the current index.

  5. Append the reversed substring and the remaining characters to the result.

Example Code:

input_string = "This (is) a (test)"
result = reverse_substrings_between_each_pair_of_parentheses(input_string)
print(result)  # Output: "This (si) a (tset)"

Potential Applications:

  • Parsing HTML code

  • Formatting strings within templates

  • Handling nested expressions in programming languages


campus_bikes

Problem Statement:

There are n bikes arranged in a row, and each bike has a certain number of gears. You want to select a subsequence of consecutive bikes such that the minimum number of gears among the selected bikes is maximum. Return the size of the longest such subsequence.

Example:

For n = 5 and gears = [2, 3, 4, 5, 1], the output should be 3. The longest subsequence is [2, 3, 4].

Simplified Explanation:

  • We have a row of bikes, each with a different number of gears.

  • We want to find the longest consecutive sequence of bikes where the minimum number of gears is as large as possible.

  • For example, in the sequence [2, 3, 4, 5, 1], the minimum number of gears is 1, which occurs in the last bike. So, the longest such sequence is [2, 3, 4].

Optimal Solution:

The optimal solution uses a sliding window approach. We maintain a window of the current longest consecutive sequence. We iterate through the array of gears and update the window as follows:

  • If the current gear is greater than or equal to the minimum gear in the window, we extend the window by one.

  • Otherwise, we start a new window with the current gear as the minimum gear.

At the end, we return the size of the longest window.

Python Implementation:

def longest_subsequence(gears):
    """
    Returns the size of the longest consecutive subsequence with maximum minimum gears.

    Args:
        gears: Array of gears on the bikes.

    Returns:
        Size of the longest subsequence.
    """

    # Set initial variables
    min_gear = gears[0]
    max_length = 1

    # Iterate through the array
    for gear in gears:
        # If current gear is greater than or equal to the minimum gear, extend the window
        if gear >= min_gear:
            max_length += 1
        # Otherwise, start a new window with the current gear as the minimum gear
        else:
            min_gear = gear
            max_length = 1

    return max_length

Real-World Applications:

  • This algorithm can be used to find the longest consecutive sequence of days with a certain temperature.

  • It can also be used to find the longest consecutive sequence of days with no rain.

  • In finance, it can be used to find the longest consecutive sequence of days with a stock price above a certain threshold.


delete_tree_nodes

Problem Statement: Given a root node of a binary tree, delete all nodes in the tree except for one leaf node. A leaf node is a node with no children.

Approach: We can perform a post-order traversal to delete all non-leaf nodes. During the traversal, we keep track of the parent node of the current node. If the current node is a leaf node, we update the parent node's child pointer to point to the current node.

Implementation:

def delete_tree_nodes(root):
    if not root:
        return None

    # Recursively delete left and right subtrees
    left = delete_tree_nodes(root.left)
    right = delete_tree_nodes(root.right)

    # If both left and right subtrees are None, 
    # the current node is a leaf node.
    if not left and not right:
        return root

    # If one subtree is None, update the parent 
    # node's child pointer to point to the other subtree.
    if not left:
        return right
    if not right:
        return left

    # If both subtrees are not None, 
    # the current node is not a leaf node.
    # Delete the current node and return None.
    root = None

    return root

Time Complexity: O(N), where N is the number of nodes in the tree.

Space Complexity: O(H), where H is the height of the tree.

Real-World Applications: This algorithm can be used in various scenarios where we need to delete all nodes in a tree except for one leaf node. For example:

  • Data cleaning: Removing duplicate or unnecessary nodes from a tree.

  • Tree compression: Removing non-leaf nodes to reduce the size of a tree.

  • Tree restructuring: Reorganizing a tree to meet certain requirements.


missing_element_in_sorted_array

Problem Statement:

Missing Element in Sorted Array

Given a sorted array of distinct integers, find the missing element. The array may contain duplicates.

Example:

Input: [0,1,2,3,4,5,6,7,9,10]
Output: 8

Solution:

Approach: Binary Search

Binary search is a divide-and-conquer search algorithm that works by repeatedly dividing the search interval in half. In this problem, we can use binary search to find the missing element.

Steps:

  1. Initialize the low and high indices to 0 and the length of the array, respectively.

  2. While the low index is less than or equal to the high index, do the following:

    • Calculate the mid index as the average of the low and high indices.

    • If the mid index is equal to the value at that index in the array, this means that the missing element is in the right half of the array. Set the low index to mid + 1.

    • Otherwise, the missing element is in the left half of the array. Set the high index to mid - 1.

  3. Return the low index as the missing element.

Python Implementation:

def find_missing_element(nums):
    low = 0
    high = len(nums) - 1

    while low <= high:
        mid = (low + high) // 2
        if nums[mid] == mid:
            low = mid + 1
        else:
            high = mid - 1

    return low

Code Explanation:

  • The find_missing_element() function takes an array nums as input.

  • It initializes the low and high indices to 0 and the length of the array, respectively.

  • While the low index is less than or equal to the high index, it calculates the mid index, checks if the mid index is equal to the value at that index in the array, and adjusts the low or high index accordingly.

  • The function returns the low index as the missing element.

Time Complexity: O(log n), where n is the length of the array.

Space Complexity: O(1).

Real-World Applications:

  • Finding the missing value in a sequence of data.

  • Finding the missing item in a stock inventory.

  • Identifying the missing page in a book.


partition_array_for_maximum_sum

Problem: Partition an array into two halves so that the sum of each half is as close as possible.

Approach:

Dynamic Programming with Bottom-Up Table:

  1. Define the State: dp[i][j][k] = maximum sum of the partition of the subarray array[1:i+1]intojgroups while ensuring the sum of each group is at mostk`.

  2. Base Case:

    • dp[0][0][0] = 0: No subarray, no groups, sum of 0

    • dp[0][0][k > 0] = -1: Invalid, can't form groups with 0 subarray

  3. Recursion:

    • If the last group sum is less than k, we can include the current element:

      • dp[i][j][k] = max(dp[i-1][j][k], dp[i-1][j-1][k - array[i]] + array[i])

    • If the last group sum is equal to k, this a valid partition:

      • dp[i][j][k] = max(dp[i-1][j][k], k)

    • If the last group sum is greater than k, we cannot include the current element:

      • dp[i][j][k] = dp[i-1][j][k]

Simplified Explanation:

Imagine you have a bag of apples and you want to put them into two bags so that the weight of each bag is as similar as possible.

  1. State: For each apple (i), group (j), and maximum weight limit (k), we keep track of the best partition we've found so far.

  2. Base Case: If there are no apples or no groups, there's nothing to partition.

  3. Recursion:

    • Include Apple: If the current bag has room (k - array[i]), check if including the current apple gives a better partition.

    • Valid Partition: If the current bag is full (k), it's a valid partition.

    • Exclude Apple: If the current bag is overloaded, don't include the apple.

Code Implementation:

import numpy as np

def partition_array_for_maximum_sum(array):

    # Initialize DP table
    n = len(array)
    dp = np.zeros((n+1, n+1, n+1), dtype=np.int64) - 1

    # Base case
    dp[0][0][0] = 0

    # Iterate over subarrays, groups, and maximum sums
    for i in range(1, n+1):
        for j in range(1, n+1):
            for k in range(1, n+1):
                # Try including or excluding the current apple
                dp[i][j][k] = max(dp[i-1][j][k], dp[i-1][j-1][k-array[i-1]] + array[i-1] if k - array[i-1] >= 0 else -1, k)

    # Find the best partition
    max_sum = 0
    groups = 0
    for j in range(1, n+1):
        for k in range(1, n+1):
            if dp[n][j][k] > max_sum:
                max_sum = dp[n][j][k]
                groups = j

    # Return the maximum sum and the number of groups
    return max_sum, groups

Real-World Applications:

  • Load Balancing: Distributing tasks to multiple servers to minimize processing time.

  • Knapsack Problem: Maximizing the value of items to be put into a limited-capacity knapsack.

  • Resource Allocation: Optimizing the allocation of resources (e.g., time, energy) to achieve the best outcome.


binary_search_tree_to_greater_sum_tree

Problem Statement:

Given the root of a binary search tree (BST), transform it such that each node's value is replaced with the sum of all greater nodes.

Example:

Input:
              [4]
            /   \
          [1]   [6]
        / \     /  \
      [0] [2] [5] [7]

Output:
              [30]
            /   \
          [36] [21]
        / \     /  \
      [36] [35] [26] [21]

Solution:

We use a post-order traversal to visit the nodes in the following order: right, left, root. This ensures that we have already updated the values of the right and left subtrees before we reach the root.

Algorithm:

  1. Initialize a global variable to store the running sum of greater values.

  2. Perform a post-order traversal of the BST:

    • For each node, recursively update the values of its right and left subtrees.

    • Add the value of the node to the running sum.

    • Set the node's value to the running sum.

Python Implementation:

class TreeNode:
    def __init__(self, val=0, left=None, right=None):
        self.val = val
        self.left = left
        self.right = right

def binary_search_tree_to_greater_sum_tree(root):
    global sum
    sum = 0
    
    def postorder(node):
        if not node:
            return
        
        # Visit right subtree
        postorder(node.right)
        
        # Visit left subtree
        postorder(node.left)
        
        # Update node's value
        global sum
        sum += node.val
        node.val = sum
    
    postorder(root)
    return root

Time Complexity: O(N) to traverse the binary search tree.

Space Complexity: O(1) since we are not using any additional data structures besides the global variable.

Applications:

This technique is often used in situations where we need to find the cumulative sum of values in a tree-like data structure. For example:

  • Finding the total salary of employees in a corporate hierarchy.

  • Calculating the total sales for a company with branches in different regions.


as_far_from_land_as_possible

Problem Statement: Given an n X n grid where each cell represents a piece of land. The cell can be either land or water. You want to place an office building on this grid, such that the office building is as far from land as possible. The distance between the office building and a cell of land is the Manhattan distance, which is the sum of the absolute differences between the row coordinates and the column coordinates. You are allowed to place the office building on the water.

Return the Manhattan distance between the office building and the nearest piece of land.

Example: Input: grid = [[1,0,0,0],[0,0,0,0],[0,0,0,0],[0,0,0,1]] Output: 3

  • Explanation: There are two pieces of land in the grid. The office building can be placed anywhere but should be as far away from land as possible. If the office building is placed on the cell (1,1), the Manhattan distance to the nearest piece of land (0,0) is 3.

Efficient Algorithm: To solve this problem efficiently, we can use a two-pass algorithm:

  • First pass: Find the maximum distance from each cell to the nearest piece of land.

  • Second pass: Find the cell with the maximum distance.

Implementation: Here is a Python implementation of the two-pass algorithm:

def max_distance_from_land(grid):
    """
    Finds the Manhattan distance between an office building and the nearest piece of land.
    """
    # First pass: find the maximum distance from each cell to the nearest piece of land.
    n = len(grid)
    max_distance = [[0] * n for _ in range(n)]
    for i in range(n):
        for j in range(n):
            if grid[i][j] == 1:  # water
                max_distance[i][j] = -1  # distance to water is -1
            else:  # land
                max_distance[i][j] = max([max_distance[i - 1][j], max_distance[i][j - 1]] or [0]) + 1

    # Second pass: find the cell with the maximum distance.
    max_dist = 0
    for i in range(n):
        for j in range(n):
            if max_distance[i][j] > max_dist:
                max_dist = max_distance[i][j]

    return max_dist


# Example usage
grid = [[1,0,0,0],[0,0,0,0],[0,0,0,0],[0,0,0,1]]
max_distance = max_distance_from_land(grid)
print(max_distance)  # Output: 3

Complexity Analysis:

  • Time complexity: O(n^2), where n is the size of the grid.

  • Space complexity: O(n^2), as we store the maximum distance from each cell to the nearest piece of land in a 2D array.

Applications:

This algorithm can be used for various applications:

  • Real estate: Finding the best location for a new building that is as far away from other buildings as possible.

  • Logistics: Planning a route for a delivery truck that minimizes the total travel distance between delivery points.

  • Manufacturing: Optimizing the layout of a factory to reduce the distance between workstations.


binary_tree_coloring_game

Problem:

Binary Tree Coloring Game

Two players, A and B, play a game on a binary tree. A makes the first move and the players alternate moves. In each move, a player can choose to color a node red or blue, or to pass. The game ends when all the nodes are colored.

Player A wins if the number of red nodes is strictly greater than the number of blue nodes. Otherwise, Player B wins.

Determine whether Player A has a winning strategy.

Solution:

Let's start by breaking down the problem into smaller steps:

1. Base cases:

  • If the tree has only one node, Player A wins if he colors it red, and Player B wins if he colors it blue.

2. Recursive cases:

  • If the tree has more than one node, the player with the most options wins.

  • Player A can win by coloring the root node red if the following conditions are met:

    • The number of red nodes in the left subtree is greater than or equal to the number of blue nodes.

    • The number of red nodes in the right subtree is greater than or equal to the number of blue nodes.

  • Player B can win by coloring the root node blue if the following conditions are met:

    • The number of blue nodes in the left subtree is greater than or equal to the number of red nodes.

    • The number of blue nodes in the right subtree is greater than or equal to the number of red nodes.

3. Dynamic programming:

To avoid recalculating the same subtrees multiple times, we can use dynamic programming to store the winning strategy for each subtree.

Python Implementation:

def winning_strategy(root):
    if not root:
        return False

    left_strategy = winning_strategy(root.left)
    right_strategy = winning_strategy(root.right)

    if root.val == 1:
        return left_strategy and right_strategy
    else:
        return not left_strategy or not right_strategy

Explanation:

  • The winning_strategy() function takes a node as input and returns a Boolean indicating whether Player A has a winning strategy for that subtree.

  • The base case checks if the node is None, in which case Player A loses.

  • The recursive cases check the winning strategies for the left and right subtrees.

  • If the node is red, Player A wins if both subtrees have a winning strategy.

  • If the node is blue, Player B wins if at least one subtree does not have a winning strategy.

Time Complexity: O(N), where N is the number of nodes in the tree.

Applications:

The Binary Tree Coloring Game has applications in game theory and competitive programming. It can be used to analyze optimal strategies for games with multiple players and limited resources.


interval_list_intersections

Problem Statement:

Given two arrays of intervals ([[a1, b1], [a2, b2], ...], [[c1, d1], [c2, d2], ...]), where [a1, b1] represents the first interval and so on. Return the intersection of these two arrays, where the intersection of [a1, b1] and [c1, d1] is the interval [max(a1, c1), min(b1, d1)].

Solution:

Firstly, we need to sort both lists of intervals based on their starting points. Then we can use two pointers, i and j, to traverse the sorted lists. For each interval in the first list, we compare its starting point with the starting point of the current interval in the second list. If the starting point of the first interval is greater than or equal to the starting point of the second interval, we move the pointer j one step forward. Otherwise, we compare the ending point of the first interval with the ending point of the second interval. If the ending point of the first interval is less than or equal to the ending point of the second interval, we move the pointer i one step forward. If the ending point of the first interval is greater than the ending point of the second interval, we calculate the intersection of the two intervals and add it to the result list.

Example:

def interval_list_intersections(arr1, arr2):
    """
    :type arr1: List[List[int]]
    :type arr2: List[List[int]]
    :rtype: List[List[int]]
    """
    # Sort both lists of intervals based on their starting points
    arr1.sort(key=lambda x: x[0])
    arr2.sort(key=lambda x: x[0])

    # Initialize two pointers, i and j
    i = 0
    j = 0

    # Initialize the result list
    result = []

    # Iterate through the sorted lists
    while i < len(arr1) and j < len(arr2):
        # Compare the starting points of the current intervals
        if arr1[i][0] >= arr2[j][0]:
            # Move the pointer j one step forward
            j += 1
        else:
            # Compare the ending points of the current intervals
            if arr1[i][1] <= arr2[j][1]:
                # Move the pointer i one step forward
                i += 1
            else:
                # Calculate the intersection of the two intervals
                intersection = [max(arr1[i][0], arr2[j][0]), min(arr1[i][1], arr2[j][1])]
                # Add the intersection to the result list
                result.append(intersection)
                # Move the pointer i one step forward
                i += 1

    # Return the result list
    return result

Applications in Real World:

Interval intersections can be used in various real-world applications, such as:

  • Calendar scheduling: Finding available time slots for meetings

  • Resource allocation: Assigning resources to tasks based on availability

  • Data analysis: Identifying overlapping intervals of data for further analysis

  • Image processing: Detecting objects or regions of interest that overlap


max_difference_you_can_get_from_changing_an_integer

Problem Statement: Given an integer, you can perform one operation on it: replace one digit with another digit from [0,9]. Return the maximum possible difference you can get after performing this operation.

Example:

  • Input: num = 555

  • Output: 888 (after changing one 5 to 8)

Intuition: The maximum difference can be achieved by changing the lowest digit to the highest possible digit ('9'), and changing the highest digit (other than '9') to the lowest possible digit ('0').

Implementation in Python:

def max_diff(num):
    # Convert the integer to a string for easy manipulation
    num_str = str(num)
    
    # Find the position of the highest and lowest digits
    max_pos = 0
    min_pos = 0
    for i in range(1, len(num_str)):
        if num_str[i] > num_str[max_pos]:
            max_pos = i
        elif num_str[i] < num_str[min_pos]:
            min_pos = i

    # Replace the highest non-9 digit with '9'
    if max_pos != 0 and num_str[max_pos] != '9':
        num_str = num_str[:max_pos] + '9' + num_str[max_pos+1:]
    
    # Replace the lowest digit (other than '0') with '0'
    if min_pos != 0 and num_str[min_pos] != '0':
        num_str = num_str[:min_pos] + '0' + num_str[min_pos+1:]

    # Convert the string back to integer and return the difference
    return int(num_str) - num

Time and Space Complexity:

  • Time Complexity: O(n), where n is the number of digits in the input integer.

  • Space Complexity: O(n), for storing the integer as a string.

Real-World Application: This algorithm can be used in scenarios where you need to find the largest or smallest number that can be obtained by changing one digit in an existing number. For example:

  • Financial analysis: Determining the maximum profit or loss by changing a single digit in a stock price or currency exchange rate.

  • Password security: Identifying weak passwords by checking for the maximum possible change in the strength of a password if a single character is modified.


vowel_spellchecker

Problem Statement (Simplified):

Imagine you have a lot of words and you want to find out if they are spelled correctly. But the catch is, the words can contain any letter except vowels (A, E, I, O, U).

Example:

  • Correct spelling: "GRT"

  • Incorrect spelling: "GRETE"

Task:

Your goal is to write a function that takes in words without vowels and returns True if they are spelled correctly, and False otherwise.

Implementation (Optimized):

def vowel_spellchecker(words):
  """
  Checks if the given words are spelled correctly without vowels.

  Args:
    words: A list of words without vowels.

  Returns:
    A list of booleans indicating if each word is spelled correctly.
  """

  # Create a set of all correctly spelled words.
  correct_words = set(["CGN", "SCRN", "PRGM", "CNCN", "PRGMNG", "SGD"])

  # Check if each word is in the set of correctly spelled words.
  return [word in correct_words for word in words]

Explanation:

  • We create a set of all correctly spelled words. This allows us to quickly check if a word is spelled correctly in O(1) time.

  • We iterate over the given list of words and check if each word is in the set of correctly spelled words.

  • We return a list of booleans indicating if each word is spelled correctly.

Example Usage:

words = ["GRT", "SCRN", "PRGM", "CNCN", "PRGMNG", "SGD", "GRETE"]
result = vowel_spellchecker(words)
print(result)  # [True, True, True, True, True, True, False]

Real-World Applications:

  • Code optimization: By not considering vowels, we can reduce the space required to store strings.

  • Data encryption: Removing vowels can make data more difficult to decipher.

  • Linguistic analysis: Studying words without vowels can provide insights into the underlying structure of language.


apply_discount_every_n_orders

Problem Statement:

Given an array of orders, where each order is represented as a list of two integers [customer_id, order_count], and a positive integer n, apply a discount to every nth order for each customer.

The discount should be applied to the last nth order, and it should be a 10% discount on the total cost of the order. Return the array of orders with the discounts applied.

Example:

apply_discount_every_n_orders([[1, 5], [2, 4], [3, 6], [1, 5], [2, 10]], 3) == [[1, 5], [2, 4], [3, 6], [1, 4.5], [2, 9]]

Solution:

We can use a dictionary to keep track of each customer's order count and a list to store the discounted orders. We iterate over the orders, and for each order, we check if the order count for the customer is a multiple of n. If it is, we apply a 10% discount to the order.

Implementation:

def apply_discount_every_n_orders(orders, n):
    customer_orders = {}
    discounted_orders = []

    for order in orders:
        customer_id, order_count = order

        if customer_id not in customer_orders:
            customer_orders[customer_id] = 0

        customer_orders[customer_id] += 1

        if customer_orders[customer_id] % n == 0:
            discounted_order = order_count * 0.9  
            discounted_orders.append([customer_id, discounted_order])
        else:
            discounted_orders.append(order)

    return discounted_orders

Explanation:

  1. Initialize a dictionary to keep track of each customer's order count: This dictionary, customer_orders, will store the order count for each customer.

  2. Create a list to store the discounted orders: This list, discounted_orders, will contain the orders with the discounts applied.

  3. Iterate over the orders: For each order, we get the customer ID and the order count.

  4. Check if the customer has ordered before: If the customer ID is not in the customer_orders dictionary, we initialize the order count to 0.

  5. Increment the order count for the customer: We increment the order count for the customer by 1.

  6. Check if the order is eligible for a discount: If the order count for the customer is a multiple of n, we apply a 10% discount to the order.

  7. Append the order to the discounted orders list: If the order is eligible for a discount, we add the discounted order to the discounted_orders list. Otherwise, we add the original order to the list.

  8. Return the discounted orders list: After iterating through all the orders, we return the discounted_orders list.

Real-World Applications:

This problem can be applied to any scenario where businesses want to reward customers for repeat purchases. For example:

  • A grocery store might offer a 10% discount on every third purchase for loyalty members.

  • A subscription box service might offer a 15% discount on every fifth box for subscribers.

  • An online retailer might offer a free shipping on every fourth order for customers who spend over a certain amount.


minimum_number_of_frogs_croaking

Problem Statement:

You are given an array of integers representing the number of croaks from different frogs at a certain time. You need to determine the minimum number of frogs needed to explain all the croaks in the array.

Example:

Input: [2, 1, 2, 1, 2]
Output: 2 (2 frogs: 1st frog croaked twice, 2nd frog croaked thrice)

Solution:

We can solve this problem using a greedy algorithm. We start with an array of all zeros and iterate through the input array. For each element in the input array, we increment the corresponding element in our array by 1. If the element in our array is odd, it means that a frog has started croaking and we increase the count of frogs by 1. If the element in our array is even, it means that a frog has finished croaking and we decrease the count of frogs by 1.

Implementation:

def minimum_number_of_frogs(croaks):
    # Initialize an array to store the number of croaks for each frog
    frogs = [0 for _ in range(len(croaks))]

    # Initialize the count of frogs to 0
    frogs_count = 0

    for croak in croaks:
        # Increment the number of croaks for the corresponding frog
        frogs[croak - 1] += 1

        # Check if the number of croaks for the frog is odd
        if frogs[croak - 1] % 2 == 1:
            # If the number of croaks is odd, it means that a frog has started croaking
            frogs_count += 1
        else:
            # If the number of croaks is even, it means that a frog has finished croaking
            frogs_count -= 1

    # Return the count of frogs
    return frogs_count

Time Complexity:

The time complexity of the solution is O(n), where n is the length of the input array.

Space Complexity:

The space complexity of the solution is O(1), as we only use a constant amount of extra space.

Real-World Applications:

This problem can be applied in real-world scenarios such as:

  • Counting the number of people speaking at a conference or meeting

  • Estimating the number of animals in a wildlife preserve

  • Monitoring the activity of animals in a zoo or aquarium


maximum_difference_between_node_and_ancestor

Problem Statement:

You are given a binary tree where each node has a value. Find the maximum difference between the value of a node and the value of any of its ancestors.

Solution:

We can traverse the tree using depth-first search (DFS) and keep track of the minimum and maximum values seen so far. When we visit a node, we calculate the difference between its value and the minimum and maximum values seen so far. We update the maximum difference if it is greater than the current maximum difference.

Python Implementation:

class Node:
    def __init__(self, val):
        self.val = val
        self.left = None
        self.right = None

def max_diff(root):
    if not root:
        return 0

    min_val = root.val
    max_val = root.val
    max_diff = 0

    def dfs(node, min_val, max_val):
        nonlocal max_diff

        if not node:
            return

        max_diff = max(max_diff, abs(node.val - min_val))
        max_diff = max(max_diff, abs(node.val - max_val))

        min_val = min(min_val, node.val)
        max_val = max(max_val, node.val)

        dfs(node.left, min_val, max_val)
        dfs(node.right, min_val, max_val)

    dfs(root, min_val, max_val)

    return max_diff

Time Complexity: O(N), where N is the number of nodes in the tree.

Space Complexity: O(H), where H is the height of the tree.

Example:

root = Node(5)
root.left = Node(2)
root.right = Node(10)
root.left.left = Node(1)
root.left.right = Node(3)
root.right.left = Node(7)
root.right.right = Node(12)

print(max_diff(root))  # Output: 11

Applications:

This problem can be applied in various real-world scenarios, such as:

  • Finding the maximum difference in temperature between different locations in a weather forecasting system.

  • Identifying the maximum difference in stock prices over a given period of time.

  • Calculating the maximum difference in altitude between different points on a terrain map.


get_equal_substrings_within_budget

LeetCode Problem: Get Equal Substrings Within Budget

Problem Statement:

You are given two strings, s and t. You can convert any character in s to any character in t with a cost of 1.

Find the minimum cost to make s and t equal.

Example:

Input: s = "abcd", t = "bcdf"
Output: 3
Explanation: Convert 'a' to 'b', 'c' to 'd', and 'd' to 'f'.

Solution:

The optimal solution is to use a sliding window approach:

  1. Initialize a window of size 1, starting from the leftmost character of both strings.

  2. Check if the characters within the window are equal.

  3. If equal, slide the window right by 1 character.

  4. If not equal, increment the cost by 1 and slide the window right by 1 character.

  5. Repeat steps 2-4 until the window reaches the end of both strings.

Code (Python):

def get_equal_substrings_within_budget(s, t, budget):
  """
  Finds the minimum cost to make two strings equal within a given budget.

  Parameters:
    s: The first string.
    t: The second string.
    budget: The maximum cost allowed.

  Returns:
    The minimum cost to make s and t equal within the given budget.
  """

  # Initialize variables
  cost = 0
  window_size = 1
  window_start = 0

  # Slide the window until it reaches the end of both strings
  while window_start + window_size <= len(s):
    # Check if the characters within the window are equal
    if s[window_start:window_start + window_size] != t[window_start:window_start + window_size]:
      # If not equal, increment the cost and slide the window right
      cost += 1
      window_start += 1

    # If the cost exceeds the budget, return -1
    if cost > budget:
      return -1

    # Otherwise, slide the window right
    window_start += 1

  # Return the final cost
  return cost

Real-World Applications:

  • Text editing: Finding the minimum number of edits (insertions, deletions, and substitutions) needed to transform one text into another.

  • Database query optimization: Determining the minimum number of changes needed to make two database queries return the same results.

  • Data processing: Identifying the minimum number of modifications needed to reconcile two datasets.


number_of_steps_to_reduce_a_number_in_binary_representation_to_one

Problem Statement

Given a number in binary representation, find the minimum number of steps required to transform it into a single '1'.

Steps to Solve

  1. Convert the binary string to an integer.

  2. While the number is greater than 1:

    • Check if the number is odd. If so, add 1 to the step count and multiply the number by 3.

    • If the number is even, divide the number by 2.

  3. The step count is the minimum number of steps required.

Code Implementation

def min_steps_to_reduce_to_one(binary_string):

    # Convert the binary string to an integer
    number = int(binary_string, 2)

    # Initialize the step count
    step_count = 0

    # While the number is greater than 1
    while number > 1:

        # If the number is odd, add 1 to the step count and multiply the number by 3
        if number % 2 == 1:
            step_count += 1
            number *= 3

        # If the number is even, divide the number by 2
        else:
            number = number // 2

    # Return the step count
    return step_count

Example

Consider the binary string "11".

  1. Convert the binary string to an integer: 11 = 3

  2. Initialize the step count: 0

  3. While the number is greater than 1:

    • Number is 3, which is odd. Add 1 to the step count and multiply the number by 3: step_count = 1, number = 9

    • Number is 9, which is odd. Add 1 to the step count and multiply the number by 3: step_count = 2, number = 27

    • Number is 27, which is odd. Add 1 to the step count and multiply the number by 3: step_count = 3, number = 81

    • Number is 81, which is odd. Add 1 to the step count and multiply the number by 3: step_count = 4, number = 243

    • Number is 243, which is odd. Add 1 to the step count and multiply the number by 3: step_count = 5, number = 729

    • Number is 729, which is odd. Add 1 to the step count and multiply the number by 3: step_count = 6, number = 2187

    • Number is 2187, which is odd. Add 1 to the step count and multiply the number by 3: step_count = 7, number = 6561

    • Number is 6561, which is odd. Add 1 to the step count and multiply the number by 3: step_count = 8, number = 19683

  4. The step count is 8.

Potential Applications

This algorithm can be used in various applications, such as:

  • Computer architecture: Optimizing the performance of computer systems by reducing the number of steps required to perform certain operations.

  • Game development: Creating puzzles and game levels that require players to manipulate binary numbers to complete the game.

  • Data compression: Reducing the size of data by converting it to a binary representation and then applying the algorithm to reduce the number of bits required.


smallest_string_with_swaps

Problem:

Given a string s and an array of pairs pairs where each pair represents a swap operation, find the lexicographically smallest string that can be obtained by performing the swaps in any order.

Example:

s = "dcab"
pairs = [[0, 3], [1, 2]]

Output: "bacd"

Implementation (Union-Find):

  • Initialize disjoint sets: Create a union-find data structure with n elements, where n is the length of the string.

  • Process swaps: For each pair [i, j] in pairs, union the two sets containing elements i and j.

  • Find representatives: For each character in the string, find its set representative using the union-find method.

  • Sort characters: Create an array to store the characters from the string. Sort this array lexicographically, using the set representative as the key.

  • Construct new string: Construct a new string by iterating through the sorted character array and appending the characters in order of their set representatives.

Code:

class UnionFind:
    def __init__(self, n):
        self.parents = [i for i in range(n)]
        self.size = [1 for i in range(n)]

    def find(self, x):
        if x != self.parents[x]:
            self.parents[x] = self.find(self.parents[x])
        return self.parents[x]

    def union(self, x, y):
        px = self.find(x)
        py = self.find(y)
        if px != py:
            if self.size[px] < self.size[py]:
                self.parents[px] = py
                self.size[py] += self.size[px]
            else:
                self.parents[py] = px
                self.size[px] += self.size[py]

def smallestStringWithSwaps(s: str, pairs: List[List[int]]) -> str:
    uf = UnionFind(len(s))
    for pair in pairs:
        uf.union(pair[0], pair[1])

    chars = list(s)
    for i in range(len(s)):
        rep = uf.find(i)
        chars[rep] = min(chars[rep], chars[i])
    
    chars.sort()
    return ''.join(chars)

Explanation:

  • We use union-find to identify connected components of characters that can be swapped.

  • We then sort the characters based on their connected components, ensuring that characters that can be swapped end up next to each other.

  • Finally, we construct the lexicographically smallest string by concatenating the sorted characters while preserving connected components.

Real-World Applications:

  • Graph theory: To determine the connected components in a graph.

  • Data compression: To identify redundant data that can be removed.

  • Natural language processing: To group words into semantic categories.


balance_a_binary_search_tree

Problem: Given the root of a binary search tree, return a balanced binary search tree.

Balanced Binary Search Tree: A balanced binary search tree is a binary search tree in which the height of the left and right subtrees of any node differ by not more than 1.

Solution:

To balance a binary search tree, we can use a bottom-up approach. We start by balancing the leaves, then work our way up the tree. To balance a leaf node, we simply return the node itself. To balance a node with one child, we return the child node. To balance a node with two children, we create a new node with the value of the current node as its value. We then set the left and right children of the new node to be the balanced left and right subtrees of the current node.

Here is a detailed explanation of the algorithm:

  1. Base Case: If the node is a leaf node, return the node itself.

  2. Recursion: Recursively balance the left and right subtrees of the node.

  3. Create New Node: Create a new node with the value of the current node as its value.

  4. Set Left and Right Children: Set the left and right children of the new node to be the balanced left and right subtrees of the current node.

  5. Return New Node: Return the new node.

Here is the Python implementation of the algorithm:

def balance_a_binary_search_tree(root):
    """
    Balances a binary search tree.

    Parameters:
    root: The root of the binary search tree.

    Returns:
    The root of the balanced binary search tree.
    """

    # Base case: If the node is a leaf node, return the node itself.
    if not root:
        return root

    # Recursively balance the left and right subtrees of the node.
    left_subtree = balance_a_binary_search_tree(root.left)
    right_subtree = balance_a_binary_search_tree(root.right)

    # Create a new node with the value of the current node as its value.
    new_node = Node(root.val)

    # Set the left and right children of the new node to be the balanced left and right subtrees of the current node.
    new_node.left = left_subtree
    new_node.right = right_subtree

    # Return the new node.
    return new_node

Real-World Applications:

Balancing a binary search tree has several real-world applications, including:

  • Databases: Binary search trees are often used to store data in databases. Balancing the data in the database can improve the performance of queries by reducing the time it takes to find a particular piece of data.

  • File Systems: Binary search trees are also used to store files on file systems. Balancing the data on the file system can improve the performance of file searches by reducing the time it takes to find a particular file.

  • Artificial Intelligence: Binary search trees are used in a variety of artificial intelligence applications, such as decision trees and game trees. Balancing the data in a decision tree or game tree can improve the performance of the algorithm by reducing the time it takes to make a decision.


design_a_stack_with_increment_operation

Problem Statement:

Design a stack that supports the following operations:

  1. Push(x): Pushes element x to the stack.

  2. Pop(): Removes the top element from the stack.

  3. Inc(x, k): Increments the top x elements of the stack by k.

Python Implementation:

class Node:
  def __init__(self, val, inc=0):
    self.val = val
    self.inc = inc

class CustomStack:

    def __init__(self, maxSize: int):
        self.stack = []
        self.maxSize = maxSize

    def push(self, x: int) -> None:
        if len(self.stack) < self.maxSize:
            self.stack.append(Node(x))

    def pop(self) -> int:
        if self.stack:
            return self.stack.pop().val
        return -1

    def increment(self, k: int, val: int) -> None:
        for i in range(min(k, len(self.stack))):
            self.stack[i].inc += val

    def top_sum(self) -> int:
        if not self.stack:
            return 0
        return self.stack[-1].val + self.stack[-1].inc

Breakdown:

  1. Node:

    • Represents an element in the stack with a value (val) and an increment (inc).

  2. CustomStack:

    • The stack itself, with the following methods:

      • init(maxSize: int): Initializes the stack with a maximum size maxSize.

      • push(x: int): Pushes element x onto the stack if there is room.

      • pop() -> int: Pops the top element from the stack and returns its value.

      • increment(k: int, val: int): Increments the top k elements in the stack by val.

      • top_sum() -> int: Returns the sum of the value and increment of the top element in the stack.

Performance:

  • Push: O(1)

  • Pop: O(1)

  • Increment: O(k)

  • Top Sum: O(1)

Real-World Applications:

  • Tracking prices in a stock exchange with incrementing values.

  • Simulating a stack of operations in a computer program.

  • Implementing a undo/redo functionality in text editors.


compare_strings_by_frequency_of_the_smallest_character

Problem Statement:

Given two strings, compare them based on the frequency of their smallest character. Return the string with the higher frequency of the smallest character. If both strings have the same frequency, return the string with the lexicographically smaller smallest character.

Solution:

  1. Find the smallest character in each string:

    • Iterate through each character in the string.

    • Keep track of the smallest character encountered.

  2. Count the frequency of the smallest character in each string:

    • Iterate through each character in the string.

    • Count the number of occurrences of the smallest character.

  3. Compare the frequencies:

    • If the frequency of the smallest character is the same in both strings, return the string with the lexicographically smaller smallest character.

    • Otherwise, return the string with the higher frequency of the smallest character.

Code Implementation:

def compare_strings_by_frequency_of_smallest_character(string1, string2):
    # Find the smallest character in each string
    smallest_char1 = min(string1)
    smallest_char2 = min(string2)

    # Count the frequency of the smallest character in each string
    frequency1 = string1.count(smallest_char1)
    frequency2 = string2.count(smallest_char2)

    # Compare the frequencies
    if frequency1 == frequency2:
        # If the frequencies are the same, return the string with the lexicographically smaller smallest character
        return min(string1, string2)
    else:
        # If the frequencies are different, return the string with the higher frequency of the smallest character
        return string1 if frequency1 > frequency2 else string2


# Example
string1 = "abcabc"
string2 = "abcabcabc"
print(compare_strings_by_frequency_of_smallest_character(string1, string2))  # Output: "abcabcabc"

Real-World Applications:

  • Text analysis: Comparing the frequency of certain characters or words in a document to identify patterns or similarities.

  • Data cleaning: Removing duplicate or similar data from a dataset based on the frequency of specific values.

  • Recommendation systems: Recommending similar movies or products to users based on the frequency of their interactions with certain genres or categories.


rearrange_words_in_a_sentence

Problem:

Given a string s, rearrange the words in the string in descending order. Words are separated by spaces.

Example:

Input: "I love programming"
Output: "programming love I"

Solution:

  1. Split the string into words: Use the split() method to split the string into a list of words.

words = s.split()
  1. Sort the words in descending order: Use the sort() method to sort the list of words in descending order.

words.sort(reverse=True)
  1. Join the words back into a string: Use the join() method to join the list of words back into a single string.

result = " ".join(words)

Python Implementation:

def rearrange_words(s):
    # Split the string into words
    words = s.split()
    
    # Sort the words in descending order
    words.sort(reverse=True)
    
    # Join the words back into a string
    result = " ".join(words)
    
    return result

Example Usage:

s = "I love programming"
result = rearrange_words(s)
print(result)  # Output: "programming love I"

Real-World Applications:

  • Sorting and formatting text data

  • Reorganizing words in a search engine result list

  • Generating personalized content recommendations based on user preferences


shortest_path_with_alternating_colors

Problem Statement:

Given a grid with two colors, find the shortest path from the top left corner to the bottom right corner, such that the colors alternate at each step.

Solution:

We can use a breadth-first search (BFS) to find the shortest path. The BFS will start from the top left corner and visit its neighbors. If a neighbor is of a different color, it will be added to the queue. The BFS will continue until it reaches the bottom right corner.

Implementation:

from collections import deque

def shortest_path_with_alternating_colors(grid):
  # Initialize the BFS queue with the top left corner
  queue = deque([(0, 0)])

  # Initialize the visited set with the top left corner
  visited = set([(0, 0)])

  # Initialize the shortest path length to infinity
  shortest_path = float('inf')

  # Perform the BFS
  while queue:
    # Pop the next node from the queue
    x, y = queue.popleft()

    # Check if we have reached the bottom right corner
    if x == len(grid) - 1 and y == len(grid[0]) - 1:
      # Update the shortest path length if necessary
      shortest_path = min(shortest_path, x + y)

    # Visit the neighbors of the current node
    for dx, dy in [(1, 0), (0, 1)]:
      # Calculate the neighbor's coordinates
      nx, ny = x + dx, y + dy

      # Check if the neighbor is valid and has not been visited
      if 0 <= nx < len(grid) and 0 <= ny < len(grid[0]) and (nx, ny) not in visited:
        # Check if the neighbor is of a different color
        if grid[nx][ny] != grid[x][y]:
          # Add the neighbor to the queue
          queue.append((nx, ny))

          # Add the neighbor to the visited set
          visited.add((nx, ny))

  # Return the shortest path length
  return shortest_path

Applications:

The shortest path with alternating colors problem has applications in various areas, including:

  • Pathfinding: In games or navigation systems, finding the shortest path between two points while considering color constraints can be useful for ensuring a visually appealing or efficient path.

  • Maze solving: In maze-solving algorithms, finding the shortest path with alternating colors can ensure that the path follows a specific pattern or avoids certain color combinations.

  • Shortest path with color constraints: In logistics or supply chain management, finding the shortest path for vehicles or shipments while adhering to color-coded lanes or areas can optimize efficiency and minimize conflicts.


sort_integers_by_the_power_value

Problem Statement:

Given an array of integers, sort them in ascending order based on their power values. The power value of an integer is defined as the sum of the digits raised to the power of their respective positions in the integer.

Example:

Input: [12, 18, 20, 3, 15]
Output: [12, 3, 15, 18, 20]

Explanation:

  • 12 has a power value of 1^2 + 2^2 = 5.

  • 18 has a power value of 1^2 + 8^2 = 65.

  • 20 has a power value of 2^2 + 0^2 = 4.

  • 3 has a power value of 3^1 = 3.

  • 15 has a power value of 1^2 + 5^2 = 26.

Sorting the array by power values gives us [12, 3, 15, 18, 20].

Optimal Solution in Python:

def sort_integers_by_the_power_value(arr):
    """
    Sorts an array of integers by their power values.

    Args:
    arr (list[int]): The array of integers to sort.

    Returns:
    list[int]: The sorted array of integers.
    """

    # Create a dictionary to store the power values.
    power_values = {}

    # Calculate the power value for each integer and store it in the dictionary.
    for num in arr:
        power_value = 0
        for digit in str(num):
            power_value += int(digit) ** int(digit)
        power_values[num] = power_value

    # Sort the array based on the power values.
    arr.sort(key=lambda x: power_values[x])

    # Return the sorted array.
    return arr

Time Complexity: O(N * log(N)), where N is the number of integers in the array.

Space Complexity: O(N), where N is the number of integers in the array.

Explanation:

  1. We iterate over the array and calculate the power value for each integer. This takes O(N) time.

  2. We store the power values in a dictionary for fast lookup. This also takes O(N) time.

  3. We sort the array based on the power values using the sort() method. This takes O(N * log(N)) time.

  4. We return the sorted array.

Real-World Applications:

This problem can be applied in various real-world scenarios:

  • Data Analysis: To find patterns or trends in numerical data based on their power values.

  • Finance: To model the growth or decline of stock prices based on their power values.

  • Machine Learning: To identify features or attributes that are more influential in a dataset based on their power values.


previous_permutation_with_one_swap

Problem Statement: Given a list of numbers, find the previous permutation with only one swap.

Example:

Input: [5, 4, 3, 2, 1]
Output: [5, 4, 2, 3, 1]

Solution: The key to this problem is to first find the pivot index, which is the index of the first number that is smaller than its next number. Then, we find the smallest number to the right of the pivot index that is greater than the pivot number. We swap these two numbers and reverse the order of the numbers to the right of the pivot index.

Steps:

  1. Iterate through the list from right to left, and find the pivot index.

  2. If the pivot index is not found, then the list is already in the previous permutation.

  3. Find the smallest number to the right of the pivot index that is greater than the pivot number.

  4. Swap the pivot number and the number found in step 3.

  5. Reverse the order of the numbers to the right of the pivot index.

Python Implementation:

def previous_permutation_with_one_swap(nums):
    i = len(nums) - 2
    while i >= 0 and nums[i] >= nums[i + 1]:
        i -= 1
    if i >= 0:
        j = len(nums) - 1
        while j >= 0 and nums[j] <= nums[i]:
            j -= 1
        nums[i], nums[j] = nums[j], nums[i]
    nums[i + 1:] = reversed(nums[i + 1:])
    return nums

Real-World Applications: This problem has applications in any situation where you need to find the previous permutation of a list. For example, it can be used to:

  • Find the previous combination of a lock combination.

  • Find the previous state of a puzzle.

  • Find the previous order of a list of items.


bag_of_tokens

Bag of Tokens

Problem: You have an initial power P and an array of tokens where each token has a positive or negative value. You can use tokens to gain power. If a token has a positive value, you can pick it up and gain its value. If a token has a negative value, you can use it to lose some of your power. However, you cannot lose more power than you currently have.

You can use a token only once, and you cannot pick up a token that you have already used.

What is the maximum total power you can gain from the given tokens?

Example 1:

Input: P = 2, tokens = [1, 2, 3]
Output: 6
Explanation: You can take the tokens in the following order: [1, 2, 3]. Your power will change as follows: [1, 3, 5, 6]. The maximum total power you can gain is 6.

Example 2:

Input: P = 10, tokens = [-3, 1, -2, 1, 5, -1]
Output: 10
Explanation: You can take the tokens in the following order: [1, 5]. Your power will change as follows: [1, 6, 11, 10]. The maximum total power you can gain is 10.

Solution:

To solve this problem, we can use a greedy algorithm. The greedy algorithm works by making the best possible decision at each step, without considering future consequences.

In this case, the best possible decision at each step is to pick up the token that has the highest positive value. If there are no positive tokens left, then we pick up the token that has the lowest negative value.

Here is the Python code for the greedy algorithm:

def get_maximum_power(P, tokens):
  """
  Args:
    P: The initial power.
    tokens: The array of tokens.

  Returns:
    The maximum total power that can be gained from the given tokens.
  """

  tokens.sort()  # Sort the tokens in ascending order.

  total_power = 0  # The total power that has been gained so far.

  for token in tokens:
    if P + token >= 0:
      P += token  # Gain the power from the token.
      total_power += token  # Add the power to the total power.
    else:
      break  # If the token has a negative value and it would reduce the power to below 0, then stop picking up tokens.

  return total_power

Time Complexity: The time complexity of this algorithm is O(n log n) where n is the number of tokens. This is because we sort the tokens in ascending order, which takes O(n log n) time.

Space Complexity: The space complexity of this algorithm is O(1), as we do not need to store any additional data structures.


most_stones_removed_with_same_row_or_column

Problem Statement:

You have a set of stones placed on a table. Each stone has a number written on it. You can remove any stone from the table if there is another stone on the table with the same number written on it.

Return the maximum number of stones that you can remove in one move.

Example:

Input: [1,2,3,4,5,1,2,3]
Output: 4

Explanation:

We can remove the stones with the numbers 1, 2, and 3 in one move.

Optimal Solution in Python:

def most_stones_removed_with_same_row_or_column(stones):
    # Create a dictionary to store the count of each stone's number.
    stone_counts = {}
    for stone in stones:
        if stone not in stone_counts:
            stone_counts[stone] = 0
        stone_counts[stone] += 1

    # Sort the dictionary by the values (counts) in descending order.
    sorted_counts = sorted(stone_counts.values(), reverse=True)

    # Remove stones until there are no more stones with the same number.
    max_stones_removed = 0
    while sorted_counts:
        current_count = sorted_counts.pop(0)
        if current_count > 1:
            max_stones_removed += current_count - 1

    return max_stones_removed

Breakdown:

  1. Create a dictionary to store the count of each stone's number. We use a dictionary to keep track of how many times each number appears in the list of stones.

  2. Sort the dictionary by the values (counts) in descending order. This allows us to easily identify which numbers have the most occurrences.

  3. Remove stones until there are no more stones with the same number. We repeatedly remove stones with the highest count until there are none left.

Example Usage:

stones = [1, 2, 3, 4, 5, 1, 2, 3]
result = most_stones_removed_with_same_row_or_column(stones)
print(result)  # Output: 4

Applications in Real World:

This problem has applications in various domains, including:

  • Scheduling: Determining the maximum number of tasks that can be scheduled simultaneously without conflicts.

  • Resource allocation: Allocating resources to maximize their utilization.

  • Data compression: Removing redundant data to reduce storage space.


broken_calculator

Problem Statement: Given a list of positive integers and a target number, find the minimum number of elements that must be removed from the list to achieve the target sum.

Brute Force Approach: The brute force approach is to try all possible combinations of elements to remove, and find the combination that results in the smallest sum closest to the target. This approach has a time complexity of O(2^N), where N is the number of elements in the list.

Optimized Approach: Dynamic Programming: The dynamic programming approach is based on the principle of breaking down the problem into smaller subproblems. We create a table where each entry represents the minimum number of elements that need to be removed to achieve a certain target sum.

To fill in the table, we iterate over the elements of the list and consider two cases:

  • Case 1: If we include the current element, we update the entry for the target sum by adding 1 to the minimum number of elements needed to achieve the target sum with the previous element.

  • Case 2: If we do not include the current element, we simply carry over the minimum number of elements needed to achieve the target sum with the previous element.

Python Implementation:

def broken_calculator(nums, target):
  """
  Finds the minimum number of elements that need to be removed from a list to achieve a target sum.

  Args:
    nums (list): The list of positive integers.
    target (int): The target sum.

  Returns:
    int: The minimum number of elements that need to be removed.
  """

  # Create a table to store the minimum number of elements needed to achieve each target sum.
  dp = [float('inf')] * (target + 1)
  dp[0] = 0

  # Iterate over the elements of the list.
  for num in nums:
    # Iterate over the target sums.
    for i in range(num, target + 1):
      # Update the entry for the target sum by adding 1 to the minimum number of elements needed to achieve the target sum with the previous element.
      dp[i] = min(dp[i], dp[i - num] + 1)

  # Return the minimum number of elements needed to achieve the target sum.
  return dp[target] if dp[target] != float('inf') else -1

Example:

nums = [3, 5, 2, 1, 4]
target = 7

result = broken_calculator(nums, target)
print(result)  # Output: 1

Explanation: To achieve the target sum of 7, we can remove the element 5, which reduces the sum to 2. We can then remove the element 1, which gives us the target sum. Therefore, the minimum number of elements that need to be removed is 2.

Applications: This problem can be applied to a variety of real-world scenarios, such as:

  • Optimizing inventory management by finding the minimum number of items that need to be removed to meet customer demand.

  • Scheduling tasks by finding the minimum number of tasks that need to be removed to meet a deadline.

  • Allocating resources by finding the minimum number of resources that need to be removed to satisfy a set of requirements.


group_the_people_given_the_group_size_they_belong_to

Problem Statement:

You are given an array of integers where each integer represents the group size of people who want to sit together in a row at a movie theatre. Return a list of groups, where each element represents the group size of the people in that group.

Brute Force Approach:

  1. Sort the array in ascending order.

  2. Iterate over the sorted array and group together elements that are equal.

  3. Append each group to the final list.

Implementation:

def group_the_people(group_sizes):
    group_sizes.sort()
    groups = []
    current_group = []
    for size in group_sizes:
        if len(current_group) < size:
            current_group.append(size)
        else:
            groups.append(current_group)
            current_group = [size]
    groups.append(current_group)
    return groups

Time Complexity: O(N log N), where N is the number of elements in the array.

Space Complexity: O(N), for the sorted array and the final list of groups.

Optimized Approach: Using a Hash Map

  1. Create a hash map to store the group size as keys and a list of people in that group as values.

  2. Iterate over the array and for each person, check if their group size is already in the hash map.

  3. If it is, append the person to the list of people in that group. Otherwise, create a new entry in the hash map with the group size as the key and a list containing the person as the value.

  4. Iterate over the hash map and append the list of people in each group to the final list of groups.

Implementation:

from collections import defaultdict

def group_the_people(group_sizes):
    groups = defaultdict(list)
    for person, size in enumerate(group_sizes):
        groups[size].append(person)
    return list(groups.values())

Time Complexity: O(N), where N is the number of elements in the array.

Space Complexity: O(N), for the hash map.

Example:

group_sizes = [3, 1, 2, 2, 3, 4, 3, 3]
groups = group_the_people(group_sizes)
print(groups)
# Output: [[3, 3, 3], [1], [2, 2], [4]]

Applications:

  • Group people for a bus or tour

  • Seat people at a concert or sporting event

  • Organize people for a team project


check_if_word_is_valid_after_substitutions

Problem Statement:

Given a string word, perform the following substitutions for each character in the word:

  • 'a' becomes 'z'

  • 'z' becomes 'a'

  • 'e' becomes 'q'

  • 'q' becomes 'e'

Return True if the resulting string is a palindrome. Otherwise, return False.

Solution:

Step 1: Create a HashMap for Substitutions

First, we create a HashMap to store the character substitutions:

substitutions = {
    'a': 'z',
    'z': 'a',
    'e': 'q',
    'q': 'e'
}

Step 2: Iterate Over the Word

Next, we iterate over each character in the word:

for char in word:
    # Check if the character is in the HashMap
    if char in substitutions:
        # Replace the character with its substitution
        word = word.replace(char, substitutions[char])

Step 3: Check if the Word is a Palindrome

Finally, we check if the resulting word is a palindrome:

if word == word[::-1]:
    return True
else:
    return False

Simplified Explanation:

Suppose we have the word "ace". We substitute the characters as follows:

  • 'a' becomes 'z'

  • 'c' remains the same

  • 'e' becomes 'q'

The resulting word is "zcq". Since "zcq" reads the same forwards and backwards, it is a palindrome. Therefore, we would return True.

Code Implementation:

def check_if_word_is_valid_after_substitutions(word):
    # Create a HashMap for substitutions
    substitutions = {
        'a': 'z',
        'z': 'a',
        'e': 'q',
        'q': 'e'
    }

    # Iterate over the word and make substitutions
    for char in word:
        if char in substitutions:
            word = word.replace(char, substitutions[char])

    # Check if the word is a palindrome
    if word == word[::-1]:
        return True
    else:
        return False

Potential Applications:

This problem could be used in various scenarios, such as:

  • Linguistics: Analyzing the structure and patterns of language

  • Text processing: Modifying and manipulating text data

  • Cryptography: Encrypting and decrypting messages


number_of_substrings_containing_all_three_characters

Problem Statement:

Given a string s, return the number of substrings that contain all three characters 'a', 'b', and 'c'.

Example:

s = "abcabc"
Output: 10
Explanation: There are 10 substrings containing all three characters 'a', 'b', and 'c': "abc", "abca", "abcab", "abcabc", "bca", "bcab", "bcabc", "cab", "cabc", and "abc".

Solution:

We can use a sliding window approach to solve this problem.

  1. Initialize two pointers, left and right, to point to the beginning of the string.

  2. Maintain a count for each of the three characters ('a', 'b', and 'c').

  3. Move the right pointer forward and check if the current substring contains all three characters.

  4. If the current substring contains all three characters, increment the count by 1.

  5. If the current substring does not contain all three characters, move the left pointer forward until it does.

  6. Repeat steps 3-5 until the right pointer reaches the end of the string.

Here is the Python code for this solution:

def number_of_substrings_containing_all_three_characters(s):
    """
    Returns the number of substrings that contain all three characters 'a', 'b', and 'c'.

    Args:
        s (str): The input string.

    Returns:
        int: The number of substrings that contain all three characters.
    """

    # Initialize the pointers and the counts.
    left = 0
    right = 0
    counts = {'a': 0, 'b': 0, 'c': 0}

    # Initialize the count.
    count = 0

    # Move the right pointer forward and check if the current substring contains all three characters.
    while right < len(s):
        counts[s[right]] += 1
        right += 1

        # Check if the current substring contains all three characters.
        while all(counts.values()):
            count += 1
            counts[s[left]] -= 1
            left += 1

    # Return the count.
    return count

Applications in Real World:

This algorithm can be used to find the number of subsequences in a string that contain all three characters 'a', 'b', and 'c'. This can be useful for tasks such as natural language processing and data mining.


largest_time_for_given_digits

Problem Statement

Given a list of four digits, find the largest possible time that can be formed using those digits.

Input

  • digits: A list of four integers representing the digits [0, 9].

Output

  • A string representing the largest possible time in the format "HH:MM".

Example

  • Input: [1, 2, 3, 4]

  • Output: "23:41"

Breakdown

To solve this problem, we can break it down into the following steps:

  1. Generate all possible permutations of the digits. There are 4! = 24 possible permutations.

  2. For each permutation, check if it is a valid time. A valid time has two digits for the hour (00 to 23) and two digits for the minutes (00 to 59).

  3. For each valid time, convert it to a string in the format "HH:MM".

  4. Return the largest valid time string.

Simplification

Step 1: Generate permutations

We can use the permutations function from the itertools module to generate all possible permutations of the digits.

import itertools

digits = [1, 2, 3, 4]
permutations = list(itertools.permutations(digits))

Step 2: Check if a permutation is valid

For each permutation, we can convert it to a string and check if it is a valid time.

def is_valid_time(permutation):
  hour = int("".join(permutation[:2]))
  minute = int("".join(permutation[2:]))
  return 0 <= hour <= 23 and 0 <= minute <= 59

Step 3: Convert a valid time to a string

For each valid time, we can convert it to a string in the format "HH:MM".

def time_to_string(time):
  return "%02d:%02d" % time

Step 4: Return the largest valid time string

Finally, we can return the largest valid time string.

def largest_time_for_given_digits(digits):
  permutations = list(itertools.permutations(digits))
  valid_times = []
  for permutation in permutations:
    if is_valid_time(permutation):
      valid_times.append(time_to_string(permutation))
  
  if valid_times:
    return max(valid_times)
  else:
    return "Invalid time"

Real-World Applications

This problem can be useful in situations where you need to find the largest or smallest possible value given a set of constraints. For example, it could be used to:

  • Find the largest possible transaction amount given a list of bills and coins.

  • Find the smallest possible number of moves to solve a puzzle.

  • Find the largest possible score for a game given a set of rules.


minimum_swaps_to_make_strings_equal

Problem Statement:

Given two strings, find the minimum number of swaps required to make them equal. Swapping two characters in a string means exchanging their positions.

Example:

Input: str1 = "abc", str2 = "acb"
Output: 1

Optimal Solution using Two-Pointers:

1. Initialize Two Pointers:

  • Create two pointers, i and j, initially set to 0.

2. Iterate Over the Strings:

  • Loop through the characters of str1 and str2:

    • Compare the characters at i and j.

    • If they are not equal, increment both i and j.

    • If they are equal, increment only j.

3. Count Swaps:

  • The number of swaps required is the difference between i and j.

Python Implementation:

def minimum_swaps(str1, str2):
    """
    Returns the minimum number of swaps to make two strings equal.

    Parameters:
        str1 (str): The first string.
        str2 (str): The second string.

    Returns:
        int: The minimum number of swaps.
    """

    n = len(str1)
    i, j, swaps = 0, 0, 0

    while i < n and j < n:
        if str1[i] != str2[j]:
            i += 1
            j += 1
            swaps += 1
        else:
            j += 1

    return swaps

Time Complexity: O(N), where N is the length of the strings.

Space Complexity: O(1), as it uses constant extra space.

Applications in Real World:

  • Data Synchronization: When synchronizing data between multiple devices, it's useful to determine the minimum number of swaps required to make the data consistent.

  • Text Editing: In text editors, finding the minimum number of swaps to transform one word into another can aid in spell checking and autocorrect features.

  • Genetic Sequence Comparison: In bioinformatics, identifying the minimum number of swaps to align genetic sequences is crucial for DNA analysis and disease diagnosis.


construct_binary_search_tree_from_preorder_traversal

Problem:

Given a preorder traversal of a binary search tree, we need to reconstruct the binary search tree.

Recursive Solution:

The preorder traversal of a binary search tree has the following property:

  • The first element is the root of the tree.

  • The left subtree consists of all elements less than the root.

  • The right subtree consists of all elements greater than the root.

Algorithm:

We can use recursion to construct the tree based on the above property:

  1. Create a new node with the first element in the preorder traversal.

  2. Find the index of the first element in the traversal that is greater than the root.

  3. The elements before this index form the left subtree, and the elements after this index form the right subtree.

  4. Recursively construct the left and right subtrees.

Python Implementation:

def construct_bst_from_preorder(preorder):
    if not preorder:
        return None

    root = TreeNode(preorder[0])
    split_index = find_split_index(preorder)

    root.left = construct_bst_from_preorder(preorder[1:split_index])
    root.right = construct_bst_from_preorder(preorder[split_index + 1:])

    return root

def find_split_index(preorder):
    for i in range(1, len(preorder)):
        if preorder[i] > preorder[0]:
            return i

Complexity Analysis:

  • Time complexity: O(N^2), where N is the number of nodes in the tree.

  • Space complexity: O(N), where N is the number of nodes in the tree.

Real-World Applications:

  • Data structures and algorithms

  • Database indexing

  • Searching and sorting

  • Decision trees


maximum_average_subtree

Problem Statement: Find the maximum average value of any subtree of a binary tree. The average value of a subtree is the sum of its values divided by the number of nodes.

Solution: We use a post-order traversal to calculate the sum and number of nodes in each subtree. The average value of a subtree is then calculated as the sum divided by the number of nodes. We then compare this average value to the maximum average value seen so far and update it if necessary.

Simplified Explanation: Imagine you have a tree with apples and you want to find the subtree with the highest average number of apples per branch. You start at the bottom branches and count how many apples and branches are in each branch. Then you move up to the next branch and do the same thing. As you move up the tree, you keep track of the highest average number of apples you've seen so far.

Code Implementation:

class TreeNode:
    def __init__(self, val=0, left=None, right=None):
        self.val = val
        self.left = left
        self.right = right

class Solution:
    def maximumAverageSubtree(self, root: TreeNode) -> float:
        self.max_avg = -float('inf')

        def postorder(node):
            if not node:
                return 0, 0

            left_sum, left_count = postorder(node.left)
            right_sum, right_count = postorder(node.right)

            total_sum = left_sum + right_sum + node.val
            total_count = left_count + right_count + 1

            self.max_avg = max(self.max_avg, total_sum / total_count)

            return total_sum, total_count

        postorder(root)
        return self.max_avg

Real-World Applications:

  • Identifying the most profitable branches of a business

  • Optimizing the performance of a computer network

  • Identifying the most efficient routes in a transportation system


camelcase_matching

Problem Statement:

Given two strings, s1 and s2, write a function that determines if s2 is a subsequence of s1 in camelCase.

CamelCase Subsequence:

A camelCase subsequence is formed by taking some or all of the characters from a camelCase string (s1). For example, "abcD" is a camelCase subsequence of "abcdEfgh".

Example:

camelMatch("abcdEfgh", "abcD") == True
camelMatch("abcdEfgh", "abcEf") == False
camelMatch("", "") == True

Solution:

1. Iterate Over s2: For each character in s2, check if it is in s1.

2. Matching Current Character: If the current character from s2 is lowercase and the current character from s1 is uppercase, the characters match.

3. Moving Ahead in s1: After matching a character, move ahead in s1 to the next character.

Code:

def camelMatch(s1: str, s2: str) -> bool:
    i1, i2 = 0, 0
    
    while i1 < len(s1) and i2 < len(s2):
        if s1[i1] == s2[i2] or (s1[i1].isupper() and s2[i2] == s1[i1].lower()):
            i1 += 1
            i2 += 1
        else:
            i1 += 1
    
    # If all characters of s2 have been matched
    return i2 == len(s2)

Explanation:

  • We initialize two pointers, i1 for s1 and i2 for s2, both starting at index 0.

  • We iterate through each character of s2.

  • For each character in s2, we check if it matches the current character in s1 (exact match or uppercase s1 with lowercase s2).

  • If they match, we advance both pointers. If they don't match, we only advance the s1 pointer (skipping non-matching characters).

  • After iterating through s2, if all its characters have been matched, we return True. Otherwise, we return False.

Applications:

Camel case matching can be useful in various scenarios:

  • Search Engine Optimization (SEO): Identifying relevant keywords in search queries.

  • Code Search: Filtering code results based on camelCase identifiers.

  • Natural Language Processing (NLP): Extracting meaningful phrases from text.

  • Data Parsing: Parsing structured data represented in camelCase format.


brace_expansion

Brace Expansion

Problem Statement: Given a string s that contains curly braces {, }, and lowercase English letters, perform the following steps:

  1. Replace {x,y} with all the possible combinations of x and y (i.e., xy, yx).

  2. Repeat step 1 until there are no more curly braces left.

Return the final expanded string.

Example:

Input: s = "{a,b}c{d,e}f"
Output: "acdf acef bcdf bcef"

Simplification:

  • Curly braces: They are used to represent a set of characters.

  • Expansion: The process of replacing curly braces with all possible combinations of characters within them.

Solution using Recursion:

def brace_expansion(s):
    if not '{' in s:
        return [s]
    
    result = []
    start = s.find('{')
    end = s.find('}')
    
    for char in s[start+1:end].split(','):
        sub_expansions = brace_expansion(s[0:start] + char + s[end+1:])
        result.extend(sub_expansions)
        
    return result

Explanation:

  1. Check if there are any more curly braces in the string. If not, return the string as is.

  2. Find the first opening and closing curly braces.

  3. Split the string inside the curly braces into a list of characters.

  4. Recursively call brace_expansion for each character and append it to the current string.

  5. Combine all the resulting strings to get the final expanded string.

Potential Applications:

  • Generating all possible combinations of a set of characters.

  • Iterating over all possible configurations in a system or program.

  • Testing different scenarios in software development.


validate_binary_tree_nodes

Validate Binary Tree Nodes

Problem:

Given an array of integers values representing the values of the nodes in a binary tree, validate if the array represents a valid binary tree with the following constraints:

  • The root node's value is 0.

  • The left child's value is always less than the parent's value.

  • The right child's value is always greater than the parent's value.

Solution:

We can use a stack to validate the binary tree:

  1. Initialize an empty stack stack.

  2. Push 0 (the root node) onto the stack.

  3. For each value v in values:

    • While the stack is not empty and v violates the binary tree constraints (i.e., v is less than or equal to the top of the stack):

      • Pop the top of the stack.

    • Push v onto the stack.

  4. If the stack is empty, the binary tree is valid. Otherwise, it is invalid.

Example:

def validate_binary_tree_nodes(values):
    stack = []
    stack.append(0)  # Initialize the stack with the root node

    for v in values:
        while stack and v <= stack[-1]:
            stack.pop()  # Pop invalid nodes from stack
        stack.append(v)  # Push valid node onto stack

    return not stack  # True if stack is empty (valid binary tree), False otherwise

Real-World Applications:

Validating a binary tree is useful in various scenarios, such as:

  • Data Structures: Ensuring that a binary tree object is well-formed and adheres to the binary tree structure constraints.

  • File Systems: Verifying that the hierarchical structure of a file system is valid and that directories and files are organized correctly.

  • Database Indexes: Confirming that a database index follows the binary tree structure and that data is stored optimally for efficient search and retrieval operations.


binary_string_with_substrings_representing_1_to_n

Problem: Given a binary string, determine if it represents the numbers 1 through n in order.

Simplified Explanation: Imagine a string of light bulbs, each representing a number. If we switch on the bulbs in order (1, 2, 3, and so on), a valid binary string would look like:

001111

where the bulbs from left to right are off, then on, and so on.

Solution:

  1. Count Leading Zeros:

    • Find the number of consecutive zeros at the beginning of the string. This is the potential gap before the start of the number sequence.

  2. Check for Continuous Ones:

    • Iterate over the string.

    • Ensure that there are no gaps between consecutive ones (i.e., the binary representation of each number is present).

  3. Verify Ending:

    • Check if the number of consecutive ones at the end of the string matches the expected number (n).

Python Implementation:

def binary_string_with_substrings_representing_1_to_n(s):
  """
  Checks if the given binary string represents the numbers 1 through n in order.

  Args:
    s (str): The binary string to check.

  Returns:
    bool: True if the string represents the numbers 1 through n in order, False otherwise.
  """

  # Count leading zeros
  leading_zeros = 0
  for char in s:
    if char == '0':
      leading_zeros += 1
    else:
      break

  # Check for continuous ones
  prev_zero = leading_zeros
  for char in s[leading_zeros:]:
    if char == '0':
      prev_zero = s.index(char)
    elif char == '1' and s.index(char) != prev_zero + 1:
      return False

  # Verify ending
  if s.count('1') != len(s) - leading_zeros:
    return False

  return True

Real-World Application: This problem finds applications in data encoding and transmission, where ensuring the integrity of a numerical sequence is crucial. For example, in barcode scanning, validating the binary representation of the product code ensures that the scanned code corresponds to a valid product.


Problem Statement:

Given the head of an immutable linked list, print the linked list in reverse.

Example:

Input: head = [1,2,3,4,5]
Output: [5,4,3,2,1]

Constraints:

  • The linked list is immutable, meaning you can't modify the list.

  • The size of the linked list is between 1 and 500.

  • 1 <= Node.val <= 1000

Solution:

This problem can be solved using recursion. The recursive solution works by calling itself on the next node in the linked list, and then printing the current node's value. This process continues until the end of the linked list is reached, at which point the values are printed in reverse order.

Here is the Python code for the recursive solution:

def print_immutable_linked_list_in_reverse(head):
  """
  Prints the values of an immutable linked list in reverse order.

  Args:
    head: The head of the linked list.
  """

  # Base case: If the linked list is empty, there's nothing to print.
  if head is None:
    return

  # Recursive case: Call the function on the next node in the linked list,
  # and then print the current node's value.
  print_immutable_linked_list_in_reverse(head.next)
  print(head.val)

Analysis:

The time complexity of the recursive solution is O(n), where n is the number of nodes in the linked list. This is because the function calls itself n times, each time taking constant time.

The space complexity of the recursive solution is also O(n), because the function call stack can grow to a maximum depth of n.

Real-World Applications:

This problem has applications in any situation where you need to print data in reverse order. For example, you could use it to print the values in a stack or queue in reverse order.

Here is an example of how you could use this problem in a real-world application:

# Define a class for a stack.
class Stack:
  def __init__(self):
    self.top = None

  def push(self, value):
    new_node = Node(value)
    new_node.next = self.top
    self.top = new_node

  def pop(self):
    if self.top is None:
      return None
    value = self.top.val
    self.top = self.top.next
    return value

  def print_in_reverse(self):
    print_immutable_linked_list_in_reverse(self.top)

# Create a stack.
stack = Stack()

# Push some values onto the stack.
stack.push(1)
stack.push(2)
stack.push(3)

# Print the values in the stack in reverse order.
stack.print_in_reverse()

Output:

3
2
1

check_completeness_of_a_binary_tree

Leetcode Problem: Check Completeness of a Binary Tree.

Problem Statement: Given a binary tree, determine if it is complete. A complete binary tree is one where all levels are completely filled except possibly the last level. The last level should have all its nodes filled from left to right.

Solution: We can use a level-order traversal to check the completeness of a binary tree.

Step-by-step Breakdown:

  1. Initialize a queue with the root node.

  2. While the queue is not empty:

    • Dequeue a node from the queue.

    • Check if the node is null:

      • If the node is null, and there are still nodes in the queue, return False.

      • If the node is null, and the queue is empty, return True.

    • Enqueue the node's left and right children to the queue.

Simplified Example:

Imagine a complete binary tree like this:

             1
           /   \
          2     3
         / \     /
        4   5   6
  1. Start with the root node (1).

  2. Enqueue its children (2 and 3).

  3. Dequeue node 1. Check if it's null (it's not), so continue.

  4. Enqueue node 2's children (4 and 5).

  5. Enqueue node 3's children (6).

  6. Dequeue node 2. Check if it's null (it's not), so continue.

  7. Enqueue node 4's children (null).

  8. Enqueue node 5's children (null).

  9. Dequeue node 3. Check if it's null (it's not), so continue.

  10. Enqueue node 6's children (null).

  11. Dequeue node 4. It's null, but there are still nodes in the queue, so return False.

Real-World Implementation:

def is_complete_binary_tree(root):
    queue = [root]

    while queue:
        node = queue.pop(0)

        if not node:
            if queue:
                return False
            else:
                return True

        queue.append(node.left)
        queue.append(node.right)

    return True

Applications:

Checking the completeness of a binary tree can be useful in various applications, such as:

  • Indexing in databases: Binary trees are used for efficient indexing in databases. Ensuring completeness helps optimize data storage and retrieval.

  • File systems: File systems often use binary trees to store data. Completeness ensures efficient disk space utilization and faster file access.

  • Min-heap data structures: Min-heaps are binary trees where each node's value is less than or equal to its children. Completeness is essential for maintaining the heap's properties.


four_divisors

Problem Statement:

Given an array nums consisting of integers, find the number of integers in the array that have exactly four distinct positive divisors.

Solution:

The divisors of a number are all the positive integers that divide it evenly. For example, the divisors of 12 are 1, 2, 3, 4, 6, and 12. A number with exactly four distinct positive divisors has the following form:

n = p^2 * q

where p and q are prime numbers.

To find the number of integers in the array with exactly four distinct positive divisors, we can iterate through the array and check if each number has the form p^2 * q. We can do this by finding the prime factors of each number using the sympy library. Here is the Python code:

import sympy

def count_integers_with_four_divisors(nums):
  count = 0
  for num in nums:
    if len(sympy.primefactors(num)) == 2:
      count += 1
  return count

Example:

nums = [24, 36, 48, 60]
result = count_integers_with_four_divisors(nums)
print(result)  # Output: 3

Applications:

This problem can be used to find properties of prime numbers and number theory. It can also be used in cryptography and other areas of mathematics.


minimum_cost_to_connect_sticks

Problem Statement: Given an array of positive integer stick lengths, determine the minimum cost to connect all the sticks into one long stick. The cost of connecting two sticks is equal to the sum of their lengths.

Example: For [2, 4, 3], the minimum cost to connect is 14.

  • Connect 2 and 3: cost 5

  • Connect 5 and 4: cost 9

  • Total cost: 5 + 9 = 14

Solution: 1. Sort the Sticks: We start by sorting the sticks in ascending order. This will help us identify the smallest sticks to connect first.

2. Initialize Minimum Cost: We initialize a variable min_cost to 0 to keep track of the total cost of connecting the sticks.

3. Greedy Approach: We iterate over the sorted sticks array and repeatedly connect the two smallest sticks until only one stick remains:

  • Find the two smallest sticks, say with lengths a and b.

  • Connect them into a single stick of length a + b.

  • Add a + b to min_cost.

  • Remove a and b from the array.

  • Insert the new stick with length a + b into the array.

Python Implementation:

def minimum_cost_to_connect_sticks(sticks):
    # Sort the sticks in ascending order
    sticks.sort()

    # Initialize minimum cost
    min_cost = 0

    # Repeatedly connect the two smallest sticks
    while len(sticks) > 1:
        # Get the two smallest sticks
        a, b = sticks[0], sticks[1]

        # Connect them into a single stick
        new_stick = a + b

        # Add the cost of connecting the sticks
        min_cost += new_stick

        # Remove the connected sticks
        sticks.pop(0)
        sticks.pop(0)

        # Insert the new stick into the sorted array
        idx = bisect.bisect_left(sticks, new_stick)
        sticks.insert(idx, new_stick)

    return min_cost

Explanation: The code follows the steps described in the solution:

  • It uses the sort() method to sort the sticks.

  • It initializes min_cost to 0.

  • It uses a while loop to repeatedly connect the two smallest sticks until only one stick remains.

  • Inside the loop, it gets the two smallest sticks, connects them, adds the cost to min_cost, and removes them from the array.

  • It uses binary search (bisect.bisect_left) to efficiently insert the new stick into the sorted array.

Applications:

  • Network Optimization: Connecting network nodes with minimum cost.

  • Data Compression: Huffman coding for lossless data compression.

  • Resource Allocation: Assigning tasks to resources optimally.


max_consecutive_ones_iii

Problem Statement:

Given an array nums of 0s and 1s, and an integer k, return the maximum number of consecutive 1s in the array if you can flip at most k 0s to 1s.

Best & Performant Solution in Python:

def max_consecutive_ones_iii(nums, k):
  left = right = max_ones = 0
  total_zeros = 0

  while right < len(nums):
    if nums[right] == 0:
      total_zeros += 1
      while total_zeros > k:
        if nums[left] == 0:
          total_zeros -= 1
        left += 1
    max_ones = max(max_ones, right - left + 1)
    right += 1

  return max_ones

Breakdown and Explanation:

  1. Sliding Window Approach:

    • We use a sliding window approach with two pointers, left and right, to track a range of consecutive 1s.

    • We move the right pointer to expand the window and check if we exceed the allowed number of flips (k).

    • If we exceed k, we move the left pointer to shrink the window until we meet the constraint again.

  2. Count Zeroes:

    • We keep track of the total number of zeros within the window using the total_zeros variable.

    • If total_zeros becomes greater than k, we know we have exceeded the allowed number of flips, and we need to adjust the window.

  3. Update Maximum Consecutive 1s:

    • After adjusting the window, we update max_ones to keep track of the maximum number of consecutive 1s we've seen so far.

  4. Move Window:

    • We continue moving the right pointer until we reach the end of the array.

Real-World Application:

This problem can be applied in practical scenarios such as:

  • Data Analysis: Finding the longest consecutive streak of successes or failures in a dataset.

  • Stock Trading: Identifying potential trading opportunities based on consecutive price increases or decreases.

  • Network Analysis: Detecting the maximum length of active connections in a computer network.


path_with_maximum_minimum_value

Path with Maximum Minimum Value

Problem Statement:

Given an m x n grid of integers, find a path from the top-left corner to the bottom-right corner such that the minimum value along the path is maximized. The path can only move down or right.

Approach:

This problem can be solved using dynamic programming. Let dp[i][j] be the maximum minimum value achievable by reaching the cell (i, j). We can compute dp[i][j] as follows:

dp[i][j] = max(dp[i-1][j], dp[i][j-1], grid[i][j])

where max is the maximum of the three values.

Implementation:

def max_minimum_path(grid):
    m, n = len(grid), len(grid[0])
    dp = [[0] * n for _ in range(m)]

    dp[0][0] = grid[0][0]

    # Fill the first row
    for j in range(1, n):
        dp[0][j] = max(dp[0][j-1], grid[0][j])

    # Fill the first column
    for i in range(1, m):
        dp[i][0] = max(dp[i-1][0], grid[i][0])

    # Fill the rest of the cells
    for i in range(1, m):
        for j in range(1, n):
            dp[i][j] = max(dp[i-1][j], dp[i][j-1], grid[i][j])

    return dp[m-1][n-1]

Example:

Input:

grid = [
    [1, 2, 3],
    [4, 5, 6],
    [7, 8, 9]
]

Output:

max_minimum_path(grid) == 5
(Path: (1, 2, 5, 8))

Applications:

This algorithm can be useful in various real-world scenarios, such as:

  • Pathfinding: Finding the best path through a maze or a complex network.

  • Resource allocation: Optimizing the distribution of resources to maximize the efficiency of a system.

  • Financial planning: Determining the optimal investment strategy to achieve a desired financial goal.


longest_repeating_substring

Problem Statement

Given a string, find the length of the longest repeating substring.

Example 1

Input: "abcabcbb" Output: 3 Explanation: The longest repeating substring is "abca".

Example 2

Input: "bbbbb" Output: 1 Explanation: The longest repeating substring is "b".

High-Level Approach

The brute-force approach is to check every possible substring and find the longest one that repeats. However, this approach is inefficient for large strings.

A more efficient approach is to use a sliding window to keep track of the longest repeating substring so far. We start with a window size of 2 and increment it by 1 until we reach the end of the string. For each window size, we check if the substring repeats by checking if the substring at the start of the window is equal to the substring at the end of the window. If it is, we update the longest repeating substring so far.

Detailed Solution

Here is the detailed solution in Python:

def longest_repeating_substring(string):
  # Initialize the longest repeating substring so far to 0.
  longest_substring = 0

  # Iterate over the string starting with a window size of 2.
  for window_size in range(2, len(string) + 1):
    # Iterate over the string with the current window size.
    for start in range(0, len(string) - window_size + 1):
      # Get the substring at the start of the window.
      substring_start = string[start:start + window_size]

      # Get the substring at the end of the window.
      substring_end = string[start + window_size - 1:start + window_size]

      # Check if the substring repeats.
      if substring_start == substring_end:
        # Update the longest repeating substring so far.
        longest_substring = max(longest_substring, window_size)

  # Return the longest repeating substring.
  return longest_substring

Complexity Analysis

  • Time complexity: O(n^3), where n is the length of the string. The outer loop iterates over the string with a window size of 2 to n, and the inner loop iterates over the string with the current window size.

  • Space complexity: O(1). No additional space is required.

Applications

The longest repeating substring algorithm can be used in a variety of applications, such as:

  • String compression: The algorithm can be used to compress strings by identifying and removing repeating substrings.

  • Sequence alignment: The algorithm can be used to align sequences of characters, such as DNA or protein sequences.

  • Pattern recognition: The algorithm can be used to identify patterns in strings, such as repeated words or phrases.


largest_values_from_labels

Problem Statement:

Given a list of strings keys and a list of integers values, where each key represents a label and each value represents the number of times the label appears, return a list of the labels with the top k highest values.

Example:

Input: keys = ["a", "b", "c", "d"], values = [1, 2, 3, 4], k = 2 Output: ["d", "c"]

Optimized Python Solution:

import heapq

def largest_values_from_labels(keys, values, k):
    # Create a dictionary mapping labels to values
    label_values = dict(zip(keys, values))

    # Convert the dictionary to a list of (value, label) tuples
    value_label_tuples = [(v, k) for k, v in label_values.items()]

    # Use heapq.nlargest to get the top k tuples
    top_k_tuples = heapq.nlargest(k, value_label_tuples)

    # Extract the labels from the tuples
    top_k_labels = [label for _, label in top_k_tuples]

    # Return the list of labels
    return top_k_labels

Breakdown:

  • We create a dictionary label_values to map each label to its corresponding value.

  • We convert the dictionary to a list of tuples (value, label) to facilitate sorting.

  • We use heapq.nlargest to retrieve the top k tuples based on their values.

  • Finally, we extract the labels from the top k tuples and return them as a list.

Real-World Applications:

This algorithm can be used in various scenarios, such as:

  • Finding the most popular products in an e-commerce website

  • Identifying the top trending hashtags on social media

  • Determining the most common words in a text document

  • Analyzing employee performance based on their performance ratings


find_k_length_substrings_with_no_repeated_characters

Problem Statement:

Given a string s, find the length of the longest substring with no repeating characters.

Approach:

We can use a sliding window technique to solve this problem efficiently. Here's a step-by-step explanation:

  1. Initialize a sliding window: Start with two pointers, left and right, both pointing to the beginning of the string (left = 0 and right = 0).

  2. Expand the window: While the right pointer is within the bounds of the string, check if the character at the right pointer has already been seen in the window. If it has, move the left pointer to the position after the last occurrence of that character.

  3. Update the maximum length: After expanding the window, check if the current window size (calculated as right - left + 1) is greater than the previously recorded maximum length. If it is, update the maximum length.

  4. Repeat: Continue expanding the window and updating the maximum length until the right pointer reaches the end of the string.

Implementation in Python:

def find_k_length_substrings_with_no_repeated_characters(s, k):
    char_count = {}
    max_length = 0

    left, right = 0, 0

    while right < len(s):
        if s[right] in char_count:
            char_count[s[right]] += 1
        else:
            char_count[s[right]] = 1

        if len(char_count) == k:  # We have found a valid substring
            max_length = max(max_length, right - left + 1)

        while len(char_count) > k:
            char_count[s[left]] -= 1
            if char_count[s[left]] == 0:
                del char_count[s[left]]
            left += 1

        right += 1

    return max_length

Real-World Applications:

The longest substring without repeating characters problem has applications in:

  • Text compression: Removing redundant characters from text can reduce its size.

  • Genome analysis: Identifying unique DNA segments can help in genetic research.

  • Spam filtering: Spam emails often contain repeated phrases or words, making this algorithm useful for detecting them.


maximum_level_sum_of_a_binary_tree

Problem: You are given a binary tree where each node contains an integer value. Find the level of the tree with the maximum sum of node values. Return the level with the maximum sum.

Example:

Input:
    1
   / \
  2   3
 / \   \
4   5   7
Output: 2
Explanation:
Level 1 has sum 1.
Level 2 has sum 2 + 3 = 5.
Level 3 has sum 4 + 5 + 7 = 16.
So, the maximum sum is 16 at level 3.

Solution: We can use a level-order traversal to solve this problem. We start at the root node and add its value to the sum of the first level. Then, we add all its children to a queue and continue to the next level. We repeat this process until we have visited all nodes in the tree.

Once we have visited all nodes, we can return the level with the maximum sum. Here is the code:

def maximum_level_sum_of_a_binary_tree(root):
  # Initialize the maximum sum and level
  max_sum = float('-inf')
  max_level = 0

  # Initialize a queue with the root node
  queue = [root]

  # Keep track of the current level
  level = 0

  # Level-order traversal
  while queue:
    # Calculate the sum of the current level
    level_sum = 0
    for node in queue:
      level_sum += node.val

    # Update the maximum sum and level if necessary
    if level_sum > max_sum:
      max_sum = level_sum
      max_level = level

    # Add the next level of nodes to the queue
    new_queue = []
    for node in queue:
      if node.left:
        new_queue.append(node.left)
      if node.right:
        new_queue.append(node.right)

    # Move to the next level
    queue = new_queue
    level += 1

  return max_level

Time Complexity: O(N), where N is the number of nodes in the tree. Space Complexity: O(N), where N is the number of nodes in the tree.

Applications: This algorithm can be used to find the level of a binary tree with the maximum sum of node values. This can be useful in applications such as:

  • Finding the optimal level of a decision tree

  • Finding the level of a binary search tree with the maximum number of nodes

  • Finding the level of a binary tree with the maximum number of leaves


web_crawler

Here is an implementation of a simple web crawling algorithm that uses Python's urllib library.

import urllib.request

# The URL of the website to crawl
url = "https://www.example.com"

# Create a Queue to store the URLs that need to be crawled
queue = [url]

# Create a set to store the URLs that have already been crawled
crawled = set()

# While there are still URLs in the queue, crawl them
while queue:
    # Get the next URL from the queue
    url = queue.pop()

    # Add the URL to the set of crawled URLs
    crawled.add(url)

    # Open the URL and read the HTML content
    with urllib.request.urlopen(url) as response:
        html = response.read()

    # Extract all the URLs from the HTML content
    urls = extract_urls(html)

    # Add the extracted URLs to the queue
    for url in urls:
        if url not in crawled and url not in queue:
            queue.append(url)

This algorithm works by first creating a queue of URLs to be crawled. It then crawls the first URL in the queue, adds it to the set of crawled URLs, and extracts all the URLs from the HTML content of the page. These extracted URLs are then added to the queue to be crawled later. The algorithm repeats this process until there are no more URLs in the queue to be crawled.

The extract_urls function is not shown in the code above, but it is responsible for extracting all the URLs from the HTML content of a page. This can be done using a regular expression or an HTML parser.

This algorithm is a simple example of a web crawler. In practice, web crawlers can be much more complex and can use a variety of techniques to improve their efficiency and performance.

Real-world applications

Web crawlers are used in a variety of applications, including:

  • Search engines: Search engines use web crawlers to index the web and make it easier for users to find information.

  • Website monitoring: Web crawlers can be used to monitor websites for changes or updates.

  • Data mining: Web crawlers can be used to extract data from websites for analysis.

  • Competitive intelligence: Web crawlers can be used to gather information about competitors' websites.

  • Spam detection: Web crawlers can be used to detect spam websites.


find_players_with_zero_or_one_losses

Problem Statement: Find the players who have lost zero or one games.

Assumptions:

  • Each player has played at least one game.

  • There are no ties.

Approach:

  1. Iterate through the list of players.

  2. For each player, count the number of games they have lost.

  3. If the number of losses is zero or one, add the player to the list of players with zero or one losses.

Implementation:

def find_players_with_zero_or_one_losses(players):
  """
  Finds the players who have lost zero or one games.

  Parameters:
    players: A list of players.

  Returns:
    A list of players who have lost zero or one games.
  """

  players_with_zero_or_one_losses = []

  for player in players:
    num_losses = player.get_num_losses()
    if num_losses == 0 or num_losses == 1:
      players_with_zero_or_one_losses.append(player)

  return players_with_zero_or_one_losses

Example:

players = [
  Player("Alice", 0),
  Player("Bob", 1),
  Player("Carol", 2),
  Player("Dave", 3),
]

players_with_zero_or_one_losses = find_players_with_zero_or_one_losses(players)

print(players_with_zero_or_one_losses)  # [Player("Alice", 0), Player("Bob", 1)]

Real-World Applications:

  • Identifying players who are new to a game and may need additional support.

  • Creating a leaderboard of players who have the best records.

  • Analyzing player performance to identify areas for improvement.


robot_bounded_in_circle

Problem Statement:

Imagine a robot starting at the origin (0, 0) on a 2D plane. Given a sequence of instructions consisting of characters "L", "R", "U", and "D", where each character represents a move in the corresponding direction (left, right, up, or down), determine if the robot will stay within a circle of radius 1 unit centered at the origin.

Solution:

The robot's position can be represented as a complex number. Each move can be represented as a multiplication by one of the four complex numbers: -1 (left), 1 (right), i (up), -i (down).

The robot will remain within the circle if and only if the final position after all moves is equal to 0. This is because the circle has a radius of 1, and the final position of the robot will be within the circle if it is at the origin (0, 0).

Python Implementation:

def robot_bounded_in_circle(instructions):
  # Represent the robot's position as a complex number.
  position = complex(0, 0)

  # Map the instructions to complex numbers.
  directions = {
    "L": -1,
    "R": 1,
    "U": 1j,
    "D": -1j
  }

  # Perform the moves.
  for instruction in instructions:
    position *= directions[instruction]

  # Return if the robot is within the circle.
  return position == complex(0, 0)

Example:

instructions = "RLU"
print(robot_bounded_in_circle(instructions))  # True

instructions = "GGLLGG"
print(robot_bounded_in_circle(instructions))  # False

Explanation:

In the first example, the robot follows the instructions "RLU". The initial position is (0, 0). After moving left ("L"), the position becomes (-1, 0). After moving right ("R"), the position becomes (0, 0). After moving up ("U"), the position becomes (0, 1). Since the final position is not equal to (0, 0), the robot does not stay within the circle.

In the second example, the robot follows the instructions "GGLLGG". The initial position is (0, 0). After moving left twice ("GG"), the position becomes (-2, 0). After moving left twice again ("LL"), the position becomes (0, 0). After moving right twice ("GG"), the position becomes (0, 0). Since the final position is equal to (0, 0), the robot stays within the circle.

Potential Applications:

This problem has applications in robotics and path planning. For example, it can be used to determine if a robot will collide with an obstacle or stay within a safe zone.


next_greater_node_in_linked_list

Problem Statement:

Given a linked list, return a corresponding linked list where each node contains the value of the next greater node in the original linked list. If a node has no greater node, return 0.

Input:

1 -> 7 -> 5 -> 1 -> 9 -> 2 -> 5 -> 1

Output:

1 -> 7 -> 5 -> 9 -> 9 -> 9 -> 5 -> 0

Breakdown:

  1. Traverse the original linked list: Iterate through the original linked list using a while loop or a for loop.

  2. Store the current node and its value: For each node, store the node in a variable current_node and its value in a variable current_value.

  3. Check for greater nodes: Use a stack to keep track of nodes with greater values. Iterate through the rest of the linked list starting from the next node of the current node. For each node in the iteration, check if its value is greater than current_value. If so, push the node into the stack.

  4. Update the current node's value: If the stack is not empty, pop the top node from the stack and set the next value of the current_node to the popped node. This indicates that the next greater node of the current_node is the popped node. If the stack is empty, it means there are no greater nodes, so set the next value of the current_node to 0.

  5. Repeat steps 2-4: Continue traversing the original linked list, updating the next value of each node based on the greater nodes found in the subsequent nodes using the stack.

  6. Return the new linked list: After traversing the entire original linked list, return the updated linked list, where each node contains the value of its next greater node or 0.

Example Implementation:

def next_greater_node(head):
    """
    :type head: ListNode
    :rtype: ListNode
    """

    # Store the original linked list
    original_list = head

    # Create a stack to store nodes with greater values
    stack = []

    # Create a new linked list for the result
    new_head = ListNode(0)
    new_list = new_head

    # Iterate through the original linked list
    while head:

        # Store the current node and its value
        current_node = head
        current_value = head.val

        # Check for greater nodes in the subsequent nodes
        while stack and stack[-1].val < current_value:
            # If there's a greater node, pop it from the stack
            popped_node = stack.pop()

            # Update the 'next' value of the popped node to the current node
            popped_node.next = current_node

        # Push the current node into the stack
        stack.append(current_node)

        # Update the 'next' value of the current node based on the stack
        if stack:
            current_node.next = stack[-1]
        else:
            current_node.next = None

        # Move to the next node in the original linked list
        head = head.next

        # Add the updated current node to the new linked list
        new_list.next = current_node
        new_list = new_list.next

    # Return the new linked list
    return new_head.next

Applications:

The solution to this problem can be applied in various real-world scenarios:

  • Finding the next greater element in an array: The algorithm can be modified to find the next greater element for each element in an array. This can be useful in stock market analysis, where we want to find the next highest stock price for each day.

  • Finding the longest increasing subsequence: The algorithm can be used to find the longest increasing subsequence of a linked list. The longest increasing subsequence is a subsequence of the original linked list such that the values are strictly increasing.

  • Finding the skyline of a set of buildings: The algorithm can be used to find the skyline of a set of buildings, where the skyline is the shape formed by the buildings when viewed from a distance.


design_file_system

Problem Statement:

Design a File System that supports the following operations:

  1. createPath(path, value): Creates a new file or directory at the given path.

  2. get(path): Returns the value of the file at the given path, or -1 if it's a directory.

Solution:

Breakdown:

  • We need a data structure to represent the file system. One good option is a dictionary, where keys are paths and values are either values (for files) or dictionaries (for directories).

  • To create a new file or directory, we can use the createPath method to add the corresponding entry to the dictionary.

  • To get the value of a file, we can use the get method to retrieve it from the dictionary. If the path corresponds to a directory, we return -1.

Python Implementation:

class FileSystem:
    def __init__(self):
        self.fs = {}

    def createPath(self, path, value):
        curr = self.fs
        for p in path.split('/')[1:]:
            curr[p] = {}
            curr = curr[p]
        curr['value'] = value

    def get(self, path):
        curr = self.fs
        for p in path.split('/')[1:]:
            if p not in curr:
                return -1
            curr = curr[p]
        return curr.get('value', -1)

Example Usage:

fs = FileSystem()
fs.createPath('/a/b/c', 123)
fs.get('/a')  # -1
fs.get('/a/b/c')  # 123

Real-World Applications:

  • File System Viewer: Visualize and navigate the file system of a computer.

  • File Manager: Manage files and folders, such as creating, deleting, and renaming them.

  • Cloud Storage: Store and access files online, using a hierarchical structure like a file system.


shortest_path_in_binary_matrix

Problem Statement:

Given a binary matrix where 0 represents an obstacle and 1 represents an open space, find the shortest path from the top-left corner (0, 0) to the bottom-right corner (m-1, n-1). You can only move right or down.

Example:

matrix = [[0, 1, 1],
         [1, 1, 0],
         [1, 1, 1]]

Output: 4
Explanation: The shortest path is (0, 0) -> (0, 1) -> (0, 2) -> (1, 2) -> (2, 2)

General Approach:

The problem can be solved using Dynamic Programming (DP). DP is a technique where we store the solutions to subproblems that are used to solve larger problems.

Implementation:

In this case, the subproblems are the shortest paths to the top-left corner of each submatrix. We can define a DP table dp where dp[i][j] represents the shortest path to the top-left corner of the submatrix ending at cell (i, j).

Initialization:

  • dp[0][0] = 1 (since there is only one way to reach the top-left corner)

  • dp[i][0] = dp[i-1][0] if matrix[i][0] == 1, otherwise dp[i][0] = 0

  • dp[0][j] = dp[0][j-1] if matrix[0][j] == 1, otherwise dp[0][j] = 0

Recurrence Relation:

For all other cells, dp[i][j] = min(dp[i-1][j], dp[i][j-1]) + 1 if matrix[i][j] == 1, otherwise dp[i][j] = 0

Final Result:

The shortest path to the bottom-right corner will be stored in dp[m-1][n-1].

Code Implementation:

def shortest_path_in_binary_matrix(matrix):
    m, n = len(matrix), len(matrix[0])
    dp = [[0] * n for _ in range(m)]

    # Initialization
    dp[0][0] = 1
    for i in range(1, m):
        dp[i][0] = dp[i-1][0] if matrix[i][0] == 1 else 0
    for j in range(1, n):
        dp[0][j] = dp[0][j-1] if matrix[0][j] == 1 else 0

    # Recurrence relation
    for i in range(1, m):
        for j in range(1, n):
            if matrix[i][j] == 1:
                dp[i][j] = min(dp[i-1][j], dp[i][j-1]) + 1
            else:
                dp[i][j] = 0

    return dp[m-1][n-1] if dp[m-1][n-1] != 0 else -1

Real-World Applications:

  • Pathfinding in robotics

  • Maze traversal

  • Network routing

  • Supply chain management


ugly_number_iii

Problem Statement:

You are given a positive integer n. An ugly number is a positive integer whose prime factors include only 2, 3, and 5. Return the nth ugly number.

Approach:

We will use a priority queue to maintain the next smallest ugly numbers:

  1. Initialization: Initialize a priority queue with [1].

  2. Iteration: While the queue has less than n elements:

    • Pop the smallest ugly number curr from the queue.

    • Multiply curr by 2, 3, and 5, and push the results into the queue (if they are not already in the queue).

  3. Return: Return the last popped element, which will be the nth ugly number.

Python Code:

import heapq

def nthUglyNumber(n):
    # Initialize the queue with [1]
    queue = [1]
    
    # Keep track of the numbers we have seen
    seen = set()
    
    # Iterate until we have found the n-th ugly number
    for i in range(1, n + 1):
        # Pop the smallest ugly number from the queue
        curr = heapq.heappop(queue)
        
        # Multiply curr by 2, 3, and 5, and push the results into the queue if we haven't seen them before
        if curr * 2 not in seen:
            heapq.heappush(queue, curr * 2)
            seen.add(curr * 2)
        if curr * 3 not in seen:
            heapq.heappush(queue, curr * 3)
            seen.add(curr * 3)
        if curr * 5 not in seen:
            heapq.heappush(queue, curr * 5)
            seen.add(curr * 5)
    
    return curr

Time Complexity:

The time complexity is O(n log n), where n is the given number. We process each ugly number exactly once, and each operation takes O(log n) time to maintain the priority queue.

Real-World Applications:

Ugly numbers are used in various fields, such as:

  • Computer Science: Generating sequences of uniformly distributed numbers, hash functions.

  • Mathematics: Number theory, combinatorial optimization.

  • Finance: Risk analysis, stock market modeling.


path_with_maximum_gold

Problem Statement

In a gold mine, each cell contains a certain amount of gold. You can move to a cell that is adjacent to your current cell (left, right, up, or down) and collect the gold in that cell. What is the maximum amount of gold you can collect if you start from any cell and move according to the given rules?

Solution

The problem can be solved using Depth First Search (DFS). We start from any cell, collect the gold in that cell, and then recursively call DFS on adjacent cells. We keep track of the maximum amount of gold collected so far.

Implementation

def path_with_maximum_gold(grid):
    if not grid:
        return 0

    nrows = len(grid)
    ncols = len(grid[0])

    max_gold = 0

    def dfs(row, col, gold):
        if row < 0 or row >= nrows or col < 0 or col >= ncols or grid[row][col] == 0:
            return

        gold += grid[row][col]
        grid[row][col] = 0  # Mark the cell as visited
        max_gold = max(max_gold, gold)

        # Recursively explore adjacent cells
        dfs(row-1, col, gold)
        dfs(row+1, col, gold)
        dfs(row, col-1, gold)
        dfs(row, col+1, gold)

        # Backtrack by setting the cell value back to its original value and resuming exploration
        grid[row][col] = gold - grid[row][col]

    # Start DFS from each cell
    for i in range(nrows):
        for j in range(ncols):
            if grid[i][j]:
                dfs(i, j, 0)

    return max_gold

Explanation

  • The function path_with_maximum_gold takes the grid as input and returns the maximum amount of gold that can be collected.

  • The function initializes the maximum gold to 0.

  • The function iterates over each cell in the grid. If the cell contains gold, the function calls the DFS function on that cell.

  • The DFS function takes the current row, column, and the amount of gold collected so far as input.

  • The DFS function checks if the current row and column are within the bounds of the grid and if the cell contains gold. If not, the function returns.

  • The DFS function adds the gold in the current cell to the total gold collected so far.

  • The DFS function marks the current cell as visited by setting its value to 0.

  • The DFS function updates the maximum gold collected so far.

  • The DFS function recursively calls itself on the adjacent cells.

  • The DFS function backtracks by setting the value of the current cell back to its original value.

Time Complexity

The time complexity of the solution is O(mn), where m is the number of rows and n is the number of columns in the grid. The DFS function is called once for each cell in the grid.

Space Complexity

The space complexity of the solution is O(mn). The DFS function uses a stack to store the current path. The maximum depth of the stack is m + n.

Applications

The problem can be applied to any situation where you need to find the maximum value in a grid. For example, it can be used to find the maximum profit in a stock market or the maximum distance you can travel in a given area.


k_closest_points_to_origin

Problem Statement: Given an array of points in the form (x, y), return the k closest points to the origin.

Best & Performant Solution: The best and most performant solution for this problem is to use a min-heap.

Breakdown and Explanation: Here's how the solution using a min-heap works:

  1. Initialize a min-heap: A min-heap is a data structure that keeps track of the smallest element. We can initialize a min-heap to store the distances of the points from the origin.

  2. Calculate distances: For each point (x, y), calculate its distance from the origin using the formula: distance = sqrt(x^2 + y^2).

  3. Insert distances into heap: Insert all the calculated distances into the min-heap. The heap will automatically sort the distances in ascending order.

  4. Extract k closest distances: Pop the top k distances from the heap. These distances belong to the k closest points to the origin.

  5. Retrieve points from distances: Once you have the k closest distances, you can retrieve the corresponding points from the original array.

Implementation:

import heapq

def k_closest_points_to_origin(points, k):
    # Initialize a min-heap to store distances
    heap = []
    
    # Calculate distances and insert into heap
    for point in points:
        x, y = point
        distance = sqrt(x**2 + y**2)
        heapq.heappush(heap, (distance, point))

    # Extract k closest distances
    closest_distances = []
    for _ in range(k):
        distance, point = heapq.heappop(heap)
        closest_distances.append(point)

    # Return k closest points
    return closest_distances

Example:

points = [(1, 2), (3, 4), (5, 6), (7, 8)]
k = 2
result = k_closest_points_to_origin(points, k)
print(result)  # [(1, 2), (3, 4)]

Real-World Applications:

This problem has practical applications in a variety of fields, such as:

  • Image processing: Finding the k most similar pixels to a given image.

  • Machine learning: Clustering data points based on their distance from each other.

  • Data analysis: Identifying potential outliers or influential points in a dataset.


build_an_array_with_stack_operations

Build an Array With Stack Operations

Problem Statement:

Given an integer array target, build an array answer of the same length such that answer[0] is equal to target[0]. For each subsequent element answer[i], answer[i] is equal to the sum of answer[i - 1] and target[i]. Note that the element answer[0] is already given.

Example:

Input: target = [1, 2, 3, 4, 5] Output: [1, 3, 6, 10, 15]

Approach Using Stack:

We can use a stack to efficiently simulate the operations required to build the answer array. The steps involved are:

  1. Initialize: Create an empty stack and push the first element of target (i.e., target[0]) onto the stack.

  2. Iterate: Iterate through the remaining elements of target (from index 1 to N-1).

  3. Pop and Sum: For each element target[i], pop the top element from the stack and add it to target[i]. The result is pushed back onto the stack.

  4. Push Remaining: Once all elements have been processed, push the remaining elements from the stack into the answer array.

Python Implementation:

def buildArray(target):
  stack = [target[0]]
  answer = []
  for i in range(1, len(target)):
    top = stack.pop()
    stack.append(top + target[i])
  while stack:
    answer.append(stack.pop())
  return answer

Time Complexity: O(N), where N is the length of the target array. Each element is processed once, either when it is pushed onto the stack or when it is popped.

Space Complexity: O(N), as the stack can store up to N elements at any point.

Real-World Applications:

This technique can be applied in any scenario where we need to accumulate or aggregate values based on sequential operations. For example:

  • Maintaining a running total in a financial accounting system.

  • Calculating the cumulative sum of weights in a physical simulation.

  • Aggregating data points in a data analysis pipeline.


last_stone_weight_ii

Problem Statement:

You have a number of stones. Each stone has a certain weight. You want to make a pile of stones that has the maximum possible weight that is still smaller than or equal to the sum of the weights of all stones.

Optimized Python Solution:

def last_stone_weight_ii(stones):
  """
  Returns the maximum possible weight of a pile of stones that is still smaller than or equal to the sum of the weights of all stones.

  Args:
    stones: A list of integers representing the weights of the stones.

  Returns:
    An integer representing the maximum possible weight of a pile of stones.
  """

  # Sort the stones in ascending order.
  stones.sort()

  # Initialize the weight of the pile to the smallest stone.
  pile_weight = stones[0]

  # Iterate over the remaining stones.
  for i in range(1, len(stones)):
    # If the current stone is heavier than the pile, add it to the pile.
    if stones[i] > pile_weight:
      pile_weight = pile_weight + stones[i]
    # Otherwise, add the current stone to the top of the pile.
    else:
      pile_weight = pile_weight + stones[i] / 2

  # Return the weight of the pile.
  return pile_weight

Explanation:

  1. Sort the stones in ascending order: This makes it easier to determine which stones should be added to the pile and which stones should be placed on top of the pile.

  2. Initialize the weight of the pile to the smallest stone: This ensures that the pile will always have a positive weight.

  3. Iterate over the remaining stones: For each stone, we determine whether it should be added to the pile or placed on top of the pile.

  4. If the current stone is heavier than the pile, add it to the pile: This ensures that the weight of the pile will always be less than or equal to the sum of the weights of all stones.

  5. Otherwise, add the current stone to the top of the pile: This ensures that the weight of the pile will be as close as possible to the sum of the weights of all stones.

  6. Return the weight of the pile: This is the maximum possible weight of a pile of stones that is still smaller than or equal to the sum of the weights of all stones.

Real-World Applications:

This problem can be applied to a variety of real-world scenarios, such as:

  • Load Balancing: When distributing tasks across a cluster of servers, it is important to ensure that each server is not overloaded. This problem can be used to determine the maximum number of tasks that can be assigned to each server while still meeting the overall performance requirements.

  • Resource Allocation: When allocating resources to a team of workers, it is important to ensure that each worker is given enough resources to complete their tasks. This problem can be used to determine the minimum amount of resources that can be given to each worker while still ensuring that all tasks are completed.

  • Scheduling: When scheduling a set of tasks, it is important to minimize the makespan, which is the time it takes to complete all tasks. This problem can be used to determine the schedule that minimizes the makespan while still ensuring that all tasks are completed.


break_a_palindrome

Problem Statement:

Given a palindromic string "s", find the minimum number of characters that you can remove to make it non-palindromic.

Implementation:

Python provides a built-in function palindrome that checks if a string is a palindrome. Here's a Python implementation that meets the problem requirements:

def break_a_palindrome(s):
    """
    Finds the minimum number of characters that need to be removed from 's' to make it non-palindromic.

    Args:
        s (str): The input palindromic string.

    Returns:
        int: The minimum number of characters to remove.
    """

    # Check if the string is already non-palindromic
    if not s == s[::-1]:
        return 0

    # Find the middle index of the string
    mid = len(s) // 2

    # Check if the string is odd-length
    if len(s) % 2 == 1:
        return 1  # Removing the middle character makes the string non-palindromic

    # Check if the first character is not equal to the second character
    if s[0] != s[1]:
        return 1  # Removing the first character makes the string non-palindromic

    # Otherwise, remove the first character that is not equal to the middle character
    for i in range(mid, len(s)):
        if s[i] != s[mid]:
            return 1

    # If no such character is found, remove the last character
    return 1

Explanation:

  • The function first checks if the string is already non-palindromic. If it is, it returns 0.

  • If the string is even-length, the middle and first characters are always equal. So, the function finds and removes the first character that is not equal to the middle character.

  • If the string is odd-length, the middle character can be removed to make the string non-palindromic.

  • If no character can be removed to make the string non-palindromic, the function removes the last character.

Real-World Application:

This problem has applications in data processing and text analysis. For example, it can be used to identify and remove duplicate text, or to find the shortest version of a string that retains its essential meaning.


string_without_aaa_or_bbb

Problem:

Given a string S, find the string S without the substrings "aaa" and "bbb".

Implementation:

def remove_aaa_bbb(s):
  """
  Removes substrings "aaa" and "bbb" from a given string.

  Args:
    s: The original string.

  Returns:
    The modified string without "aaa" and "bbb".
  """

  # Initialize an empty string for the modified string.
  result = ""

  # Iterate over the characters in the original string.
  for char in s:
    # Check if the current character is 'a' or 'b'.
    if char in ['a', 'b']:
      # Check if the current character is the first character in a substring "aaa" or "bbb".
      if result.endswith(char * 2):
        continue

    # Add the current character to the modified string.
    result += char

  # Return the modified string.
  return result

Explanation:

The function remove_aaa_bbb takes a string s as input and returns a modified string without the substrings "aaa" and "bbb".

  1. Initialize an empty string: We initialize an empty string result to store the modified string.

  2. Iterate over the characters: We iterate over each character in the original string s.

  3. Check for 'a' or 'b': If the current character is 'a' or 'b', we check if it is the first character in a substring "aaa" or "bbb". To do this, we check if the last two characters of the result string match the current character.

  4. Add to result: If the current character is not the first character in "aaa" or "bbb", we add it to the result string.

  5. Return modified string: Finally, we return the result string, which contains the original string with "aaa" and "bbb" removed.

Example:

s = "abcabcbbbaaa"
result = remove_aaa_bbb(s)
print(result)  # Output: "abcbb"

Real-World Applications:

This function can be useful in various real-world applications, such as:

  • Data cleaning: Removing unwanted patterns or substrings from datasets.

  • Text processing: Preprocessing text before performing natural language processing tasks.

  • Spam filtering: Identifying and removing spam emails that contain specific patterns.


flip_equivalent_binary_trees

Problem Statement: Given the roots of two binary trees A and B, determine if they are flip equivalents. Two binary trees are flip equivalents if they are the same tree with all their left and right children swapped.

Implementation:

def flip_equivalent_binary_trees(root1, root2):
    """
    :type root1: TreeNode
    :type root2: TreeNode
    :rtype: bool
    """
    if not root1 and not root2:
        return True
    elif not root1 or not root2:
        return False
    elif root1.val != root2.val:
        return False
    else:
        return flip_equivalent_binary_trees(root1.left, root2.right) and flip_equivalent_binary_trees(root1.right, root2.left)

Explanation: The flip_equivalent_binary_trees function takes two binary trees as input and compares them to see if they are flip equivalents. It performs a recursive depth-first search (DFS) traversal on both trees simultaneously, comparing the values of the nodes at each step. Here's a breakdown of the implementation:

  • Base cases:

    • If both root1 and root2 are None, they are considered flip equivalents and the function returns True.

    • If either root1 or root2 is None, they cannot be flip equivalents, so the function returns False.

  • Node comparison:

    • If the values of root1.val and root2.val are different, the trees cannot be flip equivalents, so the function returns False.

  • Recursive calls:

    • The function makes two recursive calls:

      • flip_equivalent_binary_trees(root1.left, root2.right): This call compares the left subtree of root1 with the right subtree of root2.

      • flip_equivalent_binary_trees(root1.right, root2.left): This call compares the right subtree of root1 with the left subtree of root2.

    • If both recursive calls return True, it means that the left and right subtrees of root1 and root2 are flip equivalents, and the function returns True.

    • If either recursive call returns False, the trees are not flip equivalents and the function returns False.

Example: Consider the following two binary trees:

A:        
             1
            / \
           2   3
          / \   
         4   5
B:
             1
            / \
           3   2
          / \   
         5   4    

These two trees are flip equivalents because they have the same structure and the values of their nodes are the same, with all their left and right children swapped. The flip_equivalent_binary_trees function would return True for these trees.

Applications: The concept of flip equivalents is not directly applicable in real-world applications. However, the recursive DFS approach used in the implementation can be applied to a variety of problems involving tree traversals, such as finding the depth of a tree, finding the maximum or minimum value in a tree, or counting the number of nodes or leaves in a tree.


number_of_ways_to_build_house_of_cards

Problem Statement: Given a certain number of cards, determine the number of ways to build a house of cards.

Breakdown:

  • Number of Cards: Denotes the number of cards available to build the house.

  • Number of Ways: Represents the distinct arrangements or ways in which the cards can be stacked to create a house.

Solution:

We can simplify this problem into two steps:

1. Base Cases:

  • If there are no cards (0 cards), there is only 1 way to build a house: not building anything.

  • If there is only 1 card, there is also only 1 way to build a house: placing the single card as the base.

2. Recursive Solution:

  • For any number of cards greater than 1, we can recursively determine the number of ways:

    • Assume the top card as the roof of the house.

    • For the remaining cards, calculate the number of ways to build the rest of the house.

    • Multiply these two values to get the total number of ways for that specific top card.

    • Repeat this process for each card that could be used as the top card.

  • The final result is the sum of the number of ways calculated for each possible top card.

Simplified Code Implementation:

def number_of_ways_to_build_house_of_cards(n):
    if n == 0:
        return 1
    elif n == 1:
        return 1
    else:
        result = 0
        for i in range(1, n + 1):
            # Calculate ways to build the house without the ith card as the top
            ways_without_ith_card = number_of_ways_to_build_house_of_cards(n - i)
            # Multiply by ways to use the ith card as the top
            result += ways_without_ith_card * i
        return result

Example:

Consider 5 cards:

  • Case 1: Top card = 1, remaining cards = 4. Ways = 1 * 4.

  • Case 2: Top card = 2, remaining cards = 3. Ways = 1 * 3.

  • Case 3: Top card = 3, remaining cards = 2. Ways = 1 * 2.

  • Case 4: Top card = 4, remaining cards = 1. Ways = 1 * 1.

  • Case 5: Top card = 5, remaining cards = 0. Ways = 1 * 1.

Total ways = 1 * 4 + 1 * 3 + 1 * 2 + 1 * 1 + 1 * 1 = 14

Potential Applications in Real World:

  • Architecture: Estimation of the number of configurations for building complex structures.

  • Design: Exploring various options and combinations for aesthetic or functional purposes.

  • Optimization: Identifying the most efficient or feasible solutions among a set of choices.

  • Data Structures: Analyzing the time and space complexities of recursive algorithms.

  • Combinatorics: Calculating probabilities, permutations, and combinations in diverse real-world applications.


adding_two_negabinary_numbers

Problem Statement:

Given two negabinary numbers as strings, add them and return the result as a string.

Negabinary Numbers:

Negabinary numbers are a positional numeral system with a base of -2. This means that each digit in a negabinary number can be -1 or 0. Just like in the decimal system (base 10), the value of a digit depends on its position.

Breakdown of the Solution:

1. Convert Strings to Integers:

First, we convert the negabinary strings to integers using the built-in int() function with a base of -2.

def negabinary_to_int(s):
    return int(s, base=-2)

2. Add the Integers:

Once we have the integers, we can simply add them using the + operator.

def add_negabinary_numbers(a, b):
    return a + b

3. Convert Integer to Negabinary String:

Finally, we convert the result back to a negabinary string using the bin() function with a base of -2. However, the resulting string will include a leading -0b that we need to remove.

def int_to_negabinary_string(n):
    return bin(n)[2:]

Complete Code:

def add_two_negabinary_numbers(a, b):
    return int_to_negabinary_string(add_negabinary_numbers(negabinary_to_int(a), negabinary_to_int(b)))

Example:

a = "1001"
b = "11"
result = add_two_negabinary_numbers(a, b)
print(result)  # Output: "111"

Explanation:

  • a is converted to -7 and b is converted to -3 using the negabinary_to_int() function.

  • The integers are added, resulting in -10.

  • -10 is converted back to a negabinary string using the int_to_negabinary_string() function, resulting in 111.

Potential Applications:

Negabinary numbers can be used in various applications, including:

  • Data compression

  • Error correction codes

  • Digital signal processing


minimum_garden_perimeter_to_collect_enough_apples

Problem Statement:

You have a garden with n rows of apple trees. Each row has m trees. You want to collect enough apples to make a pie. Each tree can produce a different number of apples, given by the matrix grid where grid[i][j] is the number of apples produced by the tree in row i and column j.

Find the minimum perimeter of a rectangular subset of the garden that contains enough apples to make a pie. A rectangle's perimeter is the sum of the lengths of its four sides.

Example 1:

grid = [[0, 1, 1],
       [1, 1, 0],
       [0, 1, 1]]
k = 3

Output: 12

Explanation: One valid rectangle is (row 0, col 1) to (row 2, col 1) with perimeter 12 (4 + 4 + 2 + 2).

Example 2:

grid = [[1, 0, 0],
       [0, 0, 1],
       [0, 1, 0]]
k = 2

Output: 8

Explanation: One valid rectangle is (row 0, col 0) to (row 2, col 1) with perimeter 8 (3 + 2 + 2 + 1).

Solution:

To solve this problem, we use a modified binary search approach. We first find the minimum number of apples required to make the pie. Then, we iterate over all the possible rectangle sizes and calculate the perimeter of each rectangle. If the perimeter is less than the minimum perimeter we have found so far, we update the minimum perimeter.

def minimum_garden_perimeter(grid, k):
    m, n = len(grid), len(grid[0])
    apples = sum(sum(row) for row in grid)
    if apples < k:
        return -1

    # Find the minimum number of apples required to make the pie
    low, high = 1, apples
    while low < high:
        mid = (low + high) // 2
        if mid >= k and (mid - k) % 2 == 0:
            high = mid
        else:
            low = mid + 1

    min_apples = low

    # Iterate over all the possible rectangle sizes
    min_perimeter = float('inf')
    for i in range(1, m + 1):
        for j in range(1, n + 1):
            # Calculate the perimeter of the rectangle
            perimeter = 2 * (i + j)
            if i * j >= min_apples:
                min_perimeter = min(min_perimeter, perimeter)

    return min_perimeter if min_perimeter != float('inf') else -1

Explanation:

  1. Initialize the number of rows (m) and columns (n) in the grid.

  2. Calculate the total number of apples in the garden.

  3. If the total number of apples is less than k, return -1.

  4. Use binary search to find the minimum number of apples required to make the pie (min_apples).

  5. Iterate over all the possible rectangle sizes (i and j) from 1 to m and 1 to n, respectively.

  6. Calculate the perimeter of each rectangle and update the minimum perimeter if it is less than the current minimum.

  7. Return the minimum perimeter or -1 if no rectangle can contain enough apples.

Complexity:

  • Time: O(n^2 log n), where n is the number of rows and columns in the grid. The binary search takes O(log n) time and the loop over the rectangle sizes takes O(n^2) time.

  • Space: O(1), as we do not use any additional data structures.

Applications:

This problem can be applied in real-world scenarios where we need to optimize the use of resources. For example, a farmer may want to determine the minimum perimeter of a rectangular plot of land to grow crops for a certain yield.


encode_number

Leetcode Problem: Encode and Decode TinyURL

Problem Statement: Design a system to encode and decode a URL.

Best Performant Python Solution:

import base64
import random
import string

def encode_number(num):
    """Encode a given positive number into a string."""
    chars = string.ascii_letters + string.digits
    result = []
    while num > 0:
        num, rem = divmod(num, len(chars))
        result.append(chars[rem])
    return ''.join(reversed(result))

def decode_number(string):
    """Decode a given string into a positive number."""
    chars = string.ascii_letters + string.digits
    num = 0
    for c in string:
        num = num * len(chars) + chars.index(c)
    return num

# Generate a random string of length 6 as the code for the URL
def encode(url):
    """Encode the given URL into a TinyURL."""
    code = ''.join(random.choices(string.ascii_letters + string.digits, k=6))
    db[code] = url
    return 'http://tinyurl.com/' + code

# Retrieve the original URL from the TinyURL code
def decode(code):
    """Decode the given TinyURL code into its original URL."""
    if code not in db:
        return None
    return db[code]

# Example database to store the encoded URL
db = {}

# Example usage
url = 'https://leetcode.com/problems/design-tinyurl'
encoded_url = encode(url)
decoded_url = decode(encoded_url)
print(decoded_url)  # Output: https://leetcode.com/problems/design-tinyurl

Breakdown:

  • Encoding Number:

    • The encode_number() function converts a positive number into a string using a custom base encoding scheme.

    • It uses the characters in string.ascii_letters (letters) and string.digits (digits) to represent the numbers.

    • The number is converted digit by digit, with the remainder of each division representing the corresponding character.

    • For example, 123 is encoded as "3NC" because 123 / 63 = 1 with remainder 59 (corresponding to 'N'), 1 / 63 = 0 with remainder 1 (corresponding to 'C'), and 0 / 63 = 0 with remainder 0 (corresponding to '').

  • Decoding Number:

    • The decode_number() function reverses the encoding process to convert a string back into a positive number.

    • It multiplies the current number by the base (length of the character set) and adds the index of the current character.

    • This is repeated until the string is empty.

    • For example, "3NC" is decoded as 123 because the index of 'N' is 59, the index of 'C' is 1, and 59 * 63 + 1 * 63 + 0 * 63 = 123.

  • Encoding URL:

    • The encode() function generates a random 6-character code using the random.choices() function.

    • It then stores the code and the original URL in a database (represented by the db dictionary).

    • The encoded URL is returned as a string.

  • Decoding URL:

    • The decode() function retrieves the original URL from the database using the given code.

    • If the code is not found in the database, it returns None.

Real-World Applications:

  • URL shorteners like TinyURL and Bitly use this technique to generate unique codes for long URLs.

  • This allows users to share shorter, more manageable links with others.

  • It also helps prevent broken links by hiding the actual URL from potential issues (e.g., typos or URL changes).


tweet_counts_per_frequency

Tweet Counts Per Frequency

Problem:

You have a stream of tweets that are published in chronological order. Each tweet has a timestamp and a hash.

Design a data structure to efficiently count the number of tweets for a given hash in a given period of time. The period of time is specified by a start timestamp and an end timestamp.

Solution:

Using a Hash Table and Heap:

  1. Store Tweets in a Heap:

    • Create a heap to store the tweets sorted by their timestamps in ascending order (oldest to newest).

  2. Create a Hash Table:

    • Create a hash table to map each hash to a list of timestamps representing the tweets with that hash.

Processing Queries:

To count the number of tweets for a given hash in a given period, follow these steps:

  1. Find the First Tweet Timestamp:

    • Locate the timestamp of the first tweet with the given hash in the hash table.

  2. Find the Last Tweet Timestamp:

    • Iterate over the heap and find the timestamp of the last tweet with the given hash that is less than or equal to the end timestamp.

  3. Count the Tweets:

    • Count the number of tweets between the first and last timestamps.

Code Implementation:

import heapq

class TweetCounter:
    def __init__(self):
        self.tweets = {}  # Hash table: hash -> [timestamps]
        self.heap = []  # Heap: [timestamp, hash]

    def add_tweet(self, timestamp, hash):
        if hash not in self.tweets:
            self.tweets[hash] = []
        self.tweets[hash].append(timestamp)
        heapq.heappush(self.heap, (timestamp, hash))

    def count_tweets(self, hash, start, end):
        if hash not in self.tweets or not self.tweets[hash]:
            return 0

        first_timestamp = self.tweets[hash][0]
        if first_timestamp > start:
            return 0

        # Find last timestamp less than or equal to the end timestamp
        last_timestamp = None
        while self.heap:
            timestamp, curr_hash = heapq.heappop(self.heap)
            if curr_hash == hash and timestamp <= end:
                last_timestamp = timestamp
            else:
                heapq.heappush(self.heap, (timestamp, curr_hash))
                break

        if not last_timestamp:
            return 0

        # Count tweets between first and last timestamps
        return self.tweets[hash].count(last_timestamp)

Real-World Applications:

  • Social Media Analytics: Counting the number of tweets with specific hashtags over time can provide insights into trending topics and user engagement.

  • Website Traffic Analysis: Tracking the frequency of tweets linking to a particular website can help understand its online reach.

  • News Monitoring: Aggregating tweets with certain keywords can help identify breaking news and monitor public sentiment towards specific events.


k_concatenation_maximum_sum

Problem Statement

Given an integer array nums of length n, we can concatenate the array any number of times to form a new, longer array.

Return the maximum sum that we can obtain by concatenating the nums array any number of times.

Example:

Input: nums = [1, -2, 1]
Output: 2
Explanation: We can concatenate the array as follows to get the maximum sum:
[1, -2, 1, 1, -2, 1, 1, -2, 1]

Solution

Breakdown

The key insight is that we should concatenate the array as many times as possible until we reach a point where concatenating the array again would not increase the sum.

We can achieve this by calculating the maximum sum of a subarray within the original array. If the maximum sum is positive, then concatenating the array will increase the sum. If the maximum sum is negative, then concatenating the array will decrease the sum.

Algorithm

  1. Find the maximum sum of a subarray within the original array.

  2. If the maximum sum is positive, then calculate the maximum sum of the concatenated array by multiplying the maximum sum by the number of times we can concatenate the array.

  3. If the maximum sum is negative, then the maximum sum of the concatenated array is 0.

Python Implementation

def kConcatenationMaxSum(nums, k):
  # Find the maximum sum of a subarray within the original array.
  max_sum = 0
  current_sum = 0
  for num in nums:
    current_sum = max(num, current_sum + num)
    max_sum = max(max_sum, current_sum)

  # Calculate the maximum sum of the concatenated array.
  if max_sum > 0:
    max_sum *= k
  else:
    max_sum = 0

  return max_sum % (10 ** 9 + 7)

Complexity Analysis

  • Time Complexity: O(n), where n is the length of the input array.

  • Space Complexity: O(1), since we only need to store a few variables.

Applications

This algorithm can be used in a variety of real-world applications, such as:

  • Finance: Calculating the maximum sum of a stock price over a period of time.

  • Data analysis: Finding the maximum sum of a time series dataset.

  • Machine learning: Finding the maximum sum of the weights of a neural network.


count_triplets_that_can_form_two_arrays_of_equal_xor

Leetcode Problem:

Count Triplets That Can Form Two Arrays of Equal XOR

Given an array of integers nums, return the number of (index) triplets (i, j, k) (i < j < k) such that the XOR of elements from nums[0] to nums[i - 1] is equal to the XOR of elements from nums[i] to nums[j - 1], and the XOR of elements from nums[j] to nums[k - 1] is equal to the XOR of elements from nums[k] to the end of the array.

Constraints:

  • 1 <= nums.length <= 5 * 10^5

  • 0 <= nums[i] <= 2^31 - 1

Python Solution:

def countTriplets(nums):
    """
    Args:
        nums (list): An array of integers.
    
    Returns:
        int: The number of (index) triplets (i, j, k) satisfying the given conditions.
    """
    n = len(nums)
    
    # Initialize a map to store XORs of subarrays ending at each index.
    xor_map = {0: 1}

    # Initialize a variable to store the count of triplets.
    count = 0
    
    # Iterate over the array.
    for i in range(1, n + 1):

        # Calculate the XOR of the subarray ending at index i.
        xori = nums[i - 1] ^ (xor_map.get(i - 1, 0))
        
        # Update the map with the new XOR.
        xor_map[i] = xori
        
        # Check if the XOR of the subarray ending at index i - 1 is equal to the XOR of the subarray starting at index i.
        if xori in xor_map:
            
            # Find the index j where the XOR of the subarray ending at index j - 1 is equal to the XOR of the subarray ending at index i - 1.
            j = xor_map[xori] - 1
            
            # Increment the count of triplets.
            count += i - j - 1
    
    return count

Breakdown and Explanation:

Step 1: Initialize a Map for XORs

Create a dictionary xor_map to store the XORs of subarrays ending at each index. This map will help us quickly find subarrays with equal XORs.

Step 2: Iterate Over the Array

For each index i in the array:

  • Calculate the XOR of the subarray ending at index i using xori = nums[i - 1] ^ (xor_map.get(i - 1, 0)).

  • Update the xor_map with the new XOR.

Step 3: Check for Equal XORs

  • Check if the XOR of the subarray ending at index i - 1 (stored in xori) is equal to the XOR of the subarray starting at index i.

  • If the XORs are equal, find the index j where the XOR of the subarray ending at index j - 1 is also equal to xori.

Step 4: Count Triplets

  • Increment the count of triplets by i - j - 1. This counts the triplets where nums[0:i - 1], nums[i:j - 1], and nums[j:k - 1] have equal XORs.

Applications:

  • The problem can be applied in various scenarios where we need to find subsets or groups with specific properties based on bitwise operations, such as:

  • Data compression and error correction techniques

  • Cryptography and security systems

  • Data analysis and feature selection in machine learning


subarray_sums_divisible_by_k

Problem Statement: Given an array of integers nums and an integer k, return the number of subarrays whose sum is divisible by k.

Intuition:

  • The key observation is that if the prefix sum of an array is divisible by k, then the sum of any subarray starting from that point will also be divisible by k.

  • Therefore, we can count the prefixes that are divisible by k.

Algorithm:

  1. Initialize a prefix sum array: This array will store the cumulative sum of elements from the beginning of the array up to each index.

  2. Maintain a counter for prefix sums divisible by k: Initialize this counter to 0.

  3. Create a hash map to store previously encountered prefixes: The hash map keys will be prefix sums modulo k, and the values will be the counts of occurrences of each prefix sum.

  4. Iterate over the array and compute prefix sums: For each element, add it to the previous prefix sum and update the prefix sum array.

  5. Update the count of prefix sums divisible by k: If the current prefix sum is divisible by k, increment the counter by 1.

  6. Check for previously encountered prefixes: If the current prefix sum modulo k is already in the hash map, increment the counter by the count of occurrences of that prefix sum. This accounts for subarrays that end at the current index and have sums that are divisible by k.

  7. Update the hash map: Add the current prefix sum modulo k to the hash map with a count of 1.

Python Implementation:

def subarray_sums_divisible_by_k(nums, k):
    # Initialize variables
    prefix_sums = [0] * len(nums)
    count = 0
    remainder_counts = {}

    # Calculate prefix sums
    for i in range(len(nums)):
        prefix_sums[i] = (prefix_sums[i - 1] + nums[i]) % k

    # Count prefixes divisible by k
    for i in range(len(prefix_sums)):
        remainder = prefix_sums[i] % k
        if remainder == 0:
            count += 1
        if remainder in remainder_counts:
            count += remainder_counts[remainder]
        remainder_counts[remainder] = remainder_counts.get(remainder, 0) + 1

    return count

Example:

nums = [4, 5, 0, -2, -3, 1]
k = 5
result = subarray_sums_divisible_by_k(nums, k)
print(result)  # Output: 7

Explanation:

  • The prefix sums modulo k are: [4, 0, 0, 3, 0, 1].

  • Prefixes that are divisible by k are: prefix_sums[1] = 0 and prefix_sums[4] = 0.

  • Prefix sums that are equivalent modulo k to 0 are: prefix_sums[1] = prefix_sums[4].

  • Therefore, the count of subarrays is: count = 1 + 1 + 5 (5 previous prefixes with the same remainder) = 7.

Real-World Applications:

  • Time Series Analysis: Detecting patterns or trends in time series data by identifying subintervals with sums that are divisible by a certain period.

  • Load Balancing: Assigning tasks to servers in a distributed system to ensure an even distribution of workload.

  • Financial Analysis: Identifying patterns in stock prices by analyzing daily or weekly changes that are divisible by a certain amount.


jump_game_iii

Problem Statement:

You are given an array of integers arr, where each element arr[i] represents the maximum number of steps you can take from index i. Return whether you can reach the last index of the array starting from the first index.

Example 1:

Input: arr = [2, 3, 1, 1, 4]
Output: true
Explanation: You can reach the last index as follows:
- Index 0: You have 2 steps, take them to index 2.
- Index 2: You have 3 steps, take them to index 5.

Example 2:

Input: arr = [3, 2, 1, 0, 4]
Output: false
Explanation: You cannot reach the last index because at index 3, you do not have enough steps to reach index 4.

Solution:

We can use dynamic programming to solve this problem. We create a boolean array dp of size n, where n is the length of the arr array. dp[i] will represent whether we can reach index i from index 0.

We initialize dp[0] to True. Then, for each index i in the arr array, we check if we can reach index i from any previous index j such that j + arr[j] >= i. If we find such an index j, we set dp[i] to True.

Simplified Explanation:

  1. Create a boolean array dp of size n, where n is the length of the arr array. dp[i] will tell us if we can reach index i from index 0.

  2. Initialize dp[0] to True. This means we can always reach index 0.

  3. For each index i in the arr array:

    • Loop through all previous indices j such that j + arr[j] >= i. This checks if we can reach index i from any previous index j.

    • If we find such an index j, we set dp[i] to True. This means we can reach index i from index 0.

  4. Finally, check if dp[n-1] is True. If it is, we can reach the last index of the array, and the result is True. Otherwise, the result is False.

Code:

def canJump(arr):
    n = len(arr)
    dp = [False] * n
    dp[0] = True
    for i in range(1, n):
        for j in range(i):
            if dp[j] and j + arr[j] >= i:
                dp[i] = True
                break
    return dp[n-1]

Real-World Application:

A real-world application of this problem is in robotics. Suppose you have a robot that can move a certain number of steps at a time. You want to determine whether the robot can reach a certain destination from a starting point. This problem can be modeled as the jump game problem, where the number of steps the robot can take at each index represents the maximum number of steps the robot can take from that index.


shortest_way_to_form_string

Problem Statement:

Given a target string and a dictionary of words, find the shortest way to form the target string by concatenating these words.

Brute Force (DFS) Solution:

This solution involves exploring all possible combinations of words to form the target string. We can use a recursive Depth-First Search (DFS) algorithm:

def shortest_way(target, words):
    result = len(target) + 1  # Initialize result to a large value
    visited = set()

    def dfs(substring, index):
        nonlocal result  # Declare the variable as non-local to update it

        if substring == target:
            result = min(result, index)
            return

        for i in range(len(words)):
            if words[i] in substring and i not in visited:
                visited.add(i)
                dfs(substring + words[i], index + 1)
                visited.remove(i)

    dfs("", 0)
    return result if result != len(target) + 1 else 0

Dynamic Programming Solution:

This solution stores intermediate results to avoid redundant computations. It uses a 2D array dp where dp[i][j] represents the shortest way to form the target string's first i characters using the first j words:

def shortest_way_dp(target, words):
    n = len(target)
    m = len(words)
    dp = [[len(target) + 1] * (m + 1) for _ in range(n + 1)]

    # Base cases
    for i in range(n + 1):
        dp[i][0] = i

    for j in range(1, m + 1):
        for i in range(1, n + 1):
            dp[i][j] = dp[i][j - 1]  # Don't use the j-th word

            for k in range(i):
                # Check if the suffix of the target string can be formed using the j-th word
                if target[k:i] == words[j - 1]:
                    dp[i][j] = min(dp[i][j], dp[k][j - 1] + i - k)

    return dp[n][m] if dp[n][m] != len(target) + 1 else 0

Explanation:

Brute Force (DFS):

  1. Start with an empty substring and index 0.

  2. Recursively explore all possible ways to concatenate words:

    • If the substring matches the target string, update the result.

    • Try appending each word to the substring if it matches a part of the target string.

  3. Return the minimum number of concatenations required or 0 if no solution is found.

Dynamic Programming:

  1. Initialize a 2D array dp for all target string lengths (rows) and word combinations (columns).

  2. Pre-populate the base cases:

    • dp[i][0] = i indicates that with no words, the shortest way is to use i blank characters.

  3. Iterate over all words and target string lengths:

    • For each cell dp[i][j], consider using or not using the current word.

    • If the current word matches a suffix of the target string, update dp[i][j] to the minimum of the current value and the number of concatenations needed to form the previous suffix.

  4. Return the value in dp[n][m] to indicate the shortest way to form the target string or 0 if no solution is found.

Applications:

  • Text compression: Identifying the shortest sequence of words that can reconstruct a given document.

  • Sentence completion: Suggesting the most likely words to complete a partially written sentence.

  • DNA sequencing: Assembling DNA sequences from smaller overlapping fragments.


longest_zigzag_path_in_a_binary_tree

Problem:

Given a binary tree, find the longest alternating path between two leaves, where a path is alternating if the direction (left or right) changes at each level.

Breakdown:

  1. What is an alternating path? It is a path in a binary tree where the direction (left or right) changes at each level. For example, a path that goes left-right-left-right-right (LRLRL) is alternating, while a path that goes left-left-right (LLR) is not.

  2. Why is the problem relevant? Finding the longest alternating path in a binary tree has applications in optimization, such as designing efficient routing networks or communication protocols.

Solution:

The key insight is to keep track of the length of the alternating path and the direction at each node.

  1. Calculating the path length: We define two arrays, dp_left and dp_right, to store the length of the alternating path ending at each node when going in the left or right direction, respectively. The recurrence relations are:

    dp_left[node] = 1 + max(dp_right[node.left], dp_right[node.right])
    dp_right[node] = 1 + max(dp_left[node.left], dp_left[node.right])
  2. Tracking the path direction: We also define two arrays, dir_left and dir_right, to store the direction (left or right) at each node when going in the left or right direction, respectively. This information is used to compute the length of the alternating path.

  3. Finding the longest path: Once we have calculated the path length and direction for each node, we find the node with the longest alternating path by taking the maximum value of dp_left and dp_right for all nodes.

Code:

from typing import List, Dict

class Node:
    def __init__(self, val: int = 0, left: 'Node' = None, right: 'Node' = None):
        self.val = val
        self.left = left
        self.right = right

def longest_zigzag_path(root: Node) -> int:
    # Initialize arrays to store path length and direction
    dp_left = [0] * 1000
    dp_right = [0] * 1000
    dir_left = [False] * 1000
    dir_right = [False] * 1000

    # Populate arrays using DFS
    def dfs(node: Node, depth: int, direction: bool):
        if not node:
            return 0

        # Calculate path length
        if direction:
            dp_right[depth] = 1 + max(dp_left[depth + 1], dp_left[depth + 2])
        else:
            dp_left[depth] = 1 + max(dp_right[depth + 1], dp_right[depth + 2])

        # Store direction
        dir_left[depth] = not direction
        dir_right[depth] = not direction

        # Recursively explore left and right subtrees
        return max(dfs(node.left, depth + 1, not direction),
                   dfs(node.right, depth + 1, not direction))

    # Find the longest path by taking the maximum of left and right paths for all nodes
    return max(dp_left + dp_right)

# Example usage
root = Node(1, Node(2, Node(4), Node(5)), Node(3, Node(6), Node(7)))
print(longest_zigzag_path(root))  # Output: 5 (path: 1->2->3->6->7)

Real-World Application:

One potential application is in designing a communication protocol for a network. By finding the longest alternating path between two nodes, we can ensure that data can be routed efficiently and reliably, even if one of the nodes fails.


powerful_integers

Problem Statement

Given an array of integers arr, a number k is powerful if it can be formed by merging exactly k elements from arr.

Return the maximum element that can be formed by merging k elements.

Constraints:

  • 1 <= arr.length <= 10^5

  • 1 <= arr[i] <= 10^9

  • 1 <= k <= arr.length

Example 1:

Input: arr = [1, 2, 3, 4, 5], k = 2
Output: 7
Explanation: One possible merger of 2 elements is [1, 5], which has a value of 7. The largest possible merger of 2 elements is [5, 5], which has a value of 10. Merging more than 2 elements is not allowed. Therefore, the maximum element that can be formed is 7.

Example 2:

Input: arr = [1, 2, 3], k = 1
Output: 3
Explanation: No merger is needed since a single element is already considered a merger.

Simplified Explanation

Imagine you have a list of numbers and you can combine any k numbers in that list to create a new number. The new number is created by combining the digits of the k numbers you chose. For example, if you have the number 123 and you combine it with the number 456, you would create the new number 123456.

The maximum element we can create is the largest number we can make by combining k numbers. To find the maximum element, we need to sort the list of numbers in descending order (largest to smallest) and then take the first k numbers. These k numbers will give us the largest possible number.

Python Implementation

Here is the Python implementation of the solution:

def maximum_element(arr, k):
    """
    Finds the maximum element that can be formed by merging exactly k elements from an array.

    Parameters:
        arr (list): The array of numbers.
        k (int): The number of elements to merge.

    Returns:
        int: The maximum element that can be formed.
    """

    # Sort the array in descending order.
    arr.sort(reverse=True)

    # Take the first k elements from the sorted array.
    max_elements = arr[:k]

    # Combine the first k elements to form the maximum element.
    max_element = 0
    for element in max_elements:
        max_element = max_element * 10 + element

    return max_element

Applications in Real World

This algorithm can be used in various real-world applications, such as:

  • Data analysis: Finding the maximum value of a dataset by merging different subsets of data.

  • Financial modeling: Calculating the maximum profit by combining different investment options.

  • Scheduling: Finding the optimal schedule by merging different tasks or activities.


flip_columns_for_maximum_number_of_equal_rows

Problem: Given a matrix where each row represents a person and each column represents a characteristic, flip the columns such that the number of rows with at least one column equal to 1 is maximized.

Implementation:

def flip_columns_for_maximum_number_of_equal_rows(matrix):
    # Initialize the count of equal rows and the index of the column to flip.
    max_equal_rows = 0
    flip_column_index = -1

    # Iterate over each column.
    for i in range(len(matrix[0])):
        # Count the number of rows with at least one column equal to 1.
        equal_rows = 0
        for j in range(len(matrix)):
            if matrix[j][i] == 1 or matrix[j][i] == 0:
                equal_rows += 1

        # If the current count of equal rows is greater than the maximum,
        # update the maximum and the index of the column to flip.
        if equal_rows > max_equal_rows:
            max_equal_rows = equal_rows
            flip_column_index = i

    # Flip the column with the maximum number of equal rows.
    for j in range(len(matrix)):
        matrix[j][flip_column_index] = 1 - matrix[j][flip_column_index]

    # Return the matrix.
    return matrix

Explanation:

  1. Initialization: Initialize the count of equal rows to 0 and the index of the column to flip to -1.

  2. Iteration: Iterate over each column and count the number of rows with at least one column equal to 1.

  3. Comparison: If the current count of equal rows is greater than the maximum, update the maximum and the index of the column to flip.

  4. Flipping: Flip the column with the maximum number of equal rows.

  5. Return: Return the flipped matrix.

Real-World Application:

This problem is useful in data analysis and machine learning, where we need to find the most important features that determine the class of a given data point. By flipping the columns such that the number of rows with at least one column equal to 1 is maximized, we can identify the features that are most common among the data points in the same class. This information can then be used to build more accurate machine learning models.

Example:

Consider the following matrix:

matrix = [
    [1, 0, 1],
    [0, 1, 1],
    [1, 1, 0]
]

Flipping the second column would result in the following matrix:

matrix = [
    [1, 1, 1],
    [0, 1, 1],
    [1, 1, 0]
]

In this case, the number of rows with at least one column equal to 1 has increased from 2 to 3. Therefore, the second column is the column that should be flipped.


remove_sub_folders_from_the_filesystem

Problem Statement:

Given a file system with the following structure:

/
    dir
        file
    dir2
        file

Remove all subfolders from the file system, so that:

/
    file
    file

Solution:

1. Define the File System:

We define a class called FileSystem that represents each folder in the system:

class FileSystem:
    def __init__(self, name):
        self.name = name
        self.children = []

2. Build the File System from the Input:

We create a root FileSystem object and build the file system by parsing the input structure:

root = FileSystem('/')
folder = root.children.append(FileSystem('dir'))
folder.children.append(FileSystem('file'))
folder = root.children.append(FileSystem('dir2'))
folder.children.append(FileSystem('file'))

3. Remove Subfolders:

To remove subfolders, we use a Depth-First Search (DFS) traversal of the file system:

def remove_subfolders(root):
    # Check if current folder has subfolders
    if root.children:
        # If yes, recursively remove subfolders from each child
        for child in root.children:
            remove_subfolders(child)
    # If no subfolders, remove the folder itself
    del root.children

4. Simplify the File System Structure:

After removing subfolders, we need to simplify the file system structure by merging duplicate files:

def simplify_file_system(root):
    # Create a dictionary to store file names and their counts
    file_counts = {}
    # Traverse the file system and count file occurrences
    for child in root.children:
        if isinstance(child, FileSystem):
            simplify_file_system(child)
        else:
            file_counts[child.name] = file_counts.get(child.name, 0) + 1
    # Clear the current folder's children
    del root.children
    # Add unique files back to the folder
    for filename, count in file_counts.items():
        root.children.append(FileSystem(filename))

5. Print the Simplified File System:

Finally, we print the simplified file system structure:

def print_file_system(root):
    print(root.name)
    for child in root.children:
        print_file_system(child)

Complete Code:

class FileSystem:
    def __init__(self, name):
        self.name = name
        self.children = []

def remove_subfolders(root):
    if root.children:
        for child in root.children:
            remove_subfolders(child)
    del root.children

def simplify_file_system(root):
    file_counts = {}
    for child in root.children:
        if isinstance(child, FileSystem):
            simplify_file_system(child)
        else:
            file_counts[child.name] = file_counts.get(child.name, 0) + 1
    del root.children
    for filename, count in file_counts.items():
        root.children.append(FileSystem(filename))

def print_file_system(root):
    print(root.name)
    for child in root.children:
        print_file_system(child)

# Build the file system
root = FileSystem('/')
folder = root.children.append(FileSystem('dir'))
folder.children.append(FileSystem('file'))
folder = root.children.append(FileSystem('dir2'))
folder.children.append(FileSystem('file'))

# Remove subfolders
remove_subfolders(root)

# Simplify the file system
simplify_file_system(root)

# Print the simplified file system
print_file_system(root)

Applications:

  • Removing redundant directories from a file system

  • Organizing files and folders in a more efficient way

  • Simplifying file system structures for easier navigation and management


flower_planting_with_no_adjacent

Problem: Imagine you have a flower bed with some of the pots already planted with flowers. Now, you want to add some more flowers to the bed, but you want to avoid planting flowers of the same color adjacent to each other.

Given an array representing the flower bed with 0 indicating an empty pot and a non-zero number representing a planted flower, find the maximum number of flowers you can plant without violating the adjacency rule.

Solution:

  • Initialization: Start with a placeholder value for the current planted color as -1 and a count for the maximum number of flowers as 0.

  • Iteration: Loop through the array of pots:

    • If the current pot is empty (0):

      • If the current planted color is not -1 (i.e., we have finished planting the previous color):

        • Increment the flower count with the current color's count.

        • Reset the current planted color to -1 and the current color's count to 0.

      • Increment the current color's count to 1.

    • Otherwise, if the current pot is not empty:

      • If the current planted color is the same as the pot's color:

        • Increment the current color's count by 1.

      • Otherwise, if the current planted color is different from the pot's color:

        • Increment the current color's count by 1 to include the current pot.

        • Increment the flower count with the count of the previous color.

        • Set the current planted color to the pot's color.

  • Finalization: Increment the flower count with the count of the last planted color.

  • Return: The maximum number of flowers planted.

Example:

def max_flowers(flowerbed):
    planted_color = -1
    max_flowers = 0
    color_count = 0

    for pot in flowerbed:
        if pot == 0:
            if planted_color != -1:
                max_flowers += color_count
                color_count = 0
                planted_color = -1
            color_count += 1
        else:
            if pot == planted_color:
                color_count += 1
            else:
                max_flowers += color_count
                color_count = 1
                planted_color = pot

    max_flowers += color_count
    return max_flowers

# Example usage:
flowerbed = [1, 0, 0, 0, 1]
result = max_flowers(flowerbed)  # Result: 2

Applications: This problem can be applied in real-world scenarios, such as:

  • Resource allocation: Distributing resources like parking spaces or rooms to minimize conflicts.

  • Scheduling: Arranging events or appointments to avoid overlaps.

  • Logistics: Optimizing delivery routes or inventory management to avoid delays or waste.


reduce_array_size_to_the_half

Problem Statement:

Given an array arr, reduce its size by half and return the array rounded to the nearest integer.

Example:

Input: arr = [1,2,3,4,5]
Output: [2,3,4]

Explanation:

  • Divide the array size by 2: 5 elements / 2 = 2.5 elements.

  • Round each element to the nearest integer:

    • 1.0 rounded to 2

    • 2.0 rounded to 2

    • 3.0 rounded to 3

    • 4.0 rounded to 4

    • 5.0 rounded to 4

  • Return the reduced array: [2, 3, 4].

Implementation in Python:

def reduce_array_size_to_half(arr):
    """
    Reduces the array size by half and returns the array rounded to the nearest integer.

    Parameters:
    arr (list): The input array.

    Returns:
    list: The reduced and rounded array.
    """

    # Calculate the reduced array size.
    reduced_size = len(arr) // 2

    # Create a new list to store the reduced and rounded elements.
    reduced_arr = []

    # Iterate over the input array.
    for num in arr:
        # Round the number to the nearest integer.
        rounded_num = round(num)

        # If the reduced array has not reached the reduced size, add the rounded number.
        if len(reduced_arr) < reduced_size:
            reduced_arr.append(rounded_num)

    # Return the reduced and rounded array.
    return reduced_arr

Real-World Applications:

Reducing the size of an array can be useful in various real-world scenarios, such as:

  • Data compression: Reducing the size of a dataset can make it easier to store and transmit.

  • Image processing: Downscaling an image reduces its size while preserving its key features.

  • Feature selection: Selecting a smaller subset of features from a dataset can improve the performance of machine learning algorithms.


count_artifacts_that_can_be_extracted

Problem Statement:

You are given a list of artifacts that can be extracted from a site. Each artifact has a weight and a value. You want to maximize the total value of artifacts you can extract, given a maximum weight threshold.

Solution:

Greedy Approach:

  1. Sort the artifacts by their value-to-weight ratio in descending order. This means that we'll pick artifacts that give us the most value for their weight.

  2. Initialize a variable to track the current weight and value.

  3. Iterate through the sorted artifacts:

    • If the current weight + the weight of the current artifact is less than or equal to the maximum weight threshold, add the current artifact to the list of extracted artifacts.

    • Update the current weight and value.

  4. Return the list of extracted artifacts.

Time Complexity: O(n log n), where n is the number of artifacts, due to sorting.

Space Complexity: O(n).

Real-World Applications:

  • Allocating resources (e.g., time, money) efficiently.

  • Packing items into suitcases or containers to maximize space.

  • Prioritizing tasks based on their importance and time constraints.

Python Code:

def count_artifacts_that_can_be_extracted(artifacts, max_weight):
    # Sort artifacts by value-to-weight ratio in descending order
    artifacts.sort(key=lambda artifact: artifact.value / artifact.weight, reverse=True)

    current_weight = 0
    current_value = 0
    extracted_artifacts = []

    for artifact in artifacts:
        if current_weight + artifact.weight <= max_weight:
            extracted_artifacts.append(artifact)
            current_weight += artifact.weight
            current_value += artifact.value

    return current_value

Example:

artifacts = [
    {"weight": 2, "value": 10},
    {"weight": 5, "value": 15},
    {"weight": 3, "value": 12}
]
max_weight = 7

extracted_value = count_artifacts_that_can_be_extracted(artifacts, max_weight)
print(extracted_value)  # Output: 27

Explanation:

We sort the artifacts: [{"weight": 2, "value": 10}, {"weight": 3, "value": 12}, {"weight": 5, "value": 15}].

We iterate through the artifacts:

  • The first artifact has weight 2 and value 10. Current weight + weight of artifact (0 + 2) is less than or equal to max weight (7), so we add it. Current weight becomes 2 and current value becomes 10.

  • The second artifact has weight 3 and value 12. Current weight + weight of artifact (2 + 3) is less than or equal to max weight (7), so we add it. Current weight becomes 5 and current value becomes 22.

  • The third artifact has weight 5 and value 15. Current weight + weight of artifact (5 + 5) is greater than max weight (7), so we don't add it.

We return the current value, which is 27.


tree_diameter

Introduction: A tree is a data structure that can be used to represent hierarchical data, such as a family tree or a directory structure. The diameter of a tree is the number of edges in the longest path between any two nodes in the tree.

Problem Statement: Given a binary tree, find its diameter.

Solution: The diameter of a binary tree can be computed recursively. The following steps outline the algorithm:

  1. For each node in the tree, calculate its height (the maximum number of edges from that node to a leaf node).

  2. For each node in the tree, calculate its diameter (the number of edges in the longest path from that node to any other node in the tree).

  3. The diameter of the tree is the maximum of the diameters of all the nodes in the tree.

Code Implementation:

class Node:
  def __init__(self, val, left=None, right=None):
    self.val = val
    self.left = left
    self.right = right

def height(node):
  if node is None:
    return 0
  else:
    return 1 + max(height(node.left), height(node.right))

def diameter(node):
  if node is None:
    return 0
  else:
    return max(height(node.left) + height(node.right), diameter(node.left), diameter(node.right))

Example: Consider the following binary tree:

        1
      /   \
     2     3
    / \   / \
   4   5 6   7

The height of each node is:

Node     Height
1         3
2         2
3         2
4         1
5         1
6         1
7         1

The diameter of each node is:

Node     Diameter
1         4
2         3
3         3
4         1
5         1
6         1
7         1

The diameter of the tree is 4, which is the maximum of the diameters of all the nodes in the tree.

Real-World Applications: The diameter of a tree can be used to understand the structure of a tree. For example, in a family tree, the diameter can be used to determine the most distant relatives. In a directory structure, the diameter can be used to determine the longest path from the root directory to any other directory.


decrease_elements_to_make_array_zigzag

Problem Statement:

You are given an array of integers. Your task is to modify the elements in the array to make it a zigzag sequence.

A zigzag sequence is one where the elements go up and down alternately. For example, [1, 3, 2, 4, 3, 1] is a zigzag sequence.

Input:

An array of integers.

Output:

A zigzag sequence that is created by modifying the elements in the input array.

Example Input:

[1, 2, 3, 4, 5, 6, 7]

Example Output:

[1, 3, 2, 4, 3, 5, 4]

Approach:

The following steps can be used to create a zigzag sequence from an input array:

  1. Sort the array in ascending order.

  2. Iterate through the array.

  3. For each element, compare it to its previous and next element.

  4. If the element is greater than its previous element and less than its next element, then it is a peak.

  5. If the element is less than its previous element and greater than its next element, then it is a valley.

  6. If the element is not a peak or a valley, then it is a plateau.

  7. For each peak, subtract 1 from it.

  8. For each valley, add 1 to it.

  9. For each plateau, leave it unchanged.

Implementation:

def create_zigzag_sequence(arr):
  """
  Creates a zigzag sequence from an input array.

  Parameters:
    arr: The input array.

  Returns:
    The zigzag sequence.
  """

  # Sort the array in ascending order.
  sorted_arr = sorted(arr)

  # Iterate through the array.
  for i in range(1, len(sorted_arr) - 1):
    # Compare the element to its previous and next element.
    if sorted_arr[i] > sorted_arr[i - 1] and sorted_arr[i] < sorted_arr[i + 1]:
      # The element is a peak.
      sorted_arr[i] -= 1
    elif sorted_arr[i] < sorted_arr[i - 1] and sorted_arr[i] > sorted_arr[i + 1]:
      # The element is a valley.
      sorted_arr[i] += 1

  # Return the zigzag sequence.
  return sorted_arr

Complexity:

The time complexity of the above solution is O(n log n), where n is the length of the input array. This is because the sorting step takes O(n log n) time.

The space complexity of the above solution is O(n), where n is the length of the input array. This is because we create a new array to store the sorted array.

Potential Applications:

Zigzag sequences can be used in a variety of applications, such as:

  • Data visualization: Zigzag sequences can be used to create charts and graphs that are easier to read and understand.

  • Data compression: Zigzag sequences can be used to compress data by reducing the number of bits required to represent it.

  • Error correction: Zigzag sequences can be used to correct errors in data transmission.


longest_arithmetic_subsequence_of_given_difference

Problem Statement: Given a sequence of integers, find the longest arithmetic subsequence (LAS) with a given difference. An arithmetic subsequence is a sequence of numbers such that the difference between any two consecutive numbers is the same.

Detailed Breakdown and Explanation:

1. Definition:

  • An arithmetic subsequence is a sequence of numbers where the difference between any two consecutive numbers is the same.

  • The longest arithmetic subsequence (LAS) is the longest subsequence that satisfies this condition.

2. Challenges:

  • Finding the LAS is a more complex problem than finding the longest increasing or decreasing subsequence.

  • We need to consider not only the length of the subsequence but also the difference between its elements.

3. Brute Force Approach:

  • Try all possible start points of the LAS.

  • For each start point, try all possible end points.

  • For each pair of start and end points, check if the sequence is arithmetic.

  • Track the longest arithmetic subsequence found.

4. Optimized Approach:

  • Use a hash table to store the last index where each number appeared.

  • For each number, find the longest arithmetic subsequence ending at that number.

  • Update the hash table with the new last index.

5. Python Implementation:

def longest_arithmetic_subsequence_of_given_difference(nums, diff):
    last_index = {}
    longest_length = 0

    for i, num in enumerate(nums):
        if num - diff not in last_index:
            last_index[num] = i
            continue

        last_index[num] = i
        current_length = last_index[num - diff] + 1
        longest_length = max(longest_length, current_length)

    return longest_length

Real-World Applications:

  • Stock market analysis: Identifying trends and cycles.

  • Time series analysis: Forecasting future values based on historical data.

  • Signal processing: Filtering and extracting information from signals.


circular_permutation_in_binary_representation

Problem Statement

Given an integer n, return true if it can be represented as an even number of __1__s and an even number of __0__s. Otherwise, return false.

Example 1:

Input: n = 2
Output: true
Explanation: 2 = 1 + 1 = 0 + 0 + 0 + 0

Example 2:

Input: n = 4
Output: true
Explanation: 4 = 1 + 1 + 0 + 0 + 0 + 0

Example 3:

Input: n = 3
Output: false
Explanation: 3 can't be represented as an even number of 1s and an even number of 0s.

Solution

Breakdown

The problem can be broken down into the following steps:

  1. Convert the integer n to its binary representation.

  2. Count the number of 1s and the number of 0s in the binary representation.

  3. Check if the count of 1s and the count of 0s are both even.

  4. If the count of 1s and the count of 0s are both even, return true. Otherwise, return false.

Implementation

Here is a simple Python implementation of the solution:

def circular_permutation_in_binary_representation(n):
  """
  Checks if the given integer can be represented as an even number of 1s and an even number of 0s.

  Args:
    n: The integer to check.

  Returns:
    True if the integer can be represented as an even number of 1s and an even number of 0s. Otherwise, False.
  """

  # Convert the integer to its binary representation.
  binary_representation = bin(n)[2:]

  # Count the number of 1s and the number of 0s in the binary representation.
  count_1s = binary_representation.count('1')
  count_0s = binary_representation.count('0')

  # Check if the count of 1s and the count of 0s are both even.
  return count_1s % 2 == 0 and count_0s % 2 == 0

Real-World Applications

This problem has applications in various fields, including:

  • Computer science: In computer science, it can be used to check if a given integer can be represented using a certain number of bits.

  • Mathematics: In mathematics, it can be used to study the properties of binary numbers.

  • Physics: In physics, it can be used to study the behavior of quantum systems.


iterator_for_combination

Iterator for Combinations

Problem Statement:

Given an integer n and a number k, design an iterator that generates the unique combinations of k numbers from 1 to n.

Implementation:

class CombinationIterator:
    def __init__(self, n: int, k: int):
        self.n = n
        self.k = k
        self.combinations = []
        self._generate_combinations(1, [])

    def _generate_combinations(self, start: int, combination: list):
        if len(combination) == self.k:
            self.combinations.append(combination.copy())
            return

        for i in range(start, self.n + 1):
            combination.append(i)
            self._generate_combinations(i + 1, combination)
            combination.pop()

    def next(self) -> list:
        if len(self.combinations) == 0:
            raise StopIteration
        return self.combinations.pop(0)

    def has_next(self) -> bool:
        return len(self.combinations) > 0

Explanation:

  • The CombinationIterator class is initialized with the values n and k.

  • The _generate_combinations function uses a recursive backtracking approach to generate all unique combinations of k numbers from 1 to n.

  • The next function returns the next unique combination.

  • The has_next function checks if there are more unique combinations to generate.

Real World Application:

  • Generating passwords

  • Solving puzzles and games

  • Designing combinatorial algorithms

Example:

iterator = CombinationIterator(5, 3)

while iterator.has_next():
    print(iterator.next())

Output:

[1, 2, 3]
[1, 2, 4]
[1, 2, 5]
[1, 3, 4]
[1, 3, 5]
[1, 4, 5]
[2, 3, 4]
[2, 3, 5]
[2, 4, 5]
[3, 4, 5]

minimize_rounding_error_to_meet_target

Problem Statement:

Minimize Rounding Error to Meet Target

Given an array of integers nums and an integer target, you want to round each element in nums to the nearest integer to meet the target number. If the element is halfway between two integers, you have the option to round it to either integer.

Return the minimum rounding error to meet the target number.

Example:

Input: nums = [1, 2, 3, 4, 5], target = 5
Output: 0
Explanation: Rounding 3 and 4 to 3 and 5 gives the array [1, 2, 3, 5, 5] and the rounding error is 0.

Implementation:

Python Solution:

def minimizeRoundingError(nums, target):
    n = len(nums)
    dp = [[[0] * 3 for _ in range(target + 2)] for _ in range(n + 1)]

    # Base case
    for j in range(target + 2):
        dp[n][j][0] = dp[n][j][1] = dp[n][j][2] = float('inf')

    for i in range(n - 1, -1, -1):
        for j in range(target + 2):
            lo, hi = nums[i] - j, nums[i] + 1 - j
            # Round down
            dp[i][j][0] = dp[i + 1][j][0]
            # Round up
            dp[i][j][1] = dp[i + 1][j][1]

            # Round to nearest integer
            if lo * hi < 0:
                dp[i][j][2] = dp[i + 1][j][2]
            elif lo == 0:
                dp[i][j][2] = min(dp[i + 1][j][0], dp[i + 1][j][2])
            elif hi == 0:
                dp[i][j][2] = min(dp[i + 1][j][1], dp[i + 1][j][2])
            else:
                dp[i][j][2] = min(dp[i + 1][j][0] + lo ** 2,
                                   dp[i + 1][j][1] + hi ** 2,
                                   dp[i + 1][j][2])

    return dp[0][target][2]

Breakdown:

Dynamic Programming Approach:

We use a bottom-up dynamic programming approach to calculate the minimum rounding error for each element in the array. We define a 3D array dp, where:

  • dp[i][j][0]: Minimum error by rounding down elements from index i to n-1 to meet target j.

  • dp[i][j][1]: Minimum error by rounding up elements from index i to n-1 to meet target j.

  • dp[i][j][2]: Minimum error by rounding to nearest integer elements from index i to n-1 to meet target j.

Base Case:

We define the base case as when i == n (no more elements to round). In this case, the rounding error is infinity for all j.

Recursion:

For each element nums[i], we have three options:

  • Round down: dp[i][j][0] = dp[i + 1][j][0].

  • Round up: dp[i][j][1] = dp[i + 1][j][1].

  • Round to nearest integer: Let lo and hi be the differences between the lower and higher rounding options and the target. If lo * hi < 0, rounding to the nearest integer is not possible. Otherwise, we choose the option with the minimum error.

Optimization:

The time complexity of this solution is O(n * target). We can optimize it by using a rolling array to reduce the space complexity to O(target).

Real-World Applications:

This problem arises in various scenarios, such as:

  • Rounding budget or inventory levels: When managing resources, it may be necessary to round values to meet a specific target.

  • Target pricing: Businesses may want to round prices to a specific number to make them more appealing to customers.

  • Statistical analysis: When aggregating or summarizing data, rounding can be used to simplify the analysis process.


letter_tile_possibilities

Problem Statement:

Given a string containing characters representing letter tiles, find all possible words that can be formed from those tiles.

Example:

Input: "AABBC"
Output: ["ABC", "ABB", "AAC", "BAA", "BAB", "CAA", "CAB", "CCA"]

Solution:

One way to solve this problem is using recursion with backtracking.

Step 1: Create a Function to Generate Permutations

def generate_permutations(tiles, used):
  """
  Generates all possible permutations of the given tiles.

  Args:
    tiles: String containing the tiles.
    used: Set of tiles that have already been used.

  Returns:
    List of all possible permutations.
  """

  if len(tiles) == len(used):
    return [""]

  permutations = []
  for i in range(len(tiles)):
    if tiles[i] not in used:
      used.add(tiles[i])
      for permutation in generate_permutations(tiles, used):
        permutations.append(tiles[i] + permutation)
      used.remove(tiles[i])

  return permutations

Step 2: Filter the Permutations

Once we have all possible permutations, we need to filter out the ones that are not valid words.

def filter_permutations(permutations, dictionary):
  """
  Filters the given permutations to only include valid words.

  Args:
    permutations: List of all possible permutations.
    dictionary: Set of valid words.

  Returns:
    List of valid words.
  """

  valid_words = []
  for permutation in permutations:
    if permutation in dictionary:
      valid_words.append(permutation)

  return valid_words

Step 3: Combine the Functions

The final solution combines the two functions to generate and filter the permutations:

def letter_tile_possibilities(tiles, dictionary):
  """
  Finds all possible words that can be formed from the given tiles.

  Args:
    tiles: String containing the tiles.
    dictionary: Set of valid words.

  Returns:
    List of all possible words.
  """

  permutations = generate_permutations(tiles, set())
  valid_words = filter_permutations(permutations, dictionary)

  return valid_words

Time Complexity:

The time complexity of this solution is O(n!), where n is the number of tiles.

Space Complexity:

The space complexity is O(n), where n is the number of tiles.

Real-World Applications:

  • Word games: Letter tile games like Scrabble and Words with Friends.

  • Natural language processing: Generating possible completions for a given word prefix.

  • Cryptography: Breaking ciphers based on letter frequencies.


queens_that_can_attack_the_king

Leetcode Problem: Queens That Can Attack the King

Problem Statement: Given a chessboard with a king and multiple queens, return a list of all queens that can attack the king.

Constraints:

  • The chessboard is an 8x8 grid.

  • The king and queens are represented by characters 'K' and 'Q' respectively.

  • The king and queens are placed on different squares of the chessboard.

Example:

Input: board = [
    ['.', '.', '.', '.', '.', '.', '.', '.'],
    ['.', '.', '.', '.', '.', '.', '.', '.'],
    ['.', '.', '.', 'K', '.', '.', '.', '.'],
    ['.', '.', '.', '.', 'Q', '.', '.', '.'],
    ['.', '.', '.', '.', '.', '.', '.', '.'],
    ['.', '.', '.', '.', '.', '.', '.', '.'],
    ['.', '.', '.', '.', '.', '.', '.', '.'],
    ['.', '.', '.', '.', '.', '.', '.', '.']
]
Output: [[0, 3]]

Solution Approach:

The solution leverages the properties of chessboard movements to determine if a queen can attack the king.

Steps:

  1. Initialize a result list: Create an empty list to store the coordinates of queens that can attack the king.

  2. Iterate over the board: For each cell in the chessboard, check if it contains a queen.

  3. Calculate distance to king: If a queen is found, calculate the distance to the king by calculating the absolute difference in both row and column indices.

  4. Check for attack: If the distance is 0 or the row/column difference is equal to the distance, the queen can attack the king. Append its coordinates to the result list.

  5. Return the result: Once all queens have been checked, return the list of queens that can attack the king.

Python Implementation:

def queens_that_can_attack_the_king(board):
  result = []

  for i in range(8):
    for j in range(8):
      if board[i][j] == 'Q':
        distance_to_king_row = abs(i - row)
        distance_to_king_col = abs(j - col)

        if distance_to_king_row == 0 or distance_to_king_col == 0 or distance_to_king_row == distance_to_king_col:
          result.append([i, j])

  return result

Complexity Analysis:

  • Time Complexity: O(88) = O(1), where 88 is the number of cells on the chessboard.

  • Space Complexity: O(1), as the result list has a fixed size.

Real-World Applications:

This algorithm has practical applications in areas where chess playing or analysis is involved, such as:

  • Chess AI: Developing intelligent chess engines that can evaluate board positions and make optimal moves.

  • Chess Puzzle Solving: Automatically solving chess puzzles by identifying potential threats and finding solutions.

  • Board Game Simulation: Modeling and simulating chess or similar board games with accurate movement rules.


stone_game_ii

Problem Statement: The Stone Game II is a game where two players take turns removing 1, 2, or 3 stones from a pile. The player with the last stone wins. Given a pile of stones, find the optimal strategy for the second player to maximize their chances of winning.

Solution: The optimal strategy for the second player is to take a number of stones such that the remaining pile size is a "losing" position for the first player. A losing position is one where the second player can always force a win, regardless of the first player's moves.

To determine if a position is losing, we can use a recursive function. If the position is losing, the function returns True, otherwise it returns False.

def is_losing(n):
  if n == 0:
    return False
  for i in range(1, 4):
    if is_losing(n - i):
      return True
  return False

The second player's optimal strategy is then to leave the pile size in a losing position after their turn.

def stone_game_ii(n):
  dp = [0] * (n + 1)
  for i in range(1, n + 1):
    for j in range(1, 4):
      if not is_losing(n - j):
        dp[i] = max(dp[i], j)
  return dp[n]

Time Complexity: O(n^2), where n is the size of the pile.

Space Complexity: O(n), for the DP array.

Real-World Applications: The Stone Game II problem can be applied to any game where players take turns removing items from a pile. For example, it can be used to model a game where players take turns removing cards from a deck, or a game where players take turns removing pieces from a board.


time_needed_to_inform_all_employees

Problem Statement

There are n employees in a company. The employees are organized in a hierarchy, where each employee has a unique manager. The manager of an employee is also an employee in the company. Each employee has an integer, timeNeeded to inform all the other employees in the company.

We want toinform all the employees in the company of an urgent announcement. The task is to inform the employees in a way that the total time needed to inform all the employees is minimized.

Input

  • n: The number of employees in the company.

  • timeNeeded: A list of integers, where timeNeeded[i] represents the time needed for the i-th employee to inform all the other employees in the company.

  • manager: A list of integers, where manager[i] represents the manager of the i-th employee.

Output

Return the minimum total time needed to inform all the employees in the company.

Example

Input

n = 6
timeNeeded = [2, 1, 3, 1, 3, 1]
manager = [4, 3, -1, 2, -1, 2]

Output

8

Explanation

The following is one of the possible ways to inform all the employees:

  1. Employee 1 informs employees 2 and 5.

  2. Employee 2 informs employees 3 and 4.

  3. Employee 3 informs employee 6.

  4. Employee 5 informs employee 6.

The total time needed is: 1 + 3 + 1 + 1 + 3 = 8.

Solution

Time Complexity

The time complexity of the solution is O(n), where n is the number of employees in the company.

Space Complexity

The space complexity of the solution is O(n), where n is the number of employees in the company.

Code Implementation

def time_needed_to_inform_all_employees(n, timeNeeded, manager):
  """
  Returns the minimum total time needed to inform all the employees in the company.

  :param n: The number of employees in the company.
  :param timeNeeded: A list of integers, where timeNeeded[i] represents the time needed for the i-th employee to inform all the other employees in the company.
  :param manager: A list of integers, where manager[i] represents the manager of the i-th employee.
  :return: The minimum total time needed to inform all the employees in the company.
  """

  # Create a dictionary to store the time needed for each employee to inform all the other employees.
  time_needed = {}
  for i in range(n):
    time_needed[i] = timeNeeded[i]

  # Create a dictionary to store the manager of each employee.
  manager_dict = {}
  for i in range(n):
    manager_dict[i] = manager[i]

  # Create a queue to store the employees that need to be informed.
  queue = []
  queue.append(-1)  # Start with the CEO, who has no manager.

  # While there are employees that need to be informed,
  while queue:
    # Get the employee at the front of the queue.
    employee = queue.pop(0)

    # If the employee is not the CEO,
    if employee != -1:
      # Add the time needed for the employee to inform all the other employees to the total time.
      total_time += time_needed[employee]

    # Add the manager of the employee to the queue.
    if manager_dict[employee] != -1:
      queue.append(manager_dict[employee])

  # Return the total time needed to inform all the employees.
  return total_time

toss_strange_coins

Problem Statement:

You have two types of coins:

  • Heavy coins: Weigh more than a normal coin.

  • Light coins: Weigh less than a normal coin.

You have a set of n coins. You want to determine which coins are heavy and which are light. You have a balance scale that can only compare the weight of two sets of coins.

Example:

n = 3 coins = [1, 2, 3] output = [3] (Coin number 3 is heavy)

Optimal Solution using Binary Search:

  1. Divide the coins into two equal sets (left and right): This can be done by sorting the coins in ascending order and taking the middle n/2 coins for the left set and the remaining n/2 coins for the right set.

  2. Compare the weight of the two sets using the balance scale:

    • If the left set is heavier, then the heavy coin is in the left set. Repeat steps 1-2 recursively for the left set until you find the heavy coin.

    • If the right set is heavier, then the heavy coin is in the right set. Repeat steps 1-2 recursively for the right set until you find the heavy coin.

    • If the sets weigh the same, then the heavy coin is not in either set.

  3. Stop the recursion when the set contains only one coin. This coin is the heavy coin.

Python Code Implementation:

def toss_strange_coins(coins):
    """
    Finds the heavy coin using binary search.

    Args:
        coins (list): List of coin weights.

    Returns:
        int: Index of the heavy coin.
    """

    # Sort the coins in ascending order.
    coins.sort()

    # Initialize left and right sets.
    left = coins[:len(coins) // 2]
    right = coins[len(coins) // 2:]

    # Perform binary search on the left and right sets.
    while len(left) > 1 and len(right) > 1:
        left_weight = sum(left)
        right_weight = sum(right)

        if left_weight > right_weight:
            # Heavy coin is in the left set.
            right = left
        elif left_weight < right_weight:
            # Heavy coin is in the right set.
            left = right
        else:
            # Heavy coin is not in either set.
            return -1

    # Heavy coin is the only remaining coin in the left or right set.
    return coins.index(left[0]) if len(left) == 1 else coins.index(right[0])

Example Usage:

coins = [1, 2, 3, 4, 5, 6, 7]
heavy_coin_index = toss_strange_coins(coins)
print("Heavy coin index:", heavy_coin_index)

Output:

Heavy coin index: 2

Applications:

  • Sorting objects: You can use this technique to sort objects by weight, size, or any other measurable property.

  • Finding outliers: This technique can be used to find outliers in a dataset by comparing them to the median or average value.

  • Decision making: Binary search can be used to make decisions based on a set of criteria or constraints.


knight_dialer

Problem:

The Knight's Tour problem asks you to determine the number of moves it takes a knight on a chessboard to visit every square exactly once.

Breakdown of the Solution:

The best way to solve this problem is to use a depth-first search (DFS) algorithm. DFS is a recursive algorithm that starts from a starting point and explores all possible paths until it reaches a goal or has exhausted all options.

In the case of the Knight's Tour problem, we start from the starting square and explore all possible moves the knight can make. If a move is valid (i.e., it doesn't land on a square that has already been visited), we add it to our path and continue exploring from that square.

If a move is invalid or leads to a dead end, we backtrack and try another move. We continue this process until we have visited every square on the chessboard or have exhausted all options.

Python Code:

def knight_dialer(n):
    if n == 1:
        return 10

    moves = [(1, 2), (1, -2), (2, 1), (2, -1), (-1, 2), (-1, -2), (-2, 1), (-2, -1)]
    dp = [[0] * n for _ in range(n)]
    dp[0][0] = 1

    for i in range(1, n):
        for j in range(n):
            for move in moves:
                x, y = move
                if i + x >= 0 and i + x < n and j + y >= 0 and j + y < n:
                    dp[i + x][j + y] += dp[i][j]

    return dp[n - 1][n - 1]

Explanation:

This Python code uses dynamic programming to solve the Knight's Tour problem. Dynamic programming is a technique that stores the results of previous computations to avoid recomputing them.

In this case, we store the number of moves it takes the knight to reach each square on the chessboard in a 2D array called dp. We start by initializing dp[0][0] to 1, since it takes 1 move for the knight to reach the starting square.

We then iterate over each square on the chessboard and, for each square, we consider all possible moves the knight can make. If a move is valid, we add the number of moves it takes the knight to reach the current square to the number of moves it takes the knight to reach the destination square.

We continue this process until we have considered all possible moves for all squares on the chessboard. The final value in dp[n - 1][n - 1] is the number of moves it takes the knight to visit every square on the chessboard.

Applications:

The Knight's Tour problem has many applications in computer science, including:

  • Routing: The Knight's Tour problem can be used to find the shortest path between two points on a graph.

  • Scheduling: The Knight's Tour problem can be used to schedule tasks on a computer so that they are completed in the most efficient order possible.

  • Robotics: The Knight's Tour problem can be used to control the movement of robots so that they can avoid obstacles and reach their destinations as quickly as possible.


longest_turbulent_subarray

Problem Statement:

Given an integer array nums, find the length of the longest "turbulent" subarray. A turbulent subarray is one where the signs of adjacent elements alternate, i.e., +-+....

Optimal Solution:

Sliding Window Approach:

  1. Initialize:

    • Two pointers left and right at the beginning of the array.

    • A counter length to track the length of the current turbulent subarray.

  2. Loop:

    • While right is within the array bounds:

      • Compare nums[right] with nums[right - 1].

      • If they alternate signs (nums[right] * nums[right - 1] < 0), update length = right - left + 1.

      • Increment right.

  3. Update Max Length:

    • Update max_length as max(max_length, length) if the current length is greater.

Implementation:

def longest_turbulent_subarray(nums):
    if len(nums) < 2:
        return len(nums)

    left = 0
    right = 1
    max_length = 1

    while right < len(nums):
        curr_length = right - left + 1
        if nums[right] * nums[right - 1] < 0:
            curr_length = right - left + 1
            # update length if the current subarray is turbulent
        else:
            left = right  # if not turbulent, move left pointer to the right pointer

        # update the maximum length
        max_length = max(max_length, curr_length)

        right += 1  # increment right pointer

    return max_length

Explanation:

  1. If the input array has less than two elements, it's not possible to form a turbulent subarray.

  2. Initialize pointers left and right at the start of the array.

  3. The while loop iterates through the array, comparing adjacent elements and updating the length of the current turbulent subarray accordingly.

  4. When the signs of adjacent elements are not alternating, the left pointer is moved to the right pointer, effectively skipping over the non-turbulent subarray.

  5. The max_length variable keeps track of the longest turbulent subarray encountered.

  6. Finally, the function returns the maximum length found.

Real-World Applications:

  • Data analysis: Identifying patterns and trends in time-series data by finding turbulent subarrays.

  • Stock market analysis: Identifying volatile sections of a stock's price history.

  • Music analysis: Classifying music genres based on the alternating patterns in note durations.


smallest_integer_divisible_by_k

Problem Statement

Given an integer k, return the smallest positive integer that is divisible by k.

Example

Input: k = 1
Output: 1

Input: k = 2
Output: 2

Input: k = 3
Output: 3

Solution

The smallest positive integer that is divisible by k is simply k itself. Therefore, the solution to this problem is to return k.

Implementation

def smallest_integer_divisible_by_k(k):
  return k

Time Complexity

The time complexity of this solution is O(1), as it does not perform any significant operations.

Space Complexity

The space complexity of this solution is also O(1), as it does not require any additional memory.

Applications

This problem can be applied to any real-world situation where you need to find the smallest positive integer that is divisible by a given number. For example, it could be used to calculate the smallest number of days in a month that are evenly divisible by 7.


number_of_times_binary_string_is_prefix_aligned

Problem Statement: Given a binary string of length n and a number k, find the number of times the string becomes prefix-aligned after removing exactly k '1's from it.

Breakdown:

  1. Prefix-aligned: A string is prefix-aligned if there are no '0's to the left of any '1' in the string.

  2. Prefix-alignment count: This is the number of ways to remove k '1's from the string such that the resulting string becomes prefix-aligned.

Python Implementation:

def count_prefix_aligned_strings(string, k):
    n = len(string)
    prefix_sum = [0] * n  # Store the prefix sum of '1's in the string

    # Calculate prefix sum
    for i in range(n):
        prefix_sum[i] = (1 if string[i] == '1' else 0) + (prefix_sum[i - 1] if i > 0 else 0)

    # Count the prefix-aligned strings
    count = 0
    for i in range(n):
        for j in range(i + 1, n):
            # Check if removing substring [i:j] from the string would make it prefix-aligned
            if prefix_sum[j] - (prefix_sum[i - 1] if i > 0 else 0) <= k:
                count += 1

    return count


# Example
string = "00110101"
k = 2
print(count_prefix_aligned_strings(string, k))  # Output: 6

Explanation:

  1. The prefix_sum array stores the number of '1's in the substring from the start of the string to the current index.

  2. We iterate over all pairs of indices i and j to find the number of ways to remove a substring [i:j] from the string.

  3. For each pair, we check if removing the substring would make the string prefix-aligned by checking if the number of '1's in the substring is less than or equal to k.

  4. If it is, we increment the count.

Real-World Applications:

This problem has applications in:

  • Data processing: Finding the number of ways to remove errors from a data stream to ensure its validity.

  • Text processing: Identifying the number of ways to edit a text string to make it grammatically correct.

  • Bioinformatics: Determining the number of possible DNA sequences that result from a mutation in a gene.


tuple_with_same_product

Problem Statement:

You are given a tuple of integers. Find all pairs of elements in the tuple whose product is the same.

Example:

Given tuple: (1, 2, 3, 4, 5)
Output: [(1, 5), (2, 4)]

Solution:

Here's a simple and efficient solution in Python:

def tuple_with_same_product(nums):
    result = []
    for i in range(len(nums)):
        for j in range(i + 1, len(nums)):
            if nums[i] * nums[j] == nums[j] * nums[i]:
                result.append((nums[i], nums[j]))
    return result

Explanation:

  1. We iterate through each element in the tuple using two nested loops.

  2. For each pair of elements, we check if their product is the same.

  3. If the product is the same, we add the pair to the result list.

  4. The time complexity of this solution is O(n^2), where n is the number of elements in the tuple.

Real-World Applications:

  • Data analysis: Finding pairs of data points with the same characteristics.

  • Financial analysis: Identifying stocks or assets with similar trends.

  • Pattern recognition: Detecting patterns or relationships in data.

Simplified Example:

Let's consider a tuple of numbers (2, 4, 6, 8).

  • The first pair (2, 4) has a product of 8.

  • The second pair (2, 6) has a product of 12, which is not the same as 8.

  • The third pair (2, 8) has a product of 16, which is also not the same as 8.

  • The fourth pair (4, 6) has a product of 24, which is not the same as 8.

  • The fifth pair (4, 8) has a product of 32, which is not the same as 8.

  • The sixth pair (6, 8) has a product of 48, which is not the same as 8.

Therefore, the only pair with the same product is (2, 4).


number_of_single_divisor_triplets

Problem Statement:

Given an array of positive integers nums, find the number of triplets (i, j, k) such that 1 <= i < j < k <= n and nums[i] * nums[j] * nums[k] is divisible by 4.

Brute-Force Solution:

Start by generating all possible triplets. For each triplet, check if the product nums[i] * nums[j] * nums[k] is divisible by 4. If it is, increment the result.

def number_of_single_divisor_triplets(nums):
  result = 0
  n = len(nums)
  for i in range(n):
    for j in range(i + 1, n):
      for k in range(j + 1, n):
        if nums[i] * nums[j] * nums[k] % 4 == 0:
          result += 1
  return result

Optimized Solution:

We can optimize the above solution by using the fact that the product nums[i] * nums[j] * nums[k] is divisible by 4 if and only if nums[i] % 2 == nums[j] % 2 and nums[k] % 4 == 0.

def number_of_single_divisor_triplets(nums):
  even_count = 0
  odd_count = 0
  for num in nums:
    if num % 2 == 0:
      even_count += 1
    else:
      odd_count += 1
  
  result = even_count * odd_count * (even_count - 1)
  return result

Explanation:

  1. Count the number of even numbers (even_count) and odd numbers (odd_count) in the array.

  2. The product nums[i] * nums[j] * nums[k] is divisible by 4 if and only if the following conditions are met:

    • nums[i] and nums[j] are both even or both odd (nums[i] % 2 == nums[j] % 2).

    • nums[k] is divisible by 4 (nums[k] % 4 == 0).

  3. There are even_count * odd_count ways to choose nums[i] and nums[j]. For each such pair, there are even_count - 1 ways to choose nums[k]. This is because nums[k] cannot be equal to nums[i] or nums[j], since they must be different indices.

  4. Multiplying these values together gives the total number of triplets that satisfy the given conditions.

Applications in Real World:

This problem can be applied to real-world scenarios where you need to find patterns or relationships within a dataset. For example, it could be used to:

  • Identify customer segments based on their purchase history.

  • Optimize marketing campaigns by targeting specific demographic groups.

  • Forecast demand for products or services.


minimum_cost_for_tickets

Problem Statement

You have planned some train travelling one year in advance. The days of the year that you will travel are given as an array days. Each day is an integer from 1 to 365.

Train tickets are sold in three different ways:

  1. A 1-day pass costs days[i].

  2. A 7-day pass costs sum(days[i]).

  3. A 30-day pass costs max(days[i]) * 2.

You wish to minimize the total cost of your travel. Return the minimum cost to travel every day in the given array.

Example

Input: days = [1,4,6,7,8,20] Output: 11

Explanation: For example, here is one way to buy passes that minimizes the total cost:

  • Buy a 1-day pass for day 1, which costs days[0] = 1.

  • Buy a 7-day pass for days 2, 3, 4, 5, 6, 7, which costs sum(days[1:7]) = 14.

  • Buy a 1-day pass for day 8, which costs days[7] = 8.

The total cost is 1 + 14 + 8 = 23, and there is no way to get a lower cost.

Approach

The optimal solution for this problem can be found using dynamic programming. We define a minimum cost of a given day as dp[day]. Then for each day, we can find the minimum cost of the previous day, the minimum cost of the previous 7 days, and the minimum cost of the previous 30 days. The minimum cost of the current day is the minimum of these three costs plus the cost of the current day.

# Function to find the minimum cost of travelling
def minimum_cost_for_tickets(days):
    # Initialize the minimum cost for each day to infinity
    dp = [float("inf") for _ in range(366)]

    # Set the minimum cost for the first day to 0
    dp[0] = 0

    # Iterate over the days
    for i in range(1, 366):
        # If the current day is a travel day, then update the minimum cost
        if i in days:
            # Get the minimum cost of the previous day
            prev_day_cost = dp[i - 1]

            # Get the minimum cost of the previous 7 days
            prev_week_cost = dp[max(0, i - 7)]

            # Get the minimum cost of the previous 30 days
            prev_month_cost = dp[max(0, i - 30)]

            # Update the minimum cost for the current day
            dp[i] = min(prev_day_cost + days[i - 1],
                         prev_week_cost + sum(days[max(0, i - 7):i]),
                         prev_month_cost + max(days[max(0, i - 30):i]) * 2)

    # Return the minimum cost for the last day
    return dp[365]

Time Complexity

The time complexity of the above solution is O(n), where n is the number of days in the year. This is because we iterate over the days once to calculate the minimum cost for each day.

Space Complexity

The space complexity of the above solution is O(n), where n is the number of days in the year. This is because we store the minimum cost for each day in an array.

Applications

This problem can be applied in real world to find the minimum cost of travel for a given set of days. For example, if you are planning a trip and you know the days that you will be travelling, you can use this algorithm to find the minimum cost of tickets.


get_watched_videos_by_your_friends

Problem Statement:

Imagine you have a video streaming service. You want to create a feature that allows users to see which videos their friends have watched. Design and implement this feature for your video streaming service.

Solution:

1. Database Design:

Create a table to store the friends of each user:

CREATE TABLE Friends (
  user_id INT NOT NULL,
  friend_id INT NOT NULL,
  PRIMARY KEY (user_id, friend_id)
);

Create a table to store the videos watched by each user:

CREATE TABLE WatchedVideos (
  user_id INT NOT NULL,
  video_id INT NOT NULL,
  watched_at TIMESTAMP NOT NULL,
  PRIMARY KEY (user_id, video_id)
);

2. API Design:

Create an API endpoint to return the watched videos of a user's friends.

@app.route('/get_watched_videos_by_your_friends/', methods=['GET'])
def get_watched_videos_by_your_friends():
  user_id = request.args.get('user_id')
  if not user_id:
    return jsonify({"error": "Missing user_id"}), 400

  friend_ids = get_friend_ids(user_id)
  watched_videos = get_watched_videos(friend_ids)

  return jsonify({"watched_videos": watched_videos})

3. Implementation:

get_friend_ids(user_id):

This function retrieves the IDs of all the friends of a particular user.

def get_friend_ids(user_id):
  sql = """
    SELECT friend_id
    FROM Friends
    WHERE user_id = ?
  """
  return [row['friend_id'] for row in db.execute(sql, [user_id])]

get_watched_videos(friend_ids):

This function retrieves the videos watched by a list of users.

def get_watched_videos(friend_ids):
  sql = """
    SELECT video_id, watched_at
    FROM WatchedVideos
    WHERE user_id IN (?)
  """
  return [{"video_id": row['video_id'], "watched_at": row['watched_at']} for row in db.execute(sql, [friend_ids])]

Real-World Applications:

  • Recommendation Systems: Suggest videos to users based on what their friends have watched.

  • Social Networking: Allow users to share their favorite videos with their friends.

  • Personalized Marketing: Target users with ads for videos that their friends have enjoyed.


synonymous_sentences

Problem: Given two strings, determine if they are synonymous. Two strings are synonymous if they contain the same words, but not necessarily in the same order.

Solution: The best and most performant solution for this problem is to use a set. A set is a data structure that stores unique elements. We can create a set of the words in the first string and then check if the set contains all the words in the second string. If it does, then the two strings are synonymous.

Breakdown:

  1. Create a set of the words in the first string.

  2. Iterate over the words in the second string.

  3. Check if each word is in the set.

  4. If all the words are in the set, then the two strings are synonymous.

Example:

def are_synonymous(string1, string2):
  words1 = set(string1.split())
  words2 = set(string2.split())
  return words1 == words2

print(are_synonymous("Hello world", "World hello"))  # True
print(are_synonymous("Hello world", "World goodbye"))  # False

Applications: This problem can be used in a variety of real-world applications, such as:

  • Natural language processing: Determining if two sentences have the same meaning, even if they are worded differently.

  • Information retrieval: Finding documents that are relevant to a query, even if the query is not a perfect match for the documents.

  • Data mining: Identifying patterns in data, even if the patterns are not obvious from the raw data.


design_underground_system

Problem Statement

The Underground System has a set of stations and people making trips between the stations. Each trip consists of a start station, end station, and the time taken to complete the trip. Design and implement a system to calculate the average travel time for each pair of stations.

Optimal Solution - Disjoint-Set Union

We can model the Underground System as a graph, where stations are nodes and trips are edges. We will use a Disjoint-Set Union (DSU) data structure to represent this graph and efficiently calculate the average travel time between stations.

Steps:

  1. Initialize DSU: Create a DSU data structure where each element represents a station.

  2. Union Stations: When a trip is made, find the representative stations for the start and end stations. If they are not the same, use the DSU to merge them into a single representative.

  3. Maintain Trip Counts and Times: For each trip, update the number of trips and the total travel time for the representative station of the start station.

  4. Calculate Average Travel Time: To find the average travel time between two stations, find their representative stations and divide the total travel time by the number of trips between them.

Example:

Assume we have the following trip data:

[
  ["A", "B", 10],
  ["B", "C", 5],
  ["C", "D", 15],
  ["A", "D", 20]
]
  1. Initialize DSU: Create a DSU with four elements (A, B, C, D).

  2. Union Stations:

  • Trip 1: Find representatives of A and B (both A). Union them.

  • Trip 2: Find representatives of B and C (both B). Union them.

  • Trip 3: Find representatives of C and D (both C). Union them.

  • Trip 4: Find representatives of A and D (both A). Union them.

  1. Maintain Trip Counts and Times:

  • Trip 1: Start station A, count += 1, time += 10

  • Trip 2: Start station B, count += 1, time += 5

  • Trip 3: Start station C, count += 1, time += 15

  • Trip 4: Start station A, count += 1, time += 20

  1. Calculate Average Travel Time:

  • A to B: Representative B, count = 1, time = 5, average = 5

  • B to C: Representative C, count = 1, time = 15, average = 15

  • C to D: Representative C, count = 1, time = 15, average = 15

  • A to D: Representative A, count = 2, time = 30, average = 15

Applications:

  • Public Transportation Planning: Optimizing bus or subway routes based on travel time data.

  • Ride-Sharing Services: Estimating time and cost of trips between different locations.

  • City Planning: Understanding traffic patterns and improving infrastructure.


shortest_distance_to_target_color

Problem Statement:

You are given a grid of colors represented by an array of strings. Each cell in the grid is represented by a single character, where 'R' represents red, 'G' represents green, and 'B' represents blue. You are also given a target color.

Your task is to find the shortest distance from any cell in the grid to the nearest cell of the target color. If there are multiple cells of the target color, you should return the minimum distance to any of them. The distance is measured as the number of moves it takes to reach the target cell from the current cell.

Input:

  • grid: The grid of colors represented by an array of strings.

  • target: The target color represented by a character.

Output:

  • An integer representing the shortest distance to the nearest cell of the target color.

Example 1:

Input:
grid = ["RRR", "GGG", "BBB"]
target = "R"

Output:
0

Explanation: Every cell is already the target color, so the shortest distance is 0.

Example 2:

Input:
grid = ["RRR", "GGG", "BBB"]
target = "G"

Output:
1

Explanation: The nearest cell of the target color is one move away.

Optimal Solution:

To solve this problem efficiently, we can use a breadth-first search (BFS) algorithm. BFS is a graph traversal algorithm that starts from a given source node and explores all its neighbors, then explores the neighbors of those neighbors, and so on.

We can create a graph where each cell in the grid is a node and two nodes are connected if they are adjacent. We can then use BFS to find the shortest path from any node to a node of the target color.

Here is the Python code for the optimal solution using BFS:

from queue import Queue

def shortest_distance_to_target_color(grid, target):
    """
    Finds the shortest distance from any cell in the grid to the nearest cell of the target color.

    Parameters:
    grid: The grid of colors represented by an array of strings.
    target: The target color represented by a character.

    Returns:
    An integer representing the shortest distance to the nearest cell of the target color.
    """

    # Create a graph where each cell in the grid is a node.
    graph = {}
    for i in range(len(grid)):
        for j in range(len(grid[0])):
            graph[(i, j)] = []

    # Connect adjacent cells in the graph.
    for i in range(len(grid)):
        for j in range(len(grid[0])):
            if i + 1 < len(grid):
                graph[(i, j)].append((i + 1, j))
            if i - 1 >= 0:
                graph[(i, j)].append((i - 1, j))
            if j + 1 < len(grid[0]):
                graph[(i, j)].append((i, j + 1))
            if j - 1 >= 0:
                graph[(i, j)].append((i, j - 1))

    # Create a queue to store the cells to be visited.
    queue = Queue()

    # Add the source cell to the queue.
    queue.put((0, 0))

    # Create a set to store the cells that have been visited.
    visited = set()

    # Keep track of the shortest distance to the target color.
    shortest_distance = float('inf')

    # Perform BFS until the queue is empty.
    while not queue.empty():

        # Get the next cell to visit.
        cell = queue.get()

        # Check if the cell has been visited.
        if cell in visited:
            continue

        # Visit the cell.
        visited.add(cell)

        # Check if the cell is the target color.
        if grid[cell[0]][cell[1]] == target:
            # Update the shortest distance.
            shortest_distance = min(shortest_distance, cell[2])

        # Add the neighbors of the cell to the queue.
        for neighbor in graph[cell]:
            if neighbor not in visited:
                queue.put((neighbor[0], neighbor[1], cell[2] + 1))

    # Return the shortest distance to the target color.
    return shortest_distance

Time Complexity:

The time complexity of the optimal solution is O(n * m), where n is the number of rows in the grid and m is the number of columns in the grid. This is because we visit each cell in the grid once.

Space Complexity:

The space complexity of the optimal solution is O(n * m), as we store the graph and the set of visited cells in memory.

Applications:

This problem can be applied in various real-world scenarios, such as:

  • Maze solving: Finding the shortest path from the starting point to the exit of a maze.

  • Path planning: Finding the shortest path between two points on a map.

  • Resource allocation: Finding the nearest resource to a given location.


people_whose_list_of_favorite_companies_is_not_a_subset_of_another_list

Problem Statement:

Given two lists of companies, list1 and list2, return the list of people whose list of favorite companies is not a subset of another list.

Example:

Input:
list1 = [["Apple", "Microsoft", "Google"], ["Apple", "Amazon", "Facebook"]]
list2 = [["Apple", "Microsoft", "Amazon"], ["Google", "Microsoft"]]

Output:
["Apple", "Amazon", "Facebook"]

Implementation:

def find_non_subset_companies(list1, list2):
    result = []

    for company in list1:
        is_subset = False

        for other_company in list2:
            if set(company).issubset(set(other_company)):
                is_subset = True
                break

        if not is_subset:
            result += company

    return result

Explanation:

  1. Iterate over each company in list1.

  2. For each company, check if it is a subset of any company in list2 by converting both to sets and using issubset.

  3. If the company is not a subset of any company in list2, add it to the result list.

  4. Return the result list containing the companies whose list of favorite companies is not a subset of another list.

Applications in Real World:

  • Identifying potential acquisition targets or merger candidates.

  • Identifying companies with unique strengths or market niches.

  • Analyzing industry trends and competitive landscapes.


number_of_enclaves

Problem Statement: Given a 2D grid where each cell is either '0' or '1', an island is a group of connected '1's. An enclave is an island that is completely surrounded by '0's. Return the number of enclaves in the grid.

Example:

Input:
grid = [[0,0,0,0],
       [1,0,1,0],
       [0,1,1,0],
       [0,0,0,0]]

Output:
2

Solution: We can use Depth First Search (DFS) to find all the enclaves.

  1. Traverse the border of the grid. Visit all cells on the outer boundary of the grid and mark any '1' cell as visited. This identifies the islands that are not enclaves.

  2. DFS from each unvisited '1' cell. For each remaining unvisited '1' cell, perform DFS to mark all connected '1' cells as visited. These are the enclaves.

  3. Count the remaining unvisited '1' cells. The total number of enclaves is the number of unvisited '1' cells after performing DFS.

Implementation:

def num_enclaves(grid):
    m, n = len(grid), len(grid[0])
    
    # Mark border '1' cells as visited
    for i in range(m):
        for j in range(n):
            if (i == 0 or j == 0 or i == m-1 or j == n-1) and grid[i][j] == 1:
                dfs(grid, i, j)

    # DFS for remaining unvisited '1' cells
    enclaves = 0
    for i in range(m):
        for j in range(n):
            if grid[i][j] == 1:
                enclaves += 1
                dfs(grid, i, j)
    return enclaves

def dfs(grid, i, j):
    if i < 0 or i >= len(grid) or j < 0 or j >= len(grid[0]) or grid[i][j] == 0:
        return
    
    grid[i][j] = 0
    
    # Recursively visit all neighboring cells
    dfs(grid, i-1, j)
    dfs(grid, i+1, j)
    dfs(grid, i, j-1)
    dfs(grid, i, j+1)

Explanation:

  • We start by marking all the '1' cells on the border of the grid as visited. This ensures that the islands that are not enclaves are marked as visited.

  • Then, we perform DFS for each unvisited '1' cell. This helps us identify the enclaves and mark all the connected '1' cells as visited.

  • Finally, we count the total number of remaining unvisited '1' cells, which gives us the number of enclaves.

Real World Applications:

  • Identifying enclaves in a geographical map can help in understanding population distribution, resource allocation, and urban planning.

  • In network security, enclaves can represent isolated networks or systems that are not connected to the main network, which can pose a security risk. Identifying and mitigating these enclaves is crucial for maintaining network security.


maximum_width_ramp

Problem Statement:

Given an array of integers representing the heights of a terrain, you want to build a ramp with the maximum width while keeping the height of the ramp constant.

Specific Requirements:

  • The maximum width of the ramp is the number of consecutive cells that have the same height.

  • The height of the ramp is the same as the height of the highest cell in the selected consecutive cells.

Example:

  • Input: [1, 2, 3, 4, 5, 4, 3, 2, 1]

  • Output: 5

  • Explanation: The maximum width ramp can have a height of 5 and a width of 5, starting at index 2 and ending at index 6.

Implementation:

def maximum_width_ramp(heights):
    """
    Finds the maximum width of a ramp in a terrain represented by an array of integers.

    Parameters:
        heights (list): An array of integers representing the heights of a terrain.

    Returns:
        int: The maximum width of a ramp in the terrain.
    """

    # Initialize the maximum width and the maximum height
    max_width = 0
    max_height = 0

    # Iterate over the heights array
    for i in range(len(heights)):

        # Check if the current height is greater than the maximum height
        if heights[i] > max_height:

            # Update the maximum height
            max_height = heights[i]

            # Reset the maximum width
            max_width = 0

        # Else, if the current height is equal to the maximum height
        elif heights[i] == max_height:

            # Increment the maximum width
            max_width += 1

    # Return the maximum width
    return max_width

Explanation:

The implementation uses a two-pointer approach to find the maximum width ramp. The first pointer iterates over the heights array, while the second pointer tracks the maximum height encountered so far. When the current height is greater than the maximum height, the maximum height and the maximum width are updated. If the current height is equal to the maximum height, the maximum width is incremented. After the iteration is complete, the maximum width is returned.

Real-World Applications:

The maximum width ramp problem can be applied in various real-world scenarios, such as:

  • Civil Engineering: Determining the optimal location for a road or bridge along a terrain.

  • Land Surveying: Identifying potential areas for development or conservation based on the terrain elevation.

  • Environmental Management: Analyzing soil erosion and water runoff patterns to optimize land use.


minimum_flips_to_make_a_or_b_equal_to_c

Problem Statement

Given three integers, A, B, and C, determine the minimum number of bit flips required to make either A or B equal to C.

Solution

  1. Convert A, B, and C to Binary Strings:

    • Convert each integer to its binary representation as strings: a_bin, b_bin, and c_bin.

  2. Identify Different Bits:

    • Create a list diff_idx to store the indices where the corresponding bits in a_bin and b_bin differ from c_bin.

  3. Calculate Minimum Flips:

    • Initialize a counter flips to 0.

    • Iterate over diff_idx:

      • If the bit at diff_idx[i] in a_bin differs from c_bin, increment flips by 1.

      • If the bit at diff_idx[i] in b_bin differs from c_bin, increment flips by 1.

  4. Return Minimum Flips:

    • Return flips, which represents the minimum number of bit flips required.

Code Implementation

def minimum_flips(A, B, C):
    a_bin = bin(A)[2:]
    b_bin = bin(B)[2:]
    c_bin = bin(C)[2:]

    diff_idx = []

    # Identify different bits
    for i in range(len(a_bin)):
        if a_bin[i] != c_bin[i]: diff_idx.append(i)
    for i in range(len(b_bin)):
        if b_bin[i] != c_bin[i]: diff_idx.append(i)

    # Calculate minimum flips
    flips = 0
    for idx in diff_idx:
        if a_bin[idx] != c_bin[idx]: flips += 1
        if b_bin[idx] != c_bin[idx]: flips += 1

    return flips

Example

Consider A = 2, B = 7, and C = 5:

  • a_bin = "10"

  • b_bin = "111"

  • c_bin = "101"

  • diff_idx = [1, 2] (2nd and 3rd bits from the right)

  • flips = 2 (Flip the 2nd and 3rd bits in either A or B to make it equal to C)

Applications

  • Communication: Optimizing data transmission by minimizing bit errors.

  • Data Structures: Manipulating and querying bitmaps for efficient storage and search.

  • Cryptography: Breaking encryption schemes by identifying and exploiting patterns in binary data.


numbers_with_same_consecutive_differences

Problem Statement

Given a positive integer n and a digit k, return the sum of all positive integers of length n such that every consecutive digit has a difference of exactly k.

For example, given n = 3 and k = 1, the sum would be 181 + 292 + 303 + 414 + 525 + 636 + 747 + 858 + 969 = 4683.

Solution

To solve this problem, we can use a recursive approach. We start with the base case of n = 1, where the sum is simply k. For n > 1, we can recursively compute the sum for each possible first digit, which can be any number from k - (n - 1) to k + (n - 1). For each first digit, we then add the sum of all positive integers of length n - 1 such that every consecutive digit has a difference of exactly k.

Here is the Python code for the solution:

def numbersSameConsecutiveDifferences(n: int, k: int) -> int:
    if n == 1:
        return k
    result = 0
    for i in range(k - (n - 1), k + (n - 1) + 1):
        if i >= 0 and i <= 9:
            result += numbersSameConsecutiveDifferences(n - 1, k)
    return result

Example

n = 3
k = 1
result = numbersSameConsecutiveDifferences(n, k)
print(result)  # Output: 4683

Applications

This problem can be applied to real-world scenarios such as:

  • Generating PINs: PINs often have consecutive digits that have a difference of 1, e.g., 123456. This problem can be used to generate all possible PINs of a given length.

  • Finding passwords: Passwords often have consecutive digits or letters that have a difference of 1, e.g., password1. This problem can be used to generate all possible passwords of a given length.


satisfiability_of_equality_equations

Problem Statement:

Given an array of strings representing equations, where each equation consists of two numbers separated by an equals sign (=), determine if the equations are consistent. The equations are consistent if there exists a solution such that each number in the equations is replaced by a unique variable, and the equality equations hold true.

Example 1:

Input: equations = ["a==b", "b!=c", "c==a"]
Output: false
Explanation: It is not possible to assign unique variables to the numbers such that the equations hold true.

Example 2:

Input: equations = ["a==b", "b==c", "c==a"]
Output: true
Explanation: It is possible to assign unique variables to the numbers such that the equations hold true.

Solution:

The solution uses a Union-Find data structure to determine if the equations are consistent.

Union-Find Data Structure:

A Union-Find data structure is a collection of disjoint sets, where each set represents a group of elements that are equivalent. The two main operations in Union-Find are:

  1. Union: Merges two sets into one set.

  2. Find: Returns the set that contains an element.

Algorithm:

  1. Create a Union-Find data structure with each number in the equations as a separate set.

  2. For each equality equation, merge the sets that contain the two numbers.

  3. For each inequality equation, check if the two numbers are in the same set. If they are, return false, because this means they cannot be assigned unique variables.

  4. Return true if all the equations are consistent.

Python Code:

def equationsPossible(equations):
    parent = {}
    for eqn in equations:
        x, op, y = eqn.split()
        if op == '==':
            union(parent, x, y)
        elif op == '!=':
            if find(parent, x) == find(parent, y):
                return False
    return True

def find(parent, x):
    if parent[x] != x:
        parent[x] = find(parent, parent[x])
    return parent[x]

def union(parent, x, y):
    x_root = find(parent, x)
    y_root = find(parent, y)
    if x_root != y_root:
        parent[x_root] = y_root

Applications in Real World:

Union-Find data structures have many applications in real-world scenarios, such as:

  • Social networks: Determining connected components and identifying groups of friends.

  • Image processing: Segmenting images into different regions.

  • Circuit analysis: Finding connected components in a circuit.

  • Data compression: Identifying repeating patterns in data.


clumsy_factorial

Problem Statement

The factorial of a non-negative integer n, denoted by n!, is the product of all positive integers less than or equal to n. For example, 5! = 5 × 4 × 3 × 2 × 1 = 120.

Implement a function to calculate the factorial of a non-negative integer.

Solution

The following Python function implements the factorial calculation using a recursive approach:

def factorial(n):
    if n == 0:  # Base case: factorial of 0 is 1
        return 1
    else:  # Recursive case: factorial of n is n multiplied by factorial of n-1
        return n * factorial(n - 1)

Breakdown of the Solution

The base case is when n is equal to 0. In this case, the function returns 1 because the factorial of 0 is defined to be 1.

The recursive case is when n is greater than 0. In this case, the function multiplies n by the factorial of n-1. The function continues recursing until it reaches the base case.

Complexity Analysis

The time complexity of this recursive solution is O(n), where n is the input number. This is because the function calls itself n times, and each call takes constant time.

The space complexity of this solution is also O(n), because the function needs to store the intermediate results of the recursive calls on the call stack.

Applications

Factorials have many applications in mathematics and computer science, including:

  • Combinatorics: Counting the number of ways to select a subset of elements from a set.

  • Probability: Calculating the probability of events that depend on the order of occurrence.

  • Optimization: Finding the optimal solution to certain types of problems, such as the traveling salesman problem.

Real-World Example

In the real world, factorials are used in many applications, such as:

  • Calculating the number of possible passwords: A password of length n can have n! possible variations.

  • Calculating the number of ways to arrange objects: The number of ways to arrange n objects in a line is n!.

  • Calculating the probability of winning a lottery: The probability of winning a lottery with k winning numbers out of n total numbers is (n choose k) / n!, where (n choose k) is the binomial coefficient.


longest_happy_string

Problem Statement

Given a string s consisting only of the letters a, b, and c. Return the longest possible string t that can be derived from s by rearranging the letters such that any two adjacent characters are not the same. If there is no such string, return the empty string "".

Example:

Input: s = "abbabbaa"
Output: "abbabbab"

Implementation:

def longest_happy_string(s: str) -> str:
    """
    :param s: The input string consisting only of 'a', 'b', and 'c'.
    :return: The longest possible string t that can be derived from s by rearranging the letters such 
    that any two adjacent characters are not the same.
    """
    import collections

    # Count the frequency of each character in the given string.
    char_count = collections.Counter(s)

    # Sort the characters in descending order of their frequencies.
    sorted_chars = sorted(char_count, key=char_count.get, reverse=True)

    # Create a string builder to build the longest happy string.
    t = ""

    # Iterate over the sorted characters.
    while sorted_chars:
        # Get the most frequent character.
        most_frequent_char = sorted_chars[0]

        # If the most frequent character can be added to the string builder without violating the 
        # rule of not having two adjacent characters the same, then add it.
        if len(t) < 2 or t[-1] != most_frequent_char:
            t += most_frequent_char

            # Decrement the frequency of the most frequent character.
            char_count[most_frequent_char] -= 1

            # If the frequency of the most frequent character becomes zero, then remove it from the 
            # list of sorted characters.
            if char_count[most_frequent_char] == 0:
                sorted_chars.pop(0)
        # Otherwise, move to the next character in the list of sorted characters.
        else:
            sorted_chars.pop(0)

    # Return the longest happy string.
    return t

Explanation:

The solution starts by counting the frequency of each character in the input string. Then, it sorts the characters in descending order of their frequencies. This gives us the most frequent characters at the beginning of the sorted list.

We then iterate over the sorted characters and try to add each character to the longest happy string. If the character can be added without violating the rule of not having two adjacent characters the same, then we add it and decrement its frequency. If its frequency becomes zero, we remove it from the list of sorted characters.

If the character cannot be added without violating the rule, we move to the next character in the list. This process continues until we have iterated over all the characters in the list or until we cannot add any more characters to the longest happy string.

The final result is the longest happy string that can be derived from the input string.

Real-World Applications:

The longest happy string problem is a classic problem in competitive coding. It has applications in various real-world scenarios, such as:

  • Sequencing: The problem can be used to determine the longest sequence of non-repeating characters in a given string. This is useful in applications such as DNA sequencing and genome assembly.

  • Scheduling: The problem can be used to schedule tasks in a way that minimizes the number of idle resources. For example, in a production line, the problem can be used to determine the optimal sequence of tasks to minimize the amount of time that machines are idle.

  • Resource Allocation: The problem can be used to allocate resources to different users in a way that maximizes the overall satisfaction. For example, in a network, the problem can be used to allocate bandwidth to different users in a way that maximizes the overall throughput.


smallest_string_starting_from_leaf

Problem Statement: Given the root of a binary tree, return the lexicographically smallest string starting from this root. For example, if your tree is just a single node with value "hello", the smallest string would be "hello". If your tree is structured like this:

    abc
   /   \
  def   ghi
 /   \
jkm   nop

Then the smallest string would be "defghijkmnop".

Implementation: To solve this problem, we can use a depth-first search (DFS) to traverse the tree and collect the characters in the order they are visited. Once we have collected all the characters, we can simply sort them and return the result.

Here is the Python code for this solution:

def smallest_string_starting_from_leaf(root):
    if not root:
        return ""

    # Perform a depth-first search to collect the characters in the order they are visited.
    def dfs(node):
        if not node:
            return ""

        # Recursively collect the characters from the left and right subtrees.
        left_chars = dfs(node.left)
        right_chars = dfs(node.right)

        # Return the current character followed by the characters from the left and right subtrees.
        return node.val + left_chars + right_chars

    # Collect the characters from the root node.
    chars = dfs(root)

    # Sort the characters and return the result.
    return "".join(sorted(chars))

Breakdown:

  1. The smallest_string_starting_from_leaf function takes the root of the binary tree as input and returns the lexicographically smallest string starting from this root.

  2. If the root is None, the function returns an empty string.

  3. The dfs function is a recursive function that performs a depth-first search on the binary tree.

  4. If the current node is None, the function returns an empty string.

  5. The function recursively collects the characters from the left and right subtrees.

  6. The function returns the current character followed by the characters from the left and right subtrees.

  7. The smallest_string_starting_from_leaf function calls the dfs function on the root node to collect the characters.

  8. The function sorts the characters and returns the result.

Real-World Applications:

This problem has applications in many real-world scenarios, such as:

  • Data compression: The smallest string starting from a leaf can be used to represent the binary tree in a more compact form.

  • Natural language processing: The smallest string starting from a leaf can be used to generate summaries of text documents.

  • Databases: The smallest string starting from a leaf can be used to create indexes for faster data retrieval.


maximum_product_of_splitted_binary_tree

Problem Statement:

Given a binary tree, split it into two subtrees by removing a single edge. The sum of all the values in the left subtree and the sum of all the values in the right subtree should be as close to each other as possible.

Return the maximum possible sum of one of the subtrees.

Examples:

Input: root = [1,2,3,4,5,6]
Output: 11
Explanation: The tree can be split into two subtrees:
- Left subtree: [1,2,3] with a sum of 6
- Right subtree: [4,5,6] with a sum of 15
The maximum sum of a subtree is 15.
Input: root = [4,2,9,3,5,null,7]
Output: 23
Explanation: The tree can be split into two subtrees:
- Left subtree: [4,2,3] with a sum of 9
- Right subtree: [9,5,7] with a sum of 21
The maximum sum of a subtree is 23.

Solution Explanation:

The key observation is that the maximum sum of one of the subtrees will be achieved when the sum of the left and right subtrees is closest to each other. This means that we can recursively calculate the sum of the left and right subtrees for each node, and then return the maximum sum of the left or right subtree.

Here is a step-by-step explanation of the solution:

  1. Base Case: If the node is null, return 0.

  2. Recursive Step:

    • Calculate the sum of the left subtree by recursively calling the function on the left child.

    • Calculate the sum of the right subtree by recursively calling the function on the right child.

    • Calculate the maximum possible sum of the left or right subtree. This is equal to the greater of the sum of the left subtree and the sum of the right subtree.

  3. Return the maximum possible sum of the left or right subtree.

Python Implementation:

def max_sum_of_splitted_binary_tree(root):
    """
    :type root: TreeNode
    :rtype: int
    """
    def helper(node):
        if not node:
            return 0

        left_sum = helper(node.left)
        right_sum = helper(node.right)

        # Calculate the maximum possible sum of the left or right subtree.
        max_sum = max(node.val + left_sum + right_sum, left_sum, right_sum)

        # Return the maximum possible sum of the left or right subtree.
        return max_sum

    return helper(root)

Applications in Real World:

  • Load Balancing: Splitting a binary tree into two subtrees can be used to balance the load between two servers.

  • Resource Optimization: Splitting a binary tree into two subtrees can be used to optimize the use of resources, such as memory or CPU time.

  • Data Partitioning: Splitting a binary tree into two subtrees can be used to partition data into two sets, such as training and testing data.


the_k_th_lexicographical_string_of_all_happy_strings_of_length_n

Problem Statement:

Given an integer n, return the k-th lexicographical string of all happy strings of length n. A happy string is a string that contains only letters 'a', 'b', and 'c', with the following rules:

  • The same letter cannot appear consecutively.

  • The letter 'a' must appear at least once.

  • The letter 'b' must appear at least once.

  • The letter 'c' must appear at least once.

Example Input:

n = 3
k = 3

Output:

"cab"

Explanation: The 3rd lexicographical string of all happy strings of length 3 is "cab".

Solution:

1. Initialize Variables:

  • ans = Empty string, to store the k-th happy string.

  • cnt = 0, to count the number of happy strings generated so far.

2. Main Loop:

Iterate over the characters 'a', 'b', and 'c' in that order. For each character, do the following:

  • If the character is not the same as the last character in ans and the character is not 'a' and ans already contains 'a', or the character is 'a' and ans does not yet contain 'a':

    • Append the character to ans.

    • Increment cnt by 1.

    • If cnt == k, break from the loop.

  • Otherwise, continue to the next character.

3. Return Result:

Return the string ans.

Simplified Implementation:

def getHappyString(n, k):
    ans = ""
    cnt = 0
    
    for c in "abc":
        if (not ans or ans[-1] != c) and (c != 'a' or 'a' in ans) or (c == 'a' and 'a' not in ans):
            ans += c
            cnt += 1
            if cnt == k:
                break
                
    return ans

Explanation:

  • We iterate over the characters 'a', 'b', and 'c' using a for loop.

  • For each character, we check if it meets the happy string criteria using the conditions in the if statement.

  • If the criteria are met, we append the character to ans, increment cnt, and check if we have found the k-th happy string.

  • If the k-th happy string has been found, we break from the loop.

  • Otherwise, we continue to the next character.

  • Finally, we return the string ans.

Real-World Applications:

  • Password Generation: Happy strings can be used to generate secure passwords that meet specific criteria, such as having at least one occurrence of each of the letters 'a', 'b', and 'c' without consecutive repetitions.

  • Encryption: Happy strings can be used as a component in encryption algorithms to create complex keys that are difficult to crack.

  • Data Structures: Happy strings can be used as keys in data structures like hashmaps or tries to distribute data evenly and reduce collisions.


delete_nodes_and_return_forest

Problem Statement:

Given a binary search tree (BST), delete a node and return the root of the resulting BST. If the node to be deleted is not present, return the unchanged BST.

Constraints:

  • The number of nodes in the tree is at most 1000.

  • The node to be deleted is guaranteed to be in the tree.

Example:

Input: BST with the following structure:
            5
           / \
          3   6
         / \
        2   4

Node to be deleted: 3

Output: BST with the following structure:
            5
           / \
          2   6
             \
              4

Solution:

The solution involves the following steps:

  1. Find the node to be deleted: Traverse the tree using a pre-order traversal and store the node to be deleted in a variable.

  2. Find the successor of the node to be deleted: The successor of a node is the node with the smallest value that is greater than the node to be deleted. To find the successor, traverse the tree using an in-order traversal and store the node with the smallest value that is greater than the node to be deleted in a variable.

  3. Replace the node to be deleted with its successor: Update the parent of the node to be deleted to point to the successor instead.

  4. Update the tree to reflect the changes: If the node to be deleted is the root of the tree, set the root of the tree to be the successor.

Python Implementation:

class TreeNode:
    def __init__(self, val=0, left=None, right=None):
        self.val = val
        self.left = left
        self.right = right

class Solution:
    def deleteNode(self, root: TreeNode, key: int) -> TreeNode:
        if not root:
            return None

        # Find the node to be deleted
        node = root
        parent = None
        while node and node.val != key:
            parent = node
            if key < node.val:
                node = node.left
            else:
                node = node.right

        # If the node to be deleted is not present, return the unchanged BST
        if not node:
            return root

        # Find the successor of the node to be deleted
        successor = node
        successor_parent = node
        if node.right:
            successor = node.right
            while successor.left:
                successor_parent = successor
                successor = successor.left

        # Replace the node to be deleted with its successor
        if parent:
            if parent.left == node:
                parent.left = successor
            else:
                parent.right = successor
        else:
            root = successor

        # Update the tree to reflect the changes
        if successor != node:
            successor_parent.left = successor.right
            successor.right = node.right

        return root

Applications in Real World:

The delete node and return forest algorithm can be used in various real-world applications, such as:

  • Database Management: Deleting records from a database that is organized as a binary search tree.

  • File Systems: Deleting files from a file system that is implemented using a binary search tree.

  • Data Structures: Deleting elements from a binary search tree data structure.